0% found this document useful (0 votes)
43 views515 pages

MI Vol 2

Here are the steps to solve this problem: 1. Given: a, b, c are positive real numbers 2. To prove: P/[(2a+b)(2a+c)] ≤ 1/a 3. Note that (2a+b)(2a+c) ≥ 4a2 4. Also, P = a + b + c ≥ a 5. Dividing both sides of the inequality in step 4 by (2a+b)(2a+c), we get: P/[(2a+b)(2a+c)] ≤ a/4a2 = 1/a Therefore, the given inequality is proved.
Copyright
© © All Rights Reserved
We take content rights seriously. If you suspect this is your content, claim it here.
Available Formats
Download as PDF, TXT or read online on Scribd
0% found this document useful (0 votes)
43 views515 pages

MI Vol 2

Here are the steps to solve this problem: 1. Given: a, b, c are positive real numbers 2. To prove: P/[(2a+b)(2a+c)] ≤ 1/a 3. Note that (2a+b)(2a+c) ≥ 4a2 4. Also, P = a + b + c ≥ a 5. Dividing both sides of the inequality in step 4 by (2a+b)(2a+c), we get: P/[(2a+b)(2a+c)] ≤ a/4a2 = 1/a Therefore, the given inequality is proved.
Copyright
© © All Rights Reserved
We take content rights seriously. If you suspect this is your content, claim it here.
Available Formats
Download as PDF, TXT or read online on Scribd
You are on page 1/ 515



                  


     !    !       "  
    #  !!            


         !      # $   % 
 &              
         '  (      " )

 
             #    # 
     $      *    "    
  +  , - ./        0  1   

   
  2 - 3       (  #    #   4
!     #        #     
5 6&    /     .       )
 !     5#         !
  "            $   
    #      #      
 "   !         # ! ! 7  
  #    "

 

     ( 7  &    


   
8!    )   (    ( !
   9   &    " 5     
        # 
#        #              
  #  !  #       "




Vasile Cîrtoaje

▀▀▀▀▀▀▀▀▀▀▀▀▀▀▀▀▀▀▀▀▀▀▀▀▀
MATHEMATICAL
INEQUALITIES
▀▀▀▀▀▀▀▀▀▀▀▀▀▀▀▀▀▀▀▀▀▀▀▀▀

Volume 2

SYMMETRIC
RATIONAL AND NONRATIONAL
INEQUALITIES

LAP LAMBERT Academic Publishing

EDITURA UNIVERSITĂŢII PETROL-GAZE DIN PLOIEŞTI


2021
Contents

1 Symmetric Rational Inequalities 1


1.1 Applications . . . . . . . . . . . . . . . . . . . . . . . . . . . . . . . . . . . 1
1.2 Solutions . . . . . . . . . . . . . . . . . . . . . . . . . . . . . . . . . . . . . 31

2 Symmetric Nonrational Inequalities 275


2.1 Applications . . . . . . . . . . . . . . . . . . . . . . . . . . . . . . . . . . . 275
2.2 Solutions . . . . . . . . . . . . . . . . . . . . . . . . . . . . . . . . . . . . . 291

3 Symmetric Power-Exponential Inequalities 439


3.1 Applications . . . . . . . . . . . . . . . . . . . . . . . . . . . . . . . . . . . 439
3.2 Solutions . . . . . . . . . . . . . . . . . . . . . . . . . . . . . . . . . . . . . 445

A Glosar 501

B Bibliography 509

i
ii Vasile Cîrtoaje
Chapter 1

Symmetric Rational Inequalities

1.1 Applications

1.1. If a, b are nonnegative real numbers, then


1 1 1
+ ≥ .
(1 + a)2 (1 + b)2 1 + ab

1.2. Let a, b, c be positive real numbers. Prove that


(a) if abc ≤ 1, then
1 1 1
+ + ≥ 1;
2a + 1 2b + 1 2c + 1
(b) if abc ≥ 1, then
1 1 1
+ + ≤ 1.
a+2 b+2 c+2

1.3. If 0 ≤ a, b, c ≤ 1, then
1 1 1 1 1 1
 ‹  ‹
2 + + ≥3 + + .
a+b b+c c+a 2a + 1 2b + 1 2c + 1

1.4. If a, b, c are nonnegative real numbers such that a + b + c ≤ 3, then

1 1 1 1 1 1
 ‹  ‹
2 + + ≥5 + + .
a+b b+c c+a 2a + 3 2b + 3 2c + 3

1
2 Vasile Cîrtoaje

1.5. If a, b, c are nonnegative real numbers, then

a2 − bc b2 − ca c 2 − ab
+ + ≥ 0.
3a + b + c 3b + c + a 3c + a + b

1.6. If a, b, c are positive real numbers, then

4a2 − b2 − c 2 4b2 − c 2 − a2 4c 2 − a2 − b2
+ + ≤ 3.
a(b + c) b(c + a) c(a + b)

1.7. Let a, b, c be nonnegative real numbers, no two of which are zero. Prove that
1 1 1 3
(a) + + ≥ ;
a2 + bc b2 + ca c 2 + ab ab + bc + ca
1 1 1 2
(b) + + ≥ .
2a2 + bc 2b2 + ca 2c 2 + ab ab + bc + ca
1 1 1 2
(c) + 2 + 2 > .
a2 + 2bc b + 2ca c + 2ab ab + bc + ca

1.8. Let a, b, c be nonnegative real numbers, no two of which are zero. Prove that

a(b + c) b(c + a) c(a + b)


+ 2 + 2 ≥ 2.
a2 + bc b + ca c + ab

1.9. Let a, b, c be nonnegative real numbers, no two of which are zero. Prove that

a2 b2 c2 a b c
+ + ≥ + + .
b +c
2 2 c +a
2 2 a +b
2 2 b+c c+a a+b

1.10. Let a, b, c be positive real numbers. Prove that

1 1 1 a b c
+ + ≥ 2 + 2 + 2 .
b+c c+a a+b a + bc b + ca c + ab

1.11. Let a, b, c be positive real numbers. Prove that

1 1 1 2a 2b 2c
+ + ≥ 2 + 2 + 2 .
b + c c + a a + b 3a + bc 3b + ca 3c + ab
Symmetric Rational Inequalities 3

1.12. Let a, b, c be nonnegative real numbers, no two of which are zero. Prove that
a b c 13 2(ab + bc + ca)
(a) + + ≥ − ;
b+c c+a a+b 6 3(a2 + b2 + c 2 )

3 p ab + bc + ca
 ‹
a b c
(b) + + − ≥ ( 3 − 1) 1 − 2 .
b+c c+a a+b 2 a + b2 + c 2

1.13. Let a, b, c be positive real numbers. Prove that


‹2
1 1 1 a+b+c

+ + ≤ .
a2 + 2bc b2 + 2ca c 2 + 2ab ab + bc + ca

1.14. Let a, b, c be nonnegative real numbers, no two of which are zero. Prove that

a2 (b + c) b2 (c + a) c 2 (a + b)
+ 2 + 2 ≥ a + b + c.
b2 + c 2 c + a2 a + b2

1.15. Let a, b, c be nonnegative real numbers, no two of which are zero. Prove that

a2 + b2 b2 + c 2 c 2 + a2 3(a2 + b2 + c 2 )
+ + ≤ .
a+b b+c c+a a + b + c)

1.16. Let a, b, c be positive real numbers. Prove that

1 1 1 9
+ 2 + 2 ≥ .
a2 + ab + b 2 b + bc + c 2 c + ca + a 2 (a + b + c)2

1.17. Let a, b, c be nonnegative real numbers, no two of which are zero. Prove that

a2 b2 c2 1
+ + ≤ .
(2a + b)(2a + c) (2b + c)(2b + a) (2c + a)(2c + b) 3

1.18. Let a, b, c be positive real numbers. Prove that


P a 1
(a) ≤ ;
(2a + b)(2a + c) a + b + c

P a3 1
(b) ≤ .
(2a2 + b2 )(2a2 + c 2 ) a + b + c
4 Vasile Cîrtoaje

1.19. If a, b, c are positive real numbers, then


X 1 1 2
≥ + .
(a + 2b)(a + 2c) (a + b + c)2 3(ab + bc + ca)

1.20. Let a, b, c be nonnegative real numbers, no two of which are zero. Prove that
1 1 1 4
(a) + + ≥ ;
(a − b)2 (b − c)2 (c − a)2 ab + bc + ca
1 1 1 3
(b) + + ≥ ;
a2 − ab + b2 b2 − bc + c 2 c 2 − ca + a2 ab + bc + ca
1 1 1 5
(c) + + ≥ .
a2 + b2 b2 + c 2 c 2 + a2 2(ab + bc + ca)

1.21. If a, b, c are positive real numbers, then

(a2 + b2 )(a2 + c 2 ) (b2 + c 2 )(b2 + a2 ) (c 2 + a2 )(c 2 + b2 )


+ + ≥ a2 + b2 + c 2 .
(a + b)(a + c) (b + c)(b + a) (c + a)(c + b)

1.22. Let a, b, c be positive real numbers such that a + b + c = 3. Prove that


1 1 1
+ + ≤ 1.
a2 + b + c b2 + c + a c 2 + a + b

1.23. Let a, b, c be nonnegative real numbers such that a + b + c = 3. Prove that

a2 − bc b2 − ca c 2 − ab
+ 2 + 2 ≥ 0.
a2 + 3 b +3 c +3

1.24. Let a, b, c be nonnegative real numbers such that a + b + c = 3. Prove that


1 − bc 1 − ca 1 − ab
+ + ≥ 0.
5 + 2a 5 + 2b 5 + 2c

1.25. Let a, b, c be positive real numbers such that a + b + c = 3. Prove that


1 1 1 3
+ 2 + 2 ≤ .
a2 + b +2 b +c +2 c +a +2 4
2 2 2
Symmetric Rational Inequalities 5

1.26. Let a, b, c be positive real numbers such that a + b + c = 3. Prove that

1 1 1 1
+ 2 + 2 ≤ .
4a2 +b +c
2 2 4b + c + a
2 2 4c + a + b
2 2 2

1.27. Let a, b, c be nonnegative real numbers such that a + b + c = 2. Prove that

bc ca ab
+ 2 + 2 ≤ 1.
a2 +1 b +1 c +1

1.28. Let a, b, c be nonnegative real numbers such that a + b + c = 1. Prove that

bc ca ab 1
+ + ≤ .
a+1 b+1 c+1 4

1.29. Let a, b, c be positive real numbers such that a + b + c = 1. Prove that

1 1 1 3
+ + ≤ .
a(2a + 1) b(2b + 1) c(2c + 1) 11abc
2 2 2

1.30. Let a, b, c be positive real numbers such that a + b + c = 3. Prove that

1 1 1
+ 3 + 3 ≤ 1.
a3 +b+c b +c+a c +a+b

1.31. Let a, b, c be positive real numbers such that a + b + c = 3. Prove that

a2 b2 c2
+ + ≥ 1.
1 + b3 + c 3 1 + c 3 + a3 1 + a3 + b3

1.32. Let a, b, c be nonnegative real numbers such that a + b + c = 3. Prove that

1 1 1 3
+ + ≤ .
6 − a b 6 − bc 6 − ca 5

1.33. Let a, b, c be nonnegative real numbers such that a + b + c = 3. Prove that

1 1 1 1
+ + ≤ .
2a2 + 7 2b2 + 7 2c 2 + 7 3
6 Vasile Cîrtoaje

1.34. Let a, b, c be nonnegative real numbers such that a + b + c = 3. Prove that

1 1 1 3
+ 2 + 2 ≥ .
2a2 + 3 2b + 3 2c + 3 5

1.35. Let a, b, c be nonnegative real numbers such that ab + bc + ca = 3. Prove


that
1 1 1 a+b+c 3
+ + ≥ + .
a+b b+c c+a 6 a+b+c

1.36. Let a, b, c be nonnegative real numbers such that ab + bc + ca = 3. Prove


that
1 1 1 3
+ 2 + 2 ≥ .
a +1 b +1 c +1 2
2

1.37. Let a, b, c be positive real numbers such that ab + bc + ca = 3. Prove that

a2 b2 c2
+ + ≥ 1.
a2 + b + c b2 + c + a c 2 + a + b

1.38. Let a, b, c be positive real numbers such that ab + bc + ca = 3. Prove that

bc + 4 ca + 4 ab + 4 bc + 2 ca + 2 ab + 2
+ 2 + 2 ≤3≤ 2 + + .
a +4 b +4 c +4
2 a + 2 b2 + 2 c 2 + 2

1.39. Let a, b, c be nonnegative real numbers such that ab + bc + ca = 3. If


p
k ≥ 2 + 3,

then
1 1 1 3
+ + ≤ .
a+k b+k c+k 1+k

1.40. Let a, b, c be nonnegative real numbers such that a2 + b2 + c 2 = 3. Prove that

a(b + c) b(c + a) c(a + b)


+ + ≤ 3.
1 + bc 1 + ca 1 + ab
Symmetric Rational Inequalities 7

1.41. Let a, b, c be positive real numbers such that a2 + b2 + c 2 = 3. Prove that

a2 + b2 b2 + c 2 c 2 + a2
+ + ≥ 3.
a+b b+c c+a

1.42. Let a, b, c be positive real numbers such that a2 + b2 + c 2 = 3. Prove that


ab bc ca 7
+ + + 2 ≤ (a + b + c).
a+b b+c c+a 6

1.43. Let a, b, c be positive real numbers such that a2 + b2 + c 2 = 3. Prove that


1 1 1 3
(a) + + ≤ ;
3 − ab 3 − bc 3 − ca 2
1 1 1
(b) + + ≤ 1;
5 − 2ab 5 − 2bc 5 − 2ca
1 1 1 3
(c) p +p +p ≤p .
6 − ab 6 − bc 6 − ca 6−1

1.44. Let a, b, c be positive real numbers such that a2 + b2 + c 2 = 3. Prove that


1 1 1 3
+ + ≥ .
1+a 5 1+ b 5 1+c 5 2

1.45. Let a, b, c be positive real numbers such that abc = 1. Prove that
1 1 1
+ 2 + 2 ≥ 1.
a2 +a+1 b + b+1 c +c+1

1.46. Let a, b, c be positive real numbers such that abc = 1. Prove that
1 1 1
+ 2 + 2 ≤ 3.
a2 −a+1 b − b+1 c −c+1

1.47. Let a, b, c be positive real numbers such that abc = 1. Prove that
3+a 3+ b 3+c
+ + ≥ 3.
(1 + a)2 (1 + b)2 (1 + c)2
8 Vasile Cîrtoaje

1.48. Let a, b, c be positive real numbers such that abc = 1. Prove that

7 − 6a 7 − 6b 7 − 6c
+ + ≥ 1.
2 + a2 2 + b2 2 + c 2

1.49. Let a, b, c be positive real numbers such that abc = 1. Prove that

a6 b6 c6
+ + ≥ 1.
1 + 2a5 1 + 2b5 1 + 2c 5

1.50. Let a, b, c be positive real numbers such that abc = 1. Prove that

a b c 1
+ 2 + 2 ≤ .
a2 +5 b +5 c +5 2

1.51. Let a, b, c be positive real numbers such that abc = 1. Prove that

1 1 1 2
+ + + ≥ 1.
(1 + a)2 (1 + b)2 (1 + c)2 (1 + a)(1 + b)(1 + c)

1.52. Let a, b, c be nonnegative real numbers such that

1 1 1 3
+ + = .
a+b b+c c+a 2
Prove that
3 2 1
≥ + 2 .
a+b+c ab + bc + ca a + b2 + c 2

1.53. Let a, b, c be nonnegative real numbers such that

7(a2 + b2 + c 2 ) = 11(ab + bc + ca).

Prove that
51 a b c
≤ + + ≤ 2.
28 b+c c+a a+b

1.54. Let a, b, c be nonnegative real numbers, no two of which are zero. Prove that

1 1 1 10
+ 2 + 2 ≥ .
a2 +b 2 b +c 2 c +a 2 (a + b + c)2
Symmetric Rational Inequalities 9

1.55. Let a, b, c be nonnegative real numbers, no two of which are zero. Prove that
1 1 1 3
+ + ≥ .
a2 − ab + b2 b2 − bc + c 2 c 2 − ca + a2 max{ab, bc, ca}

1.56. Let a, b, c be nonnegative real numbers, no two of which are zero. Prove that
a(2a + b + c) b(2b + c + a) c(2c + a + b)
+ + ≥ 6.
b2 + c 2 c 2 + a2 a2 + b2

1.57. Let a, b, c be nonnegative real numbers, no two of which are zero. Prove that

a2 (b + c)2 b2 (c + a)2 c 2 (a + b)2


+ 2 + 2 ≥ 2(ab + bc + ca).
b2 + c 2 c + a2 a + b2

1.58. If a, b, c are positive real numbers, then


1 1 1
 ‹  ‹
X a a b c
3 +5 + + ≥8 + + .
b2 − bc + c 2 bc ca ab a b c

1.59. Let a, b, c be nonnegative real numbers, no two of which are zero. Prove that
1 1 1
 ‹
(a) 2abc + + + a2 + b2 + c 2 ≥ 2(ab + bc + ca);
a+b b+c c+a

a2 b2 c2 3(a2 + b2 + c 2 )
(b) + + ≤ .
a+b b+c c+a 2(a + b + c)

1.60. Let a, b, c be nonnegative real numbers, no two of which are zero. Prove that
a2 − bc b2 − ca c 2 − ab 3(ab + bc + ca)
(a) + 2 + 2 + ≥ 3;
b2 + c 2 c + a2 a + b2 a2 + b2 + c 2
a2 b2 c2 ab + bc + ca 5
(b) + + + ≥ ;
b2 + c 2 c 2 + a2 a2 + b2 a2 + b2 + c 2 2
a2 + bc b2 + ca c 2 + ab ab + bc + ca
(c) + + ≥ + 2.
b2 + c 2 c 2 + a2 a2 + b2 a2 + b2 + c 2

1.61. Let a, b, c be nonnegative real numbers, no two of which are zero. Prove that

a2 b2 c2 (a + b + c)2
+ + ≥ .
b2 + c 2 c 2 + a2 a2 + b2 2(ab + bc + ca)
10 Vasile Cîrtoaje

1.62. Let a, b, c be nonnegative real numbers, no two of which are zero. Prove that

2ab 2bc 2ca a2 + b2 + c 2 5


+ + + ≥ .
(a + b)2 (b + c)2 (c + a)2 ab + bc + ca 2

1.63. Let a, b, c be nonnegative real numbers, no two of which are zero. Prove that

ab bc ca 1 ab + bc + ca
+ + + ≥ 2 .
(a + b)2 (b + c)2 (c + a)2 4 a + b2 + c 2

1.64. Let a, b, c be nonnegative real numbers, no two of which are zero. Prove that

3ab 3bc 3ca ab + bc + ca 5


+ + ≤ + .
(a + b)2 (b + c)2 (c + a)2 a2 + b2 + c 2 4

1.65. Let a, b, c be nonnegative real numbers, no two of which are zero. Prove that
a3 + abc b3 + abc c 3 + abc
(a) + + ≥ a2 + b2 + c 2 ;
b+c c+a a+b
a3 + 2abc b3 + 2abc c 3 + 2abc 1
(b) + + ≥ (a + b + c)2 ;
b+c c+a a+b 2
a3 + 3abc b3 + 3abc c 3 + 3abc
(c) + + ≥ 2(ab + bc + ca).
b+c c+a a+b

1.66. Let a, b, c be nonnegative real numbers, no two of which are zero. Prove that

a3 + 3abc b3 + 3abc c 3 + 3abc


+ + ≥ a + b + c.
(b + c)2 (c + a)2 (a + b)2

1.67. Let a, b, c be nonnegative real numbers, no two of which are zero. Prove that
a3 + 3abc b3 + 3abc c 3 + 3abc 3
(a) + + ≥ ;
(b + c)3 (c + a)3 (a + b)3 2
3a3 + 13abc 3b3 + 13abc 3c 3 + 13abc
(b) + + ≥ 6.
(b + c)3 (c + a)3 (a + b)3
Symmetric Rational Inequalities 11

1.68. Let a, b, c be nonnegative real numbers, no two of which are zero. Prove that
a3 b3 c3 3
(a) + + + ab + bc + ca ≥ (a2 + b2 + c 2 );
b+c c+a a+b 2
2a2 + bc 2b2 + ca 2c 2 + ab 9(a2 + b2 + c 2 )
(b) + + ≥ .
b+c c+a a+b 2(a + b + c)

1.69. Let a, b, c be nonnegative real numbers, no two of which are zero. Prove that

a(b + c) b(c + a) c(a + b)


+ + ≥ 2.
b2 + bc + c 2 c 2 + ca + a2 a2 + ab + b2

1.70. Let a, b, c be nonnegative real numbers, no two of which are zero. Prove that

a(b + c) b(c + a) c(a + b) Y  a − b ‹2


+ + ≥2+4 .
b2 + bc + c 2 c 2 + ca + a2 a2 + ab + b2 a+b

1.71. Let a, b, c be nonnegative real numbers, no two of which are zero. Prove that

ab − bc + ca bc − ca + ab ca − ab + bc 3
+ + ≥ .
b +c
2 2 c +a
2 2 a +b
2 2 2

1.72. Let a, b, c be nonnegative real numbers, no two of which are zero. If k > −2,
then
X ab + (k − 1)bc + ca 3(k + 1)
≥ .
b2 + kbc + c 2 k+2

1.73. Let a, b, c be nonnegative real numbers, no two of which are zero. If k > −2,
then
X 3bc − a(b + c) 3
≤ .
b + kbc + c
2 2 k+2

1.74. Let a, b, c be nonnegative real numbers such that ab + bc + ca = 3. Prove


that
ab + 1 bc + 1 ca + 1 4
+ + ≥ .
a2 + b2 b2 + c 2 c 2 + a2 3
12 Vasile Cîrtoaje

1.75. Let a, b, c be nonnegative real numbers such that ab + bc + ca = 3. Prove


that
5ab + 1 5bc + 1 5ca + 1
+ + ≥ 2.
(a + b)2 (b + c)2 (c + a)2

1.76. Let a, b, c be nonnegative real numbers, no two of which are zero. Prove that

a2 − bc b2 − ca c 2 − ab
+ + ≥ 0.
2b2 − 3bc + 2c 2 2c 2 − 3ca + 2a2 2a2 − 3ab + 2b2

1.77. Let a, b, c be nonnegative real numbers, no two of which are zero. Prove that

2a2 − bc 2b2 − ca 2c 2 − ab
+ + ≥ 3.
b2 − bc + c 2 c 2 − ca + a2 a2 − ab + b2

1.78. Let a, b, c be nonnegative real numbers, no two of which are zero. Prove that

a2 b2 c2
+ + ≥ 1.
2b2 − bc + 2c 2 2c 2 − ca + 2a2 2a2 − ab + 2b2

1.79. Let a, b, c be nonnegative real numbers, no two of which are zero. Prove that
1 1 1 9
+ + ≥ .
4b2 − bc + 4c 2 4c 2 − ca + 4a2 4a2 − ab + 4b2 7(a2 + b2 + c 2 )

1.80. Let a, b, c be nonnegative real numbers, no two of which are zero. Prove that

2a2 + bc 2b2 + ca 2c 2 + ab 9
+ 2 + 2 ≥ .
b2 + c 2 c + a2 a + b2 2

1.81. Let a, b, c be nonnegative real numbers, no two of which are zero. Prove that

2a2 + 3bc 2b2 + 3ca 2c 2 + 3ab


+ + ≥ 5.
b2 + bc + c 2 c 2 + ca + a2 a2 + ab + b2

1.82. Let a, b, c be nonnegative real numbers, no two of which are zero. Prove that

2a2 + 5bc 2b2 + 5ca 2c 2 + 5ab 21


+ + ≥ .
(b + c)2 (c + a)2 (a + b)2 4
Symmetric Rational Inequalities 13

1.83. Let a, b, c be nonnegative real numbers, no two of which are zero. If k > −2,
then
X 2a2 + (2k + 1)bc 3(2k + 3)
≥ .
b2 + kbc + c 2 k+2

1.84. Let a, b, c be nonnegative real numbers, no two of which are zero. If k > −2,
then
X 3bc − 2a2 3
≤ .
b + kbc + c
2 2 k+2

1.85. If a, b, c are nonnegative real numbers, no two of which are zero, then

a2 + 16bc b2 + 16ca c 2 + 16ab


+ 2 + 2 ≥ 10.
b2 + c 2 c + a2 a + b2

1.86. If a, b, c are nonnegative real numbers, no two of which are zero, then

a2 + 128bc b2 + 128ca c 2 + 128ab


+ + ≥ 46.
b2 + c 2 c 2 + a2 a2 + b2

1.87. If a, b, c are nonnegative real numbers, no two of which are zero, then

a2 + 64bc b2 + 64ca c 2 + 64ab


+ + ≥ 18.
(b + c)2 (c + a)2 (a + b)2

1.88. Let a, b, c be nonnegative real numbers, no two of which are zero. If k ≥ −1,
then
X a2 (b + c) + kabc
≥ a + b + c.
b2 + kbc + c 2

−3
1.89. Let a, b, c be nonnegative real numbers, no two of which are zero. If k ≥ ,
2
then
X a3 + (k + 1)abc
≥ a + b + c.
b2 + kbc + c 2

1.90. Let a, b, c be nonnegative real numbers, no two of which are zero. If k > 0,
then
2a k − b k − c k 2b k − c k − a k 2c k − a k − b k
+ 2 + 2 ≥ 0.
b2 − bc + c 2 c − ca + a2 a − ab + b2
14 Vasile Cîrtoaje

1.91. If a, b, c are the lengths of the sides of a triangle, then


b+c−a c+a−b a+b−c 2(a + b + c)
(a) + + ≥ ;
b2 − bc + c 2 c 2 − ca + a2 a2 − ab + b2 a2 + b2 + c 2
2bc − a2 2ca − b2 2ab − c 2
(b) + + ≥ 0.
b2 − bc + c 2 c 2 − ca + a2 a2 − ab + b2

1.92. If a, b, c are nonnegative real numbers, then


a2 b2 c2 1
(a) + + ≤ ;
5a + (b + c)
2 2 5b + (c + a)
2 2 5c + (a + b)
2 2 3
a3 b3 c3 1
(b) + + ≤ .
13a + (b + c)
3 3 13b + (c + a)
3 3 13c + (a + b)
3 3 7

1.93. If a, b, c are nonnegative real numbers, then

b2 + c 2 − a2 c 2 + a2 − b2 a2 + b2 − c 2 1
+ + ≥ .
2a + (b + c)
2 2 2b + (c + a)
2 2 2c + (a + b)
2 2 2

1.94. Let a, b, c be positive real numbers. If k > 0, then

3a2 − 2bc 3b2 − 2ca 3c 2 − 2ab 3


+ + ≤ .
ka2 + (b − c)2 kb2 + (c − a)2 kc 2 + (a − b)2 k

p Let a, b, c be nonnegative real numbers, no two of which are zero. If k ≥


1.95.
3 + 7, then
a b c 9
(a) + 2 + 2 ≥ ;
a2 + kbc b + kca c + kab (1 + k)(a + b + c)
1 1 1 9
(b) + + ≥ .
ka2 + bc kb2 + ca kc 2 + ab (k + 1)(ab + bc + ca)

1.96. Let a, b, c be nonnegative real numbers, no two of which are zero. Prove that

1 1 1 6
+ 2 + 2 ≥ 2 .
2a2 + bc 2b + ca 2c + ab a + b + c + ab + bc + ca
2 2
Symmetric Rational Inequalities 15

1.97. Let a, b, c be nonnegative real numbers, no two of which are zero. Prove that

1 1 1 1
+ + ≥ .
22a2 + 5bc 22b + 5ca 22c + 5ab (a + b + c)2
2 2

1.98. Let a, b, c be nonnegative real numbers, no two of which are zero. Prove that

1 1 1 8
+ 2 + 2 ≥ .
2a2 + bc 2b + ca 2c + ab (a + b + c)2

1.99. Let a, b, c be nonnegative real numbers, no two of which are zero. Prove that

1 1 1 12
+ + ≥ .
a2 + bc b2 + ca c 2 + ab (a + b + c)2

1.100. Let a, b, c be nonnegative real numbers, no two of which are zero. Prove
that
1 1 1 1 2
(a) + 2 + 2 ≥ 2 + ;
a2 + 2bc b + 2ca c + 2ab a +b +c
2 2 ab + bc + ca
a(b + c) b(c + a) c(a + b) ab + bc + ca
(b) + 2 + 2 ≥1+ 2 .
a + 2bc b + 2ca c + 2ab
2 a + b2 + c 2

1.101. Let a, b, c be nonnegative real numbers, no two of which are zero. Prove
that
a b c a+b+c
(a) + 2 + 2 ≤ ;
a2 + 2bc b + 2ca c + 2ab ab + bc + ca
a(b + c) b(c + a) c(a + b) a2 + b2 + c 2
(b) + + ≤ 1 + .
a2 + 2bc b2 + 2ca c 2 + 2ab ab + bc + ca

1.102. Let a, b, c be nonnegative real numbers, no two of which are zero. Prove
that
a b c a+b+c
(a) + 2 + 2 ≥ 2 ;
2a2 + bc 2b + ca 2c + ab a + b2 + c 2
b+c c+a a+b 6
(b) + 2 + 2 ≥ .
2a + bc 2b + ca 2c + ab
2 a+b+c
16 Vasile Cîrtoaje

1.103. Let a, b, c be nonnegative real numbers, no two of which are zero. Prove
that
a(b + c) b(c + a) c(a + b) (a + b + c)2
+ 2 + 2 ≥ 2 .
a2 + bc b + ca c + ab a + b2 + c 2

1.104. Let a, b, c be nonnegative real numbers, no two of which are zero. If k > 0,
then
p p p p
b2 + c 2 + 3bc c 2 + a2 + 3ca a2 + b2 + 3ab 3(2 + 3)
+ + ≥ .
a2 + kbc b2 + kca c 2 + kab 1+k

1.105. Let a, b, c be nonnegative real numbers, no two of which are zero. Prove
that
1 1 1 8 6
+ 2 + 2 + 2 ≥ .
a +b
2 2 b +c 2 c +a 2 a +b +c
2 2 ab + bc + ca

1.106. If a, b, c are the lengths of the sides of a triangle, then


a(b + c) b(c + a) c(a + b)
+ 2 + 2 ≤ 2.
a + 2bc b + 2ca c + 2ab
2

1.107. If a, b, c are real numbers, then


a2 − bc b2 − ca c 2 − ab
+ + ≥ 0.
2a2 + b2 + c 2 2b2 + c 2 + a2 2c 2 + a2 + b2

1.108. If a, b, c are nonnegative real numbers, then


3a2 − bc 3b2 − ca 3c 2 − ab 3
+ + ≤ .
2a2 + b2 + c 2 2b2 + c 2 + a2 2c 2 + a2 + b2 2

1.109. If a, b, c are nonnegative real numbers, then


(b + c)2 (c + a)2 (a + b)2
+ + ≥ 2.
4a2 + b2 + c 2 4b2 + c 2 + a2 4c 2 + a2 + b2

1.110. If a, b, c are positive real numbers, then


P 1 3
(a) ≤ ;
11a2 + 2b2 + 2c 2 5(ab + bc + ca)
1 1 1
+
P
(b) ≤ .
4a2 + b2 + c 2 2(a2 + b2 + c 2 ) ab + bc + ca
Symmetric Rational Inequalities 17

1.111. If a, b, c are nonnegative real numbers such that ab + bc + ca = 3, then


p p p
a b c 3
+ + ≥ .
b+c c+a a+b 2

1.112. If a, b, c are nonnegative real numbers such that ab + bc + ca ≥ 3, then


1 1 1 1 1 1
+ + ≥ + + .
2+a 2+ b 2+c 1+ b+c 1+c+a 1+a+ b

1.113. If a, b, c are the lengths of the sides of a triangle, then


a2 − bc b2 − ca c 2 − ab
(a) + + ≤ 0;
3a2 + b2 + c 2 3b2 + c 2 + a2 3c 2 + a2 + b2
a4 − b2 c 2 b4 − c 2 a2 c 4 − a2 b2
(b) + + ≤ 0.
3a4 + b4 + c 4 3b4 + c 4 + a4 3c 4 + a4 + b4

1.114. If a, b, c are the lengths of the sides of a triangle, then


bc ca ab 1
+ 2 + 2 ≥ .
4a2 +b +c
2 2 4b + c + a
2 2 4c + a + b
2 2 2

1.115. If a, b, c are the lengths of the sides of a triangle, then


1 1 1 9
+ 2 + 2 ≤ .
b2 +c 2 c +a 2 a +b 2 2(ab + bc + ca)

1.116. If a, b, c are the lengths of the sides of a triangle, then


a+b b+c c+a
(a) + + > 5;
a−b b−c c−a
a2 + b2 b2 + c 2 c 2 + a2
(b) + + ≥ 3.
a2 − b2 b2 − c 2 c 2 − a2

1.117. If a, b, c are the lengths of the sides of a triangle, then


b+c c+a a+b
 ‹
a b c
+ + +3≥6 + + .
a b c b+c c+a a+b
18 Vasile Cîrtoaje

1.118. Let a, b, c be nonnegative real numbers, no two of which are zero. Prove
that X 3a(b + c) − 2bc 3
≥ .
(b + c)(2a + b + c) 2

1.119. Let a, b, c be nonnegative real numbers, no two of which are zero. Prove
that X a(b + c) − 2bc
≥ 0.
(b + c)(3a + b + c)

1.120. Let a, b, c be positive real numbers such that a2 + b2 + c 2 ≥ 3. Prove that

a5 − a2 b5 − b2 c5 − c2
+ + ≥ 0.
a5 + b2 + c 2 b5 + c 2 + a2 c 5 + a2 + b2

1.121. Let a, b, c be positive real numbers such that a2 + b2 + c 2 = a3 + b3 + c 3 .


Prove that
a2 b2 c2 3
+ + ≥ .
b+c c+a a+b 2

1.122. If a, b, c ∈ [0, 1], then


a b c
+ + ≤ 1.
bc + 2 ca + 2 ab + 2

1.123. Let a, b, c be positive real numbers such that a + b + c = 2. Prove that

1 1 1
 ‹
5(1 − ab − bc − ca) + + + 9 ≥ 0.
1 − ab 1 − bc 1 − ca

1.124. Let a, b, c be nonnegative real numbers such that a + b + c = 2. Prove that

2 − a2 2 − b2 2 − c2
+ + ≤ 3.
2 − bc 2 − ca 2 − ab

1.125. Let a, b, c be nonnegative real numbers such that a + b + c = 3. Prove that

3 + 5a2 3 + 5b2 3 + 5c 2
+ + ≥ 12.
3 − bc 3 − ca 3 − ab
Symmetric Rational Inequalities 19

1.126. Let a, b, c be nonnegative real numbers such that a + b + c = 2. If


−1 7
≤m≤ ,
7 8
then
a2 + m b2 + m c2 + m 3(4 + 9m)
+ + ≥ .
3 − 2bc 3 − 2ca 3 − 2ab 19

1.127. Let a, b, c be nonnegative real numbers such that a + b + c = 3. Prove that

47 − 7a2 47 − 7b2 47 − 7c 2
+ + ≥ 60.
1 + bc 1 + ca 1 + ab

1.128. Let a, b, c be nonnegative real numbers such that a + b + c = 3. Prove that

26 − 7a2 26 − 7b2 26 − 7c 2 57
+ + ≤ .
1 + bc 1 + ca 1 + ab 2

1.129. If a, b, c are nonnegative real numbers, then


X 5a(b + c) − 6bc
≤ 3.
a2 + b2 + c 2 + bc

1.130. Let a, b, c be nonnegative real numbers, no two of which are zero, and let

a2 + b2 + c 2
x= .
ab + bc + ca
Prove that
a b c 1 1
(a) + + + ≥x+ ;
b+c c+a a+b 2 x
4
 ‹
a b c
(b) 6 + + ≥ 5x + ;
b+c c+a a+b x
3 1 1
 ‹
a b c
(c) + + − ≥ x− .
b+c c+a a+b 2 3 x

1.131. If a, b, c are real numbers, then


1 1 1 9
+ 2 + 2 ≤ .
a2 + 7(b + c ) b + 7(c + a ) c + 7(a + b ) 5(a + b + c)2
2 2 2 2 2 2
20 Vasile Cîrtoaje

1.132. If a, b, c are real numbers, then


bc ca ab 3
+ 2 + 2 ≤ .
3a2 +b +c
2 2 3b + c + a
2 2 3c + a + b
2 2 5

1.133. If a, b, c are real numbers such that a + b + c = 3, then


1 1 1 1
+ + ≤ .
8 + 5(b2 + c 2 ) 8 + 5(c 2 + a2 ) 8 + 5(a2 + b2 ) 6

1.134. If a, b, c are real numbers, then


(a + b)(a + c) (b + c)(b + a) (c + a)(c + b) 4
+ 2 + 2 ≤ .
a + 4(b + c ) b + 4(c + a ) c + 4(a + b ) 3
2 2 2 2 2 2 2

1.135. Let a, b, c be nonnegative real numbers, no two of which are zero. Prove
that X 1 1
≤ .
(b + c)(7a + b + c) 2(ab + bc + ca)

1.136. Let a, b, c be nonnegative real numbers, no two of which are zero. Prove
that X 1 9
≤ .
b + c + 4a(b + c) 10(ab + bc + ca)
2 2

1.137. Let a, b, c be nonnegative real numbers, no two of which are zero. If a +


b + c = 3, then
1 1 1 9
+ + ≤ .
3 − ab 3 − bc 3 − ca 2(ab + bc + ca)

1.138. If a, b, c are nonnegative real numbers such that a + b + c = 3, then


bc ca ab 3
+ 2 + 2 ≤ .
a2 +a+6 b + b+6 c +c+6 8

1.139. If a, b, c are nonnegative real numbers such that ab + bc + ca = 3, then


1 1 1 1
+ 2 + 2 ≥ .
8a2 − 2bc + 21 8b − 2ca + 21 8c − 2ab + 21 9
Symmetric Rational Inequalities 21

1.140. Let a, b, c be real numbers, no two of which are zero. Prove that
a2 + bc b2 + ca c 2 + ab (a + b + c)2
(a) + + ≥ ;
b2 + c 2 c 2 + a2 a2 + b2 a2 + b2 + c 2
a2 + 3bc b2 + 3ca c 2 + 3ab 6(ab + bc + ca)
(b) + 2 + 2 ≥ .
b2 + c 2 c + a2 a + b2 a2 + b2 + c 2

1.141. Let a, b, c be real numbers, no two of which are zero. If ab + bc + ca ≥ 0,


then
a(b + c) b(c + a) c(a + b) 3
+ 2 + 2 ≥ .
b +c
2 2 c +a 2 a +b 2 10

1.142. If a, b, c are positive real numbers such that abc > 1, then

1 1 4
+ ≥ .
a + b + c − 3 abc − 1 ab + bc + ca − 3

1.143. Let a, b, c be nonnegative real numbers, no two of which are zero. Prove
that
X (4b2 − ac)(4c 2 − ab) 27
≤ abc.
b+c 2

1.144. Let a, b, c be nonnegative real numbers, no two of which are zero, such that

a + b + c = 3.

Prove that
a b c 2
+ + ≥ .
3a + bc 3b + ca 3c + ab 3

1.145. Let a, b, c be positive real numbers such that

1 1 1
 ‹
(a + b + c) + + = 10.
a b c

Prove that
19 a b c 5
≤ + + ≤ .
12 b+c c+a a+b 3
22 Vasile Cîrtoaje

1.146. Let a, b, c be nonnegative real numbers, no two of which are zero, such that
a + b + c = 3. Prove that
9 a b c
< + + ≤ 1.
10 2a + bc 2b + ca 2c + ab

1.147. Let a, b, c be nonnegative real numbers, no two of which are zero. Prove
that
a3 b3 c3 a3 + b3 + c 3
+ + ≤ .
2a2 + bc 2b2 + ca 2c 2 + ab a2 + b2 + c 2

1.148. If a, b, c are positive real numbers, then

a3 b3 c3 a+b+c
+ + ≥ .
4a2 + bc 4b2 + ca 4c 2 + ab 5

1.149. If a, b, c are positive real numbers, then

1 1 1 3
+ + ≥ .
(2 + a)2 (2 + b)2 (2 + c)2 6 + ab + bc + ca

1.150. If a, b, c are positive real numbers, then

1 1 1 3
+ + ≥ .
1 + 3a 1 + 3b 1 + 3c 3 + abc

1.151. Let a, b, c be real numbers, no two of which are zero. If 1 < k ≤ 3, then

2ab 2bc 2ca


 ‹ ‹ ‹
k+ 2 k+ 2 k+ 2 ≥ (k − 1)(k2 − 1).
a +b 2 b +c 2 c +a 2

1.152. If a, b, c are non-zero and distinct real numbers, then

1 1 1 1 1 1 1 1 1
• ˜  ‹
+ + +3 + + ≥4 + + .
a2 b2 c 2 (a − b)2 (b − c)2 (c − a)2 ab bc ca
Symmetric Rational Inequalities 23

1.153. Let a, b, c be positive real numbers, and let

a b b c c a
A= + + k, B= + + k, C= + + k,
b a c b a b
where −2 < k ≤ 4. Prove that
1 1 1 1 4
+ + ≤ + .
A B C k + 2 A+ B + C − k − 2

1.154. If a, b, c are nonnegative real numbers, no two of which are zero, then

1 1 1 1 1 1
+ + ≥ + + .
b2 + bc + c 2 c 2 + ca + a2 a2 + ab + b2 2a2 + bc 2b2 + ca 2c 2 + ab

1.155. If a, b, c are nonnegative real numbers such that a + b + c ≤ 3, then


1 1 1 1 1 1
(a) + + ≥ + + ;
2a + 1 2b + 1 2c + 1 a + 2 b + 2 c + 2
1 1 1 1 1 1
(b) + + ≥ 2 + 2 + 2 .
2ab + 1 2bc + 1 2ca + 1 a + 2 b + 2 c + 2

1.156. If a, b, c are nonnegative real numbers such that a + b + c = 4, then

1 1 1 1 1 1
+ + ≥ 2 + 2 + 2 .
ab + 2 bc + 2 ca + 2 a + 2 b + 2 c + 2

1.157. If a, b, c are nonnegative real numbers, no two of which are zero, then
ab + bc + ca (a − b)2 (b − c)2 (c − a)2
(a) + ≤ 1;
a2 + b2 + c 2 (a2 + b2 )(b2 + c 2 )(c 2 + a2 )
ab + bc + ca (a − b)2 (b − c)2 (c − a)2
(b) + ≤ 1.
a2 + b2 + c 2 (a2 − ab + b2 )(b2 − bc + c 2 )(c 2 − ca + a2 )

1.158. If a, b, c are nonnegative real numbers, no two of which are zero, then

a2 + b2 + c 2 9(a − b)2 (b − c)2 (c − a)2


≥1+ .
ab + bc + ca (a + b)2 (b + c)2 (c + a)2
24 Vasile Cîrtoaje

1.159. If a, b, c are nonnegative real numbers, no two of which are zero, then

a2 + b2 + c 2 p (a − b)2 (b − c)2 (c − a)2


≥ 1 + (1 + 2)2 2 .
ab + bc + ca (a + b2 )(b2 + c 2 )(c 2 + a2 )

1.160. If a, b, c are nonnegative real numbers, no two of which are zero, then

2 2 2 5 5 5
+ + ≥ + + .
a + b b + c c + a 3a + b + c 3b + c + a 3c + a + b

1.161. If a, b, c are real numbers, no two of which are zero, then


8a2 + 3bc 8b2 + 3ca 8c 2 + 3ab
(a) + + ≥ 11;
b2 + bc + c 2 c 2 + ca + a2 a2 + ab + b2
8a2 − 5bc 8b2 − 5ca 8c 2 − 5ab
(b) + + ≥ 9.
b2 − bc + c 2 c 2 − ca + a2 a2 − ab + b2

1.162. If a, b, c are real numbers, no two of which are zero, then

4a2 + bc 4b2 + ca 4c 2 + ab
+ + ≥ 1.
4b2 + 7bc + 4c 2 4c 2 + 7ca + 4a2 4a2 + 7ab + 4b2

1.163. If a, b, c are real numbers, no two of which are equal, then

1 1 1 27
+ + ≥ .
(a − b)2 (b − c)2 (c − a)2 4(a + b + c − ab − bc − ca)
2 2 2

1.164. If a, b, c are real numbers, no two of which are zero, then

1 1 1 14
+ 2 + 2 ≥ .
a2 − ab + b 2 b − bc + c 2 c − ca + a 2 3(a + b2 + c 2 )
2

1.165. If a, b, c are real numbers, then

a2 + bc b2 + ca c 2 + ab 1
+ + ≥ .
2a + b + c
2 2 2 a + 2b + c
2 2 2 a + b + 2c
2 2 2 6
Symmetric Rational Inequalities 25

1.166. If a, b, c are real numbers, then

2b2 + 2c 2 + 3bc 2c 2 + 2a2 + 3ca 2a2 + 2b2 + 3ab 3


+ + ≥ .
(a + 3b + 3c)2 (b + 3c + 3a)2 (c + 3a + 3b)2 7

1.167. If a, b, c are nonnegative real numbers, then

6b2 + 6c 2 + 13bc 6c 2 + 6a2 + 13ca 6a2 + 6b2 + 13ab


+ + ≤ 3.
(a + 2b + 2c)2 (b + 2c + 2a)2 (c + 2a + 2b)2

1.168. If a, b, c are nonnegative real numbers such that a + b + c = 3, then

3a2 + 8bc 3b2 + 8ca 3c 2 + 8ab


+ + ≤ 3.
9 + b2 + c 2 9 + c 2 + a2 9 + a2 + b2

1.169. If a, b, c are nonnegative real numbers such that a + b + c = 3, then

5a2 + 6bc 5b2 + 6ca 5c 2 + 6ab


+ + ≥ 3.
9 + b2 + c 2 9 + c 2 + a2 9 + a2 + b2

1.170. If a, b, c are nonnegative real numbers such that a + b + c = 3, then


1 1 1 3
+ + ≤ .
a2 + bc + 12 b2 + ca + 12 c 2 + ab + 12 14

1.171. If a, b, c are nonnegative real numbers, no two of which are zero, then
1 1 1 45
+ 2 + 2 ≥ .
a2 +b 2 b +c 2 c +a 2 8(a + b + c ) + 2(ab + bc + ca)
2 2 2

1.172. If a, b, c are real numbers, no two of which are zero, then

a2 − 7bc b2 − 7ca c 2 − 7ab 9(ab + bc + ca)


+ 2 + 2 + ≥ 0.
b2 + c 2 a + b2 a + b2 a2 + b2 + c 2

1.173. If a, b, c are nonnegative real numbers, no two of which are zero, then

a2 − 4bc b2 − 4ca c 2 − 4ab 9(ab + bc + ca) 9


+ 2 + 2 + ≥ .
b2 + c 2 c + a2 a + b2 a2 + b2 + c 2 2
26 Vasile Cîrtoaje

1.174. If a, b, c are real numbers such that abc 6= 0, then

(b + c)2 (c + a)2 (a + b)2 10(a + b + c)2


+ + ≥ 2 + .
a2 b2 c2 3(a2 + b2 + c 2 )

1.175. Let a, b, c be real numbers, no two of which are zero. If ab + bc + ca ≥ 0,


then
a b c 3
(a) + + ≥ ;
b+c c+a a+b 2
(b) i f ab ≤ 0, then
a b c
+ + ≥ 2.
b+c c+a a+b

1.176. If a, b, c are nonnegative real numbers, then


a b c ab + bc + ca
+ + ≥ .
7a + b + c 7b + c + a 7c + a + b (a + b + c)2

1.177. If a, b, c are positive real numbers such that abc = 1, then


a+b+c 1 1 1 8
+ + + ≥ .
30 a+1 b+1 c+1 5

1.178. Let f be a real function defined on an interval I, and let x, y, s ∈ I such that
x + m y = (1 + m)s, where m > 0. Prove that the inequality

f (x) + m f ( y) ≥ (1 + m) f (s)

holds if and only if


h(x, y) ≥ 0,
where
g(x) − g( y) f (u) − f (s)
h(x, y) = , g(u) = .
x−y u−s

1.179. Let a, b, c ≤ 8 be real numbers such that a + b + c = 3. Prove that


13a − 1 13b − 1 13c − 1 3
+ 2 + ≤ .
a2 + 23 b + 23 c 2 + 23 2
Symmetric Rational Inequalities 27

3
1.180. Let a, b, c 6= be nonnegative real numbers such that a + b + c = 3. Prove
4
that
1−a 1− b 1−c
+ + ≥ 0.
(4a − 3)2 (4b − 3)2 (4c − 3)2

1.181. If a, b, c are the lengths of the sides of a triangle, then

a2 b2 c2 1
+ + ≥ .
4a2 + 5bc 4b2 + 5ca 4c 2 + 5ab 3

1.182. If a, b, c are the lengths of the sides of a triangle, then

1 1 1 3
+ + ≥ .
7a2 + b2 + c 2 7b2 + c 2 + a2 7c 2 + a2 + b2 (a + b + c)2

1.183. Let a, b, c be the lengths of the sides of a triangle. If k > −2, then
X a(b + c) + (k + 1)bc 3(k + 3)
≤ .
b2 + kbc + c 2 k+2

1.184. Let a, b, c be the lengths of the sides of a triangle. If k > −2, then
X 2a2 + (4k + 9)bc 3(4k + 11)
≤ .
b2 + kbc + c2 k+2

1.185. If a, b, c are nnonnegative numbers such that abc = 1, then

1 1 1 1
+ + + ≥ 1.
(a + 1)2 (b + 1)2 (c + 1)2 2(a + b + c − 1)

1.186. If a, b, c are positive real numbers such that

a ≤ b ≤ c, a2 bc ≥ 1,

then
1 1 1 3
+ + ≥ .
1+a 3 1+ b 3 1+c 3 1 + abc
28 Vasile Cîrtoaje

1.187. If a, b, c are positive real numbers such that

a ≤ b ≤ c, a2 c ≥ 1,

then
1 1 1 3
+ + ≥ .
1 + a3 1 + b3 1 + c 3 1 + abc

1.188. If a, b, c are positive real numbers such that

a ≤ b ≤ c, 2a + c ≥ 3,

then
1 1 1 3
+ + ≥ 2 .
3 + a2 3 + b2 3 + c 2 3 + a+b+c
3

1.189. If a, b, c are positive real numbers such that

a ≤ b ≤ c, 9a + 8b ≥ 17,

then
1 1 1 3
+ + ≥ 2 .
3 + a2 3 + b2 3 + c 2 3 + a+b+c
3

1.190. Let a, b, c, d be positive real numbers such that abcd = 1. Prove that
X 1
≤ 1.
1 + ab + bc + ca

1.191. Let a, b, c, d be positive real numbers such that abcd = 1. Prove that

1 1 1 1
+ + + ≥ 1.
(1 + a)2 (1 + b)2 (1 + c)2 (1 + d)2

1
1.192. Let a, b, c, d 6= be positive real numbers such that abcd = 1. Prove that
3
1 1 1 1
+ + + ≥ 1.
(3a − 1)2 (3b − 1)2 (3c − 1)2 (3d − 1)2
Symmetric Rational Inequalities 29

1.193. Let a, b, c, d be positive real numbers such that abcd = 1. Prove that
1 1 1 1
+ + + ≥ 1.
1 + a + a2 + a3 1 + b + b2 + b3 1 + c + c 2 + c 3 1 + d + d 2 + d 3

1.194. Let a, b, c, d be positive real numbers such that abcd = 1. Prove that
1 1 1 1
+ + + ≥ 1.
1 + a + 2a 2 1 + b + 2b 2 1 + c + 2c 2 1 + d + 2d 2

1.195. Let a, b, c, d be positive real numbers such that abcd = 1. Prove that
1 1 1 1 9 25
+ + + + ≥ .
a b c d a+b+c+d 4

1.196. If a, b, c, d are real numbers such that a + b + c + d = 0, then


(a − 1)2 (b − 1)2 (c − 1)2 (d − 1)2
+ + 2 + ≤ 4.
3a2 + 1 3b2 + 1 3c + 1 3d 2 + 1

1.197. If a, b, c, d ≥ −5 such that a + b + c + d = 4, then


1−a 1− b 1−c 1−d
+ + + ≥ 0.
(1 + a)2 (1 + b)2 (1 + c)2 (1 + d)2

1.198. Let a1 , a2 , . . . , an be positive real numbers such that a1 + a2 + · · · + an = n.


Prove that X 1 1
≤ .
(n + 1)a1 + a2 + · · · + an
2 2 2 2

1.199. Let a1 , a2 , . . . , an be real numbers such that a1 + a2 + · · · + an = 0. Prove that

(a1 + 1)2 (a2 + 1)2 (an + 1)2 n


+ + · · · + ≥ .
a12 + n − 1 a22 + n − 1 an2 + n − 1 n − 1

1.200. Let a1 , a2 , . . . , an be positive real numbers such that a1 a2 · · · an = 1. Prove


that
1 1 1
(a) + + ··· + ≥ 1;
1 + (n − 1)a1 1 + (n − 1)a2 1 + (n − 1)an
1 1 1
(b) + + ··· + ≤ 1.
a1 + n − 1 a2 + n − 1 an + n − 1
30 Vasile Cîrtoaje

1.201. Let a1 , a2 , . . . , an be positive real numbers such that a1 a2 · · · an = 1. Prove


that
1 1 1
+ + ··· + ≥ 1.
1 − a1 + na1 1 − a2 + na2
2 2
1 − an + nan2

1.202. Let a1 , a2 , . . . , an be positive real numbers such that

k(n − k − 1)
a1 , a2 , . . . , an ≥ , k>1
kn − k − 1
and
a1 a2 · · · an = 1.
Prove that
1 1 1 n
+ + ··· + ≤ .
a1 + k a2 + k an + k 1 + k

1.203. If a1 , a2 , . . . , an ≥ 0, then

1 1 1 n
+ + ··· + ≥ .
1 + na1 1 + na2 1 + nan n + a1 a2 · · · an
Symmetric Rational Inequalities 31

1.2 Solutions

P 1.1. If a, b are nonnegative real numbers, then


1 1 1
+ ≥ .
(1 + a)2 (1 + b)2 1 + ab
First Solution. Use the Cauchy-Schwarz inequality as follows:
1 1 1 (b + a)2 1
+ − ≥ −
(1 + a)2 (1 + b)2 1 + ab b2 (1 + a)2 + a2 (1 + b)2 1 + ab
ab[a2 + b2 − 2(a + b) + 2]
=
(1 + ab)[b2 (1 + a)2 + a2 (1 + b)2 ]
ab[(a − 1)2 + (b − 1)2 ]
= ≥ 0.
(1 + ab)[b2 (1 + a)2 + a2 (1 + b)2 ]
The equality holds for a = b = 1.
Second Solution. By the Cauchy-Schwarz inequality, we have
1 1
 ‹  ‹
(a + b) a + ≥ (a + 1) ,
2
(a + b) + b ≥ (1 + b)2 ,
b a
hence
1 1 1 1 1
+ ≥ + = .
(1 + a)2 (1 + b)2 (a + b)(a + 1/b) (a + b)(1/a + b) 1 + ab
Third Solution. The desired inequality follows from the identity
1 1 1 ab(a − b)2 + (1 − ab)2
+ − = .
(1 + a)2 (1 + b)2 1 + ab (1 + a)2 (1 + b)2 (1 + ab)
Remark. Replacing a by a/x and b by and b/x, where x is a positive number, we
get the inequality
1 1 1
+ ≥ 2 ,
(x + a) 2 (x + b)2 x + ab
which is valid for any x, a, b ≥ 0.

P 1.2. Let a, b, c be positive real numbers. Prove that


(a) if abc ≤ 1, then
1 1 1
+ + ≥ 1;
2a + 1 2b + 1 2c + 1
(b) if abc ≥ 1, then
1 1 1
+ + ≤ 1.
a+2 b+2 c+2
32 Vasile Cîrtoaje

Solution. (a) Use the substitution

kx 2 k y2 kz 2
a= , b= , c= ,
yz zx xy

where x, y, z > 0 and 0 < k ≤ 1. Applying the Cauchy-Schwarz inequality, we have


P 2
X 1 X yz X yz yz
= ≥ ≥P = 1.
a+2 2kx + yz
2 2x + yz
2 yz(2x 2 + yz)

The equality holds for a = b = c = 1.


(b) The desired inequality follows from the inequality in (a) by replacing a, b, c
with 1/a, 1/b, 1/c, respectively. The equality holds for a = b = c = 1.

P 1.3. If 0 ≤ a, b, c ≤ 1, then

1 1 1 1 1 1
 ‹  ‹
2 + + ≥3 + + .
a+b b+c c+a 2a + 1 2b + 1 2c + 1

Solution. Write the inequality as E(a, b, c) ≥ 0, assume that 0 ≤ a ≤ b ≤ c ≤ 1


and show that
E(a, b, c) ≥ E(a, b, 1) ≥ E(a, 1, 1) ≥ 0.
The inequality E(a, b, c) ≥ E(a, b, 1) is equivalent to

1 1 1 1 1 1
 ‹  ‹  ‹
2 − +2 − −3 − ≥ 0,
b+c b+1 c+a 1+a 2c + 1 3

1 1 1
• ˜
(1 − c) + − ≥ 0.
(b + c)(b + 1) (c + a)(1 + a) 2c + 1
We have
1 1 1 1 1 1
+ − ≥ + −
(b + c)(b + 1) (c + a)(1 + a) 2c + 1 (1 + c)(1 + 1) (c + 1)(1 + 1) 2c + 1
c
= > 0.
(c + 1)(2c + 1)

The inequality E(a, b, 1) ≥ E(a, 1, 1) is equivalent to

1 1 1 1 1 1
 ‹  ‹  ‹
2 − +2 − −3 − ,
a+ b a+1 1+ b 2 2b + 1 3

2 1 2
• ˜
(1 − b) + − ≥ 0.
(a + b)(a + 1) 1 + b 2b + 1
Symmetric Rational Inequalities 33

We have
2 1 2 2 1 2
+ − ≥ + −
(a + b)(a + 1) 1 + b 2b + 1 (1 + b)(1 + 1) 1 + b 2b + 1
2b
= > 0.
(1 + b)(2b + 1)

Finally,
2a(1 − a)
E(a, 1, 1) = ≥ 0.
(a + 1)(2a + 1)
The equality holds for a = b = c = 1, and also for a = 0 and b = c = 1 (or any
cyclic permutation).

P 1.4. If a, b, c are nonnegative real numbers such that a + b + c ≤ 3, then

1 1 1 1 1 1
 ‹  ‹
2 + + ≥5 + + .
a+b b+c c+a 2a + 3 2b + 3 2c + 3

Solution. It suffices to prove the homogeneous inequality


X 2 5
‹
− ≥ 0.
b + c 3a + b + c
We use the SOS (sum-of-squares) method. Without loss of generality, assume that

a ≥ b ≥ c.

Write the inequality as follows:


X 2a − b − c
≥ 0,
(b + c)(3a + b + c)
X a−b X a−c
+ ≥ 0,
(b + c)(3a + b + c) (b + c)(3a + b + c)
X a−b X b−a
+ ≥ 0,
(b + c)(3a + b + c) (c + a)(3b + c + a)
1 1
X  ‹
(a − b) − ≥ 0,
(b + c)(3a + b + c) (c + a)(3b + c + a)
X
(a − b)2 (a + b − c)(a + b)(3c + a + b) ≥ 0.
Consider the nontrivial case a > b + c. Since a + b − c > 0, it suffices to show that

(a − c)2 (a + c − b)(a + c)(3b + c + a) ≥ (b − c)2 (a − b − c)(b + c)(3a + b + c).


34 Vasile Cîrtoaje

This inequality is true since


(a − c)2 ≥ (b − c)2 , a+c−b≥a−b−c
and
(a + c)(3b + c + a) ≥ (b + c)(3a + b + c).
The last inequality is equivalent to
(a − b)(a + b − c) ≥ 0.
The equality holds for a = b = c = 1, and also for a = b = 3/2 and c = 0 (or any
cyclic permutation).

P 1.5. If a, b, c are nonnegative real numbers, then


a2 − bc b2 − ca c 2 − ab
+ + ≥ 0.
3a + b + c 3b + c + a 3c + a + b

Solution. We use the SOS method. Without loss of generality, assume that
a ≥ b ≥ c.
We have
X a2 − bc X (a − b)(a + c) + (a − c)(a + b)
2 =
3a + b + c 3a + b + c
X (a − b)(a + c) X (b − a)(b + c)
= +
3a + b + c 3b + c + a
X (a − b)2 (a + b − c)
=
(3a + b + c)(3b + c + a)
Since a + b − c ≥ 0, it suffices to show that
(b − c)2 (b + c − a)(3a + b + c) + (c − a)2 (c + a − b)(3b + c + a) ≥ 0;
that is,
(a − c)2 (c + a − b)(3b + c + a) ≥ (b − c)2 (a − b − c)(3a + b + c).
For the nontrivial case a > b + c, we can get this inequality by multiplying the
obvious inequalities
c + a − b ≥ a − b − c,
b2 (a − c)2 ≥ a2 (b − c)2 ,
a(3b + c + a) ≥ b(3a + b + c),
a ≥ b.
The equality holds for a = b = c, and also for a = b and c = 0 (or any cyclic
permutation).
Symmetric Rational Inequalities 35

P 1.6. If a, b, c are positive real numbers, then

4a2 − b2 − c 2 4b2 − c 2 − a2 4c 2 − a2 − b2
+ + ≤ 3.
a(b + c) b(c + a) c(a + b)

(Vasile Cîrtoaje, 2006)

Solution. We use the SOS method. Write the inequality as follows:


X• 4a2 − b2 − c 2
˜
1− ≥ 0,
a(b + c)
X b2 + c 2 − 4a2 + a(b + c)
≥ 0,
a(b + c)
X (b2 − a2 ) + a(b − a) + (c 2 − a2 ) + a(c − a)
≥ 0,
a(b + c)
X (b − a)(2a + b) + (c − a)(2a + c)
≥ 0,
a(b + c)
X (b − a)(2a + b) X (a − b)(2b + a)
+ ≥ 0,
a(b + c) b(c + a)
X
c(a + b)(a − b)2 (bc + ca − ab) ≥ 0.
Without loss of generality, assume that

a ≥ b ≥ c.

Since ca + ab − bc > 0, it suffices to show that

b(c + a)(c − a)2 (ab + bc − ca) + c(a + b)(a − b)2 (bc + ca − ab) ≥ 0,

that is,

b(c + a)(a − c)2 (ab + bc − ca) ≥ c(a + b)(a − b)2 (ab − bc − ca).

For the nontrivial case ab − bc − ca > 0, this inequality follows by multiplying the
inequalities
ab + bc − ca > ab − bc − ca,
(a − c)2 ≥ (a − b)2 ,
b(c + a) ≥ c(a + b).
The equality holds for a = b = c
36 Vasile Cîrtoaje

P 1.7. Let a, b, c be nonnegative real numbers, no two of which are zero. Prove that
1 1 1 3
(a) + 2 + 2 ≥ ;
a2 + bc b + ca c + ab ab + bc + ca
1 1 1 2
(b) + 2 + 2 ≥ .
2a2 + bc 2b + ca 2c + ab ab + bc + ca
1 1 1 2
(c) + 2 + 2 > .
a2 + 2bc b + 2ca c + 2ab ab + bc + ca
(Vasile Cîrtoaje, 2005)

Solution. (a) Since


ab + bc + ca a(b + c − a)
= 1 + ,
a2 + bc a2 + bc
we can write the inequality as

a(b + c − a) b(c + a − b) c(a + b − c)


+ + ≥ 0.
a2 + bc b2 + ca c 2 + ab
Without loss of generality, assume that

a = min{a, b, c}.

Since b + c − a > 0, it suffices to show that


b(c + a − b) c(a + b − c)
+ ≥ 0.
b2 + ca c 2 + ab
This is equivalent to each of the following inequalities

(b2 + c 2 )a2 − (b + c)(b2 − 3bc + c 2 )a + bc(b − c)2 ≥ 0,

(b − c)2 a2 − (b + c)(b − c)2 a + bc(b − c)2 + abc(2a + b + c) ≥ 0,


(b − c)2 (a − b)(a − c) + abc(2a + b + c) ≥ 0.
The last inequality is obviously true. The equality holds for a = 0 and b = c (or
any cyclic permutation thereof).
(b) Using the identities

2a2 + bc = a(2a − b − c) + ab + bc + ca,

2b2 + ca = b(2b − c − a) + ab + bc + ca,


2c 2 + ab = c(2c − a − b) + ab + bc + ca,
we can write the inequality as
1 1 1
+ + ≥ 2,
1+ x 1+ y 1+z
Symmetric Rational Inequalities 37

where
a(2a − b − c) b(2b − c − a) c(2c − a − b)
x= , y= , z= .
ab + bc + ca ab + bc + ca ab + bc + ca
Without loss of generality, assume that a = min{a, b, c}. Since

1
x ≤ 0, ≥ 1,
1+ x
it suffices to show that
1 1
+ ≥ 1.
1+ y 1+z
This is equivalent to
1 ≥ yz,
(ab + bc + ca)2 ≥ bc(2b − c − a)(2c − a − b),
a2 (b2 + bc + c 2 ) + 3abc(b + c) + 2bc(b − c)2 ≥ 0.
The last inequality is obviously true. The equality holds for a = 0 and b = c (or
any cyclic permutation thereof).
(c) According to the identities

a2 + 2bc = (a − b)(a − c) + ab + bc + ca,

b2 + 2ca = (b − c)(b − a) + ab + bc + ca,


c 2 + 2ab = (c − a)(c − b) + ab + bc + ca,
we can write the inequality as

1 1 1
+ + > 2,
1+ x 1+ y 1+z

where
(a − b)(a − c) (b − c)(b − a) (c − a)(c − b)
x= , y= , z= .
ab + bc + ca ab + bc + ca ab + bc + ca
Since
x y + yz + z x = 0
and
−(a − b)2 (b − c)2 (c − a)2
x yz = ≤ 0,
(ab + bc + ca)3
we have
1 1 1 1 − 2x yz
+ + −2= > 0.
1+ x 1+ y 1+z (1 + x)(1 + y)(1 + z)
38 Vasile Cîrtoaje

P 1.8. Let a, b, c be nonnegative real numbers, no two of which are zero. Prove that
a(b + c) b(c + a) c(a + b)
+ 2 + 2 ≥ 2.
a2 + bc b + ca c + ab
(Pham Kim Hung, 2006)
Solution. Without loss of generality, assume that a ≥ b ≥ c and write the inequality
as
b(c + a) (a − b)(a − c) (a − c)(b − c)
≥ + .
b2 + ca a2 + bc c 2 + ab
Since
(a − b)(a − c) (a − b)a a−b
≤ ≤
a2 + bc a2 + bc a
and
(a − c)(b − c) a(b − c) b−c
≤ 2 ≤ ,
c + ab
2 c + ab b
it suffices to show that
b(c + a) a − b b − c
≥ + .
b2 + ca a b
This inequality is equivalent to

b2 (a − b)2 − 2abc(a − b) + a2 c 2 + ab2 c ≥ 0,

(ab − b2 − ac)2 + ab2 c ≥ 0.


The equality holds for for a = b and c = 0 (or any cyclic permutation).

P 1.9. Let a, b, c be nonnegative real numbers, no two of which are zero. Prove that
a2 b2 c2 a b c
+ + ≥ + + .
b +c
2 2 c +a
2 2 a +b
2 2 b+c c+a a+b
(Vasile Cîrtoaje, 2002)
Solution. Use the SOS method. We have
X  a2 ab(a − b) + ac(a − c)
‹ X
a
− =
b2 + c 2 b + c (b2 + c 2 )(b + c)
X ab(a − b) X ba(b − a)
= +
(b + c )(b + c)
2 2 (c + a2 )(c + a)
2

X ab(a − b)2
= (a2 + b2 + c 2 + ab + bc + ca) ≥ 0.
(b2 + c 2 )(c 2 + a2 )(b + c)(c + a)
The equality holds for a = b = c, and also for a = 0 and b = c (or any cyclic
permutation).
Symmetric Rational Inequalities 39

P 1.10. Let a, b, c be positive real numbers. Prove that

1 1 1 a b c
+ + ≥ 2 + 2 + 2 .
b+c c+a a+b a + bc b + ca c + ab

First Solution. Without loss of generality, assume that a = min{a, b, c}. Since
X 1 X a X 1 a
‹
− = −
b+c a2 + bc b + c a2 + bc
X (a − b)(a − c)
=
(b + c)(a2 + bc)

and (a − b)(a − c) ≥ 0, it suffices to show that

(b − c)(b − a) (c − a)(c − b)
+ ≥ 0.
(c + a)(b + ca) (a + b)(c 2 + ab)
2

This inequality is equivalent to

(b − c)[(b2 − a2 )(c 2 + ab) + (a2 − c 2 )(b2 + ca)] ≥ 0,

a(b − c)2 (b2 + c 2 − a2 + ab + bc + ca) ≥ 0.


The last inequality is clearly true. The equality holds for a = b = c.
Second Solution. Since
X 1 X• b c
˜ X •
1 1
˜
= + = a + ,
b+c (b + c)2 (b + c)2 (a + b)2 (a + c)2

we can write the inequality as


X • 1 1 1
˜
a + − ≥ 0.
(a + b)2 (a + c)2 a2 + bc

This is true since, according to Remark from P 1.1, we have

1 1 1
+ − ≥ 0.
(a + b)2 (a + c)2 a2 + bc

P 1.11. Let a, b, c be positive real numbers. Prove that

1 1 1 2a 2b 2c
+ + ≥ 2 + 2 + 2 .
b + c c + a a + b 3a + bc 3b + ca 3c + ab
(Vasile Cîrtoaje, 2005)
40 Vasile Cîrtoaje

Solution. Since
X 1 X 2a X 1 2a
‹
− = −
b+c 3a2 + bc b + c 3a2 + bc
X (a − b)(a − c) + a(2a − b − c)
= ,
(b + c)(3a2 + bc)
it suffices to show that X (a − b)(a − c)
≥0
(b + c)(3a2 + bc)
and X a(2a − b − c)
≥ 0.
(b + c)(3a2 + bc)
In order to prove the first inequality, assume that a = min{a, b, c}. Since
(a − b)(a − c) ≥ 0,
it is enough to show that
(b − c)(b − a) (c − a)(c − b)
+ ≥ 0.
(c + a)(3b + ca) (a + b)(3c 2 + ab)
2

This is equivalent to the obvious inequality


a(b − c)2 (b2 + c 2 − a2 + 3ab + bc + 3ca) ≥ 0.
The second inequality can be proved by the SOS method. We have
X a(2a − b − c) X a(a − b) + a(a − c)
=
(b + c)(3a2 + bc) (b + c)(3a2 + bc)
X a(a − b) X b(b − a)
= +
(b + c)(3a + bc)
2 (c + a)(3b2 + ca)
• ˜
X a b
= (a − b) −
(b + c)(3a2 + bc) (c + a)(3b2 + ca)
X c(a − b)2 [(a − b)2 + c(a + b)]
= ≥ 0.
(b + c)(c + a)(3a2 + bc)(3b2 + ca)
The equality holds for a = b = c.

P 1.12. Let a, b, c be nonnegative real numbers, no two of which are zero. Prove that

a b c 13 2(ab + bc + ca)
(a) + + ≥ − ;
b+c c+a a+b 6 3(a2 + b2 + c 2 )
3 p ab + bc + ca
 ‹
a b c
(b) + + − ≥ ( 3 − 1) 1 − 2 .
b+c c+a a+b 2 a + b2 + c 2
(Vasile Cîrtoaje, 2006)
Symmetric Rational Inequalities 41

Solution. (a) We use the SOS method. Rewrite the inequality as

3 2 ab + bc + ca
 ‹
a b c
+ + − ≥ 1− 2 .
b+c c+a a+b 2 3 a + b2 + c 2

Since
X a 1 (a − b) + (a − c)
‹ X
− =
b+c 2 2(b + c)
X a−b X b−a
= +
2(b + c) 2(c + a)
Xa−b 1 1
‹
= −
2 b+c c+a
X (a − b)2
=
2(b + c)(c + a)

and
2 ab + bc + ca (a − b)2
 ‹ X
1− 2 = ,
3 a + b2 + c 2 3(a2 + b2 + c 2 )
the inequality can be restated as

1 1
X • ˜
(a − b)2
− ≥ 0.
2(b + c)(c + a) 3(a2 + b2 + c 2 )

This is true since

3(a2 + b2 + c 2 ) − 2(b + c)(c + a) = (a + b − c)2 + 2(a − b)2 ≥ 0.

The equality holds for a = b = c.


(b) Let
p = a + b + c, q = ab + bc + ca, r = abc.
We have
X a X a  X 1
= +1 −3= p −3
b+c b+c b+c
p(p2 + q)
= − 3.
pq − r

According to P 3.57-(a) in Volume 1, for fixed p and q, the product r is minimum


when a = 0 or b = c. Therefore, it suffices to prove the inequality for a = 0 and
for b = c = 1.
Case 1: a = 0. The original inequality can be written as

b c 3 p
 ‹
bc
+ − ≥ ( 3 − 1) 1 − 2 .
c b 2 b + c2
42 Vasile Cîrtoaje

It suffices to show that


b c 3 bc
+ − ≥1− 2 .
c b 2 b + c2
Denoting
b2 + c 2
t= , t ≥ 2,
bc
this inequality becomes
3 1
t− ≥1− ,
2 t
(t − 2)(2t − 1) ≥ 0.
Case 2: b = c = 1. The original inequality becomes as follows:

2 3 p 2a + 1
 ‹
a
+ − ≥ ( 3 − 1) 1 − 2 ,
2 a+1 2 a +2
p
(a − 1)2 ( 3 − 1)(a − 1)2
≥ ,
2(a + 1) a2 + 2
p
(a − 1)2 (a − 3 + 1)2 ≥ 0.
a
The equality holds for a = b = c, and for p = b = c (or any cyclic permuta-
3−1
tion).

P 1.13. Let a, b, c be positive real numbers. Prove that


‹2
1 1 1 a+b+c

+ + ≤ .
a2 + 2bc b2 + 2ca c 2 + 2ab ab + bc + ca

(Vasile Cîrtoaje, 2006)

First Solution. Assume that a ≥ b ≥ c and write the inequality as

(a + b + c)2 X  ab + bc + ca ‹
−3≥ −1 ,
ab + bc + ca a2 + 2bc

(a − b)2 + (b − c)2 + (a − b)(b − c) X (a − b)(a − c)


+ ≥ 0.
ab + bc + ca a2 + 2bc
Since
(a − b)(a − c) ≥ 0, (c − a)(c − b) ≥ 0,
it suffices to show that
(ab + bc + ca)(a − b)(b − c)
(a − b)2 + (b − c)2 + (a − b)(b − c) − ≥ 0.
b2 + 2ca
Symmetric Rational Inequalities 43

This inequality is equivalent to


(a − b)2 (b − c)2
(a − b) + (b − c) −
2 2
≥ 0,
b2 + 2ca
c(a − b)2 (2a + 2b − c)
(b − c)2 + ≥ 0.
b2 + 2ca
Clearly, the last inequality is true. The equality holds for a = b = c.
Second Solution. Assume that a ≥ b ≥ c and write the desired inequality as
(a + b + c)2 X  ab + bc + ca ‹
−3≥ −1 ,
ab + bc + ca a2 + 2bc
1 X X (a − b)(a − c)
(a − b)(a − c) + ≥ 0,
ab + bc + ca a2 + 2bc
X ab + bc + ca
‹
1+ (a − b)(a − c) ≥ 0.
a2 + 2bc
Since (c − a)(c − b) ≥ 0 and a − b ≥ 0, it suffices to prove that
ab + bc + ca ab + bc + ca
 ‹  ‹
1+ (a − c) + 1 + (c − b) ≥ 0.
a2 + 2bc b2 + 2ca
Write this inequality as
a−c c−b
 ‹
a − b + (ab + bc + ca) 2 + ≥ 0,
a + 2bc b2 + 2ca
(ab + bc + ca)(3ac + 3bc − ab − 2c 2
• ˜
(a − b) 1 + ≥ 0.
(a2 + 2bc)(b2 + 2ca)
Since a − b ≥ 0 and 2ac + 3bc − 2c 2 > 0, it is enough to show that
(ab + bc + ca)(ac − ab)
1+ ≥ 0.
(a2 + 2bc)(b2 + 2ca)
We have
(ab + bc + ca)(ac − ab) (ab + bc + ca)(ac − ab)
1+ ≥1+
(a + 2bc)(b + 2ca)
2 2 a2 (b2 + ca)
(a + b)c 2 + (a2 − b2 )c
= > 0.
a(b2 + ca)

P 1.14. Let a, b, c be nonnegative real numbers, no two of which are zero. Prove that
a2 (b + c) b2 (c + a) c 2 (a + b)
+ 2 + 2 ≥ a + b + c.
b2 + c 2 c + a2 a + b2
(Darij Grinberg, 2004)
44 Vasile Cîrtoaje

First Solution. Use the SOS method. We have


X a2 (b + c) X X • a2 (b + c) ˜
− a = − a
b2 + c 2 b2 + c 2
X ab(a − b) + ac(a − c)
=
b2 + c 2
X ab(a − b) X ba(b − a)
= +
b2 + c 2 c 2 + a2
X ab(a + b)(a − b)2
= ≥ 0.
(b2 + c 2 )(c 2 + a2 )
The equality holds for a = b = c, and also for a = 0 and b = c (or any cyclic
permutation).
Second Solution. By virtue of the Cauchy-Schwarz inequality, we have
P 2
X a2 (b + c) a2 (b + c)
≥P .
b2 + c 2 a2 (b + c)(b2 + c 2 )

Then, it suffices to show that


”X —2 €X Š ”X —
a2 (b + c) ≥ a a2 (b + c)(b2 + c 2 ) .

Let p = a + b + c and q = ab + bc + ca. Since


”X —2
a2 (b + c) = (pq − 3abc)2
= p2 q2 − 6abcpq + 9a2 b2 c 2

and
X X
a2 (b + c)(b2 + c 2 ) = (b + c)[(a2 b2 + b2 c 2 + c 2 a2 ) − b2 c 2 ]
X
= 2p(a2 b2 + b2 c 2 + c 2 a2 ) − b2 c 2 (p − a)
= p(a2 b2 + b2 c 2 + c 2 a2 ) + abcq = p(q2 − 2abcp) + abcq,

the inequality can be written as

p2 q2 − 6abcpq + 9a2 b2 c 2 ≥ p2 (q2 − 2abcp) + abcpq,

abc(2p3 + 9abc − 7pq) ≥ 0.


Using Schur’s inequality
p3 + 9abc − 4pq ≥ 0,
we have
2p3 + 9abc − 7pq ≥ p(p2 − 3q) ≥ 0.
Symmetric Rational Inequalities 45

P 1.15. Let a, b, c be nonnegative real numbers, no two of which are zero. Prove that

a2 + b2 b2 + c 2 c 2 + a2 3(a2 + b2 + c 2 )
+ + ≤ .
a+b b+c c+a a + b + c)

Solution. Use the SOS method.


First Solution. Multiplying by 2(a + b + c), the inequality successively becomes:
X a  2
1+ (b + c 2 ) ≤ 3(a2 + b2 + c 2 ),
b+c
X a X
(b2 + c 2 ) ≤ a2 ,
b+c
X  b2 + c 2
‹
a a− ≥ 0,
b+c
X ab(a − b) − ac(c − a)
≥ 0,
b+c
X ab(a − b) X ba(a − b)
− ≥ 0,
b+c c+a
X ab(a − b)2
≥ 0.
(b + c)(c + a)
The equality holds for a = b = c, and also for a = 0 and b = c (or any cyclic
permutation).
Second Solution. Subtracting a + b + c from the both sides, the desired inequality
becomes as follows:
3(a2 + b2 + c 2 ) X  a2 + b2 a + b ‹
− (a + b + c) ≥ − ,
a+b+c a+b 2
X (a − b)2 X (a − b)2
≥ ,
a+b+c 2(a + b)
X (a + b − c)(a − b)2
≥ 0.
a+b
Without loss of generality, assume that a ≥ b ≥ c. Since a + b − c ≥ 0, it suffices to
prove that
(a + c − b)(a − c)2 (a − b − c)(b − c)2
≥ .
a+c b+c
This inequality is true because

a−c b−c
a + c − b ≥ a − b − c, a − c ≥ b − c, ≥ .
a+c b+c
The last inequality reduces to c(a − b) ≥ 0.
46 Vasile Cîrtoaje

Third Solution. Write the inequality as follows:


X • 3(a2 + b2 ) a2 + b2
˜
− ≥ 0,
2(a + b + c) a+b
X (a2 + b2 )(a + b − 2c)
≥ 0,
a+b
X (a2 + b2 )(a − c) X (a2 + b2 )(b − c)
+ ≥ 0,
a+b a+b
X (a2 + b2 )(a − c) X (b2 + c 2 )(c − a)
+ ≥ 0,
a+b b+c
X (a − c)2 (ab + bc + ca − b2 )
≥ 0.
(a + b)(b + c)
It suffices to prove that
X (a − c)2 (ab + bc − ca − b2 )
≥ 0.
(a + b)(b + c)

Since
ab + bc − ca − b2 = (a − b)(b − c),
this inequality is equivalent to
X c−a
(a − b)(b − c)(c − a) ≥ 0,
(a + b)(b + c)

which is true because X c−a


= 0.
(a + b)(b + c)

P 1.16. Let a, b, c be positive real numbers. Prove that

1 1 1 9
+ + ≥ .
a2 + ab + b2 b2 + bc + c 2 c 2 + ca + a2 (a + b + c)2

(Vasile Cîrtoaje, 2000)

First Solution. Due to homogeneity, we may assume that

a + b + c = 1.

Let q = ab + bc + ca. Since

b2 + bc + c 2 = (a + b + c)2 − a(a + b + c) − (ab + bc + ca) = 1 − a − q,


Symmetric Rational Inequalities 47

we can write the inequality as


X 1
≥ 9,
1−a−q

9q3 − 6q2 − 3q + 1 + 9abc ≥ 0.

From Schur’s inequality

(a + b + c)3 + 9abc ≥ 4(a + b + c)(ab + bc + ca),

we get
1 + 9abc − 4q ≥ 0.

Therefore,

9q3 − 6q2 − 3q + 1 + 9abc = (1 + 9abc − 4q) + q(3q − 1)2 ≥ 0.

The equality holds for a = b = c.


Second Solution. Multiplying by a2 + b2 + c 2 + ab + bc + ca, the inequality can be
written as
X a 9(ab + bc + ca)
(a + b + c) + ≥ 6.
b + bc + c
2 2 (a + b + c)2
By the Cauchy-Schwarz inequality, we have

X a (a + b + c)2 a+b+c
≥ = .
b + bc + c a(b + bc + c ) ab + bc + ca
P
2 2 2 2

Then, it suffices to show that

(a + b + c)2 9(ab + bc + ca)


+ ≥ 6.
ab + bc + ca (a + b + c)2

This follows immediately from the AM-GM inequality.

P 1.17. Let a, b, c be nonnegative real numbers, no two of which are zero. Prove that

a2 b2 c2 1
+ + ≤ .
(2a + b)(2a + c) (2b + c)(2b + a) (2c + a)(2c + b) 3

(Tigran Sloyan, 2005)


48 Vasile Cîrtoaje

First Solution. The inequality is equivalent to each of the inequalities


X• a2
˜
a
− ≤ 0,
(2a + b)(2a + c) 3(a + b + c)
X a(a − b)(a − c)
≥ 0.
(2a + b)(2a + c)
Due to symmetry, we may consider

a ≥ b ≥ c.

Since c(c − a)(c − b) ≥ 0, it suffices to prove that


a(a − b)(a − c) b(b − c)(b − a)
+ ≥ 0.
(2a + b)(2a + c) (2b + c)(2b + a)
This is equivalent to the obvious inequality

(a − b)2 [(a + b)(2ab − c 2 ) + c(a2 + b2 + 5ab)] ≥ 0.

The equality holds for a = b = c, and also for a = b and c = 0 (or any cyclic
permutation).
Second Solution (by Vo Quoc Ba Can). Apply the Cauchy-Schwarz inequality in the
following manner
9a2 (2a + a)2 2a a2
= ≤ + .
(2a + b)(2a + c) 2a(a + b + c) + (2a2 + bc) a + b + c 2a2 + bc
Then,
X 9a2 X a2
≤2+ ≤ 3.
(2a + b)(2a + c) 2a2 + bc
For the nontrivial case a, b, c > 0, the right inequality is equivalent to
X 1
≤ 1,
2 + bc/a2
which follows immediately from P 1.2-(b).
Remark. From the inequality in P 1.17 and Hölder’s inequality
a2
•X ˜ ”X Æ —2
a(2a + b)(2a + c) ≥ (a + b + c)3 ,
(2a + b)(2a + c)
we get the following result:
• If a, b, c are nonnegative real numbers such that a + b + c = 3, then
Æ Æ Æ
a(2a + b)(2a + c) + b(2b + c)(2b + a) + c(2c + a)(2c + bc) ≥ 9,
3 3
 ‹
with equality for a = b = c = 1, and for (a, b, c) = 0, , (or any cyclic permutation).
2 2
Symmetric Rational Inequalities 49

P 1.18. Let a, b, c be positive real numbers. Prove that


P a 1
(a) ≤ ;
(2a + b)(2a + c) a + b + c
P a3 1
(b) ≤ .
(2a2 + b2 )(2a2 + c 2 ) a + b + c
(Vasile Cîrtoaje, 2005)
Solution. (a) Write the inequality as
X •1 a(a + b + c)
˜
− ≥ 0,
3 (2a + b)(2a + c)
X (a − b)(a − c)
≥ 0.
(2a + b)(2a + c)
Assume that
a ≥ b ≥ c.
Since (a − b)(a − c) ≥ 0, it suffices to prove that
(b − c)(b − a) (a − c)(b − c)
+ ≥ 0.
(2b + c)(2b + a) (2c + a)(2c + b)
In addition, since b − c ≥ 0 and a − c ≥ a − b ≥ 0, it is enough to show that
1 1
≥ .
(2c + a)(2c + b) (2b + c)(2b + a)
This is equivalent to the obvious inequality

(b − c)(a + 4b + 4c) ≥ 0.

The equality holds for a = b = c.


(b) We obtain the desired inequality by summing the inequalities
a3 a
≤ ,
(2a2 + b2 )(2a2 + c 2 ) (a + b + c)2
b3 b
≤ ,
(2b + c )(2b + a ) (a + b + c)2
2 2 2 2

c3 c
≤ ,
(2c + a )(2c + b ) (a + b + c)2
2 2 2 2

which are consequences of the Cauchy-Schwarz inequality. For example, from

(a2 + a2 + b2 )(c 2 + a2 + a2 ) ≥ (ac + a2 + ba)2 ,

the first inequality follows. The equality holds for a = b = c.


50 Vasile Cîrtoaje

P 1.19. If a, b, c are positive real numbers, then


X 1 1 2
≥ + .
(a + 2b)(a + 2c) (a + b + c)2 3(ab + bc + ca)

Solution. Write the inequality as follows:


X• 1 1 2 2
˜
− ≥ − ,
(a + 2b)(a + 2c) (a + b + c)2 3(ab + bc + ca) (a + b + c)2

X (b − c)2 X (b − c)2
≥ ,
(a + 2b)(a + 2c) 3(ab + bc + ca)
X b−c
(a − b)(b − c)(c − a) ≥ 0.
(a + 2b)(a + 2c)
Since

b−c X• b−c b−c


X ˜
= −
(a + 2b)(a + 2c) (a + 2b)(a + 2c) 3(ab + bc + ca)
(a − b)(b − c)(c − a) X 1
= ,
3(ab + bc + ca) (a + 2b)(a + 2c)

the desired inequality is equivalent to the obvious inequality


X 1
(a − b)2 (b − c)2 (c − a)2 ≥ 0.
(a + 2b)(a + 2c)

The equality holds for a = b, or b = c, or c = a.

P 1.20. Let a, b, c be nonnegative real numbers, no two of which are zero. Prove that

1 1 1 4
(a) + + ≥ ;
(a − b)2 (b − c)2 (c − a)2 ab + bc + ca

1 1 1 3
(b) + 2 + 2 ≥ ;
a2 − ab + b 2 b − bc + c 2 c − ca + a 2 ab + bc + ca

1 1 1 5
(c) + 2 + 2 ≥ .
a2 +b 2 b +c 2 c +a 2 2(ab + bc + ca)
Symmetric Rational Inequalities 51

Solution. Let
ab + bc + ca ab + bc + ca ab + bc + ca
Ek (a, b, c) = + + ,
a2 − kab + b2 b2 − kbc + c 2 c 2 − kca + a2
where k ∈ [0, 2]. We will prove that
Ek (a, b, c) ≥ αk ,
where
5 − 2k


 , 0≤k≤1
2−k
αk = .
 2 + k,
 1≤k≤2

Assume that a ≤ b ≤ c and show that


Ek (a, b, c) ≥ Ek (0, b, c) ≥ αk .
The left inequality is true because
Ek (a, b, c) − Ek (0, b, c)
=
a
b2 + (1 + k)bc − ac b+c c 2 + (1 + k)bc − ab
= + 2 +
b(a2 − kab + b2 ) b − kbc + c 2 c(c 2 − kca + a2 )
bc − ac b+c bc − ab
> + + > 0.
b(a2 − kab + b2 ) b2 − kbc + c 2 c(c 2 − kca + a2 )
In order to prove the right inequality, Ek (0, b, c) ≥ αk , where
bc b c
Ek (0, b, c) = + + ,
b2 − kbc + c 2 c b
we well use the AM-GM inequality. Thus, for k ∈ [1, 2], we have
bc b2 − kbc + c 2
Ek (0, b, c) = + + k ≥ 2 + k.
b2 − kbc + c 2 bc
Also, for k ∈ [0, 1], we have
bc b2 − kbc + c 2
Ek (0, b, c) = +
b2 − kbc + c 2 (2 − k)2 bc
1
• ˜ ‹
b c k
+ 1− + +
(2 − k)2 c b (2 − k)2
2 1 5 − 2k
• ˜
k
≥ +2 1− + = .
2−k (2 − k)2 (2 − k)2 2−k
b c
For k ∈ [1, 2], the equality holds when a = 0 and + = 1 + k (or any cyclic
c b
permutation). For k ∈ [0, 1], the equality holds when a = 0 and b = c (or any
cyclic permutation).
52 Vasile Cîrtoaje

P 1.21. If a, b, c are positive real numbers, then

(a2 + b2 )(a2 + c 2 ) (b2 + c 2 )(b2 + a2 ) (c 2 + a2 )(c 2 + b2 )


+ + ≥ a2 + b2 + c 2 .
(a + b)(a + c) (b + c)(b + a) (c + a)(c + b)
(Vasile Cîrtoaje, 2011)
Solution. Using the identity

(a2 + b2 )(a2 + c 2 ) = b2 c 2 + a2 (a2 + b2 + c 2 ),

we can write the inequality as follows:

b2 c 2 a2
X • X ˜
≥ (a + b + c ) 1 −
2 2 2
,
(a + b)(a + c) (a + b)(a + c)
X
b2 c 2 (b + c) ≥ 2abc(a2 + b2 + c 2 ),
X X
a3 (b2 + c 2 ) ≥ 2 a3 bc,
X
a3 (b − c)2 ≥ 0.
The equality holds for a = b = c.

P 1.22. Let a, b, c be positive real numbers such that a + b + c = 3. Prove that


1 1 1
+ 2 + 2 ≤ 1.
a2 +b+c b +c+a c +a+b

First Solution. By virtue of the Cauchy-Schwarz inequality, we have

(a2 + b + c)(1 + b + c) ≥ (a + b + c)2 .

Therefore,
X 1 X 1+ b+c 3 + 2(a + b + c)
≤ = = 1.
a2 + b + c (a + b + c)2 (a + b + c)2
The equality occurs for a = b = c = 1.
Second Solution. Rewrite the inequality as
1 1 1
+ + ≤ 1.
a2 − a + 3 b2 − b + 3 c 2 − c + 3
We see that the equality holds for a = b = c = 1. Thus, if there exists a real number
k such that
1 1
 ‹
≤k+ −k a
a2 − a + 3 3
Symmetric Rational Inequalities 53

for all a ∈ [0, 3], then


1 X• 1 1
X  ‹ ˜  ‹X
≤ k+ − k a = 3k + −k a = 1.
a2 − a + 3 3 3
We have
1 1 (a − 1) f (a)
‹
k+ −k a− 2 = ,
3 a − a + 3 3(a2 − a + 3)
where
f (a) = (1 − 3k)a2 + 3ka + 3(1 − 3k).
From f (1) = 0, we get k = 4/9. Thus, setting k = 4/9, we get
1 1 (a − 1)2 (3 − a)
 ‹
k+ −k a− 2 = ≥ 0.
3 a −a+3 9(a2 − a + 3)

P 1.23. Let a, b, c be real numbers such that a + b + c = 3. Prove that


a2 − bc b2 − ca c 2 − ab
+ 2 + 2 ≥ 0.
a2 + 3 b +3 c +3
(Vasile Cîrtoaje, 2005)
Solution. Apply the SOS method. We have
X a2 − bc X (a − b)(a + c) + (a − c)(a + b)
2 =
a2 + 3 a2 + 3
X (a − b)(a + c) X (b − a)(b + c)
= +
a2 + 3 ‹b + 3
2

a+c b+c
X 
= (a − b) 2 − 2
a +3 b +3
X (a − b)2
= (3 − ab − bc − ca) ≥ 0.
(a2 + 3)(b2 + 3)
Thus, it suffices to show that

3 − ab − bc − ca ≥ 0.

This follows immediately from the known inequality

(a + b + c)2 ≥ 3(ab + bc + ca),

which is equivalent to

(a − b)2 + (b − c)2 + (c − a)2 ≥ 0.

The equality holds for a = b = c = 1.


54 Vasile Cîrtoaje

P 1.24. Let a, b, c be nonnegative real numbers such that a + b + c = 3. Prove that


1 − bc 1 − ca 1 − ab
+ + ≥ 0.
5 + 2a 5 + 2b 5 + 2c

Solution. We apply the SOS method. Since

9(1 − bc) = (a + b + c)2 − 9bc,

we can write the inequality as


X a2 + b2 + c 2 + 2a(b + c) − 7bc
≥ 0.
5 + 2a
From
(a − b)(a + k b + mc) + (a − c)(a + kc + mb) =
= 2a2 − k(b2 + c 2 ) + (k + m − 1)a(b + c) − 2mbc,
choosing k = −2 and m = 7, we get

(a − b)(a − 2b + 7c) + (a − c)(a − 2c + 7b) = 2[a2 + b2 + c 2 + 2a(b + c) − 7bc].

Therefore, the desired inequality becomes as follows:


X (a − b)(a − 2b + 7c) X (a − c)(a − 2c + 7b)
+ ≥ 0,
5 + 2a 5 + 2a
X (a − b)(a − 2b + 7c) X (b − a)(b − 2a + 7c)
+ ≥ 0,
5 + 2a 5 + 2b
X
(a − b)(5 + 2c)[(5 + 2b)(a − 2b + 7c) − (5 + 2a)(b − 2a + 7c)] ≥ 0,
X
(a − b)2 (5 + 2c)(15 + 4a + 4b − 14c) ≥ 0,
X
(a − b)2 (5 + 2c)(a + b − c) ≥ 0.
Without loss of generality, assume that a ≥ b ≥ c. Clearly, it suffices to show that

(a − c)2 (5 + 2b)(a + c − b) ≥ (b − c)2 (5 + 2a)(a − b − c).

Since a − c ≥ b − c ≥ 0 and a + c − b ≥ a − b − c, we only need to show that

(a − c)(5 + 2b) ≥ (b − c)(5 + 2a).

Indeed,
(a − c)(5 + 2b) − (b − c)(5 + 2a) = (a − b)(5 + 2c) ≥ 0.
The equality holds for a = b = c = 1, and for a = b = 3/2 and c = 0 (or any cyclic
permutation).
Symmetric Rational Inequalities 55

P 1.25. Let a, b, c be positive real numbers such that a + b + c = 3. Prove that

1 1 1 3
+ + ≤ .
a2 + b2 + 2 b2 + c 2 + 2 c 2 + a2 + 2 4

(Vasile Cîrtoaje, 2006)

Solution. Since
2 a2 + b2
= 1 − ,
a2 + b2 + 2 a2 + b2 + 2
we may write the inequality as

a2 + b2 b2 + c 2 c 2 + a2 3
+ + ≥ .
a2 + b2 + 2 b2 + c 2 + 2 c 2 + a2 + 2 2
By the Cauchy-Schwarz inequality, we have
Pp 2
X a2 + b2 a2 + b2
≥P
a2 + b2 + 2 (a2 + b2 + 2)
Pp
2 a +2 (a2 + b2 )(a2 + c 2 )
P 2
=
2 a2 + 6
P

2 a + 2 (a + bc)
P 2 P 2

2 a2 + 6
P

3 a2 + 9 3
P
= P = .
2 a2 + 6 2

The equality holds for a = b = c = 1.

P 1.26. Let a, b, c be positive real numbers such that a + b + c = 3. Prove that

1 1 1 1
+ 2 + 2 ≤ .
4a2 +b +c
2 2 4b + c + a
2 2 4c + a + b
2 2 2

(Vasile Cîrtoaje, 2007)

Solution. According to the Cauchy-Schwarz inequality, we have

9 (a + b + c)2
=
4a2 + b2 + c 2 2a2 + (a2 + b2 ) + (a2 + c 2 )
1 b2 c2
≤ + 2 + .
2 a + b2 a2 + c 2
56 Vasile Cîrtoaje

Therefore,

9 3 X b2 c2
X  ‹
≤ + +
4a2 + b2 + c 2 2 a2 + b2 a2 + c 2
3 X b2 a2 3 9
 ‹
= + + = + 3 = .
2 a2 + b2 b2 + a2 2 2

The equality holds for a = b = c = 1.

P 1.27. Let a, b, c be nonnegative real numbers such that a + b + c = 2. Prove that

bc ca ab
+ + ≤ 1.
a2 + 1 b2 + 1 c 2 + 1
(Pham Kim Hung, 2005)

Solution. Let

p = a + b + c = 2, q = ab + bc + ca, q ≤ p2 /3 = 4/3.

If a = 0, then the inequality reduces to 4ab ≤ (a + b)2 . Otherwise, for a, b, c > 0,


write the inequality as
X 1 1
≤ ,
a(a2 + 1) abc
X1 a
‹
1
− 2 ≤ ,
a a +1 abc
X a 1 1 1 1
≥ + + − ,
a + 1 a b c abc
2

X a q−1
≥ ,
a2 + 1 abc
Using the inequality
2
≥ 2 − a,
a +1
2

which is equivalent to
a(a − 1)2 ≥ 0,
we get
X a X a(2 − a) X a(b + c)
≥ = = q.
a2 + 1 2 2
Therefore, it suffices to prove that

1 + abcq ≥ q.
Symmetric Rational Inequalities 57

By Schur’s inequality of degree four, we have

(p2 − q)(4q − p2 ) (4 − q)(q − 1)


abc ≥ = .
6p 3

Thus,
q(4 − q)(q − 1) (3 − q)(q − 1)2
1 + abcq − q ≥ 1 + −q = ≥ 0.
3 3
The equality holds if a = 0 and b = c = 1 (or any cyclic permutation).

P 1.28. Let a, b, c be nonnegative real numbers such that a + b + c = 1. Prove that

bc ca ab 1
+ + ≤ .
a+1 b+1 c+1 4
(Vasile Cîrtoaje, 2009)

First Solution. We have


X bc X bc
=
a+1 (a + b) + (c + a)
1X 1 1
 ‹
≤ bc +
4 a+b c+a
1 X bc 1 X bc
= +
4 a+b 4 c+a
1 X bc 1 X ca
= +
4 a+b 4 a+b
1 X bc + ca 1 X 1
= = c= .
4 a+b 4 4
The equality holds for a = b = c = 1/3, and for a = 0 and b = c = 1/2 (or any
cyclic permutation).
Second Solution. It is easy to check that the inequality is true if one of a, b, c is
zero. Otherwise, write the inequality as
1 1 1 1
+ + ≤ .
a(a + 1) b(b + 1) c(c + 1) 4abc
Since
1 1 1
= − ,
a(a + 1) a a + 1
we may write the required inequality as
1 1 1 1 1 1 1
+ + ≥ + + − .
a + 1 b + 1 c + 1 a b c 4abc
58 Vasile Cîrtoaje

In virtue of the Cauchy-Schwarz inequality, we have

1 1 1 9 9
+ + ≥ = .
a + 1 b + 1 c + 1 (a + 1) + (b + 1) + (c + 1) 4

Therefore, it suffices to prove that

9 1 1 1 1
≥ + + − .
4 a b c 4abc
This is equivalent to Schur’s inequality

(a + b + c)3 + 9abc ≥ 4(a + b + c)(ab + bc + ca).

P 1.29. Let a, b, c be positive real numbers such that a + b + c = 1. Prove that

1 1 1 3
+ + ≤ .
a(2a2 + 1) b(2b2 + 1) c(2c 2 + 1) 11abc

(Vasile Cîrtoaje, 2009)

Solution. Since
1 1 2a
= − 2 ,
a(2a + 1) a 2a + 1
2

we can write the inequality as


X 2a 1 1 1 3
≥ + + − .
2a + 1 a b c 11abc
2

By the Cauchy-Schwarz inequality, we have

( a)2
P
X a 1
≥P = .
2a + 1
2 a(2a + 1) 2(a + b + c 3 ) + 1
2 3 3

Therefore, it suffices to show that

2 11q − 3
≥ ,
2(a3 + b3 + c 3 ) + 1 11abc

where
1 1
q = ab + bc + ca, q ≤ (a + b + c)2 = .
3 3
Since

a3 + b3 + c 3 = 3abc + (a + b + c)3 − 3(a + b + c)(ab + bc + ca) = 3abc + 1 − 3q,


Symmetric Rational Inequalities 59

we need to prove that

22abc ≥ (11q − 3)(6abc + 3 − 6q),

or, equivalently,
2(20 − 33q)abc ≥ 3(11q − 3)(1 − 2q).
From Schur’s inequality

(a + b + c)3 + 9abc ≥ 4(a + b + c)(ab + bc + ca),

we get
9abc ≥ 4q − 1.
Thus,
2(20 − 33q)abc − 3(11q − 3)(1 − 2q) ≥
2(20 − 33q)(4q − 1)
≥ − 3(11q − 3)(1 − 2q)
9
330q2 − 233q + 41 (1 − 3q)(41 − 110q)
= = ≥ 0.
9 9
This completes the proof. The equality holds for a = b = c = 1/3.

P 1.30. Let a, b, c be positive real numbers such that a + b + c = 3. Prove that

1 1 1
+ + ≤ 1.
a3 + b + c b3 + c + a c 3 + a + b
(Vasile Cîrtoaje, 2009)

Solution. Write the inequality in the form

1 1 1
+ 3 + 3 ≤ 1.
a3 −a+3 b − b+3 c −c+3
Assume that a ≥ b ≥ c. There are two cases to consider.
Case 1: 2 ≥ a ≥ b ≥ c. The desired inequality follows by adding the inequalities

1 5 − 2a 1 5 − 2b 1 5 − 2c
≤ , 3 ≤ , 3 ≤ .
a3 −a+3 9 b − b+3 9 c −c+3 9
These inequalities are true since

1 5 − 2a (a − 1)2 (a − 2)(2a + 3)
− = ≤ 0.
a3 − a + 3 9 9(a3 − a + 3)
60 Vasile Cîrtoaje

Case 2: a > 2. From a + b + c = 3, we get b + c < 1. Since


X 1 1 1 1 1 1 1
< 3 + + < + + ,
a3 −a+3 a −a+3 3− b 3−c 9 3− b 3−c
it suffices to prove that
1 1 8
+ ≤ .
3− b 3−c 9
We have
1 1 8 −3 − 15(1 − b − c) − 8bc
+ − = < 0.
3− b 3−c 9 9(3 − b)(3 − c)
The equality holds for a = b = c = 1.

P 1.31. Let a, b, c be positive real numbers such that a + b + c = 3. Prove that

a2 b2 c2
+ + ≥ 1.
1 + b3 + c 3 1 + c 3 + a3 1 + a3 + b3

Solution. Using the Cauchy-Schwarz inequality, we have


P 2
X a 2 a2
≥P ,
1 + b3 + c 3 a2 (1 + b3 + c 3 )

and it remains to show that


X
(a2 + b2 + c 2 )2 ≥ (a2 + b2 + c 2 ) + a2 b2 (a + b).

Let
p = a + b + c, q = ab + bc + ca, q ≤ 3.
Since a2 + b2 + c 2 = 9 − 2q and
X X X
a b (a + b) =
2 2
a b (3 − c) = 3
2 2
a2 b2 − qabc = 3q2 − (q + 18)abc,

the desired inequality can be written as

(9 − 2q)2 ≥ (9 − 2q) + 3q2 − (q + 18)abc,

q2 − 34q + 72 + (q + 18)abc ≥ 0.
This inequality is clearly true for q ≤ 2. Consider further that 2 < q ≤ 3. By Schur’s
inequality of degree four, we get

(p2 − q)(4q − p2 ) (9 − q)(4q − 9)


abc ≥ = .
6p 18
Symmetric Rational Inequalities 61

Therefore
(q + 18)(9 − q)(4q − 9)
q2 − 34q + 72 + (q + 18)abc ≥ q2 − 34q + 72 +
18
(3 − q)(4q2 + 21q − 54)
= ≥ 0.
18
The equality holds for a = b = c = 1.

P 1.32. Let a, b, c be nonnegative real numbers such that a + b + c = 3. Prove that

1 1 1 3
+ + ≤ .
6 − a b 6 − bc 6 − ca 5

Solution. Rewrite the inequality as

108 − 48(ab + bc + ca) + 13abc(a + b + c) − 3a2 b2 c 2 ≥ 0,

4[9 − 4(ab + bc + ca) + 3abc] + abc(1 − abc) ≥ 0.


By the AM-GM inequality,
‹3
a+b+c

1= ≥ abc.
3
Consequently, it suffices to show that

9 − 4(ab + bc + ca) + 3abc ≥ 0.

We see that the homogeneous form of this inequality is just Schur’s inequality of
third degree

(a + b + c)3 + 9abc ≥ 4(a + b + c)(ab + bc + ca).

The equality holds for a = b = c = 1, as well as for a = 0 and b = c = 3/2 (or any
cyclic permutation).

P 1.33. Let a, b, c be nonnegative real numbers such that a + b + c = 3. Prove that

1 1 1 1
+ + ≤ .
2a2 + 7 2b2 + 7 2c 2 + 7 3
(Vasile Cîrtoaje, 2005)
62 Vasile Cîrtoaje

Solution. Use the mixing variables method. Assume that a = max{a, b, c} and
prove that
1
E(a, b, c) ≤ E(a, s, s) ≤ ,
3
where
b+c
s= , 0 ≤ s ≤ 1,
2
1 1 1
E(a, b, c) = 2 + 2 + 2 .
2a + 7 2b + 7 2c + 7
We have
1 1 1 1
 ‹  ‹
E(a, s, s) − E(a, b, c) = − + −
2s2 + 7 2b2 + 7 2s2 + 7 2c 2 + 7
1 (b − c)(b + s) (c − b)(c + s)
• ˜
= 2 +
2s + 7 2b2 + 7 2c 2 + 7
(b − c)2 (7 − 4s2 − 2bc)
= .
(2s2 + 7)(2b2 + 7)(2c 2 + 7)

Since bc ≤ s2 ≤ 1, it follows that

7 − 4s2 − 2bc = 1 + 4(1 − s2 ) + 2(1 − bc) > 0,

hence E(a, s, s) ≥ E(a, b, c). Also,

1 1 4(s − 1)2 (2s − 1)2


− E(a, s, s) = − E(3 − 2s, s, s) = ≥ 0.
3 3 3(2a2 + 7)(2s2 + 7)

The equality holds for a = b = c = 1, as well as for a = 2 and b = c = 1/2 (or any
cyclic permutation).

P 1.34. Let a, b, c be nonnegative real numbers such that a + b + c = 3. Prove that

1 1 1 3
+ + ≥ .
2a2 + 3 2b2 + 3 2c 2 + 3 5
(Vasile Cîrtoaje, 2005)

First Solution (by Nguyen Van Quy). Write the inequality as


X 1 1
‹
2
− 2 ≤ ,
3 2a + 3 5
X a2 3
≤ .
2a + 5 5
2
Symmetric Rational Inequalities 63

Using the Cauchy-Schwarz inequality gives


25 25
= 2
3(2a + 3) 6a + (a + b + c)2
2

(2 + 2 + 1)2
=
2(2a2 + bc) + 2a(a + b + c) + a2 + b2 + c 2
22 22 1
≤ + + 2 ,
2(2a + bc) 2a(a + b + c) a + b2 + c 2
2

hence
X 25a2 X 2a2 X 2a X a2
≤ + +
3(2a2 + 3) 2a2 + bc a+b+c a2 + b2 + c 2
X 2a2
= + 3.
2a2 + bc
Therefore, it suffices to show that
X a2
≤ 1.
2a2 + bc
For the nontrivial case a, b, c > 0, this is equivalent to
X 1
≤ 1,
2 + bc/a2

which follows immediately from P 1.2-(b). The equality holds for a = b = c = 1,


as well as for a = 0 and b = c = 3/2 (or any cyclic permutation).
Second Solution. First, we can check that the desired inequality becomes an equal-
ity for a = b = c = 1, and for a = 0 and b = c = 3/2. Consider then the inequality
f (x) ≥ 0, where
1 −4x
f (x) = − A − B x, f 0 (x) = − B.
2x 2+3 (2x 2 + 3)2

The conditions f (1) = 0 and f 0 (1) = 0 involve A = 9/25 and B = −4/25. Also, the
conditions f (3/2) = 0 and f 0 (3/2) = 0 involve A = 22/75 and B = −8/75. Using
these values of A and B, we obtain the identities

1 9 − 4x 2(x − 1)2 (4x − 1)


− = ,
2x 2 + 3 25 25(2x 2 + 3)

1 22 − 8x (2x − 3)2 (4x + 1)


− = ,
2x 2 + 3 75 75(2x 2 + 3)
and the inequalities
1 9 − 4x 1
≥ , x≥ ,
2x 2+3 25 4
64 Vasile Cîrtoaje

1 22 − 8x
≥ , x ≥ 0.
2x 2+3 75
Without loss of generality, assume that a ≥ b ≥ c.
1
Case 1: a ≥ b ≥ c ≥ . By summing the inequalities
4

1 9 − 4a 1 9 − 4b 1 9 − 4c
≥ , ≥ , ≥ ,
2a2+3 25 2b2+3 25 2c 2+3 25
we get
1 1 1 27 − 4(a + b + c) 3
+ 2 + 2 ≥ = .
2a2 + 3 2b + 3 2c + 3 25 5

1
Case 2: a ≥ b ≥ ≥ c. We have
4
X 1 22 − 8a 22 − 8b 1
≥ + +
2a2 + 3 75 75 2c 2 + 3
44 − 8(a + b) 1 20 + 8c 1
= + 2 = + 2 .
75 2c + 3 75 2c + 3

Therefore, it suffices to show that

20 + 8c 1 3
+ 2 ≥ ,
75 2c + 3 5

which is equivalent to the obvious inequality

c(8c 2 − 25c + 12) ≥ 0.

1
Case 3: a ≥ ≥ b ≥ c. We have
4
X 1 1 1 2 3
> 2 + 2 ≥ > .
2a2+ 3 2b + 3 2c + 3 1/8 + 3 5

P 1.35. Let a, b, c be nonnegative real numbers such that ab + bc + ca = 3. Prove


that
1 1 1 a+b+c 3
+ + ≥ + .
a+b b+c c+a 6 a+b+c
(Vasile Cîrtoaje, 2007)
Symmetric Rational Inequalities 65

First Solution. Denoting

x = a + b + c, x ≥ 3,

we have

1 1 1 (a + b + c)2 + ab + bc + ca x2 + 3
+ + = = .
a + b b + c c + a (a + b + c)(ab + bc + ca) − abc 3x − abc

Then, the inequality becomes

x2 + 3 x 3
≥ + ,
3x − abc 6 x

3(x 3 + 9abc − 12x) + abc(x 2 − 9) ≥ 0.


This inequality is true since

x 2 − 9 ≥ 0, x 3 + 9abc − 12x ≥ 0.

The last inequality is just Schur’s inequality of degree three

(a + b + c)3 + 9abc ≥ 4(a + b + c)(ab + bc + ca).


p
The equality holds for a = b = c = 1, and for a = 0 and b = c = 3 (or any cyclic
permutation).
Second Solution. We apply the SOS method. Write the inequality as follows:

1 1 1 a+b+c 3
+ + ≥ + ,
a + b b + c c + a 2(ab + bc + ca) a + b + c

1 1 1 (a + b + c)2
 ‹
2(a + b + c) + + ≥ + 6,
a+b b+c c+a ab + bc + ca
1 1 1 (a + b + c)2
 ‹
[(a + b) + (b + c) + (c + a)] + + −9≥ − 3,
a+b b+c c+a ab + bc + ca
X (b − c)2 1 X
≥ (b − c)2 ,
(a + b)(c + a) 2(ab + bc + ca)
X ab + bc + ca − a2
(b − c)2 ≥ 0,
(a + b)(c + a)
X 3 − a2
(b − c)2 ≥ 0,
3 + a2
Without loss of generality, assume that a ≥ b ≥ c. Since 3 − c 2 ≥ 0, it suffices to
show that
3 − a2 3 − b2
(b − c)2
+ (c − a)2 ≥ 0.
3+a 2 3+ b 2
66 Vasile Cîrtoaje

Having in view that

3 − b2 = ab + bc + ca − b2 ≥ b(a − b) ≥ 0, (c − a)2 ≥ (b − c)2 ,

it is enough to prove that


3 − a2 3 − b2
+ ≥ 0.
3 + a2 3 + b2
This is true since
3 − a2 3 − b2 2(9 − a2 b2 ) 2c(a + b)(3 + ab)
+ = = ≥ 0.
3+a 2 3+ b 2 (3 + a )(3 + b )
2 2 (3 + a2 )(3 + b2 )

P 1.36. Let a, b, c be nonnegative real numbers such that ab + bc + ca = 3. Prove


that
1 1 1 3
+ + ≥ .
a2 + 1 b2 + 1 c 2 + 1 2
(Vasile Cîrtoaje, 2005)

First Solution. After expanding, the inequality can be restated as

a2 + b2 + c 2 + 3 ≥ a2 b2 + b2 c 2 + c 2 a2 + 3a2 b2 c 2 .

From

(a + b + c)(ab + bc + ca) − 9abc = a(b − c)2 + b(c − a)2 + c(a − b)2 ≥ 0,

we get
a + b + c ≥ 3abc.
So, it suffices to show that

a2 + b2 + c 2 + 3 ≥ a2 b2 + b2 c 2 + c 2 a2 + abc(a + b + c).

This is equivalent to the homogeneous inequalities

(a b+ bc +ca)(a2 + b2 +c 2 )+(ab+ bc +ca)2 ≥ 3(a2 b2 + b2 c 2 +c 2 a2 )+3abc(a+ b+c),

ab(a2 + b2 ) + bc(b2 + c 2 ) + ca(c 2 + a2 ) ≥ 2(a2 b2 + b2 c 2 + c 2 a2 ),


ab(a − b)2 + bc(b − c)2 + ca(c − a)2 ≥ 0.
p
The equality holds for a = b = c = 1, and for a = 0 and b = c = 3 (or any cyclic
permutation).
Second Solution. Without loss of generality, assume that

a = min{a, b, c}, bc ≥ 1.
Symmetric Rational Inequalities 67

From

(a + b + c)(ab + bc + ca) − 9abc = a(b − c)2 + b(c − a)2 + c(a − b)2 ≥ 0,

we get
a + b + c ≥ 3abc.
The desired inequality follows by summing the inequalities

1 1 2
+ ≥ ,
b2 + 1 c 2 + 1 bc + 1
1 2 3
+ ≥ .
a2 + 1 bc + 1 2
We have
1 1 2 b(c − b) c(b − c)
+ 2 − = 2 + 2
b2 + 1 c + 1 bc + 1 (b + 1)(bc + 1) (c + 1)(bc + 1)
(b − c)2 (bc − 1)
= 2 ≥0
(b + 1)(c 2 + 1)(bc + 1)

and

1 2 3 a2 − bc + 3 − 3a2 bc a(a + b + c − 3abc)


+ − = = ≥ 0.
a + 1 bc + 1 2
2 2(a + 1)(bc + 1)
2 2(a2 + 1)(bc + 1)

Third Solution. Since

1 a2 1 b2 1 c2
= 1 − , = 1 − , = 1 − ,
a2 + 1 a2 + 1 b2 + 1 b2 + 1 c 2 + 1 c2 + 1
we can rewrite the inequality as

a2 b2 c2 3
+ + ≤ ,
a +1 b +1 c +1 2
2 2 2

or, in the homogeneous form,


X a2 1
≤ .
3a + ab + bc + ca 2
2

According to the Cauchy-Schwarz inequality, we have

4a2 (a + a)2 a a2
= ≤ + ,
3a2 + ab + bc + ca a(a + b + c) + (2a2 + bc) a + b + c 2a2 + bc

hence
X 4a2 X a2
≤ 1 + .
3a2 + a b + bc + ca 2a2 + bc
68 Vasile Cîrtoaje

It suffices to show that


X a2
≤ 1.
2a2 + bc
For the nontrivial case a, b, c > 0, this is equivalent to
X 1
≤ 1,
2 + bc/a2

which follows immediately from P 1.2-(b).


Remark. We can write the inequality in P 1.36 in the homogeneous form

1 1 1 3
2
+ 2
+ 2
≥ .
3a 3b 3c 2
1+ 1+ 1+
ab + bc + ca ab + bc + ca ab + bc + ca
1 1 1
Substituting a, b, c by , , , respectively, we get
x y z

x y z 3
+ + ≥ .
3 yz 3z x 3x y 2
x+ y+ z+
x + y +z x + y +z x + y +z

So, we find the following result.


• If x, y, z are positive real numbers such that x + y + z = 3, then

x y z 3
+ + ≥ .
x + yz y + z x z + x y 2

P 1.37. Let a, b, c be positive real numbers such that ab + bc + ca = 3. Prove that

a2 b2 c2
+ + ≥ 1.
a2 + b + c b2 + c + a c 2 + a + b
(Vasile Cîrtoaje, 2005)

Solution. We apply the Cauchy-Schwarz inequality in the following way


2
a3/2 + b3/2 + c 3/2 a3 + 2 (ab)3/2
P P
X a2
≥ = .
a2 + b + c a(a2 + b + c) a3 + 6
P P

Then, we still have to show that

(ab)3/2 + (bc)3/2 + (ca)3/2 ≥ 3.


Symmetric Rational Inequalities 69

By the AM-GM inequality, we have

(ab)3/2 + (ab)3/2 + 1 1 3ab 1


(ab) 3/2
= − ≥ − ,
2 2 2 2
hence
3 3
(ab)3/2 + (bc)3/2 + (ca)3/2 ≥ (ab + bc + ca) − = 3.
2 2
The equality holds for a = b = c = 1.

P 1.38. Let a, b, c be positive real numbers such that ab + bc + ca = 3. Prove that

bc + 4 ca + 4 ab + 4 bc + 2 ca + 2 ab + 2
+ 2 + 2 ≤3≤ 2 + + .
a +4 b +4 c +4
2 a + 2 b2 + 2 c 2 + 2
(Vasile Cîrtoaje, 2007)

Solution. More general, using the SOS method, we will show that

bc + k ca + k ab + k
 ‹
(k − 3) 2 + + 2 −3 ≤0
a + k b2 + k c +k

for k > 0. This inequality is equivalent to


X a2 − bc
(k − 3) ≥ 0.
a2 + k
Since
X a2 − bc X (a − b)(a + c) + (a − c)(a + b)
2 =
a2 + k a2 + k
X (a − b)(a + c) X (b − a)(b + c)
= +
a2 + k b2 + k
X (a − b)2
= (k − ab − bc − ca)
(a2 + k)(b2 + k)
X (a − b)2
= (k − 3) ,
(a2 + k)(b2 + k)

we have
X a2 − bc X (a − b)2
2(k − 3) = (k − 3) 2
≥ 0.
a2 + k (a2 + k)(b2 + k)
The equality in both inequalities holds for a = b = c = 1.
70 Vasile Cîrtoaje

P 1.39. Let a, b, c be nonnegative real numbers such that ab + bc + ca = 3. If


p
k ≥ 2 + 3,

then
1 1 1 3
+ + ≤ .
a+k b+k c+k 1+k
(Vasile Cîrtoaje, 2007)

Solution. Let us denote


p = a + b + c, p ≥ 3.
By expanding, the inequality becomes

k(k − 2)p + 3abc ≥ 3(k − 1)2 .

Since this inequality is true for p ≥ 3(k − 1)2 /(k2 − 2k), consider further that

3(k − 1)2
p≤ .
k(k − 2)
From Schur’s inequality

(a + b + c)3 + 9abc ≥ 4(ab + bc + ca)(a + b + c),

we get
9abc ≥ 12p − p3 .
Therefore, it suffices to prove that

3k(k − 2)p + 12p − p3 ≥ 9(k − 1)2 ,

or, equivalently,
(p − 3)[(3(k − 1)2 − p2 − 3p] ≥ 0.
Thus, it remains to prove that

3(k − 1)2 − p2 − 3p ≥ 0.
p
Since p ≤ 3(k − 1)2 /(k2 − 2k) and k ≥ 2 + 3, we have

2 2 9(k − 1)4
2 9(k − 1)2
3(k − 1) − p − 3p ≥ 3(k − 1) − 2 −
k (k − 2)2 k(k − 2)
3(k − 1) (k − 3)(k − 4k + 1)
2 2 2
= ≥ 0.
k2 (k − 2)2
p
The equality holds for a =pb = c = 1. In the case k = 2 + 3, the equality holds
also for a = 0 and b = c = 3 (or any cyclic permutation).
Symmetric Rational Inequalities 71

P 1.40. Let a, b, c be nonnegative real numbers such that a2 + b2 + c 2 = 3. Prove that

a(b + c) b(c + a) c(a + b)


+ + ≤ 3.
1 + bc 1 + ca 1 + ab
(Vasile Cîrtoaje, 2010)

Solution. Write the inequality in the homogeneous forms


X a(b + c)
≤ 1,
a2 + b2 + c 2 + 3bc
X• a(b + c)
˜
a
− ≤ 0,
a2 + b2 + c 2 + 3bc a + b + c
X a(a − b)(a − c)
≥ 0.
a2 + b2 + c 2 + 3bc
Without loss of generality, assume that a ≥ b ≥ c. Then, it suffices to prove that

a(a − b)(a − c) b(b − c)(b − a)


+ 2 ≥ 0,
a2+ b + c + 3bc a + b2 + c 2 + 3ca
2 2

which is true if
a(a − c) b(b − c)
≥ .
a2 + b2 + c 2 + 3bc a2 + b2 + c 2 + 3ca
Since
a(a − c) ≥ b(b − c)
and
1 1
≥ 2 ,
a2 + b2 + c + 3bc
2 a + b + c 2 + 3ca
2
p
the conclusion follows. The equality holds for a = b = c = 1, and for a = b = 3/2
and c = 0 (or any cyclic permutation).

P 1.41. Let a, b, c be positive real numbers such that a2 + b2 + c 2 = 3. Prove that

a2 + b2 b2 + c 2 c 2 + a2
+ + ≥ 3.
a+b b+c c+a
(Cezar Lupu, 2005)

First Solution. We apply the SOS method. Write the inequality in the homoge-
neous forms
X  b2 + c 2 b + c ‹ Æ
− ≥ 3(a2 + b2 + c 2 ) − a − b − c,
b+c 2
72 Vasile Cîrtoaje

(b − c)2
P
X (b − c)2
≥p .
2(b + c) 3(a2 + b2 + c 2 ) + a + b + c
Since Æ
3(a2 + b2 + c 2 ) + a + b + c ≥ 2(a + b + c) > 2(b + c),
the conclusion follows. The equality holds for a = b = c = 1.
Second Solution. By virtue of the Cauchy-Schwarz inequality, we have
Pp 2
a2 + b2
Pp
2 a2 + 2 (a2 + b2 )(a2 + c 2 )
P
X a2 + b2
≥ P =
a+b (a + b)
P
2 a
P 2
2 a + 2 (a + bc) 3 a +
P 2 P 2 P 2
a
≥ P = P
2 a 2 a
P 2 2
9+
P
a a−3
= P =3+ P ≥ 3.
2 a 2 a

P 1.42. Let a, b, c be positive real numbers such that a2 + b2 + c 2 = 3. Prove that


ab bc ca 7
+ + + 2 ≤ (a + b + c).
a+b b+c c+a 6
(Vasile Cîrtoaje, 2011)

Solution. We apply the SOS method. Write the inequality as


X 4bc
‹
3 b+c− ≥ 8(3 − a − b − c).
b+c
Since
4bc (b − c)2
b+c− =
b+c b+c
and
9 − (a + b + c)2 3(a2 + b2 + c 2 ) − (a + b + c)2
3−a− b−c = =
3+a+ b+c 3+a+ b+c
1 X
= (b − c)2 ,
3+a+ b+c
we can write the inequality as

Sa (b − c)2 + S b (c − a)2 + Sc (a − b)2 ≥ 0,

where
3 8
Sa = − .
b+c 3+a+ b+c
Symmetric Rational Inequalities 73

Without loss of generality, assume that a ≥ b ≥ c, which involves Sa ≥ S b ≥ Sc . If

S b + Sc ≥ 0,

then
Sa ≥ S b ≥ 0,
hence

Sa (b − c)2 + S b (c − a)2 + Sc (a − b)2 ≥ S b (c − a)2 + Sc (a − b)2


≥ (S b + Sc )(a − b)2 ≥ 0.

By the Cauchy-Schwarz inequality, we have

1 1 16
 ‹
S b + Sc = 3 + −
a+c a+b 3+a+ b+c
12 16
≥ −
(a + c) + (a + b) 3 + a + b + c
4(9 − 5a − b − c)
= .
(2a + b + c)(3 + a + b + c)

Therefore,we only need to show that

9 ≥ 5a + b + c.

This follows immediately from the Cauchy-Schwarz inequality

(25 + 1 + 1)(a2 + b2 + c 2 ) ≥ (5a + b + c)2 .

Thus, the proof is completed. The equality holds for a = b = c = 1, and also for
a = 5/3 and b = c = 1/3 (or any cyclic permutation).

P 1.43. Let a, b, c be positive real numbers such that a2 + b2 + c 2 = 3. Prove that

1 1 1 3
(a) + + ≤ ;
3 − ab 3 − bc 3 − ca 2
1 1 1
(b) + + ≤ 1;
5 − 2ab 5 − 2bc 5 − 2ca
1 1 1 3
(c) p +p +p ≤p .
6 − ab 6 − bc 6 − ca 6−1

(Vasile Cîrtoaje, 2005)


74 Vasile Cîrtoaje

Solution. (a) Since

3 ab 2ab
=1+ =1+ 2
3 − ab 3 − ab a + b + 2c 2 + (a − b)2
2

2ab (a + b)2
≤1+ 2 ≤ 1 + ,
a + b2 + 2c 2 2(a2 + b2 + 2c 2 )

it suffices to prove that

(a + b)2 (b + c)2 (c + a)2


+ + ≤ 3.
a2 + b2 + 2c 2 b2 + c 2 + 2a2 c 2 + a2 + 2b2
By the Cauchy-Schwarz inequality, we have

(a + b)2 (a + b)2 a2 b2
= ≤ + .
a2 + b2 + 2c 2 (a2 + c 2 ) + (b2 + c 2 ) a2 + c 2 b2 + c 2

Thus,
X (a + b)2 X a2 X b2 X a2 X c2
≤ + = + = 3.
a2 + b2 + 2c 2 a2 + c 2 b2 + c 2 a2 + c 2 c 2 + a2
The equality holds for a = b = c = 1.
(b) Write the inequality in the homogeneous form
X a2 + b2 + c 2
≤ 1.
5(a2 + b2 + c 2 ) − 6bc

Since
2(a2 + b2 + c 2 ) 3a2 + 3(b − c)2
= 1 − ,
5(a2 + b2 + c 2 ) − 6bc 5(a2 + b2 + c 2 ) − 6bc
the inequality is equivalent to
X a2 + (b − c)2 1
≥ .
5(a + b + c ) − 6bc
2 2 2 3

Assume that
a ≥ b ≥ c.
By the Cauchy-Schwarz inequality, we have
P 2
a + 2 ab
P 2 P
X a2 a
≥P = P .
5(a2 + b2 + c 2 ) − 6bc [5(a2 + b2 + c 2 ) − 6bc] 15 a2 − 6 ab
P

X (b − c)2 [(b − c) + (a − c) + (a − b)]2 4(a − c)2


≥ = P .
5(a2 + b2 + c 2 ) − 6bc [5(a2 + b2 + c 2 ) − 6bc]
P P
15 a2 − 6 ab
Symmetric Rational Inequalities 75

Therefore, it suffices to show that

a2 + 2 ab + 4(a − c)2
P P
1
P P ≥ ,
2
15 a − 6 ab 3

which is equivalent to
X X
ab + (a − c) ≥
2
a2 ,

(a − b)(b − c) ≥ 0.

(c) According to P 1.32, the following inequality holds

1 1 1 3
+ + ≤ .
6 − a2 b2 6 − b2 c 2 6 − c 2 a2 5

Since p
2 6 1 1
= p + p ,
6 − a2 b2 6 − ab 6 + ab
this inequality becomes
p
X 1 X 1 6 6
p + p ≤ .
6 − ab 6 + ab 5

Thus, it suffices to show that


X 1 3
p ≥p .
6 + ab 6+1

Since ab + bc + ca ≤ a2 + b2 + c 2 = 3, by the Cauchy-Schwarz inequality, we have


X 1 9 9 3
p ≥Pp = p ≥p .
6 + ab ( 6 + ab) 3 6 + ab + bc + ca 6+1

The equality holds for a = b = c = 1.

P 1.44. Let a, b, c be positive real numbers such that a2 + b2 + c 2 = 3. Prove that

1 1 1 3
+ + ≥ .
1+a 5 1+ b 5 1+c 5 2

(Vasile Cîrtoaje, 2007)


76 Vasile Cîrtoaje

Solution. Let a = min{a, b, c}. There are two cases to consider.


1
Case 1: a ≥ . The desired inequality follows by summing the inequalities
2
8 8 8
≥ 9 − 5a2 , ≥ 9 − 5b2 , ≥ 9 − 5c 2 .
1 + a5 1 + b5 1 + c5
To obtain these inequalities, we consider the inequality
8
≥ p + qx 2 ,
1+ x 5

where the real coefficients p and q will be determined such that (x − 1)2 is a factor
of the polynomial
P(x) = 8 − (1 + x 5 )(p + qx 2 ).
It is easy to check that P(1) = 0 involves p + q = 4, hence

P(x) = 4(2 − x 2 − x 7 ) − p(1 − x 2 + x 5 − x 7 ) = (1 − x)Q(x),

where

Q(x) = 4(2 + 2x + x 2 + x 3 + x 4 + x 5 + x 6 ) − p(1 + x + x 5 + x 6 ).

In addition, Q(1) = 0 involves p = 9, hence

P(x) = (1 − x)2 (5x 5 + 10x 4 + 6x 3 + 2x 2 − 2x − 1)


= (1 − x)2 [x 5 + (2x − 1)(2x 4 + 6x 3 + 6x 2 + 4x + 1)].

1
Clearly, we have P(x) ≥ 0 for x ≥ .
2
1
Case 2: a ≤ . Write the desired inequality as
2
1 1 b5 c 5 − 1
− ≥ .
1 + a5 2 (1 + b5 )(1 + c 5 )
Since
1 1 32 1 31
− ≥ − =
1+a 5 2 33 2 66
and p
(1 + b5 )(1 + c 5 ) ≥ (1 + b5 c 5 )2 ,
it suffices to show that
p
31(1 + b5 c 5 )2 ≥ 66(b5 c 5 − 1).

For the nontrivial case bc > 1, this inequality is equivalent to


p p
31(1 + b5 c 5 ) ≥ 66( b5 c 5 − 1),
Symmetric Rational Inequalities 77

bc ≤ (97/35)2/5 .
Indeed, from
3 = a2 + b2 + c 2 > b2 + c 2 ≥ 2bc,
we get
bc < 3/2 < (97/35)2/5 .
This completes the proof. The equality holds for a = b = c = 1.

P 1.45. Let a, b, c be positive real numbers such that abc = 1. Prove that

1 1 1
+ 2 + 2 ≥ 1.
a2 +a+1 b + b+1 c +c+1

First Solution. Using the substitution


yz zx xy
a= , b= , c= ,
x2 y2 z2

where x, y, z are positive real numbers, the inequality becomes


X x4
≥ 1.
x 4 + x 2 yz + y 2 z 2

By the Cauchy-Schwarz inequality, we have


P 2
x2 x + 2 y 2z2
P 4 P
4
X x
≥P =P .
x 4 + x 2 yz + y 2 z 2 (x 4 + x 2 yz + y 2 z 2 ) x 4 + x yz x + y 2 z 2
P P

Therefore, it suffices to show that


X X
y 2 z 2 ≥ x yz x,

x 2 ( y − z)2 ≥ 0. The equality holds for a = b = c = 1.


P
which is equivalent to
Second Solution. Using the substitution
y z x
a= , b= , c= ,
x y z

where x, y, z > 0, we need to prove that

x2 y2 z2
+ + ≥ 1.
x 2 + x y + y 2 y 2 + yz + z 2 z 2 + z x + z 2
78 Vasile Cîrtoaje

Since
x 2 (x 2 + y 2 + z 2 + x y + yz + z x) x 2 z(x + y + z)
= x 2
+ ,
x2 + x y + y2 x2 + x y + y2
multiplying by x 2 + y 2 + z 2 + x y + yz + z x, the inequality can be written as
X x 2z x y + yz + z x
≥ .
x +xy+ y
2 2 x + y +z

By the Cauchy-Schwarz inequality, we have


P 2
X x 2z xz x y + yz + z x
≥ = .
x2 + x y + y2 z(x 2 + x y + y 2 ) x + y +z
P

Remark. The inequality in P 1.45 is a particular case of the following more general
inequality (Vasile Cîrtoaje, 2009).
• Let a1 , a2 , . . . , an (n ≥ 3) be positive real numbers such that a1 a2 · · · an = 1. If
p, q ≥ 0 such that p + q = n − 1, then
i=n
X 1
≥ 1.
i=1
1 + pai + qai2

P 1.46. Let a, b, c be positive real numbers such that abc = 1. Prove that
1 1 1
+ + ≤ 3.
a2 − a + 1 b2 − b + 1 c 2 − c + 1

First Solution. Since


1 1 2(a2 + 1) 2a4
+ = = 2 − ,
a2 − a + 1 a2 + a + 1 a4 + a2 + 1 a4 + a2 + 1
we can rewrite the inequality as
X 1 X a4
+ 2 ≥ 3.
a2 + a + 1 a4 + a2 + 1
Thus, it suffices to show that
X 1
≥1
a2 +a+1
and
X a4
≥ 1.
a4 + a2 + 1
Symmetric Rational Inequalities 79

The first inequality is just the inequality in P 1.45, while the second follows from
the first by substituting a, b, c with a−2 , b−2 , c −2 , respectively. The equality holds
for a = b = c = 1.
Second Solution. Write the inequality as
X 4 1
‹
− ≥ 1,
3 a2 − a + 1
X (2a − 1)2
≥ 3.
a2 − a + 1
Let p = a + b + c and q = ab + bc + ca. By the Cauchy-Schwarz inequality, we have
P 2
X (2a − 1)2 2 a−3 (2p − 3)2
≥ = .
a2 − a + 1 (a2 − a + 1) p2 − 2q − p + 3
P

Thus, it suffices to show that

(2p − 3)2 ≥ 3(p2 − 2q − p + 3),

which is equivalent to
p2 + 6q − 9p ≥ 0.
From the known inequality

(ab + bc + ca)2 ≥ 3abc(a + b + c),

we get q2 ≥ 3p. Using this inequality and the AM-GM inequality, we find
p Æ
p2 + 6q = p2 + 3q + 3q ≥ 3 9p2 q2 ≥ 3 9p2 (3p) = 9p.
3 3

P 1.47. Let a, b, c be positive real numbers such that abc = 1. Prove that
3+a 3+ b 3+c
+ + ≥ 3.
(1 + a)2 (1 + b)2 (1 + c)2

Solution. Using the inequality in P 1.1, we have


X 3+a X 2 X 1
= +
(1 + a)2 (1 + a)2 1+a
X• 1 1 1
˜ X
= + +
(1 + a)2 (1 + b)2 1+c
X 1 X ab
≥ + = 3.
1 + ab 1 + ab
The equality holds for a = b = c = 1.
80 Vasile Cîrtoaje

P 1.48. Let a, b, c be positive real numbers such that abc = 1. Prove that

7 − 6a 7 − 6b 7 − 6c
+ + ≥ 1.
2 + a2 2 + b2 2 + c 2
(Vasile Cîrtoaje, 2008)

Solution. Write the inequality as

7 − 6a 7 − 6b 7 − 6c
 ‹  ‹  ‹
+1 + +1 + + 1 ≥ 4,
2 + a2 2 + b2 2 + c2

(3 − a)2 (3 − b)2 (3 − c)2


+ + ≥ 4.
2 + a2 2 + b2 2 + c2
Substituting a, b, c by 1/a, 1/b, 1/c, respectively, we need to prove that abc = 1
involves
(3a − 1)2 (3b − 1)2 (3c − 1)2
+ + ≥ 4.
2a2 + 1 2b2 + 1 2c 2 + 1
By the Cauchy-Schwarz inequality, we have
P 2
9 a2 + 18 ab − 18 a + 9
P P P
X (3a − 1)2 3 a−3
≥ P = .
2a2 + 1 (2a2 + 1) 2 a2 + 3
P

Thus, it suffices to prove that


X X X € X Š
9 a2 + 18 ab − 18 a+9≥4 2 a2 + 3 ,

which is equivalent to
f (a) + f (b) + f (c) ≥ 3,
where
1
‹ 
f (x) = x + 18 2
−x .
x
We use the mixing variables technique. Without loss of generality, assume that

a = max{a, b, c}, a ≥ 1, bc ≤ 1.

Since
p 2 1
p p  ‹
f (b) + f (c) − 2 f ( bc) = (b − c) + 18( b − c)
2
− 1 ≥ 0,
bc
it suffices to show that p
f (a) + 2 f ( bc) ≥ 3,
which is equivalent to

1 p
 ‹
f (x ) + 2 f
2
≥ 3, x= a,
x
Symmetric Rational Inequalities 81

x 6 − 18x 4 + 36x 3 − 3x 2 − 36x + 20 ≥ 0,


(x − 1)2 (x − 2)2 (x + 1)(x + 5) ≥ 0.
The equality holds for a = b = c = 1, and also for a = 1/4 and b = c = 2 (or any
cyclic permutation).

P 1.49. Let a, b, c be positive real numbers such that abc = 1. Prove that

a6 b6 c6
+ + ≥ 1.
1 + 2a5 1 + 2b5 1 + 2c 5
(Vasile Cîrtoaje, 2008)

Solution. Using the substitutions


v v v
3 x
t 2 t
3 y2 t z2
a= b= c=
3
, , ,
yz zx xy

the inequality becomes


X x4
p ≥ 1.
y 2 z 2 + 2x 3 3 x yz
By the Cauchy-Schwarz inequality, we have

( x 2 )2 ( x 2 )2
P P
X x4
p ≥P p =P p P .
y 2 z 2 + 2x 3 3 x yz ( y 2 z 2 + 2x 3 3 x yz) x 2 y 2 + 2 3 x yz x 3

Therefore, we only need to show that


€X Š2 X p X
x2 ≥ x 2 y 2 + 2 3 x yz x 3.

Since, by the AM-GM inequality,


p
x + y + z ≥ 3 3 x yz,

it suffices to prove that


X X X X
3( x ) ≥3
2 2
x y + 2(
2 2
x)( x 3 );

that is, X X X
x4 + 3 x2 y2 ≥ 2 x y(x 2 + y 2 ),
X
(x − y)4 ≥ 0.
The equality holds for a = b = c = 1.
82 Vasile Cîrtoaje

P 1.50. Let a, b, c be positive real numbers such that abc = 1. Prove that

a b c 1
+ 2 + 2 ≤ .
a2 +5 b +5 c +5 2
(Vasile Cîrtoaje, 2008)

Solution. Let
a b c
F (a, b, c) = + 2 + 2 .
a2 +5 b +5 c +5
Without loss of generality, assume that a = min{a, b, c}.
Case 1: a ≤ 1/5. We have

a b c 1 1 1
F (a, b, c) < + p + p ≤ +p < .
5 2 5b 2 2 5c 2 25 5 2

Case 2: a > 1/5. Use the mixing variables method. We will show that

1
F (a, b, c) ≤ F (a, x, x) ≤ ,
2
where p p
x= bc, a = 1/x 2 , x< 5.
The left inequality, F (a, b, c) ≤ F (a, x, x), is equivalent to
p p
( b − c)2 [10x(b + c) + 10x 2 − 25 − x 4 ] ≥ 0.

This is true since

10x(b + c) + 10x 2 − 25 − x 4 ≥ 20x 2 + 10x 2 − 25x 2 − x 4 = x 2 (5 − x 2 ) > 0.

1
The right inequality, F (a, x, x) ≤ , is equivalent to
2

(x − 1)2 (5x 4 − 10x 3 − 2x 2 + 6x + 5) ≥ 0.

It is also true since

5x 4 − 10x 3 − 2x 2 + 6x + 5 = 5(x − 1)4 + 2x(5x 2 − 16x + 13)

and p
5x 2 + 13 ≥ 2 65x 2 > 16x.
The equality holds for a = b = c = 1.
Symmetric Rational Inequalities 83

P 1.51. Let a, b, c be positive real numbers such that abc = 1. Prove that
1 1 1 2
+ + + ≥ 1.
(1 + a)2 (1 + b)2 (1 + c)2 (1 + a)(1 + b)(1 + c)
(Pham Van Thuan, 2006)

First Solution. There are two of a, b, c either greater than or equal to 1, or less
than or equal to 1. Let b and c be these numbers; that is, (1 − b)(1 − c) ≥ 0. Since
1 1 1
+ ≥
(1 + b)2 (1 + c)2 1 + bc
(see P 1.1), it suffices to show that
1 1 2
+ + ≥ 1.
(1 + a)2 1 + bc (1 + a)(1 + b)(1 + c)
This inequality is equivalent to

b2 c 2 1 2bc
+ + ≥ 1,
(1 + bc) 2 1 + bc (1 + bc)(1 + b)(1 + c)
which can be written in the obvious form
bc(1 − b)(1 − c)
≥ 0.
(1 + bc)(1 + b)(1 + c)
The equality holds for a = b = c = 1.
Second Solution. Setting

a = yz/x 2 , b = z x/ y 2 , c = x y/z 2 ,

where x, y, z > 0, the inequality becomes


X x4 2x 2 y 2 z 2
+ ≥ 1.
(x 2 + yz)2 (x 2 + yz)( y 2 + z x)(z 2 + x y)
By the Cauchy-Schwarz inequality, we have
X x4 X x4 2x 2 y 2 z 2
≥ = 1 − .
(x 2 + yz)2 (x 2 + y 2 )(x 2 + z 2 ) (x 2 + y 2 )( y 2 + z 2 )(z 2 + x 2 )

Then, it suffices to show that

(x 2 + y 2 )( y 2 + z 2 )(z 2 + x 2 ) ≥ (x 2 + yz)( y 2 + z x)(z 2 + x y).

This inequality follows by multiplying the inequalities

(x 2 + y 2 )(x 2 + z 2 ) ≥ (x 2 + yz)2 ,
84 Vasile Cîrtoaje

( y 2 + z 2 )( y 2 + x 2 ) ≥ ( y 2 + z x)2 ,
(z 2 + x 2 )(z 2 + y 2 ) ≥ (z 2 + x y)2 .
Third Solution. We make the substitution
1 1+ x 1 1+ y 1 1+z
= , = , = ,
1+a 2 1+ b 2 1+c 2
which is equivalent to

1− x 1− y 1−z
a= , b= , c= ,
1+ x 1+ y 1+z

where
−1 < x, y, z < 1, x + y + z + x yz = 0.
The desired inequality becomes

(1 + x)2 + (1 + y)2 + (1 + z)2 + (1 + x)(1 + y)(1 + z) ≥ 4,

x 2 + y 2 + z 2 + (x + y + z)2 + 4(x + y + z) ≥ 0.
By virtue of the AM-GM inequality, we have

x 2 + y 2 + z 2 + (x + y + z)2 + 4(x + y + z) = x 2 + y 2 + z 2 + x 2 y 2 z 2 − 4x yz
p
≥ 4 x 4 y 4 z 4 − 4x yz = 4|x yz| − 4x yz ≥ 0.
4

P 1.52. Let a, b, c be nonnegative real numbers such that

1 1 1 3
+ + = .
a+b b+c c+a 2
Prove that
3 2 1
≥ + 2 .
a+b+c ab + bc + ca a + b2 + c 2

Solution. Write the inequality in the homogeneous form

2 1 1 1 2 1
 ‹
+ + ≥ + 2 .
a+b+c a+b b+c c+a ab + bc + ca a + b2 + c 2

Due to homogeneity, we may assume that

a + b + c = 1, 0 ≤ a, b, c < 1.
Symmetric Rational Inequalities 85

Denote q = ab + bc + ca. From the known inequality (a + b + c)2 ≥ 3(ab + bc + ca),


we get
1 − 3q ≥ 0.
Rewrite the desired inequality as follows:

1 1 1 2 1
 ‹
2 + + ≥ + ,
1−c 1−a 1− b q 1 − 2q

2(q + 1) 2 − 3q
≥ ,
q − abc q(1 − 2q)
q2 (1 − 4q) + (2 − 3q)abc ≥ 0.
By Schur’s inequality, we have

(a + b + c)3 + 9abc ≥ 4(a + b + c)(ab + bc + ca),

1 − 4q ≥ −9abc.
Then,

q2 (1 − 4q) + (2 − 3q)abc ≥ −9q2 abc + (2 − 3q)abc


= (1 − 3q)(2 + 3q)abc ≥ 0.

5
The equality holds for a = b = c = 1, and for a = 0 and b = c = (or any cyclic
3
permutation).

P 1.53. Let a, b, c be nonnegative real numbers such that

7(a2 + b2 + c 2 ) = 11(ab + bc + ca).

Prove that
51 a b c
≤ + + ≤ 2.
28 b+c c+a a+b

Solution. Due to homogeneity, we may assume that b + c = 2. Let us denote

x = bc, 0 ≤ x ≤ 1.

By the hypothesis 7(a2 + b2 + c 2 ) = 11(ab + bc + ca), we get

7a2 − 22a + 28
x= .
25
86 Vasile Cîrtoaje

Notice that the condition x ≤ 1 involves


1
≤ a ≤ 3.
7
Since
a b c a a(b + c) + (b + c)2 − 2bc
+ + = +
b+c c+a a+b b+c a2 + (b + c)a + bc
a 2(a + 2 − x) 4a3 + 27a + 11
= + 2 = ,
2 a + 2a + x 8a2 + 7a + 7
the required inequalities become

51 4a3 + 27a + 11
≤ ≤ 2.
28 8a2 + 7a + 7
We have
4a3 + 27a + 11 51 (7a − 1)(4a − 7)2
− = ≥0
8a2 + 7a + 7 28 28(8a2 + 7a + 7)
and
4a3 + 27a + 11 (3 − a)(2a − 1)2
2− = ≥ 0.
8a2 + 7a + 7 8a2 + 7a + 7

This completes the proof. The left inequality becomes an equality for 7a = b = c
a
(or any cyclic permutation), while the right inequality is an equality for = b = c
3
(or any cyclic permutation).

P 1.54. Let a, b, c be nonnegative real numbers, no two of which are zero. Prove that

1 1 1 10
+ 2 + 2 ≥ .
a2 +b 2 b +c 2 c +a 2 (a + b + c)2

Solution. Assume that a = min{a, b, c}, and denote


a a
x = b+ , y =c+ .
2 2
Since
a2 + b2 ≤ x 2 , b2 + c 2 ≤ x 2 + y 2 , c 2 + a2 ≤ y 2 ,
(a + b + c)2 = (x + y)2 ≥ 4x y,
it suffices to show that
1 1 1 5
+ 2 + 2≥ .
x 2 x +y 2 y 2x y
Symmetric Rational Inequalities 87

We have
1 1 1 5 1 1 2 1 1
 ‹  ‹
+ + − = + − + −
x 2 x 2 + y 2 y 2 2x y x2 y2 x y x 2 + y 2 2x y
(x − y)2 (x − y)2
= −
x2 y2 2x y(x 2 + y 2 )
(x − y)2 (2x 2 − x y + 2 y 2 )
= ≥ 0.
2x 2 y 2 (x 2 + y 2 )

The equality holds for a = 0 and b = c (or any cyclic permutation).

P 1.55. Let a, b, c be nonnegative real numbers, no two of which are zero. Prove that

1 1 1 3
+ 2 + 2 ≥ .
a2 − ab + b 2 b − bc + c 2 c − ca + a 2 max{ab, bc, ca}

Solution. Assume that

a = min{a, b, c}, bc = max{ab, bc, ca}.

Since
1 1 1 1 1 1
+ 2 + 2 ≥ 2+ 2 + 2,
a2 − ab + b 2 b − bc + c 2 c − ca + a 2 b b − bc + c 2 c
it suffices to show that
1 1 1 3
+ 2 + 2≥ .
b 2 b − bc + c 2 c bc
We have
1 1 1 3 (b − c)4
+ + − = ≥ 0.
b2 b2 − bc + c 2 c 2 bc b2 c 2 (b2 − bc + c 2 )
The equality holds for a = b = c, and also a = 0 and b = c (or any cyclic permuta-
tion).

P 1.56. Let a, b, c be nonnegative real numbers, no two of which are zero. Prove that

a(2a + b + c) b(2b + c + a) c(2c + a + b)


+ + ≥ 6.
b2 + c 2 c 2 + a2 a2 + b2
88 Vasile Cîrtoaje

Solution. By the Cauchy-Schwarz inequality, we have


P 2
X a(2a + b + c) a(2a + b + c)
≥P .
b2 + c 2 a(2a + b + c)(b2 + c 2 )

Thus, we still need to show that


€X X Š2 X
2 a2 + ab ≥3 a(2a + b + c)(b2 + c 2 ),

which is equivalent to
X X X X
2 a4 + 2abc a+ ab(a2 + b2 ) ≥ 6 a2 b2 .

We can obtain this inequality by adding Schur’s inequality of degree four


X X X
a + abc
4
a≥ ab(a2 + b2 )

and X X
ab(a2 + b2 ) ≥ 2 a2 b2 ,
multiplied by 2 and 3, respectively. The equality occurs for a = b = c, and for a = 0
and b = c (or any cyclic permutation).

P 1.57. Let a, b, c be nonnegative real numbers, no two of which are zero. Prove that

a2 (b + c)2 b2 (c + a)2 c 2 (a + b)2


+ 2 + 2 ≥ 2(ab + bc + ca).
b2 + c 2 c + a2 a + b2

Solution. We apply the SOS method. Since

a2 (b + c)2 ) 2a2 bc
= a 2
+ ,
b2 + c 2 b2 + c 2
we can write the inequality as

2bc
€X X Š X  ‹
2 2
2 a − ab − a 1− 2 ≥ 0,
b + c2
X X a2 (b − c)2
(b − c)2 − ≥ 0,
b2 + c 2
X a2
‹
1− 2 (b − c)2 ≥ 0.
b +c 2
Symmetric Rational Inequalities 89

c2
Without loss of generality, assume that a ≥ b ≥ c. Since 1 − > 0, it suffices
a2 + b2
to prove that

a2 b2
 ‹  ‹
1− (b − c) + 1 − 2
2
(a − c)2 ≥ 0,
b +c
2 2 c +a 2

which is equivalent to

(a2 − b2 + c 2 )(a − c)2 (a2 − b2 − c 2 )(b − c)2


≥ .
a2 + c 2 b2 + c 2
This inequality follows by multiplying the inequalities

(a − c)2 (b − c)2
a2 − b2 + c 2 ≥ a2 − b2 − c 2 , ≥ .
a2 + c 2 b2 + c 2
The latter inequality is true since

(a − c)2 (b − c)2 2bc 2ac 2c(a − b)(ab − c 2 )


− = − = ≥ 0.
a2 + c 2 b2 + c 2 b2 + c 2 a2 + c 2 (b2 + c 2 )(a2 + c 2 )

The equality occurs for a = b = c, and for a = b and c = 0 (or any cyclic permuta-
tion).

P 1.58. If a, b, c are positive real numbers, then

1 1 1
 ‹  ‹
X a a b c
3 +5 + + ≥8 + + .
b2 − bc + c 2 bc ca ab a b c

(Vasile Cîrtoaje, 2011)

Solution. In order to apply the SOS method, we multiply the inequality by abc
and write it as follows:
€X X Š X  bc
‹
2 2
8 a − bc − 3 a 1− 2 ≥ 0,
b − bc + c 2
X X a2 (b − c)2
4 (b − c)2 − 3 ≥ 0,
b2 − bc + c 2
X (b − c)2 (4b2 − 4bc + 4c 2 − 3a2 )
≥ 0.
b2 − bc + c 2
Without loss of generality, assume that a ≥ b ≥ c. Since

4a2 − 4ab + 4b2 − 3c 2 = (2a − b)2 + 3(b2 − c 2 ) ≥ 0,


90 Vasile Cîrtoaje

it suffices to prove that


(a − c)2 (4a2 − 4ac + 4c 2 − 3b2 ) (b − c)2 (3a2 − 4b2 + 4bc − 4c 2 )
≥ .
a2 − ac + c 2 b2 − bc + c 2
Notice that
4a2 − 4ac + 4c 2 − 3b2 = (a − 2c)2 + 3(a2 − b2 ) ≥ 0.
Thus, the desired inequality follows by multiplying the inequalities
4a2 − 4ac + 4c 2 − 3b2 ≥ 3a2 − 4b2 + 4bc − 4c 2
and
(a − c)2 (b − c)2
≥ 2 .
a2 − ac + c 2 b − bc + c 2
The first inequality is equivalent to
(a − 2c)2 + (b − 2c)2 ≥ 0.
Also, we have
(a − c)2 (b − c)2 bc ac
− = 2 − 2
a − ac + c
2 2 b − bc + c
2 2 b − bc + c 2 a − ac + c 2
c(a − b)(ab − c 2 )
= 2 ≥ 0.
(b − bc + c 2 )(a2 − ac + c 2 )
The equality occurs for a = b = c, and for 2a = b = c (or any cyclic permutation).

P 1.59. Let a, b, c be nonnegative real numbers, no two of which are zero. Prove that

1 1 1
 ‹
(a) 2abc + + + a2 + b2 + c 2 ≥ 2(ab + bc + ca);
a+b b+c c+a
a2 b2 c2 3(a2 + b2 + c 2 )
(b) + + ≤ .
a+b b+c c+a 2(a + b + c)

Solution. (a) First Solution. We have


X 1 X X a(2bc + ab + ac)
2abc + a2 =
b+c b+c
X ab(a + c) X ac(a + b)
= +
b+c b+c
X ab(a + c) X ba(b + c)
= +
b+ c ‹c + a X
X a+c b+c
= ab + ≥2 ab.
b+c a+c
Symmetric Rational Inequalities 91

The equality occurs for a = b = c, and for a = 0 and b = c (or any cyclic permuta-
tion).
Second Solution. Write the inequality as
X  2abc ‹
+ a − ab − ac ≥ 0.
2
b+c
We have
X  2abc ‹ X ab(a − b) + ac(a − c)
+ a − ab − ac =
2
b+c b+c
X ab(a − b) X ba(b − a)
= +
b+c c+a
X ab(a − b)2
= ≥ 0.
(b + c)(c + a)
(b) Since
X a2 X ab
‹ X ab
= a− =a+b+c− ,
a+b a+b a+b
we can write the desired inequality as
X ab 3(a2 + b2 + c 2 )
+ ≥ a + b + c.
a+b 2(a + b + c)
Multiplying by 2(a + b + c), the inequality can be written as
X a 
2 1+ bc + 3(a2 + b2 + c 2 ) ≥ 2(a + b + c)2 ,
b+c
or X 1
2abc + a2 + b2 + c 2 ≥ 2(ab + bc + ca),
b+c
which is just the inequality in (a).

P 1.60. Let a, b, c be nonnegative real numbers, no two of which are zero. Prove that

a2 − bc b2 − ca c 2 − ab 3(ab + bc + ca)
(a) + 2 + 2 + ≥ 3;
b2 + c 2 c + a2 a + b2 a2 + b2 + c 2
a2 b2 c2 ab + bc + ca 5
(b) + + + 2 ≥ ;
b +c
2 2 c +a
2 2 a +b
2 2 a + b2 + c 2 2
a2 + bc b2 + ca c 2 + ab ab + bc + ca
(c) + + ≥ + 2.
b2 + c 2 c 2 + a2 a2 + b2 a2 + b2 + c 2
(Vasile Cîrtoaje, 2014)
92 Vasile Cîrtoaje

Solution. (a) Use the SOS method. Write the inequality as follows:
X  2a2 2bc ab + bc + ca
‹ X ‹  ‹
−1 + 1− 2 −6 1− 2 ≥ 0,
b2 + c 2 b + c2 a + b2 + c 2
X 2a2 − b2 − c 2 X (b − c)2 X (b − c)2
+ −3 ≥ 0.
b2 + c 2 b2 + c 2 a2 + b2 + c 2
Since
X 2a2 − b2 − c 2 X a2 − b2 X a2 − c 2 X a2 − b2 X b2 − a2
= + = +
b2 + c 2 b2 + c 2 b2 + c 2 b2 + c 2 c 2 + a2
X (a − b )
2 2 2 X (b − c )
2 2 2
= = ,
(b2 + c 2 )(c 2 + a2 ) (a2 + b2 )(a2 + c 2 )

we can write the inequality as


X
(b − c)2 Sa ≥ 0,

where
(b + c)2 1 3
Sa = 2 + 2 − 2 .
(a + b )(a + c ) b + c
2 2 2 2 a + b2 + c 2
It suffices to show that Sa , S b , Sc ≥ 0 for all nonnegative real numbers a, b, c, no
two of which are zero. Denoting x 2 = b2 + c 2 , we have

x 2 + 2bc 1 3
Sa = + 2− 2 ,
a +a x +b c
4 2 2 2 2 x a + x2
and the inequality Sa ≥ 0 becomes

(a2 − 2x 2 )b2 c 2 + 2x 2 (a2 + x 2 )bc + (a2 + x 2 )(a2 − x 2 )2 ≥ 0.

Clearly, this is true if


−2x 2 b2 c 2 + 2x 4 bc ≥ 0.
Indeed,

−2x 2 b2 c 2 + 2x 4 bc = 2x 2 bc(x 2 − bc) = 2bc(b2 + c 2 )(b2 + c 2 − bc) ≥ 0.

The equality occurs for a = b = c, and for a = 0 and b = c (or any cyclic permuta-
tion).

(b) First Solution. We get the desired inequality by summing the inequality
in (a) and the inequality

bc ca ab 1 2(ab + bc + ca)
+ 2 + 2 + ≥ .
b2 +c 2 c +a 2 a +b 2 2 a2 + b2 + c 2
Symmetric Rational Inequalities 93

This inequality is equivalent to


X  2bc 4(ab + bc + ca)
‹
+1 ≥ + 2,
b +c
2 2 a2 + b2 + c 2
X (b + c)2
2(a + b + c)2
≥ .
b2 + c 2 a2 + b2 + c 2
By the Cauchy-Schwarz inequality, we have
P 2
X (b + c)2 (b + c) 2(a + b + c)2
≥ = .
b2 + c 2 (b2 + c 2 ) a2 + b2 + c 2
P

The equality occurs for a = b = c, and for a = 0 and b = c (or any cyclic permuta-
tion).
Second Solution. Let

p = a + b + c, q = ab + bc + ca, r = abc.

By the Cauchy-Schwarz inequality, we have


P 2 2
X a2 a (p2 − 2q)2
≥ = .
b2 + c 2 a2 (b2 + c 2 ) 2(q2 − 2pr)
P

Therefore, it suffices to show that

(p2 − 2q)2 2q
2
+ 2 ≥ 5. (*)
q − 2pr p − 2q

Consider the following cases: p2 ≥ 4q and 3q ≤ p2 < 4q.


Case 1: p2 ≥ 4q. The inequality (*) is true if

(p2 − 2q)2 2q
2
+ 2 ≥ 5,
q p − 2q

which is equivalent to the obvious inequality


 
(p2 − 4q) (p2 − q)2 − 2q2 ≥ 0.

Case 2: 3q ≤ p2 < 4q. Using Schur’s inequality of degree four

6pr ≥ (p2 − q)(4q − p2 ),

the inequality (*) is true if

3(p2 − 2q)2 2q
+ 2 ≥ 5,
3q − (p − q)(4q − p ) p − 2q
2 2 2
94 Vasile Cîrtoaje

which is equivalent to the obvious inequality

(p2 − 3q)(p2 − 4q)(2p2 − 5q) ≤ 0.

Third Solution (by Nguyen Van Quy). Write the inequality (*) from the preceding
solution as follows:
(a2 + b2 + c 2 )2 2(ab + bc + ca)
+ ≥ 5,
a2 b2 + b2 c 2 + c 2 a2 a2 + b2 + c 2
(a2 + b2 + c 2 )2 2(ab + bc + ca)
− 3 ≥ 2 − ,
a2 b2 + b2 c 2 + c 2 a2 a2 + b2 + c 2
a4 + b4 + c 4 − a2 b2 − b2 c 2 − c 2 a2 2(a2 + b2 + c 2 − ab − bc − ca)
≥ .
a2 b2 + b2 c 2 + c 2 a2 a2 + b2 + c 2
Since
X
2(a b + b c + c a ) ≤
2 2 2 2 2 2
ab(a2 + b2 ) ≤ (ab + bc + ca)(a2 + b2 + c 2 ),

it suffices to prove that

a4 + b4 + c 4 − a2 b2 − b2 c 2 − c 2 a2
≥ a2 + b2 + c 2 − ab − bc − ca,
ab + bc + ca
which is just Schur’s inequality of degree four

a4 + b4 + c 4 + abc(a + b + c) ≥ ab(a2 + b2 ) + bc(b2 + c 2 ) + ca(c 2 + a2 ).

(c) We get the desired inequality by summing the inequality in (a) and the
inequality
2bc 2ca 2ab 4(ab + bc + ca)
+ + + 1 ≥ ,
b2 + c 2 c 2 + a2 a2 + b2 a2 + b2 + c 2
which was proved at the first solution of (b). The equality occurs for a = b = c,
and for a = 0 and b = c (or any cyclic permutation).

P 1.61. Let a, b, c be nonnegative real numbers, no two of which are zero. Prove that

a2 b2 c2 (a + b + c)2
+ + ≥ .
b2 + c 2 c 2 + a2 a2 + b2 2(ab + bc + ca)

Solution. Applying the Cauchy-Schwarz inequality, we get


P 2 2
X a2 a (a2 + b2 + c 2 )2
≥ = .
b2 + c 2 a2 (b2 + c 2 ) 2(a2 b2 + b2 c 2 + c 2 a2 )
P
Symmetric Rational Inequalities 95

Therefore, it suffices to show that


(a2 + b2 + c 2 )2 (a + b + c)2
≥ ,
2(a2 b2 + b2 c 2 + c 2 a2 ) 2(ab + bc + ca)
which is equivalent to

(a2 + b2 + c 2 )2 (a + b + c)2
− 3 ≥ − 3,
a2 b2 + b2 c 2 + c 2 a2 ab + bc + ca
a4 + b4 + c 4 − a2 b2 − b2 c 2 − c 2 a2 a2 + b2 + c 2 − ab − bc − ca
≥ .
a2 b2 + b2 c 2 + c 2 a2 ab + bc + ca
Since a2 b2 + b2 c 2 + c 2 a2 ≤ (ab + bc + ca)2 , it suffices to show that

a4 + b4 + c 4 − a2 b2 − b2 c 2 − c 2 a2 ≥ (a2 + b2 + c 2 − ab − bc − ca)(ab + bc + ca),

which is just Schur’s inequality of degree four

a4 + b4 + c 4 + abc(a + b + c) ≥ ab(a2 + b2 ) + bc(b2 + c 2 ) + ca(c 2 + a2 ).

The equality holds for a = b = c, and also for a = 0 and b = c (or any cyclic
permutation).

P 1.62. Let a, b, c be nonnegative real numbers, no two of which are zero. Prove that

2ab 2bc 2ca a2 + b2 + c 2 5


+ + + ≥ .
(a + b)2 (b + c)2 (c + a)2 ab + bc + ca 2
(Vasile Cîrtoaje, 2006)
First Solution. We use the SOS method. Write the inequality as follows:

a2 + b2 + c 2 X •1 2bc
˜
−1≥ − ,
ab + bc + ca 2 (b + c)2
X (b − c)2 X (b − c)2
≥ ,
ab + bc + ca (b + c)2
(b − c)2 Sa + (c − a)2 S b + (a − b)2 Sc ≥ 0,
where
ab + bc + ca ab + bc + ca ab + bc + ca
Sa = 1 − , Sb = 1 − , Sc = 1 − .
(b + c)2 (c + a)2 (a + b)2
Without loss of generality, assume that a ≥ b ≥ c. We have Sc > 0 and
(c + a)(c + b) a − b
Sb ≥ 1 − = ≥ 0.
(c + a)2 c+a
96 Vasile Cîrtoaje

If b2 Sa + a2 S b ≥ 0, then
X a2
(b − c)2 Sa ≥ (b − c)2 Sa + (c − a)2 S b ≥ (b − c)2 Sa + 2 (b − c)2 S b
b
(b − c) (b Sa + a S b )
2 2 2
= ≥ 0.
b2
We have
 ‹2
 a 2 
b
b Sa + a S b = a + b − (ab + bc + ca)
2 2 2 2
+
b+cc+a
 ‹2  
b a 2
≥ a + b − (b + c)(c + a))
2 2
+
b+c c+a
b+c c+a
 ‹  
= a2 1 − + b2 1 −
c+a b+c
(a − b)2 (ab + bc + ca)
= ≥ 0.
(b + c)(c + a)
The equality occurs for a = b = c, and for a = b and c = 0 (or any cyclic permuta-
tion).
Second Solution. Multiplying by ab + bc + ca, the inequality becomes
X 2a2 b2 X 1 5
+ 2abc + a2 + b2 + c 2 ≥ (ab + bc + ca),
(a + b) 2 a+b 2
X 1 X1 • X 4ab ˜
2a bc + a + b + c − 2(ab + bc + ca) −
2 2 2
ab 1 − ≥ 0.
a+b 2 (a + b)2
According to the second solution of P 1.59-(a), we can write the inequality as fol-
lows:
X ab(a − b)2 X ab(a − b)2
− ≥ 0,
(b + c)(c + a) 2(a + b)2
(b − c)2 Sa + (c − a)2 S b + (a − b)2 Sc ≥ 0,
where
bc
Sa = [2(b + c)2 − (a + b)(a + c)].
b+c
Without loss of generality, assume that a ≥ b ≥ c. We have Sc > 0 and
ac ac
Sb = [2(a + c)2 − (a + b)(b + c)] ≥ [2(a + c)2 − (2a)(a + c)]
a+c a+c
2ac 2 (a + c)
= ≥ 0.
a+c
If Sa + S b ≥ 0, then
X
(b − c)2 Sa ≥ (b − c)2 Sa + (a − c)2 S b ≥ (b − c)2 (Sa + S b ) ≥ 0.
Symmetric Rational Inequalities 97

The inequality Sa + S b ≥ 0 is equivalent to

ac bc
[2(a + c)2 − (a + b)(b + c)] ≥ [(a + b)(a + c) − 2(b + c)2 ].
a+c b+c
Since
ac bc
≥ ,
a+c b+c
it suffices to show that

2(a + c)2 − (a + b)(b + c) ≥ (a + b)(a + c) − 2(b + c)2 .

This is true since is equivalent to

(a − b)2 + 2c(a + b) + 4c 2 ≥ 0.

P 1.63. Let a, b, c be nonnegative real numbers, no two of which are zero. Prove that

ab bc ca 1 ab + bc + ca
+ + + ≥ 2 .
(a + b)2 (b + c)2 (c + a)2 4 a + b2 + c 2

(Vasile Cîrtoaje, 2011)

First Solution. We use the SOS method. Write the inequality as follows:

ab + bc + ca X 1
• ˜
bc
1− 2 ≥ − ,
a + b2 + c 2 4 (b + c)2
X (b − c)2 X (b − c)2
2 ≥ ,
a2 + b2 + c 2 (b + c)2
a2 + b2 + c 2
X • ˜
(b − c) 2 −
2
≥ 0.
(b + c)2
Since
a2 + b2 + c 2 2bc − a2  a 2
2− =1+ ≥1− ,
(b + c)2 (b + c)2 b+c
it suffices to show that

(b − c)2 Sa + (c − a)2 S b + (a − b)2 Sc ≥ 0,

where ‹2
 a 2 
b  c 2
Sa = 1 − , Sb = 1 − , Sc = 1 − .
b+c c+a a+b
98 Vasile Cîrtoaje

Without loss of generality, assume that a ≥ b ≥ c. Since S b ≥ 0 and Sc > 0, if


b2 Sa + a2 S b ≥ 0, then
X a2
(b − c)2 Sa ≥ (b − c)2 Sa + (c − a)2 S b ≥ (b − c)2 Sa + 2 (b − c)2 S b
b
(b − c)2 (b2 Sa + a2 S b )
= ≥ 0.
b2
We have
ab 2 ab 2
‹  ‹
b Sa + a S b = a + b −
2 2 2 2

b+c c+a
  ‹2  •  a 2 ˜
b
= a 1−
2
+ b 1−
2
≥ 0.
b+c c+a

The equality occurs for a = b = c, and for a = b and c = 0 (or any cyclic permuta-
tion).
Second Solution. Since (a + b)2 ≤ 2(a2 + b2 ), it suffices to prove that
X ab 1 ab + bc + ca
+ ≥ 2 ,
2(a2 +b ) 4
2 a + b2 + c 2
which is equivalent to
X 2ab 4(ab + bc + ca)
+1≥ ,
a +b
2 2 a2 + b2 + c 2
X (a + b)2 4(ab + bc + ca)
≥2+ ,
a2 + b2 a2 + b2 + c 2
X (a + b)2 2(a + b + c)2
≥ 2 .
a2 + b2 a + b2 + c 2
The last inequality follows immediately by the Cauchy-Schwarz inequality

[ (a + b)]2
P
X (a + b)2
≥ P .
a2 + b2 (a2 + b2 )

Remark. The following generalization of the inequalities in P 1.62 and P 1.63


holds:
• Let a, b, c be nonnegative real numbers, no two of which are zero. If 0 ≤ k ≤ 2,
then
X 4ab a2 + b2 + c 2 ab + bc + ca
+ k ≥ 3k − 1 + 2(2 − k) .
(a + b)2 ab + bc + ca a2 + b2 + c 2
with equality for a = b = c, and for a = 0 and b = c (or any cyclic permutation).
Symmetric Rational Inequalities 99

P 1.64. Let a, b, c be nonnegative real numbers, no two of which are zero. Prove that
3ab 3bc 3ca ab + bc + ca 5
+ + ≤ 2 + .
(a + b)2 (b + c)2 (c + a)2 a + b2 + c 2 4
(Vasile Cîrtoaje, 2011)
Solution. We use the SOS method. Write the inequality as follows:
X •1 ab + bc + ca
˜
bc
3 − ≥ 1 − ,
4 (b + c)2 a2 + b2 + c 2
X (b − c)2 X (b − c)2
3 ≥ 2 ,
(b + c)2 a2 + b2 + c 2
(b − c)2 Sa + (c − a)2 S b + (a − b)2 Sc ≥ 0,
where
3(a2 + b2 + c 2 ) 3(a2 + b2 + c 2 ) 3(a2 + b2 + c 2 )
Sa = − 2, S b = − 2, S c = − 2.
(b + c)2 (c + a)2 (a + b)2
Without loss of generality, assume that a ≥ b ≥ c. Since Sa > 0 and
a2 + 3b2 + c 2 − 4ac (a − 2c)2 + 3(b2 − c 2 )
Sb = = ≥ 0,
(c + a)2 (c + a)2
if S b + Sc ≥ 0, then
X
(b − c)2 Sa ≥ (c − a)2 S b + (a − b)2 Sc ≥ (a − b)2 (S b + Sc ) ≥ 0.
Using the Cauchy-Schwarz Inequality, we have
1 1
• ˜
S b + Sc = 3(a + b + c )
2 2 2
+ −4
(c + a)2 (a + b)2
12(a2 + b2 + c 2 ) 4(a − b − c)2 + 4(b − c)2
≥ − 4 = ≥ 0.
(c + a)2 + (a + b)2 (c + a)2 + (a + b)2
a
The equality occurs for a = b = c, and for = b = c (or any cyclic permutation).
2

P 1.65. Let a, b, c be nonnegative real numbers, no two of which are zero. Prove that

a3 + abc b3 + abc c 3 + abc


(a) + + ≥ a2 + b2 + c 2 ;
b+c c+a a+b
a3 + 2abc b3 + 2abc c 3 + 2abc 1
(b) + + ≥ (a + b + c)2 ;
b+c c+a a+b 2
a3 + 3abc b3 + 3abc c 3 + 3abc
(c) + + ≥ 2(ab + bc + ca).
b+c c+a a+b
100 Vasile Cîrtoaje

Solution. (a) First Solution. Write the inequality as


X  a3 + abc ‹
2
− a ≥ 0,
b+c
X a(a − b)(a − c)
≥ 0.
b+c
Assume that a ≥ b ≥ c. Since (c − a)(c − b) ≥ 0 and
a(a − b)(a − c) b(b − c)(b − a) (a − b)2 (a2 + b2 + c 2 + ab)
+ = ≥ 0,
b+c b+c (b + c)(c + a)
the conclusion follows. The equality occurs for a = b = c, and for a = b and c = 0
(or any cyclic permutation).
(b) Taking into account the inequality in (a), it suffices to show that
abc abc abc 1
+ + + a2 + b2 + c 2 ≥ (a + b + c)2 ,
b+c c+a a+b 2
which is just the inequality (a) from P 1.59. The equality occurs for a = b = c, and
for a = b and c = 0 (or any cyclic permutation).
(c) The desired inequality follows by adding the inequality in (a) and the in-
equality (a) from P 1.59. The equality occurs for a = b = c, and for a = b and
c = 0 (or any cyclic permutation).

P 1.66. Let a, b, c be nonnegative real numbers, no two of which are zero. Prove that
a3 + 3abc b3 + 3abc c 3 + 3abc
+ + ≥ a + b + c.
(b + c)2 (c + a)2 (a + b)2
(Vasile Cîrtoaje, 2005)
Solution. We use the SOS method. We have
X a3 + 3abc X X • a3 + 3abc ˜ X 3
a − a(b2 − bc + c 2 )
− a = − a =
(b + c)2 (b + c)2 (b + c)2
X a3 (b + c) − a(b3 + c 3 ) X ab(a2 − b2 ) + ac(a2 − c 2 )
= =
(b + c)3 (b + c)3
X ab(a2 − b2 ) X ba(b2 − a2 ) X ab(a2 − b2 )[(c + a)3 − (b + c)3 ]
= + =
(b + c)3 (c + a)3 (b + c)3 (c + a)3
X ab(a + b)(a − b)2 [(c + a)2 + (c + a)(b + c) + (b + c)2 ]
= ≥ 0.
(b + c)3 (c + a)3
The equality occurs for a = b = c, and for a = 0 and b = c (or any cyclic permuta-
tion).
Symmetric Rational Inequalities 101

P 1.67. Let a, b, c be nonnegative real numbers, no two of which are zero. Prove that

a3 + 3abc b3 + 3abc c 3 + 3abc 3


(a) + + ≥ ;
(b + c)3 (c + a)3 (a + b)3 2

3a3 + 13abc 3b3 + 13abc 3c 3 + 13abc


(b) + + ≥ 6.
(b + c)3 (c + a)3 (a + b)3

(Vasile Cîrtoaje and Ji Chen, 2005)

Solution. (a) First Solution. Use the SOS method. We have


X a3 + 3abc X a(b + c)2 + a(a2 + bc − b2 − c 2 )
=
(b + c)3 (b + c)3
X a X a3 − a(b2 − bc + c 2 )
= +
b+c (b + c)3
3 X a3 (b + c) − a(b3 + c 3 )
≥ +
2 (b + c)4
3 X ab(a2 − b2 ) + ac(a2 − c 2 )
= +
2 (b + c)4
3 X ab(a2 − b2 ) X ba(b2 − a2 )
= + +
2 (b + c)4 (c + a)4
3 X ab(a + b)(a − b)[(c + a)4 − (b + c)4 ]
= + ≥ 0.
2 (b + c)4 (c + a)4

The equality occurs for a = b = c.


Second Solution. Assume that a ≥ b ≥ c. Since

a3 + 3abc b3 + 3abc c 3 + 3abc


≥ ≥
b+c c+a a+b
and
1 1 1
≥ ≥ ,
(b + c)2 (c + a)2 (a + b)2
by Chebyshev’s inequality, we get
X a3 + 3abc 1 X a3 + 3abc X
 ‹
1
≥ .
(b + c)3 3 b+c (b + c)2

Thus, it suffices to show that

a + 3abc X
X 3
1 9
‹
≥ .
b+c (b + c)2 2
102 Vasile Cîrtoaje

We can obtain this inequality by multiplying the known inequality (Iran-1996)


X 1 9

(b + c)2 4(ab + bc + ca)
and the inequality (c) from P 1.65.
(b) We have
X 3a3 + 13abc X 3a(b + c)2 + 4abc + 3a(a2 + bc − b2 − c 2 )
=
(b + c)3 (b + c)3
X 3a X 1 X a3 − a(b2 − bc + c 2 )
= + 4abc +3 .
b+c (b + c)3 (b + c)3
Since X1 3

(b + c)3 (a + b)(b + c)(c + a)
(by the AM-GM inequality) and
X a3 − a(b2 − bc + c 2 ) X a3 (b + c) − a(b3 + c 3 )
=
(b + c)3 (b + c)4
X ab(a2 − b2 ) + ac(a2 − c 2 ) X ab(a2 − b2 ) X ba(b2 − a2 )
= = +
(b + c)4 (b + c)4 (c + a)4
X ab(a + b)(a − b)[(c + a)4 − (b + c)4 ]
= ≥ 0,
(b + c)4 (c + a)4
it suffices to prove that
X 3a 12abc
+ ≥ 6.
b + c (a + b)(b + c)(c + a)
This inequality is equivalent to the third degree Schur’s inequality
X
a + b + c + 3abc ≥
3 3 3
ab(a + b).

The equality occurs for a = b = c, and for a = 0 and b = c (or any cyclic permuta-
tion).

P 1.68. Let a, b, c be nonnegative real numbers, no two of which are zero. Prove that

a3 b3 c3 3
(a) + + + ab + bc + ca ≥ (a2 + b2 + c 2 );
b+c c+a a+b 2
2a2 + bc 2b2 + ca 2c 2 + ab 9(a2 + b2 + c 2 )
(b) + + ≥ .
b+c c+a a+b 2(a + b + c)
(Vasile Cîrtoaje, 2006)
Symmetric Rational Inequalities 103

Solution. (a) We apply the SOS method. Write the inequality as


X  2a3 ‹ X
2
−a ≥ (a − b)2 .
b+c
Since
X  2a3 a (a − b) + a2 (a − c)
‹ X 2
−a =
2
b+c b+c
X a2 (a − b) X b2 (b − a) X (a − b)2 (a2 + b2 + ab + bc + ca)
= + = ,
b+c c+a (b + c)(c + a)
we can write the inequality as

(b − c)2 Sa + (c − a)2 S b + (a − b)2 Sc ≥ 0,

where

Sa = (b + c)(b2 + c 2 − a2 ), S b = (c + a)(c 2 + a2 − b2 ), Sc = (a + b)(a2 + b2 − c 2 ).

Without loss of generality, assume that a ≥ b ≥ c. Since S b ≥ 0, Sc ≥ 0 and

Sa + S b = (a + b)(a − b)2 + c 2 (a + b + 2c) ≥ 0,

we have
X
(b − c)2 Sa ≥ (b − c)2 Sa + (a − c)2 S b ≥ (b − c)2 (Sa + S b ) ≥ 0.

The equality occurs for a = b = c, and for a = b and c = 0 (or any cyclic permuta-
tion).
(b) Multiplying by a + b + c, the inequality can be written as
X a  9
1+ (2a2 + bc) ≥ (a2 + b2 + c 2 ),
b+c 2
X 2a3 + abc 5
+ ab + bc + ca ≥ (a2 + b2 + c 2 ).
b+c 2
This inequality follows using the inequality in (a) and the first inequality from P
1.59. The equality occurs for a = b = c, and for a = b and c = 0 (or any cyclic
permutation).

P 1.69. Let a, b, c be nonnegative real numbers, no two of which are zero. Prove that

a(b + c) b(c + a) c(a + b)


+ 2 + 2 ≥ 2.
b2 + bc + c 2 c + ca + a 2 a + ab + b2
104 Vasile Cîrtoaje

First Solution. Apply the SOS method. We have

a(b + c) a(b + c)(a + b + c)


•X ˜ X• ˜
(a + b + c) −2 = − 2a
b2 + bc + c 2 b2 + bc + c 2
X a(ab + ac − b2 − c 2 ) X ab(a − b) − ca(c − a)
= =
b2 + bc + c 2 b2 + bc + c 2
X ab(a − b) X ab(a − b)
= −
b2 + bc + c 2 c 2 + ca + a2
X ab(a − b)2
= (a + b + c) ≥ 0.
(b2 + bc + c 2 )(c 2 + ca + a2 )
The equality occurs for a = b = c, and for a = 0 and b = c (or any cyclic permuta-
tion).
Second Solution. By the AM-GM inequality, we have

4(b2 + bc + c 2 )(ab + bc + ca) ≤ (b2 + bc + c 2 + ab + bc + ca)2


= (b + c)2 (a + b + c)2 .

Thus,
Xa(b + c) X a(b + c)(ab + bc + ca)
=
b2 + bc + c 2 (b2 + bc + c 2 )(ab + bc + ca)
X 4a(ab + bc + ca) 4(ab + bc + ca) X a
≥ = ,
(b + c)(a + b + c)2 (a + b + c)2 b+c
and it suffices to show that
X a (a + b + c)2
≥ .
b+c 2(ab + bc + ca)

This follows immediately from the Cauchy-Schwarz inequality


X a (a + b + c)2
≥ P .
b+c a(b + c)

Third Solution. By the Cauchy-Schwarz inequality, we have


X a(b + c) (a + b + c)2
≥ X a(b2 + bc + c 2 ) .
b2 + bc + c 2
b+c
Thus, it is enough to show that
X a(b2 + bc + c 2 )
(a + b + c)2 ≥ 2 .
b+c
Symmetric Rational Inequalities 105

Since
a(b2 + bc + c 2 )
 ‹
bc abc
=a b+c− = ab + ca − ,
b+c b+c b+c
X a(b2 + bc + c 2 ) 
1 1 1
‹
= 2(ab + bc + ca) − abc + + ,
b+c b+c c+a a+b
this inequality is equivalent to

1 1 1
 ‹
2abc + + + a2 + b2 + c 2 ≥ 2(ab + bc + ca),
b+c c+a a+b

which is just the inequality (a) from P 1.59.


Fourth Solution. By direct calculation, we can write the inequality as
X X
ab(a4 + b4 ) ≥ a2 b2 (a2 + b2 ),

which is equivalent to the obvious inequality


X
ab(a − b)(a3 − b3 ) ≥ 0.

P 1.70. Let a, b, c be nonnegative real numbers, no two of which are zero. Prove that

a(b + c) b(c + a) c(a + b) Y  a − b ‹2


+ + ≥2+4 .
b2 + bc + c 2 c 2 + ca + a2 a2 + ab + b2 a+b

(Vasile Cîrtoaje, 2011)

Solution. For b = c = 1, the inequality reduces to a(a − 1)2 ≥ 0. Assume further


that
a > b > c.
As we have shown in the first solution of the preceding P 1.69,
X a(b + c) X bc(b − c)2
− 2 = .
b2 + bc + c 2 (a2 + ab + b2 )(a2 + ac + c 2 )

Therefore, it remains to show that


X bc(b − c)2 Y  a − b ‹2
≥ 4 .
(a2 + ab + b2 )(a2 + ac + c 2 ) a+b

Since
(a2 + ab + b2 )(a2 + ac + c 2 ) ≤ (a + b)2 (a + c)2 ,
106 Vasile Cîrtoaje

it suffices to show that


X bc(b − c)2 Y  a − b ‹2
≥4 ,
(a + b)2 (a + c)2 a+b
which is equivalent to
X bc(b + c)2
≥ 4.
(a − b)2 (a − c)2
We have
X bc(b + c)2 ab(a + b)2

(a − b)2 (a − c)2 (a − c)2 (b − c)2
ab(a + b)2 (a + b)2
≥ = ≥ 4.
a2 b2 ab
The equality occurs for a = b = c, and for a = b and c = 0 (or any cyclic permuta-
tion).

P 1.71. Let a, b, c be nonnegative real numbers, no two of which are zero. Prove that
ab − bc + ca bc − ca + ab ca − ab + bc 3
+ + ≥ .
b +c
2 2 c +a
2 2 a +b
2 2 2

Solution. Use the SOS method. We have


X  ab − bc + ca 1 ‹ X (b + c)(2a − b − c)
− =
b2 + c 2 2 2(b2 + c 2 )
X (b + c)(a − b) X (b + c)(a − c)
= +
2(b2 + c 2 ) 2(b2 + c 2 )
X (b + c)(a − b) X (c + a)(b − a)
= +
2(b2 + c 2 ) 2(c 2 + a2 )
X (a − b)2 (ab + bc + ca − c 2 )
= .
2(b2 + c 2 )(c 2 + a2 )
Since
ab + bc + ca − c 2 = (b − c)(c − a) + 2ab ≥ (b − c)(c − a),
it suffices to show that
X
(a2 + b2 )(a − b)2 (b − c)(c − a) ≥ 0.
This inequality is equivalent to
X
(a − b)(b − c)(c − a) (a − b)(a2 + b2 ) ≥ 0,

(a − b)2 (b − c)2 (c − a)2 ≥ 0.


The equality occurs for a = b = c, and for a = 0 and b = c (or any cyclic permuta-
tion).
Symmetric Rational Inequalities 107

P 1.72. Let a, b, c be nonnegative real numbers, no two of which are zero. If k > −2,
then X ab + (k − 1)bc + ca 3(k + 1)
≥ .
b2 + kbc + c 2 k+2
(Vasile Cîrtoaje, 2005)

First Solution. Apply the SOS method. Write the inequality as


X • ab + (k − 1)bc + ca k + 1 ˜
− ≥ 0,
b2 + kbc + c 2 k+2
X A
≥ 0,
b + kbc + c 2
2

where
A = (b + c)(2a − b − c) + k(ab + ac − b2 − c 2 ).
Since

A =(b + c)[(a − b) + (a − c)] + k[b(a − b) + c(a − c)]


= (a − b)[(k + 1)b + c] + (a − c)[(k + 1)c + b],

the inequality is equivalent to


X (a − b)[(k + 1)b + c] X (a − c)[(k + 1)c + b]
+ ≥ 0,
b2 + kbc + c 2 b2 + kbc + c 2
X (a − b)[(k + 1)b + c] X (b − a)[(k + 1)a + c]
+ ≥ 0,
b2 + kbc + c 2 c 2 + kca + a2
X
(b − c)2 R a Sa ≥ 0,
where
R a = b2 + kbc + c 2 , Sa = a(b + c − a) + (k + 1)bc.
Without loss of generality, assume that

a ≥ b ≥ c.

Case 1: k ≥ −1. Since Sa ≥ a(b + c − a), it suffices to show that


X
a(b + c − a)(b − c)2 R a ≥ 0.

We have
X
a(b + c − a)(b − c)2 R a ≥ a(b + c − a)(b − c)2 R a + b(c + a − b)(c − a)2 R b
≥ (b − c)2 [a(b + c − a)R a + b(c + a − b)R b ].

Thus, it is enough to prove that

a(b + c − a)R a + b(c + a − b)R b ≥ 0.


108 Vasile Cîrtoaje

Since b + c − a ≥ −(c + a − b), we have

a(b + c − a)R a + b(c + a − b)R b ≥ (c + a − b)(bR b − aR a )


= (c + a − b)(a − b)(ab − c 2 ) ≥ 0.

Case 2: −2 < k ≤ 1. Since

Sa = (a − b)(c − a) + (k + 2)bc ≥ (a − b)(c − a),

we have X X
(b − c)2 R a Sa ≥ (a − b)(b − c)(c − a) (b − c)R a .
From
X X
(b − c)R a = (b − c)[b2 + bc + c 2 − (1 − k)bc]
X X
= (b3 − c 3 ) − (1 − k) bc(b − c)
= (1 − k)(a − b)(b − c)(c − a),

we get
X
(a − b)(b − c)(c − a) (b − c)R a = (1 − k)(a − b)2 (b − c)2 (c − a)2 ≥ 0.

This completes the proof. The equality occurs for a = b = c, and also for a = b and
c = 0 (or any cyclic permutation).
Second Solution. Use the highest coefficient method (see P 3.76 in Volume 1). Let

p = a + b + c, q = ab + bc + ca.

Write the inequality in the form f6 (a, b, c) ≥ 0, where


X
f6 (a, b, c) =(k + 2) [a(b + c) + (k − 1)bc](a2 + kab + b2 )(a2 + kac + c 2 )
Y
− 3(k + 1) (b2 + kbc + c 2 ).

Since
a(b + c) + (k − 1)bc = (k − 2)bc + q,
(a2 + kab + b2 )(a2 + kac + c 2 ) = (p2 − 2q + kab − c 2 )(p2 − 2q + kac − b2 ),
f6 (a, b, c) has the same highest coefficient A as

(k + 2)(k − 2)P2 (a, b, c) − 3(k + 1)P4 (a, b, c),

where
X Y
P2 (a, b, c) = bc(kab − c 2 )(kac − b2 ), P4 (a, b, c) = (b2 + kbc + c 2 ).
Symmetric Rational Inequalities 109

According to Remark 2 from the proof of P 2.75 in Volume 1,

A = (k + 2)(k − 2)P2 (1, 1, 1) − 3(k + 1)(k − 1)3 = −9(k − 1)2 .

Since A ≤ 0, according to P 3.76-(a) in Volume 1, it suffices to prove the original


inequality for b = c = 1, and for a = 0.
For b = c = 1, the inequality becomes as follows:
2a + k − 1 2(ka + 1) 3(k + 1)
+ 2 ≥ ,
k+2 a + ka + 1 k+2
a−k−2 ka + 1
+ 2 ≥ 0,
k+2 a + ka + 1
a(a − 1)2
≥ 0.
(k + 2)(a2 + ka + 1)
For a = 0, the inequality becomes:
(k − 1)bc b c 3(k + 1)
+ + ≥ ,
b2+ c + kbc c
2 b k+2
k−1 3(k + 1) b c
+x≥ , x = + , x ≥ 2,
x +k k+2 c b
(x − 2)[(k + 2)x + k + k + 1]
2
≥ 0,
(k + 2)(x + k)
(b − c)2 [(k + 2)(b2 + c 2 ) + (k2 + k + 1)bc] ≥ 0.
Remark. For k = 1 and k = 0, from P 1.72, we get the inequalities in P 1.69 and
P 1.71, respectively. Besides, for k = 2, we get the well-known inequality (Iran
1996):
1 1 1 9
+ + ≥ .
(a + b)2 (b + c)2 (c + a)2 4(ab + bc + ca)

P 1.73. Let a, b, c be nonnegative real numbers, no two of which are zero. If k > −2,
then X 3bc − a(b + c) 3
≤ .
b2 + kbc + c 2 k+2
(Vasile Cîrtoaje, 2011)

Solution. Write the inequality in P 1.72 as


X• ab + (k − 1)bc + ca 3
˜
1− ≤ ,
b + kbc + c
2 2 k+2
X b2 + c 2 + bc − a(b + c) 3
≤ .
b + kbc + c
2 2 k+2
110 Vasile Cîrtoaje

Since b2 + c 2 ≥ 2bc, we get


X 3bc − a(b + c) 3
≤ ,
b2 + kbc + c2 k+2
which is just the desired inequality. The equality occurs for a = b = c.

P 1.74. Let a, b, c be nonnegative real numbers such that ab + bc + ca = 3. Prove


that
ab + 1 bc + 1 ca + 1 4
+ 2 + 2 ≥ .
a +b
2 2 b +c 2 c +a 2 3

Solution. Write the inequality in the homogeneous form E(a, b, c) ≥ 4, where

4ab + bc + ca 4bc + ca + ab 4ca + ab + bc


E(a, b, c) = + + .
a2 + b2 b2 + c 2 c 2 + a2
Without loss of generality, assume that a = min{a, b, c}. We will show that

E(a, b, c) ≥ E(0, b, c) ≥ 4.

We have
E(a, b, c) − E(0, b, c) 4b2 + c(b − a) b+c 4c 2 + b(c − a)
= + + > 0,
a b(a2 + b2 ) b2 + c 2 c(c 2 + a2 )

b 4bc c (b − c)4
E(0, b, c) − 4 = + 2 + − 4 = ≥ 0.
c b + c2 b bc(b2 + c 2 )
p
The equality holds for a = 0 and b = c = 3 (or any cyclic permutation).

P 1.75. Let a, b, c be nonnegative real numbers such that ab + bc + ca = 3. Prove


that
5ab + 1 5bc + 1 5ca + 1
+ + ≥ 2.
(a + b)2 (b + c)2 (c + a)2

Solution. Write the inequality as E(a, b, c) ≥ 6, where

16ab + bc + ca 16bc + ca + ab 16ca + ab + bc


E(a, b, c) = + + .
(a + b)2 (b + c)2 (c + a)2
Without loss of generality, assume that

a ≤ b ≤ c.
Symmetric Rational Inequalities 111

Case 1: 16b2 ≥ c(a + b). We will show that

E(a, b, c) ≥ E(0, b, c) ≥ 6.

Indeed,
E(a, b, c) − E(0, b, c) 16b2 − c(a + b) 1 16c 2 − b(a + c)
= + + > 0,
a b(a + b)2 b+c c(c + a)2
b 16bc c (b − c)4
E(0, b, c) − 6 = + + −6= ≥ 0.
c (b + c)2 b bc(b + c)2
Case 2: 16b2 < c(a + b). We have
16ab + bc + ca 16ab + 16b2 2(5b − 3a)
E(a, b, c) − 6 > − 6 > − 6 = > 0.
(a + b)2 (a + b)2 a+b
p
The equality holds for a = 0 and b = c = 3 (or any cyclic permutation).

P 1.76. Let a, b, c be nonnegative real numbers, no two of which are zero. Prove that

a2 − bc b2 − ca c 2 − ab
+ + ≥ 0.
2b2 − 3bc + 2c 2 2c 2 − 3ca + 2a2 2a2 − 3ab + 2b2
(Vasile Cîrtoaje, 2005)
Solution. The hint is applying the Cauchy-Schwarz inequality after we made the
numerators of the fractions to be nonnegative and as small as possible. Thus, we
write the inequality as
X a2 − bc
‹
+ 1 ≥ 3,
2b2 − 3bc + 2c 2
X a2 + 2(b − c)2
≥ 3.
2b2 − 3bc + 2c 2
Without loss of generality, assume that

a ≥ b ≥ c.

Using the Cauchy-Schwarz inequality gives


P 2 2
a + 2 a2 b2
P 4 P
X a2 a
≥P = P
2b2 − 3bc + 2c 2 a2 (2b2 − 3bc + 2c 2 ) 4 a2 b2 − 3abc a
P

and
X (b − c)2 [a(b − c) + b(a − c) + c(a − b)]2 4b2 (a − c)2
≥ = P .
2b2 − 3bc + 2c 2 a2 (2b2 − 3bc + 2c 2 )
P P
4 a2 b2 − 3abc a
112 Vasile Cîrtoaje

Therefore, it suffices to show that

a + 2 a2 b2 + 8b2 (a − c)2
P 4 P
P P ≥ 3.
4 a2 b2 − 3abc a

By Schur’s inequality of degree four, we have


X X X X
a4 + abc a≥ ab(a2 + b2 ) ≥ 2 a2 b2 .

Thus,it is enough to prove that

4 a2 b2 − abc a + 8b2 (a − c)2


P P
P P ≥ 3,
4 a2 b2 − 3abc a

which is equivalent to
X X
abc a + b2 (a − c)2 ≥ a2 b2 ,

ac(a − b)(b − c) ≥ 0.
The equality holds for a = b = c, and also for a = 0 and b = c (or any cyclic
permutation).

P 1.77. Let a, b, c be nonnegative real numbers, no two of which are zero. Prove that

2a2 − bc 2b2 − ca 2c 2 − ab
+ + ≥ 3.
b2 − bc + c 2 c 2 − ca + a2 a2 − ab + b2
(Vasile Cîrtoaje, 2005)

Solution. Write the inequality such that the numerators of the fractions are non-
negative and as small as possible:
X  2a2 − bc ‹
+ 1 ≥ 6,
b2 − bc + c 2
X 2a2 + (b − c)2
≥ 6.
b2 − bc + c 2
Applying the Cauchy-Schwarz inequality, we get
P P 2
X 2a2 + (b − c)2 4 2 a2 − ab
≥P .
b2 − bc + c 2 [2a2 + (b − c)2 ](b2 − bc + c 2 )

Thus, we still have to prove that


€ X X Š2 X
2 2 a2 − ab ≥ 3 [2a2 + (b − c)2 ](b2 − bc + c 2 ).
Symmetric Rational Inequalities 113

This inequality is equivalent to


X X X X
2 a4 + 2abc a+ ab(a2 + b2 ) ≥ 6 a2 b2 .

We can obtain it by summing up Schur’s inequality of degree four


X X X
a4 + abc a≥ ab(a2 + b2 )

and X X
ab(a2 + b2 ) ≥ 2 a2 b2 ,
multiplied by 2 and 3, respectively. The equality holds for a = b = c, and for a = 0
and b = c (or any cyclic permutation).

P 1.78. Let a, b, c be nonnegative real numbers, no two of which are zero. Prove that

a2 b2 c2
+ + ≥ 1.
2b2 − bc + 2c 2 2c 2 − ca + 2a2 2a2 − ab + 2b2
(Vasile Cîrtoaje, 2005)

Solution. By the Cauchy-Schwarz inequality, we have


P 2 2
X a2 a
≥P .
2b − bc + 2c
2 2 a (2b − bc + 2c 2 )
2 2

Therefore, it suffices to show that


€X Š2 X
a2 ≥ a2 (2b2 − bc + 2c 2 ),

which is equivalent to
X X X
a4 + abc a≥2 a2 b2 .

This inequality follows by adding Schur’s inequality of degree four


X X X
a + abc
4
a≥ ab(a2 + b2 )

and X X
ab(a2 + b2 ) ≥ 2 a2 b2 .
The equality holds for a = b = c, and for a = 0 and b = c (or any cyclic permuta-
tion).
114 Vasile Cîrtoaje

P 1.79. Let a, b, c be nonnegative real numbers, no two of which are zero. Prove that

1 1 1 9
+ 2 + 2 ≥ .
4b2 − bc + 4c 2 4c − ca + 4a 2 4a − ab + 4b 2 7(a + b2 + c 2 )
2

(Vasile Cîrtoaje, 2005)

Solution. Use the SOS method. Without loss of generality, assume that

a ≥ b ≥ c.

Write the inequality as


X • 7(a2 + b2 + c 2 ) ˜
− 3 ≥ 0,
4b2 − bc + 4c 2
X 7a2 − 5b2 − 5c 2 + 3bc
≥ 0,
4b2 − bc + 4c 2
X 5(2a2 − b2 − c 2 ) − 3(a2 − bc)
≥ 0.
4b2 − bc + 4c 2
Since
2a2 − b2 − c 2 = (a − b)(a + b) + (a − c)(a + c),
and
2(a2 − bc) = (a − b)(a + c) + (a − c)(a + b)
we have

10(2a2 − b2 − c 2 ) − 6(a2 − bc) =


= (a − b)[10(a + b) − 3(a + c)] + (a − c)[10(a + c) − 3(a + b)]
= (a − b)(7a + 10b − 3c) + (a − c)(7a + 10c − 3b).

Thus, we can write the desired inequality as follows:


X (a − b)(7a + 10b − 3c) X (a − c)(7a + 10c − 3b)
+ ≥ 0,
4b2 − bc + 4c 2 4b2 − bc + 4c 2
X (a − b)(7a + 10b − 3c) X (b − a)(7b + 10a − 3c)
+ ≥ 0,
4b2 − bc + 4c 2 4c 2 − ca + 4a2
X (a − b)2 (28a2 + 28b2 − 9c 2 + 68ab − 19bc − 19ca)
,
(4b2 − bc + 4c 2 )(4c 2 − ca + 4a2 )
X (a − b)2 [(b − c)(28b + 9c) + a(28a + 68b − 19c)]
,
(4b2 − bc + 4c 2 )(4c 2 − ca + 4a2 )
X
(a − b)2 R c Sc ≥ 0,
Symmetric Rational Inequalities 115

where

R a = 4b2 − bc + 4c 2 , R b = 4c 2 − ca + 4a2 , R c = 4a2 − ab + 4b2 ,

Sa = (c − a)(28c + 9a) + b(28b + 68c − 19a),


S b = (a − b)(28a + 9b) + c(28c + 68a − 19b),
Sc = (b − c)(28b + 9c) + a(28a + 68b − 19c).
Since S b ≥ 0, Sc > 0 and R c ≥ R b ≥ R a > 0, we have
X
(b − c)2 R a Sa ≥ (b − c)2 R a Sa + (a − c)2 R b S b
≥ (b − c)2 R a Sa + (b − c)2 R a S b
= (b − c)2 R a (Sa + S b ).

Thus, we only need to show that Sa + S b ≥ 0. Indeed,

Sa + S b = 19(a − b)2 + 49(a − b)c + 56c 2 ≥ 0.

The equality holds for a = b = c, and for a = b and c = 0 (or any cyclic permuta-
tion).

P 1.80. Let a, b, c be nonnegative real numbers, no two of which are zero. Prove that

2a2 + bc 2b2 + ca 2c 2 + ab 9
+ 2 + 2 ≥ .
b2 + c 2 c + a2 a + b2 2
(Vasile Cîrtoaje, 2005)

First Solution. We apply the SOS method. Since


X • 2(2a2 + bc) ˜ X 2a2 − b2 − c 2 X (b − c)2
−3 =2 −
b2 + c 2 b2 + c 2 b2 + c 2

and
X 2a2 − b2 − c 2 X a2 − b2 X a2 − c 2 X a2 − b2 X b2 − a2
= + = +
b2 + c 2 b2 + c 2
b2 + c 2 b2 + c 2 c 2 + a2

1 1 (a2 − b2 )2
X  ‹ X
= (a − b ) 2
2 2
− =
b + c 2 c 2 + a2 (b2 + c 2 )(c 2 + a2 )
X (a − b)2 (a2 + b2 )
≥ ,
(b2 + c 2 )(c 2 + a2 )
116 Vasile Cîrtoaje

we can write the inequality as


X (b − c)2 (b2 + c 2 ) X (b − c)2
2 ≥ ,
(c 2 + a2 )(a2 + b2 ) b2 + c 2
or
(b − c)2 Sa + (c − a)2 S b + (a − b)2 Sc ≥ 0,
where
Sa = 2(b2 + c 2 )2 − (c 2 + a2 )(a2 + b2 ).
Without loss of generality, assume that a ≥ b ≥ c, which involves Sa ≤ S b ≤ Sc . If

Sa + S b ≥ 0,

then
Sc ≥ S b ≥ 0,
hence

(b − c)2 Sa + (c − a)2 S b + (a − b)2 Sc ≥ (b − c)2 Sa + (a − c)2 S b


≥ (b − c)2 (Sa + S b ) ≥ 0.

We have
Sa + S b = (a2 − b2 )2 + 2c 2 (a2 + b2 + 2c 2 ) ≥ 0.
The equality holds for a = b = c, and for a = b and c = 0 (or any cyclic permuta-
tion).

Second Solution. Since


2b2 c 2
bc ≥ ,
b2 + c 2
we have
X 2a2 + bc X 2a2 + 2b2 c 2
b2 +c 2
X 1
≥ = 2(a2 b2 + b2 c 2 + c 2 a2 ) .
b2 + c 2 b2 + c 2 (b2 + c 2 )2

Therefore, it suffices to show that


X 1 9
≥ ,
(b2 +c )
2 2 4(a b + b2 c 2 + c 2 a2 )
2 2

which is just the known Iran-1996 inequality (see Remark from P 1.72).

Third Solution. We get the desired inequality by summing the inequality in P


1.60-(a), namely

2a2 − 2bc 2b2 − 2ca 2c 2 − 2ab 6(ab + bc + ca)


+ 2 + 2 + ≥ 6,
b2 + c 2 c + a2 a + b2 a2 + b2 + c 2
Symmetric Rational Inequalities 117

and the inequality


3bc 3ca 3ab 3 6(ab + bc + ca)
+ 2 + 2 + ≥ .
b2+c 2 c +a 2 a +b 2 2 a2 + b2 + c 2
This inequality is equivalent to
X  2bc 4(ab + bc + ca)
‹
+ 1 ≥ + 2,
b2 + c 2 a2 + b2 + c 2
X (b + c)2 2(a + b + c)2
≥ .
b2 + c 2 a2 + b2 + c 2
By the Cauchy-Schwarz inequality, we have
P 2
X (b + c)2 (b + c) 2(a + b + c)2
≥ = .
b2 + c 2 (b2 + c 2 ) a2 + b2 + c 2
P

P 1.81. Let a, b, c be nonnegative real numbers, no two of which are zero. Prove that

2a2 + 3bc 2b2 + 3ca 2c 2 + 3ab


+ + ≥ 5.
b2 + bc + c 2 c 2 + ca + a2 a2 + ab + b2
(Vasile Cîrtoaje, 2005)
Solution. We apply the SOS method. Write the inequality as
X • 3(2a2 + 3bc) ˜
− 5 ≥ 0,
b2 + bc + c 2
or
X 6a2 + 4bc − 5b2 − 5c 2
≥ 0.
b2 + bc + c 2
Since
2a2 − b2 − c 2 = (a − b)(a + b) + (a − c)(a + c)
and
2(a2 − bc) = (a − b)(a + c) + (a − c)(a + b),
we have
6a2 + 4bc − 5b2 − 5c 2 = 5(2a2 − b2 − c 2 ) − 4(a2 − bc)
= (a − b)[5(a + b) − 2(a + c)] + (a − c)[5(a + c) − 2(a + b)]
= (a − b)(3a + 5b − 2c) + (a − c)(3a + 5c − 2b).
Thus, we can write the desired inequality as follows:
X (a − b)(3a + 5b − 2c) X (a − c)(3a + 5c − 2b)
+ ≥ 0,
b2 + bc + c 2 b2 + bc + c 2
118 Vasile Cîrtoaje

X (a − b)(3a + 5b − 2c) X (b − a)(3b + 5a − 2c)


+ ≥ 0,
b2 + bc + c 2 c 2 + ca + a2
X (a − b)2 (3a2 + 3b2 − 4c 2 + 8ab + bc + ca)
≥ 0,
(b2 + bc + c 2 )(c 2 + ca + a2 )
(b − c)2 Sa + (c − a)2 S b + (a − b)2 Sc ≥ 0,
where
Sa = (b2 + bc + c 2 )(−4a2 + 3b2 + 3c 2 + ab + 8bc + ca),
S b = (c 2 + ca + a2 )(−4b2 + 3c 2 + 3a2 + bc + 8ca + ab),
Sc = (a2 + ab + b2 )(−4c 2 + 3a2 + 3b2 + ca + 8ab + bc).
Assume that a ≥ b ≥ c. Since Sc > 0,

S b = (c 2 + ca + a2 )[(a − b)(3a + 4b) + c(8a + b + 3c)] ≥ 0,

Sa + S b ≥ (b2 + bc + c 2 )(b − a)(3b + 4a) + (c 2 + ca + a2 )(a − b)(3a + 4b)


= (a − b)2 [3(a + b)(a + b + c) + ab − c 2 ] ≥ 0,

we have

(b − c)2 Sa + (c − a)2 S b + (a − b)2 Sc ≥ (b − c)2 Sa + (a − c)2 S b


≥ (b − c)2 (Sa + S b ) ≥ 0.

The equality holds for a = b = c, and for a = 0 and b = c (or any cyclic permuta-
tion).

P 1.82. Let a, b, c be nonnegative real numbers, no two of which are zero. Prove that

2a2 + 5bc 2b2 + 5ca 2c 2 + 5ab 21


+ + ≥ .
(b + c)2 (c + a)2 (a + b)2 4

(Vasile Cîrtoaje, 2005)

Solution. Use the SOS method.Write the inequality as follows:


X • 2a2 + 5bc 7
˜
− ≥ 0,
(b + c)2 4
X 4(a2 − b2 ) + 4(a2 − c 2 ) − 3(b − c)2
≥ 0,
(b + c)2
X b2 − c 2 X c 2 − b2 X (b − c)2
4 +4 −3 ≥ 0,
(c + a)2 (a + b)2 (b + c)2
Symmetric Rational Inequalities 119

X (b − c)2 (b + c)(2a + b + c) X (b − c)2


4 −3 ≥ 0.
(c + a)2 (a + b)2 (b + c)2
Substituting b + c = x, c + a = y and a + b = z, we can rewrite the inequality in
the form
( y − z)2 S x + (z − x)2 S y + (x − y)2 Sz ≥ 0,
where

S x = 4x 3 ( y + z) − 3 y 2 z 2 , S y = 4 y 3 (z + x) − 3z 2 x 2 , Sz = 4z 3 (x + y) − 3x 2 y 2 .

Without loss of generality, assume that

0 < x ≤ y ≤ z, z ≤ x + y,

which involves S x ≤ S y ≤ Sz . If

S x + S y ≥ 0,

then
Sz ≥ S y ≥ 0,
hence

( y − z)2 S x + (z − x)2 S y + (x − y)2 Sz ≥ ( y − z)2 S x + (z − x)2 S y


≥ ( y − z)2 (S x + S y ) ≥ 0.

We have

S x + S y = 4x y(x 2 + y 2 ) + 4(x 3 + y 3 )z − 3(x 2 + y 2 )z 2


≥ 4x y(x 2 + y 2 ) + 4(x 3 + y 3 )z − 3(x 2 + y 2 )(x + y)z
= 4x y(x 2 + y 2 ) + (x 2 − 4x y + y 2 )(x + y)z.

For the nontrivial case x 2 − 4x y + y 2 < 0, we get

S x + S y ≥ 4x y(x 2 + y 2 ) + (x 2 − 4x y + y 2 )(x + y)2


≥ 2x y(x + y)2 + (x 2 − 4x y + y 2 )(x + y)2
= (x − y)2 (x + y)2 .

The equality holds for a = b = c, and for a = 0 and b = c (or any cyclic permuta-
tion).

P 1.83. Let a, b, c be nonnegative real numbers, no two of which are zero. If k > −2,
then
X 2a2 + (2k + 1)bc 3(2k + 3)
≥ .
b2 + kbc + c 2 k+2
(Vasile Cîrtoaje, 2005)
120 Vasile Cîrtoaje

First Solution. There are two cases to consider.


Case 1: −2 < k ≤ −1/2. Write the inequality as
X • 2a2 + (2k + 1)bc 2k + 1 ˜ 6
− ≥ ,
b + kbc + c
2 2 k+2 k+2
X 2(k + 2)a2 − (2k + 1)(b − c)2
≥ 6.
b2 + kbc + c 2
Since 2(k + 2)a2 − (2k + 1)(b − c)2 ≥ 0 for −2 < k ≤ −1/2, we can apply the
Cauchy-Schwarz inequality. Thus, it suffices to show that
 2
2(k + 2) a2 − (2k + 1) (b − c)2
P P
≥ 6,
[2(k + 2)a2 − (2k + 1)(b − c)2 ](b2 + kbc + c 2 )
P

which is equivalent to each of the following inequalities

2[(1 − k) a2 + (2k + 1) ab]2


P P
≥ 3,
[2(k + 2)a2 − (2k + 1)(b − c)2 ](b2 + kbc + c 2 )
P

X X X X
2(k + 2) a4 + 2(k + 2)abc a − (2k + 1) ab(a2 + b2 ) ≥ 6 a2 b2 ,
”X X X — X
2(k + 2) a4 + abc a− ab(a2 + b2 ) + 3 ab(a − b)2 ≥ 0.
The last inequality is true since, by Schur’s inequality of degree four, we have
X X X
a + abc
4
a− ab(a2 + b2 ) ≥ 0.

Case 2: k ≥ −9/5. Use the SOS method. Without loss of generality, assume that
a ≥ b ≥ c. Write the inequality as
X • 2a2 + (2k + 1)bc 2k + 3 ˜
− ≥ 0,
b2 + kbc + c 2 k+2
X 2(k + 2)a2 − (2k + 3)(b2 + c 2 ) + 2(k + 1)bc
≥ 0,
b2 + kbc + c 2
X (2k + 3)(2a2 − b2 − c 2 ) − 2(k + 1)(a2 − bc)
≥ 0.
b2 + kbc + c 2
Since
2a2 − b2 − c 2 = (a − b)(a + b) + (a − c)(a + c)
and
2(a2 − bc) = (a − b)(a + c) + (a − c)(a + b),
we have
(2k + 3)(2a2 − b2 − c 2 ) − 2(k + 1)(a2 − bc) =
= (a − b)[(2k + 3)(a + b) − (k + 1)(a + c)] + (a − c)[(2k + 3)(a + c) − (k + 1)(a + b)]
Symmetric Rational Inequalities 121

= (a − b)[(k + 2)a + (2k + 3)b − (k + 1)c] + (a − c)[(k + 2)a + (2k + 3)c − (k + 1)b)].
Thus, we can write the desired inequality as
X (a − b)[(k + 2)a + (2k + 3)b − (k + 1)c]
+
b2 + kbc + c 2
X (a − c)[(k + 2)a + (2k + 3)c − (k + 1)b]
+ ≥ 0,
b2 + kbc + c 2
or X (a − b)[(k + 2)a + (2k + 3)b − (k + 1)c]
+
b2 + kbc + c 2
X (b − a)[(k + 2)b + (2k + 3)a − (k + 1)c]
+ ≥ 0,
c 2 + kca + a2
or
(b − c)2 R a Sa + (c − a)2 R b S b + (a − b)2 R c Sc ≥ 0,
where

R a = b2 + kbc + c 2 , R b = c 2 + kca + a2 , R c = a2 + kab + b2 ,

Sa = (k + 2)(b2 + c 2 ) − (k + 1)2 a2 + (3k + 5)bc + (k2 + k − 1)a(b + c)


= −(a − b)[(k + 1)2 a + (k + 2)b] + c[(k2 + k − 1)a + (3k + 5)b + (k + 2)c],
S b = (k + 2)(c 2 + a2 ) − (k + 1)2 b2 + (3k + 5)ca + (k2 + k − 1)b(c + a)
= (a − b)[(k + 2)a + (k + 1)2 b] + c[(3k + 5)a + (k2 + k − 1)b + (k + 2)c],
Sc = (k + 2)(a2 + b2 ) − (k + 1)2 c 2 + (3k + 5)ab + (k2 + k − 1)c(a + b)
= (k + 2)(a2 + b2 ) + (3k + 5)ab + c[(k2 + k − 1)(a + b) − (k + 1)2 c]
≥ (5k + 9)ab + c[(k2 + k − 1)(a + b) − (k + 1)2 c].
We have S b ≥ 0, since for the nontrivial case

(3k + 5)a + (k2 + k − 1)b + (k + 2)c < 0,

we get

S b ≥ (a − b)[(k + 2)a + (k + 1)2 b] + b[(3k + 5)a + (k2 + k − 1)b + (k + 2)c]

= (k + 2)(a2 − b2 ) + (k + 2)2 ab + (k + 2)bc > 0.


Also, we have Sc ≥ 0 for k ≥ −9/5, since

(5k + 9)ab + c[(k2 + k − 1)(a + b) − (k + 1)2 c] ≥


≥ (5k + 9)ac + c[(k2 + k − 1)(a + b) − (k + 1)2 c]
= (k + 2)(k + 4)ac + (k2 + k − 1)bc − (k + 1)2 c 2
≥ (2k2 + 7k + 7)bc − (k + 1)2 c 2
≥ (k + 2)(k + 3)c 2 ≥ 0.
122 Vasile Cîrtoaje

Therefore, it suffices to show that R a Sa + R b S b ≥ 0. From

bR b − aR a = (a − b)(ab − c 2 ) ≥ 0,

we get  a 
R a Sa + R b S b ≥ R a Sa + S b .
b
Thus, it suffices to show that
a
Sa + S b ≥ 0.
b
We have

bSa + aS b = (k + 2)(a + b)(a − b)2 + c f (a, b, c)


≥ 2(k + 2)b(a − b)2 + c f (a, b, c),

hence
a c
Sa + S b ≥ 2(k + 2)(a − b)2 + f (a, b, c),
b b
where

f (a, b, c) = b[(k2 + k − 1)a + (3k + 5)b] + a[(3k + 5)a + (k2 + k − 1)b]

+(k + 2)c(a + b) = (3k + 5)(a2 + b2 ) + 2(k2 + k − 1)ab + (k + 2)c(a + b).


For the nontrivial case f (a, b, c) < 0, we have
a
Sa + S b ≥ 2(k + 2)(a − b)2 + f (a, b, c)
b

≥ 2(k + 2)(a − b)2 + (3k + 5)(a2 + b2 ) + 2(k2 + k − 1)ab


= (5k + 9)(a2 + b2 ) + 2(k2 − k − 5)ab ≥ 2(k + 2)2 ab ≥ 0.
The proof is completed. The equality holds for a = b = c, and for a = b and c = 0
(or any cyclic permutation).
Second Solution. We use the highest coefficient method (see P 3.76 in Volume 1).
Let
p = a + b + c, q = ab + bc + ca.
Write the inequality as f6 (a, b, c) ≥ 0, where
X
f6 (a, b, c) = (k + 2) [2a2 + (2k + 1)bc](a2 + kab + b2 )(a2 + kac + c 2 )
Y
−3(2k + 3) (b2 + kbc + c 2 ).
Since

(a2 + kab + b2 )(a2 + kac + c 2 ) = (p2 − 2q + kab − c 2 )(p2 − 2q + kac − b2 ),


Symmetric Rational Inequalities 123

f6 (a, b, c) has the same highest coefficient A as

(k + 2)P2 (a, b, c) − 3(2k + 3)P4 (a, b, c),

where X
P2 (a, b, c) = [2a2 + (2k + 1)bc](kab − c 2 )(kac − b2 ),
Y
P4 (a, b, c) = (b2 + kbc + c 2 ).
According to Remark 2 from the proof of P 2.75 in Volume 1, we have

A = (k + 2)P2 (1, 1, 1) − 3(2k + 3)(k − 1)3 = 9(2k + 3)(k − 1)2 .

On the other hand,

f6 (a, 1, 1) = 2(k + 2)a(a2 + ka + 1)(a − 1)2 (a + k + 2) ≥ 0,


f6 (0, b, c)
= 2(k + 2)(b2 + c 2 )2 + 2(k + 2)2 bc(b2 + c 2 ) + (4k2 + 6k − 1)b2 c 2 .
(b − c)2
For −2 < k ≤ −3/2, we have A ≤ 0. According to P 3.76-(a) in Volume 1, it
suffices to show that f6 (a, 1, 1) ≥ 0 and f6 (0, b, c) ≥ 0 for all a, b, c ≥ 0. The first
condition is clearly satisfied. The second condition is satisfied for all k > −2 since

2(k + 2)(b2 + c 2 )2 + (4k2 + 6k − 1)b2 c 2 ≥ [8(k + 2) + 4k2 + 6k − 1]b2 c 2


= (4k2 + 14k + 15)b2 c 2 ≥ 0.

For k > −3/2, when A > 0, we will apply the highest coefficient cancellation
method. Consider two cases: p2 ≤ 4q and p2 > 4q.

Case 1: p2 ≤ 4q. Since


f6 (1, 1, 1) = f6 (0, 1, 1) = 0,
define the homogeneous function

P(a, b, c) = abc + B(a + b + c)3 + C(a + b + c)(ab + bc + ca)

such that P(1, 1, 1) = P(0, 1, 1) = 0; that is,


1 4
P(a, b, c) = abc + (a + b + c)3 − (a + b + c)(ab + bc + ca).
9 9
We will prove the sharper inequality g6 (a, b, c) ≥ 0, where

g6 (a, b, c) = f6 (a, b, c) − 9(2k + 3)(k − 1)2 P 2 (a, b, c).

Clearly, g6 (a, b, c) has the highest coefficient A = 0. Then, according to Remark 1


from the proof of P 3.76 in Volume 1, it suffices to prove that g6 (a, 1, 1) ≥ 0 for
0 ≤ a ≤ 4. We have
a(a − 1)2
P(a, 1, 1) = ,
9
124 Vasile Cîrtoaje

hence
a(a − 1)2 g(a)
g6 (a, 1, 1) = f6 (a, 1, 1) − 9(2k + 3)(k − 1) P (a, 1, 1) =
2 2
,
9
where
g(a) = 18(k + 2)(a2 + ka + 1)(a + k + 2) − (2k + 3)(k − 1)2 a(a − 1)2 .
Since a2 + ka + 1 ≥ (k + 2)a, it suffices to show that
18(k + 2)2 (a + k + 2) ≥ (2k + 3)(k − 1)2 (a − 1)2 .
Also, since (a − 1)2 ≤ 2a + 1, it is enough to prove that h(a) ≥ 0, where
h(a) = 18(k + 2)2 (a + k + 2) − (2k + 3)(k − 1)2 (2a + 1).
Since h(a) is a linear function, the inequality h(a) ≥ 0 is true if h(0) ≥ 0 and
h(4) ≥ 0. Setting x = 2k + 3, x > 0, we get
1
h(0) = 18(k + 2)3 − (2k + 3)(k − 1)2 = (8x 3 + 37x 2 + 2x + 9) > 0.
4
Also,
1
h(4) = 2(k + 2)2 (k + 6) − (2k + 3)(k − 1)2 = 3(7k2 + 20k + 15) > 0.
9

Case 2: p2 > 4q. We will prove the sharper inequality g6 (a, b, c) ≥ 0, where
g6 (a, b, c) = f6 (a, b, c) − 9(2k + 3)(k − 1)2 a2 b2 c 2 .
We see that g6 (a, b, c) has the highest coefficient A = 0. According to Remark 1
from the proof of P 3.76 in Volume 1, it suffices to prove that g6 (a, 1, 1) ≥ 0 for
a > 4 and g6 (0, b, c) ≥ 0 for all b, c ≥ 0. We have
g6 (a, 1, 1) = f6 (a, 1, 1) − 9(2k + 3)(k − 1)2 a2
= a[2(k + 2)(a2 + ka + 1)(a − 1)2 (a + k + 2) − 9(2k + 3)(k − 1)2 a].
Since
a2 + ka + 1 > (k + 2)a, (a − 1)2 > 9,
it suffices to show that
2(k + 2)2 (a + k + 2) ≥ (2k + 3)(k − 1)2 .
Indeed,
2(k + 2)2 (a + k + 2) − (2k + 3)(k − 1)2 > 2(k + 2)2 (k + 6) − (2k + 3)(k − 1)2
= 3(7k2 + 20k + 15) > 0.
Also,
g6 (0, b, c) = f6 (0, b, c) ≥ 0.
Symmetric Rational Inequalities 125

P 1.84. Let a, b, c be nonnegative real numbers, no two of which are zero. If k > −2,
then
X 3bc − 2a2 3
≤ .
b2 + kbc + c 2 k+2
(Vasile Cîrtoaje, 2011)

First Solution. Write the inequality as


X • 2a2 − 3bc 3
˜
6
+ ≥ ,
b + kbc + c
2 2 k+2 k+2
X 2(k + 2)a2 + 3(b − c)2
≥ 6.
b2 + kbc + c 2
Applying the Cauchy-Schwarz inequality, it suffices to show that
 2
2(k + 2) a2 + 3 (b − c)2
P P
≥ 6,
[2(k + 2)a2 + 3(b − c)2 ](b2 + kbc + c 2 )
P

which is equivalent to each of the following inequalities


 P 2
2 (k + 5) a2 − 3 ab
P
≥ 3,
[2(k + 2)a2 + 3(b − c)2 ](b2 + kbc + c 2 )
P

X X X X
2(k + 8) a4 + 2(2k + 19) a2 b2 ≥ 6(k + 2)abc a + 21 ab(a2 + b2 ),
2(k + 2) f (a, b, c) + 3g(a, b, c) ≥ 0,
where X X X
f (a, b, c) = a4 + 2 a2 b2 − 3abc a,
X X X
g(a, b, c) = 4 a4 + 10 a2 b2 − 7 ab(a2 + b2 ).
We need to show that f (a, b, c) ≥ 0 and g(a, b, c) ≥ 0. Indeed,
€X Š2 X €X Š2 X
f (a, b, c) = a2 − 3abc a≥ ab − 3abc a≥0

and
X
g(a, b, c) = [2(a4 + b4 ) + 10a2 b2 − 7ab(a2 + b2 )]
X
= (a − b)2 (2a2 − 3ab + 2b2 ) ≥ 0.

The equality occurs for a = b = c.


Second Solution. Write the inequality in P 1.83 as
X• 2a2 + (2k + 1)bc 3
˜
2− ≤ ,
b2 + kbc + c 2 k+2
126 Vasile Cîrtoaje

X 2(b2 + c 2 ) − bc − 2a2 3
≤ .
b2 + kbc + c 2 k+2
Since b + c ≥ 2bc, we get
2 2

X 3bc − 2a2 3
≤ ,
b2 + kbc + c 2 k+2
which is just the desired inequality.

P 1.85. If a, b, c are nonnegative real numbers, no two of which are zero, then
a2 + 16bc b2 + 16ca c 2 + 16ab
+ 2 + 2 ≥ 10.
b2 + c 2 c + a2 a + b2
(Vasile Cîrtoaje, 2005)
Solution. Assume that a ≤ b ≤ c and denote
a2 + 16bc b2 + 16ca c 2 + 16ab
E(a, b, c) = + 2 + 2 .
b2 + c 2 c + a2 a + b2
Consider two cases.
Case 1: 16b3 ≥ ac 2 . We will show that
E(a, b, c) ≥ E(0, b, c) ≥ 10.
We have
a2 a(16c 3 − ab2 ) a(16b3 − ac 2 )
E(a, b, c) − E(0, b, c) = + + 2 2 ≥ 0.
b2 + c 2 c 2 (c 2 + a2 ) b (a + b2 )
Also,
16bc b2 c 2
E(0, b, c) − 10 = + + − 10
b2 + c 2 c 2 b2
(b − c)4 (b2 + c 2 + 4bc)
= ≥ 0.
b2 c 2 (b2 + c 2 )
Case 2: 16b3 ≤ ac 2 . It suffices to show that
c 2 + 16ab
≥ 10.
a2 + b2
Indeed,
16b3
+ 16ab
c 2 + 16ab a
− 10 ≥ − 10
a2 + b2 a2 + b2
16b
= − 10 ≥ 16 − 10 > 0.
a
This completes the proof. The equality holds for a = 0 and b = c (or any cyclic
permutation).
Symmetric Rational Inequalities 127

P 1.86. If a, b, c are nonnegative real numbers, no two of which are zero, then

a2 + 128bc b2 + 128ca c 2 + 128ab


+ + ≥ 46.
b2 + c 2 c 2 + a2 a2 + b2
(Vasile Cîrtoaje, 2005)

Solution. Let
a ≤ b ≤ c,
a2 + 128bc b2 + 128ca c 2 + 128ab
E(a, b, c) = + + .
b2 + c 2 c 2 + a2 a2 + b2
Consider two cases.
Case 1: 128b3 ≥ ac 2 . We will show that

E(a, b, c) ≥ E(0, b, c) ≥ 46.

We have

a2 a(128c 3 − ab2 ) a(128b3 − ac 2 )


E(a, b, c) − E(0, b, c) = + + ≥ 0.
b2 + c 2 c 2 (c 2 + a2 ) b2 (a2 + b2 )

Also,

128bc b2 c 2
E(0, b, c) − 46 = 2 + + − 46
b + c 2 c 2 b2
(b2 + c 2 − 4bc)2 (b2 + c 2 + 8bc)
= ≥ 0.
b2 c 2 (b2 + c 2 )

Case 2: 128b3 ≤ ac 2 . It suffices to show that

c 2 + 128ab
≥ 46.
a2 + b2
Indeed,

128b3
+ 128ab
c 2 + 128ab a
− 46 ≥ − 46
a2 + b2 a2 + b2
128b
= − 46 ≥ 128 − 46 > 0.
a
b c
This completes the proof. The equality holds for a = 0 and + = 4 (or any cyclic
c b
permutation).
128 Vasile Cîrtoaje

P 1.87. If a, b, c are nonnegative real numbers, no two of which are zero, then

a2 + 64bc b2 + 64ca c 2 + 64ab


+ + ≥ 18.
(b + c)2 (c + a)2 (a + b)2
(Vasile Cîrtoaje, 2005)

Solution. Let
a ≤ b ≤ c,
a2 + 64bc b2 + 64ca c 2 + 64ab
E(a, b, c) = + + .
(b + c)2 (c + a)2 (a + b)2
Consider two cases.
Case 1: 64b3 ≥ c 2 (a + 2b). We will show that

E(a, b, c) ≥ E(0, b, c) ≥ 18.

We have
a2 a[64c 3 − b2 (a + 2c)] a[64b3 − c 2 (a + 2b)]
E(a, b, c) − E(0, b, c) = + +
(b + c)2 c 2 (c + a)2 b2 (a + b)2
≥ 0.

Also,

64bc b2 c 2
E(0, b, c) − 18 = + + − 18
(b + c)2 c 2 b2
(b − c)4 (b2 + c 2 + 6bc)
= ≥ 0.
b2 c 2 (b + c)2

Case 2: 64b3 ≤ c 2 (a + 2b). It suffices to show that

c 2 + 64ab
≥ 18.
(a + b)2
Indeed,

64b3
+ 64ab
c 2 + 64ab a + 2b
− 18 ≥ − 18
(a + b)2 (a + b)2
64b 64
= − 18 ≥ − 18 > 0.
a + 2b 3
This completes the proof. The equality holds for a = 0 and b = c (or any cyclic
permutation).
Symmetric Rational Inequalities 129

P 1.88. Let a, b, c be nonnegative real numbers, no two of which are zero. If k ≥ −1,
then
X a2 (b + c) + kabc
≥ a + b + c.
b2 + kbc + c 2

Solution. We apply the SOS method. Write the inequality as follows:


X • a2 (b + c) + kabc ˜
− a ≥ 0,
b2 + kbc + c 2
X a(ab + ac − b2 − c 2 )
≥ 0,
b2 + kbc + c 2
X ab(a − b) X ac(a − c)
+ ≥ 0,
b2 + kbc + c 2 b2 + kbc + c 2
X ab(a − b) X ba(b − a)
+ ≥ 0,
b2 + kbc + c 2 c 2 + kca + a2
X
ab(a2 + kab + b2 )(a + b + kc)(a − b)2 ≥ 0.
Without loss of generality, assume that

a ≥ b ≥ c.

Since a + b + kc ≥ a + b − c > 0, it suffices to show that

b(b2 + kbc + c 2 )(b + c + ka)(b − c)2 + a(c 2 + kca + a2 )(c + a + kb)(c − a)2 ≥ 0.

Since
c + a + kb ≥ c + a − b ≥ 0, c 2 + kca + a2 ≥ b2 + kbc + c 2 ,
it is enough to prove that

b(b + c + ka)(b − c)2 + a(c + a + kb)(c − a)2 ≥ 0.

We have
b(b + c + ka)(b − c)2 + a(c + a + kb)(c − a)2 ≥
≥ [b(b + c + ka) + a(c + a + kb)](b − c)2
= [a2 + b2 + 2kab + c(a + b)](b − c)2
≥ [(a − b)2 + c(a + b)](b − c)2 ≥ 0.
The equality holds for a = b = c, and for a = b and c = 0 (or any cyclic permuta-
tion).
130 Vasile Cîrtoaje

−3
P 1.89. Let a, b, c be nonnegative real numbers, no two of which are zero. If k ≥ ,
2
then
X a3 + (k + 1)abc
≥ a + b + c.
b2 + kbc + c 2
(Vasile Cîrtoaje, 2009)
Solution. Use the SOS method. Write the inequality as follows:
X • a3 + (k + 1)abc ˜ X a3 − a(b2 − bc + c 2 )
− a ≥ 0, ≥ 0,
b2 + kbc + c 2 b2 + kbc + c 2
X (b + c)a3 − a(b3 + c 3 ) X ab(a2 − b2 ) + ac(a2 − c 2 )
≥ 0, ≥ 0,
(b + c)(b2 + kbc + c 2 ) (b + c)(b2 + kbc + c 2 )
X ab(a2 − b2 ) X ba(b2 − a2 )
+ ≥ 0,
(b + c)(b2 + kbc + c 2 ) (c + a)(c 2 + kca + a2 )
X
(a2 − b2 )2 ab(a2 + kab + b2 )[a2 + b2 + ab + (k + 1)c(a + b + c)] ≥ 0,
X
(b2 − c 2 )2 bc(b2 + kbc + c 2 )Sa ≥ 0,
where
Sa = b2 + c 2 + bc + (k + 1)a(a + b + c).
Without loss of generality, assume that
a ≥ b ≥ c.
Since Sc > 0, it suffices to show that
(b2 − c 2 )2 b(b2 + kbc + c 2 )Sa + (c 2 − a2 )2 a(c 2 + kca + a2 )S b ≥ 0.
Since
(c 2 − a2 )2 ≥ (b2 − c 2 )2 , a ≥ b,
c + kca + a − (b + kbc + c ) = (a − b)(a + b + kc) ≥ 0,
2 2 2 2

1
S b = a2 + c 2 + ac + (k + 1)b(a + b + c) ≥ a2 + c 2 + ac − b(a + b + c)
2
(a − b)(2a + b) + c(2a + 2c − b)
= ≥ 0,
2
it is enough to show that Sa + S b ≥ 0. Indeed,
Sa + S b = a2 + b2 + 2c 2 + c(a + b) + (k + 1)(a + b)(a + b + c)
1
≥ a2 + b2 + 2c 2 + c(a + b) − (a + b)(a + b + c)
2
(a − b) + c(a + b + 4c)
2
= ≥ 0.
2
This completes the proof. The equality holds for a = b = c, and also for a = b and
c = 0 (or any cyclic permutation).
Symmetric Rational Inequalities 131

P 1.90. Let a, b, c be nonnegative real numbers, no two of which are zero. If k > 0,
then
2a k − b k − c k 2b k − c k − a k 2c k − a k − b k
+ 2 + 2 ≥ 0.
b2 − bc + c 2 c − ca + a2 a − ab + b2
(Vasile Cîrtoaje, 2004)

Solution. Let
X = bk − c k , Y = c k − ak , Z = ak − bk ,

A = b2 − bc + c 2 , B = c 2 − ca + a2 , C = a2 − ab + b2 .

Without loss of generality, assume that a ≥ b ≥ c. This involves

A ≤ B, A ≤ C, X ≥ 0, Z ≥ 0.

Since
X 2a k − b k − c k X + 2Z X − Z 2X + Z
= + −
b2 − bc + c2 A B‹ C
1 1 2 2 1 1
‹
=X + − +Z − − ,
A B C A B C

it suffices to prove that


1 1 2
+ − ≥ 0.
A B C
Write this inequality as
1 1 1 1
− ≥ − ,
A C C B
that is,

(a − c)(a + c − b)(a2 − ac + c 2 ) ≥ (b − c)(a − b − c)(b2 − bc + c 2 ).

For the nontrivial case a > b + c, this inequality follows from

a − c ≥ b − c,

a + c − b ≥ a − b − c,

a2 − ac + c 2 > b2 − bc + c 2 .

This completes the proof. The equality holds for a = b = c, and for a = b and c = 0
(or any cyclic permutation).
132 Vasile Cîrtoaje

P 1.91. If a, b, c are the lengths of the sides of a triangle, then

b+c−a c+a−b a+b−c 2(a + b + c)


(a) + 2 + 2 ≥ 2 ;
b2− bc + c 2 c − ca + a 2 a − ab + b 2 a + b2 + c 2

2bc − a2 2ca − b2 2ab − c 2


(b) + + ≥ 0.
b2 − bc + c 2 c 2 − ca + a2 a2 − ab + b2
(Vasile Cîrtoaje, 2009)

Solution. (a) By the Cauchy-Schwarz inequality, we get


P 2
X b+c−a (b + c − a)
≥P
b2 − bc + c 2 (b + c − a)(b2 − bc + c 2 )
P 2
a
= P .
2 a3 − a2 (b + c) + 3abc
P

On the other hand, from

(b + c − a)(c + a − b)(a + b − c) ≥ 0,

we get X X
2abc ≤ a2 (b + c) − a3 ,
hence
P  P 2
a3 + a2 (b + c)
P P
X X a a
2 a3 − a2 (b + c) + 3abc ≤ = .
2 2
Therefore, P
X b+c−a 2 a
≥ P .
b2 − bc + c 2 a2
The equality holds for a degenerate triangle with a = b + c (or any cyclic permuta-
tion).
(b) Since
2bc − a2 (b − c)2 + (b + c)2 − a2
= − 2,
b2 − bc + c 2 b2 − bc + c 2
we can write the inequality as
X (b − c)2 X b+c−a
+ (a + b + c) ≥ 6.
b2 − bc + c 2 b2 − bc + c 2
Using the inequality in (a), it suffices to prove that
X (b − c)2 2(a + b + c)2
+ ≥ 6.
b2 − bc + c 2 a2 + b2 + c 2
Symmetric Rational Inequalities 133

Write this inequality as


X (b − c)2 X 2(b − c)2
≥ ,
b2 − bc + c 2 a2 + b2 + c 2
X (b − c)2 (a − b + c)(a + b − c)
≥ 0.
b2 − bc + c 2
Clearly, the last inequality is true. The equality holds for degenerate triangles with
either a/2 = b = c (or any cyclic permutation), or a = 0 and b = c (or any cyclic
permutation).
Remark. The following generalization of the inequality in (b) holds (Vasile Cîrtoaje,
2009):
• Let a, b, c be the lengths of the sides of a triangle. If k ≥ −1, then
X 2(k + 2)bc − a2
≥ 0.
b2 + kbc + c 2
with equality for a = 0 and b = c (or any cyclic permutation).

P 1.92. If a, b, c are nonnegative real numbers, then

a2 b2 c2 1
(a) + + ≤ ;
5a2 + (b + c)2 5b2 + (c + a)2 5c 2 + (a + b)2 3
a3 b3 c3 1
(b) + + ≤ .
13a + (b + c)
3 3 13b + (c + a)
3 3 13c + (a + b)
3 3 7
(Vo Quoc Ba Can and Vasile Cîrtoaje, 2009)

Solution. (a) Apply the Cauchy-Schwarz inequality in the following manner

9 (1 + 2)2 1 2
= ≤ 2 + 2 .
5a + (b + c)
2 2 (a + b + c ) + 2(2a + bc) a + b + c
2 2 2 2 2 2 2a + bc
Then,
X 9a2 X a2 X 2a2 X a2
≤ + = 1 + 2 ,
5a2 + (b + c)2 a2 + b2 + c 2 2a2 + bc 2a2 + bc
and it remains to show that
X a2
≤ 1.
2a2 + bc
For the nontrivial case a, b, c > 0, this is equivalent to
X 1
≤ 1,
2 + bc/a2
134 Vasile Cîrtoaje

which follows immediately from P 1.2-(b). The equality holds for a = b = c, and
for a = 0 and b = c (or any cyclic permutation).
(b) By the Cauchy-Schwarz inequality, we have

49 (1 + 6)2
=
13a3 + (b + c)3 (a3 + b3 + c 3 ) + 12a3 + 3bc(b + c))
1 36
≤ 3 + ,
a + b3 + c 3 12a3 + 3bc(b + c)
hence
X 49a3 X a3 X 36a3
≤ +
13a3 + (b + c)3 a3 + b3 + c 3 12a3 + 3bc(b + c)
X 12a3
=1+ .
4a3 + bc(b + c)
Thus, it suffices to show that
X 2a3
≤ 1.
4a3 + bc(b + c)
For the nontrivial case a, b, c > 0, this is equivalent to
X 1
≤ 1.
2 + bc(b + c)/(2a3 )
Since Y Y p
bc(b + c)/(2a ) ≥
3
bc bc/a3 = 1,
the inequality follows immediately from P 1.2-(b). The equality holds for a = b = c,
and for a = 0 and b = c (or any cyclic permutation).

P 1.93. If a, b, c are nonnegative real numbers, then

b2 + c 2 − a2 c 2 + a2 − b2 a2 + b2 − c 2 1
+ + ≥ .
2a2 + (b + c)2 2b2 + (c + a)2 2c 2 + (a + b)2 2
(Vasile Cîrtoaje, 2011)

Solution. We apply the SOS method. Write the inequality as follows:


X • b2 + c 2 − a2 1
˜
− ≥ 0,
2a2 + (b + c)2 6
X 5(b2 + c 2 − 2a2 ) + 2(a2 − bc)
≥ 0,
2a2 + (b + c)2
Symmetric Rational Inequalities 135

X 5(b2 − a2 ) + 5(c 2 − a2 ) + (a − b)(a + c) + (a − c)(a + b)


≥ 0,
2a2 + (b + c)2
X (b − a)[5(b + a) − (a + c)] X (c − a)[5(c + a) − (a + b)]
+ ≥ 0,
2a2 + (b + c)2 2a2 + (b + c)2
X (b − a)[5(b + a) − (a + c)] X (a − b)[5(a + b) − (b + c)]
+ ≥ 0,
2a2 + (b + c)2 2b2 + (c + a)2
X
(a − b)2 [2c 2 + (a + b)2 ][2(a2 + b2 ) + c 2 + 3ab − 3c(a + b)] ≥ 0,
X
(b − c)2 R a Sa ≥ 0,
where

R a = 2a2 + (b + c)2 , Sa = a2 + 2(b2 + c 2 ) + 3bc − 3a(b + c).

Without loss of generality, assume that a ≥ b ≥ c. We have

S b = b2 + 2(c 2 + a2 ) + 3ca − 3b(c + a) = (a − b)(2a − b) + 2c 2 + 3c(a − b) ≥ 0,

Sc = c 2 + 2(a2 + b2 ) + 3ab − 3c(a + b) ≥ 7ab − 3c(a + b) ≥ 3a(b − c) + 3b(a − c) ≥ 0,


Sa + S b = 3(a − b)2 + 4c 2 ≥ 0.
Since X
(b − c)2 R a Sa ≥ (b − c)2 R a Sa + (c − a)2 R b S b

= (b − c)2 R a (Sa + S b ) + [(c − a)2 R b − (b − c)2 R a ]S b ,


it suffices to prove that
(a − c)2 R b ≥ (b − c)2 R a .
We can get this by multiplying the inequalities

b2 (a − c)2 ≥ a2 (b − c)2

and
a2 R b ≥ b2 R a .
The equality holds for a = b = c, and for a = b and c = 0 (or any cyclic permuta-
tion).

P 1.94. Let a, b, c be positive real numbers. If k > 0, then

3a2 − 2bc 3b2 − 2ca 3c 2 − 2ab 3


+ + ≤ .
ka + (b − c)
2 2 kb + (c − a)
2 2 kc + (a − b)
2 2 k

(Vasile Cîrtoaje, 2011)


136 Vasile Cîrtoaje

Solution. Use the SOS method. Write the inequality as follows:


X•1 3a2 − 2bc
˜
− ≥ 0,
k ka2 + (b − c)2
X b2 + c 2 − 2a2 + 2(k − 1)(bc − a2 )
≥ 0;
ka2 + (b − c)2
X (b2 − a2 ) + (c 2 − a2 ) + (k − 1)[(a + b)(c − a) + (a + c)(b − a)]
≥ 0;
ka2 + (b − c)2
X (b − a)[b + a + (k − 1)(a + c)] X (c − a)[c + a + (k − 1)(a + b)]
+ ≥ 0;
ka2 + (b − c)2 ka2 + (b − c)2
X (b − a)[b + a + (k − 1)(a + c)] X (a − b)[a + b + (k − 1)(b + c)]
+ ≥ 0;
ka2 + (b − c)2 kb2 + (c − a)2
X
(a − b)2 [kc 2 + (a − b)2 ][(k − 1)c 2 + 2c(a + b) + (k2 − 1)(ab + bc + ca)] ≥ 0.
For k ≥ 1, the inequality is clearly true. Consider further that 0 < k < 1. Since

(k − 1)c 2 + 2c(a + b) + (k2 − 1)(ab + bc + ca) >

> −c 2 + 2c(a + b) − (ab + bc + ca) = (b − c)(c − a),


it suffices to prove that
X
(a − b)(b − c)(c − a) (a − b)[kc 2 + (a − b)2 ] ≥ 0.

Since
X X X
(a − b)[kc 2 + (a − b)2 ] = k (a − b)c 2 + (a − b)3
= (3 − k)(a − b)(b − c)(c − a),

we have X
(a − b)(b − c)(c − a) (a − b)[kc 2 + (a − b)2 ] =
= (3 − k)(a − b)2 (b − c)2 (c − a)2 ≥ 0.
This completes the proof. The equality holds for a = b = c.

p Let a, b, c be nonnegative real numbers, no two of which are zero. If k ≥


P 1.95.
3 + 7, then
a b c 9
(a) + + ≥ ;
a2 + kbc b2 + kca c 2 + kab (1 + k)(a + b + c)
1 1 1 9
(b) + 2 + 2 ≥ .
ka2 + bc kb + ca kc + ab (k + 1)(ab + bc + ca)
(Vasile Cîrtoaje, 2005)
Symmetric Rational Inequalities 137

Solution. (a) Assume that a = max{a, b, c}. Setting


b+c
t= , t ≤ a,
2
by the Cauchy-Schwarz inequality, we get

b c (b + c)2 4t 2
+ ≥ =
b2 + kca c 2 + kab b(b2 + kca) + c(c 2 + kab) 8t 3 − 6bc t + 2kabc
2t 2 2t 2 2
= 3 ≥ 3 = .
4t + (ka − 3t)bc 4t + (ka − 3t)t 2 t + ka
On the other hand,
a a
≥ .
a2 + kbc a2 + kt 2
Therefore, it suffices to prove that
a 2 9
+ ≥ ,
a2 + kt 2 t + ka (k + 1)(a + 2t)
which is equivalent to

(a − t)2 [(k2 − 6k + 2)a + k(4k − 5)t] ≥ 0.

This inequality is true, since k2 − 6k + 2 ≥ 0 and 4k − 5 > 0. The equality holds for
a = b = c.
(b) For a = 0, the inequality becomes
1 1 k(8 − k)
+ ≥ .
b2 c 2 (k + 1)bc
We have
1 1 k(8 − k) 2 k(8 − k) k2 − 6k + 2
+ − ≥ − = ≥ 0.
b2 c 2 (k + 1)bc bc (k + 1)bc (k + 1)bc
For a, b, c > 0, the desired inequality follows from the inequality in (a) by substi-
1/a, 1/b, 1/c, respectively. The equality holds for a = b = c. In
tuting a, b, c with p
the case k = 3 + 7, the equality also holds for a = 0 and b = c (or any cyclic
permutation).

P 1.96. Let a, b, c be nonnegative real numbers, no two of which are zero. Prove that
1 1 1 6
+ 2 + 2 ≥ 2 .
2a2 + bc 2b + ca 2c + ab a + b + c + ab + bc + ca
2 2

(Vasile Cîrtoaje, 2005)


138 Vasile Cîrtoaje

Solution. Applying the Cauchy-Schwarz inequality, we have


P 2
X 1 (b + c) 4(a + b + c)2
≥ = .
2a2 + bc (b + c)2 (2a2 + bc) (b + c)2 (2a2 + bc)
P P

Thus, it suffices to show that


X
2(a + b + c)2 (a2 + b2 + c 2 + ab + bc + ca) ≥ 3 (b + c)2 (2a2 + bc),
which is equivalent to
X X X X
2 a4 + 3 ab(a2 + b2 ) + 2abc a ≥ 10 a2 b2 .
This follows by adding Schur’s inequality
X X X
2 a4 + 2abc a≥2 ab(a2 + b2 )
to the inequality X X
5 ab(a2 + b2 ) ≥ 10 a2 b2 .
The equality holds for a = b = c, and also for a = 0 and b = c (or any cyclic
permutation).

P 1.97. Let a, b, c be nonnegative real numbers, no two of which are zero. Prove that
1 1 1 1
+ + ≥ .
22a2 + 5bc 22b2 + 5ca 22c 2 + 5ab (a + b + c)2
(Vasile Cîrtoaje, 2005)
Solution. Applying the Cauchy-Schwarz inequality, we have
P 2
X 1 (b + c) 4(a + b + c)2
≥ = .
22a2 + 5bc (b + c)2 (22a2 + 5bc) (b + c)2 (22a2 + 5bc)
P P

Thus, it suffices to show that


X
4(a + b + c)4 ≥ (b + c)2 (22a2 + 5bc),
which is equivalent to
X X X X
4 a4 + 11 a b(a2 + b2 ) + 4abc a ≥ 30 a2 b2 .
This follows by adding Schur’s inequality
X X X
4 a4 + 4abc a≥4 ab(a2 + b2 )
to the inequality X X
15 ab(a2 + b2 ) ≥ 30 a2 b2 .
The equality holds for a = b = c.
Symmetric Rational Inequalities 139

P 1.98. Let a, b, c be nonnegative real numbers, no two of which are zero. Prove that
1 1 1 8
+ 2 + 2 ≥ .
2a2 + bc 2b + ca 2c + ab (a + b + c)2
(Vasile Cîrtoaje, 2005)
First Solution. Applying the Cauchy-Schwarz inequality, we have
P 2
X 1 (b + c) 4(a + b + c)2
≥ = .
2a2 + bc (b + c)2 (2a2 + bc) (b + c)2 (2a2 + bc)
P P

Thus, it suffices to show that


X
(a + b + c)4 ≥ 2 (b + c)2 (2a2 + bc),

which is equivalent to
X X X X
a4 + 2 ab(a2 + b2 ) + 4abc a≥6 a2 b2 .

We will prove the sharper inequality


X X X X
a4 + 2 ab(a2 + b2 ) + abc a≥6 a2 b2 .

This follows by adding Schur’s inequality


X X X
a4 + abc a≥ ab(a2 + b2 )

to the inequality X X
3 ab(a2 + b2 ) ≥ 6 a2 b2 .
The equality holds for a = 0 and b = c (or any cyclic permutation).
Second Solution. Without loss of generality, we may assume that a ≥ b ≥ c. Since
the equality holds for c = 0 and a = b, when
1 1 1
= 2 = 2 ,
2a2 + bc 2b + ca 4c + 2ab
write the inequality as
1 1 1 1 8
+ + + ≥ ,
2a2 + bc 2b2 + ca 4c 2 + 2ab 4c 2 + 2ab (a + b + c)2
then apply the Cauchy-Schwarz inequality. Thus, it suffices to prove that
16 8
≥ ,
(2a2 + bc) + (2b2 + ca) + (4c + 2ab) + (4c + 2ab) (a + b + c)2
2 2

which is equivalent to the obvious inequality

c(a + b − 2c) ≥ 0.
140 Vasile Cîrtoaje

P 1.99. Let a, b, c be nonnegative real numbers, no two of which are zero. Prove that

1 1 1 12
+ 2 + 2 ≥ .
a2 + bc b + ca c + ab (a + b + c)2

(Vasile Cîrtoaje, 2005)

Solution. Write the inequality such that the numerators of the fractions are non-
negative and as small as possible:
X• 1 1
˜
9
− ≥ ,
a + bc (a + b + c)
2 2 (a + b + c)2
X (a + b + c)2 − a2 − bc
≥ 9.
a2 + bc
Assuming that a + b + c = 1, the inequality becomes
X 1 − a2 − bc
≥ 9.
a2 + bc
By the Cauchy-Schwarz inequality, we have
P 2
X 1 − a2 − bc (1 − a2 − bc)
≥P .
a2 + bc (1 − a2 − bc)(a2 + bc)

Then, it suffices to prove that


€ X X Š2 X X
3− a2 − bc ≥9 (a2 + bc) − 9 (a2 + bc)2 ,

which is equivalent to

(1 − 4q)(4 − 7q) + 36abc ≥ 0, q = ab + bc + ca.

For q ≤ 1/4, this inequality is clearly true. Consider further that q > 1/4. By
Schur’s inequality of degree three

(a + b + c)3 + 9abc ≥ 4(a + b + c)(ab + bc + ca),

we get 1 + 9abc ≥ 4q, and hence 36abc ≥ 16q − 4. Thus,

(1 − 4q)(4 − 7q) + 36abc ≥ (1 − 4q)(4 − 7q) + 16q − 4 = 7q(4q − 1) > 0.

The equality holds for a = 0 and b = c (or any cyclic permutation).


Symmetric Rational Inequalities 141

P 1.100. Let a, b, c be nonnegative real numbers, no two of which are zero. Prove
that
1 1 1 1 2
(a) + 2 + 2 ≥ 2 + ;
a2 + 2bc b + 2ca c + 2ab a +b +c
2 2 ab + bc + ca
a(b + c) b(c + a) c(a + b) ab + bc + ca
(b) + 2 + 2 ≥1+ 2 .
a + 2bc b + 2ca c + 2ab
2 a + b2 + c 2
(Darij Grinberg and Vasile Cîrtoaje, 2005)

Solution. (a) Write the inequality as

(b + 2ca)(c 2 + 2ab)
P 2
ab + bc + ca + 2a2 + 2b2 + 2c 2
≥ .
(a2 + 2bc)(b2 + 2ca)(c 2 + 2ab) (a2 + b2 + c 2 )(ab + bc + ca)
Since
X
(b2 + 2ca)(c 2 + 2ab) = (ab + bc + ca)(ab + bc + ca + 2a2 + 2b2 + 2c 2 ),

it suffices to show that

(a2 + b2 + c 2 )(ab + bc + ca)2 ≥ (a2 + 2bc)(b2 + 2ca)(c 2 + 2ab),

which is just the inequality (a) in P 2.16 in Volume 1. The equality holds for a = b,
or b = c, or c = a.
(b) Write the inequality in (a) as
X ab + bc + ca ab + bc + ca
≥2+ ,
a2 + 2bc a2 + b2 + c 2
or X a(b + c) X bc ab + bc + ca
+ ≥2+ 2 .
a + 2bc
2 a + 2bc
2 a + b2 + c 2
The desired inequality follows by adding this inequality to
X bc
1≥ .
a2 + 2bc
The last inequality is equivalent to
X a2
≥ 1,
a2 + 2bc
which follows by applying the AM-GM inequality as follows:
X a2 X a2
≥ = 1.
a2 + 2bc a2 + b2 + c 2
The equality holds for a = b = c.
142 Vasile Cîrtoaje

P 1.101. Let a, b, c be nonnegative real numbers, no two of which are zero. Prove
that
a b c a+b+c
(a) + 2 + 2 ≤ ;
a2 + 2bc b + 2ca c + 2ab ab + bc + ca
a(b + c) b(c + a) c(a + b) a2 + b2 + c 2
(b) + + ≤ 1 + .
a2 + 2bc b2 + 2ca c 2 + 2ab ab + bc + ca
(Vasile Cîrtoaje, 2008)
Solution. (a) Use the SOS method. Write the inequality as
X  ab + bc + ca
‹
a 1− ≥ 0,
a2 + 2bc
X a(a − b)(a − c)
≥ 0.
a2 + 2bc
Assume that a ≥ b ≥ c. Since (c − a)(c − b) ≥ 0, it suffices to show that
a(a − b)(a − c) b(b − a)(b − c)
+ ≥ 0.
a2 + 2bc b2 + 2ca
This inequality is equivalent to

c(a − b)2 [2a(a − c) + 2b(b − c) + 3ab] ≥ 0,

which is clearly true. The equality holds for a = b = c, and for a = b and c = 0 (or
any cyclic permutation).
(b) Since
a(b + c) a(a + b + c) a2
= − ,
a2 + 2bc a2 + 2bc a2 + 2bc
we can write the inequality as
X a a2 + b2 + c 2 X a2
(a + b + c) ≤ 1 + + .
a2 + 2bc ab + bc + ca a2 + 2bc
According to the inequality in (a), it suffices to show that
(a + b + c)2 a2 + b2 + c 2 X a2
≤1+ + ,
ab + bc + ca ab + bc + ca a2 + 2bc
which is equivalent to
X a2
≥ 1.
a2 + 2bc
Indeed,
X a2 X a2
≥ = 1.
a2 + 2bc a2 + b2 + c 2
The equality holds for a = b = c.
Symmetric Rational Inequalities 143

P 1.102. Let a, b, c be nonnegative real numbers, no two of which are zero. Prove
that
a b c a+b+c
(a) + 2 + 2 ≥ 2 ;
2a2 + bc 2b + ca 2c + ab a + b2 + c 2
b+c c+a a+b 6
(b) + 2 + 2 ≥ .
2a + bc 2b + ca 2c + ab
2 a+b+c
(Vasile Cîrtoaje, 2008)
Solution. Assume that
a ≥ b ≥ c.

(a) Multiplying by a + b + c, we can write the inequality as follows:


X a(a + b + c) (a + b + c)2
≥ 2 ,
2a2 + bc a + b2 + c 2
(a + b + c)2 X a(a + b + c)
• ˜
3− 2 ≥ 1− ,
a + b2 + c 2 2a2 + bc
X X (a − b)(a − c)
2 (a − b)(a − c) ≥ (a2 + b2 + c 2 ) ,
2a2 + bc
X 3a2 − (b − c)2
(a − b)(a − c) ≥ 0,
2a2 + bc
3 f (a, b, c) + (a − b)(b − c)(c − a)g(a, b, c) ≥ 0,
where
X a2 (a − b)(a − c) b−c
X
f (a, b, c) = , g(a, b, c) = .
2a2+ bc 2a2 + bc
It suffices to show that f (a, b, c) ≥ 0 and g(a, b, c) ≤ 0. We have
a2 (a − b)(a − c) b2 (b − a)(b − c)
f (a, b, c) ≥ +
2a2 + bc 2b2 + ca
a (a − b)(b − c) b (b − a)(b − c)
2 2
≥ +
2a2 + bc 2b2 + ca
a c(a − b) (b − c)(a + ab + b2 )
2 2 2
= ≥ 0.
(2a2 + bc)(2b2 + ca)
Also,
b−c (a − b) + (b − c) a−b
g(a, b, c) = − + 2
2a + bc
2 2b + ca
2
‹ 2c + ab 
1 1 1 1
 ‹
= (a − b) − + (b − c) −
2c 2 + ab 2b2 + ca 2a2 + bc 2b2 + ca
(a − b)(b − c) 2b + 2c − a 2b + 2a − c
• ˜
= − =
2b2 + ca 2c 2 + ab 2a2 + bc
2(a − b)(b − c)(c − a)(a2 + b2 + c 2 − ab − bc − ca)
= ≤ 0.
(2a2 + bc)(2b2 + ca)(2c 2 + ab)
144 Vasile Cîrtoaje

The equality holds for a = b = c, and for a = b and c = 0 (or any cyclic permuta-
tion).
(b) We apply the SOS method. Write the inequality as follows:
X • (b + c)(a + b + c) ˜
− 2 ≥ 0,
2a2 + bc
X (b2 + a b − 2a2 ) + (c 2 + ca − 2a2 )
≥ 0,
2a2 + bc
X (b − a)(b + 2a) + (c − a)(c + 2a)
≥ 0,
2a2 + bc
X (b − a)(b + 2a) X (a − b)(a + 2b)
+ ≥ 0,
2a2 + bc 2b2 + ca
a + 2b b + 2a
X  ‹
(a − b) − ≥ 0,
2b2 + ca 2a2 + bc
X
(a − b)2 (2c 2 + ab)(a2 + b2 + 3ab − ac − bc) ≥ 0.
It suffices to show that
X
(a − b)2 (2c 2 + ab)(a2 + b2 + 2ab − ac − bc) ≥ 0,

which is equivalent to
X
(a − b)2 (2c 2 + ab)(a + b)(a + b − c) ≥ 0.

This inequality is true if

(b − c)2 (2a2 + bc)(b + c)(b + c − a) + (c − a)2 (2b2 + ca)(c + a)(c + a − b) ≥ 0;

that is,

(a − c)2 (2b2 + ca)(a + c)(a + c − b) ≥ (b − c)2 (2a2 + bc)(b + c)(a − b − c).

Since
a + c ≥ b + c, a + c − b ≥ a − b − c,
it is enough to prove that

(a − c)2 (2b2 + ca) ≥ (b − c)2 (2a2 + bc).

We can obtain this inequality by multiplying the inequalities

b2 (a − c)2 ≥ a2 (b − c)2

and
a2 (2b2 + ca) ≥ b2 (2a2 + bc).
The equality holds for a = b = c, and for a = b and c = 0 (or any cyclic permuta-
tion).
Symmetric Rational Inequalities 145

P 1.103. Let a, b, c be nonnegative real numbers, no two of which are zero. Prove
that
a(b + c) b(c + a) c(a + b) (a + b + c)2
+ 2 + 2 ≥ 2 .
a2 + bc b + ca c + ab a + b2 + c 2
(Pham Huu Duc, 2006)

Solution. Assume that a ≥ b ≥ c and write the inequality as follows:

(a + b + c)2 X ab + ac
 ‹
3− 2 ≥ 1− 2 ,
a + b2 + c 2 a + bc
X X (a − b)(a − c)
2 (a − b)(a − c) ≥ (a2 + b2 + c 2 ) ,
a2 + bc
X (a − b)(a − c)(a + b − c)(a − b + c)
≥ 0.
a2 + bc
It suffices to show that
(b − c)(b − a)(b + c − a)(b − c + a) (c − a)(c − b)(c + a − b)(c − a + b)
+ ≥ 0,
b2 + ca c 2 + ab
which is equivalent to the obvious inequality

(b − c)2 (c − a + b)2 (a2 + bc)


≥ 0.
(b2 + ca)(c 2 + ab)
The equality holds for a = b = c, and for a = b and c = 0 (or any cyclic permuta-
tion).

P 1.104. Let a, b, c be nonnegative real numbers, no two of which are zero. If k > 0,
then
p p p p
b2 + c 2 + 3bc c 2 + a2 + 3ca a2 + b2 + 3ab 3(2 + 3)
+ + ≥ .
a2 + kbc b2 + kca c 2 + kab 1+k
(Vasile Cîrtoaje, 2013)

Solution. We use the highest coefficient method. Write the inequality in the form
f6 (a, b, c) ≥ 0, where
X p
f6 (a, b, c) = (1 + k) (b2 + c 2 + 3bc)(b2 + kca)(c 2 + kab)
p
−3(2 + 3 )(a2 + kbc)(b2 + kca)(c 2 + kab).
Clearly, f6 (a, b, c) has the same highest coefficient A as
p
(1 + k)P2 (a, b, c) − 3(2 + 3 )P3 (a, b, c),
146 Vasile Cîrtoaje

where Xp
P2 (a, b, c) = ( 3bc − a2 )(b2 + kca)(c 2 + kab),

P3 (a, b, c) = (a2 + kbc)(b2 + kca)(c 2 + kab).


According to Remark 2 from the proof of P 2.75 in Volume 1, we have
p
A = (1 + k)P2 (1, 1, 1) − 3(2 + 3 )P3 (1, 1, 1)
p p
= 3( 3 − 1)(1 + k)3 − 3(2 + 3 )(1 + k)3 = −9(1 + k)3 .

Since A ≤ 0, according to P 3.76-(a) in Volume 1, it suffices to prove the original


inequality for b = c = 1 and for a = 0.
In the first case (b = c = 1), the inequality is equivalent to
p p p
2 + 3 2(a2 + 3a + 1) 3(2 + 3)
+ ≥ ,
a2 + k ka + 1 1+k
p p
2(a2 + 3a + 1) (2 + 3)(3a2 + 2k − 1)
≥ ,
ka + 1 (k + 1)(a2 + k)
– p  p 2 ™
+
 
3 1 3
(a − 1)2 (k + 1)a2 − 1 + (k − 2)a + k − ≥ 0.
2 2
For the nontrivial case k > 2, we have
p 2 p 
1+ 3 1+ 3
 
p
(k + 1)a + k −
2
≥2 k+1 k− a
2 2
p  p 
p 1+ 3
 
3
≥2 3 k− a ≥ 1+ (k − 2)a.
2 2
In the second case (a = 0), the original inequality can be written as
p
1 b c p 3(2 + 3)
‹  2
c2
 ‹
b
+ + 3 + 2 + 2 ≥ .
k c b c b 1+k
It suffices to show that
p
1 p 3(2 + 3)
(2 + 3) + 2 ≥ ,
k 1+k
which is equivalent to
p 2
1+ 3

k− ≥ 0.
2
p
1+ 3
The equality holds for a = b = c. If k = , then the equality holds also for
2
a = 0 and b = c (or any cyclic permutation).
Symmetric Rational Inequalities 147

P 1.105. Let a, b, c be nonnegative real numbers, no two of which are zero. Prove
that
1 1 1 8 6
+ + + ≥ .
a2 + b2 b2 + c 2 c 2 + a2 a2 + b2 + c 2 ab + bc + ca
(Vasile Cîrtoaje, 2013)

Solution. Multiplying by a2 + b2 + c 2 , the inequality becomes

a2 b2 c2 6(a2 + b2 + c 2 )
+ + + 11 ≥ .
b2 + c 2 c 2 + a2 a2 + b2 ab + bc + ca
Since
a2 b2 c2
 ‹
+ 2 + 2 (a2 b2 + b2 c 2 + c 2 a2 ) =
b +c
2 2 c +a 2 a +b 2

1 1 1
 ‹
=a +b +c +a b c
4 4 4 2 2 2
+ 2 + 2 ≥ a4 + b4 + c 4 ,
a +b
2 2 b +c 2 c +a 2

it suffices to show that

a4 + b4 + c 4 6(a2 + b2 + c 2 )
+ 11 ≥ ,
a2 b2 + b2 c 2 + c 2 a2 ab + bc + ca
which is equivalent to

(a2 + b2 + c 2 )2 6(a2 + b2 + c 2 )
+ 9 ≥ .
a2 b2 + b2 c 2 + c 2 a2 ab + bc + ca
Clearly, it is enough to prove that
‹2
a2 + b2 + c 2 6(a2 + b2 + c 2 )

+9≥ ,
ab + bc + ca ab + bc + ca

which is
‹2
a2 + b2 + c 2

−3 ≥ 0.
ab + bc + ca
b c
The equality holds for a = 0 and + = 3 (or any cyclic permutation).
c b

P 1.106. If a, b, c are the lengths of the sides of a triangle, then

a(b + c) b(c + a) c(a + b)


+ 2 + 2 ≤ 2.
a + 2bc b + 2ca c + 2ab
2

(Vo Quoc Ba Can and Vasile Cîrtoaje, 2010)


148 Vasile Cîrtoaje

Solution. Write the inequality as


X ab + ac
‹
1− 2 ≥ 1,
a + 2bc
X a2 + 2bc − ab − ac
≥ 1.
a2 + 2bc
Since

a2 + 2bc − ab − ac = bc − (a − c)(b − a) ≥ |a − c||b − a| − (a − c)(b − a) ≥ 0,

by the Cauchy-Schwarz inequality, we have


P 2 2
X a2 + 2bc − ab − ac (a + 2bc − ab − ac)
≥P .
a2 + 2bc (a2 + 2bc)(a2 + 2bc − ab − ac)
Thus, it suffices to prove that
X
(a2 + b2 + c 2 )2 ≥ (a2 + 2bc)(a2 + 2bc − ab − ac),

which reduces to the obvious inequality

ab(a − b)2 + bc(b − c)2 + ca(c − a)2 ≥ 0.

The equality holds for an equilateral triangle, and for a degenerate triangle with
a = 0 and b = c (or any cyclic permutation).

P 1.107. If a, b, c are real numbers, then


a2 − bc b2 − ca c 2 − ab
+ + ≥ 0.
2a2 + b2 + c 2 2b2 + c 2 + a2 2c 2 + a2 + b2
(Nguyen Anh Tuan, 2005)
First Solution. Rewrite the inequality as
X 1 a2 − bc 3
‹
− 2 ≤ ,
2 2a + b + c 2 2 2
X (b + c)2
≤ 3.
2a2 + b2 + c 2
If two of a, b, c are zero, then the inequality is trivial. Otherwise, applying the
Cauchy-Schwarz inequality, we get
X (b + c)2 X (b + c)2 X  b2 c2
‹
= ≤ +
2a2 + b2 + c 2 (a2 + b2 ) + (a2 + c 2 ) a2 + b2 a2 + c 2
X b2 X a2
= + = 3.
a2 + b2 b2 + a2
Symmetric Rational Inequalities 149

The equality holds for a = b = c.


Second Solution. Use the SOS method. We have
X a2 − bc X (a − b)(a + c) + (a − c)(a + b)
2 =
2a2 + b2 + c 2 2a2 + b2 + c 2
X (a − b)(a + c) X (b − a)(b + c)
= +
2a2 + b2 + c 2 2b2 + c 2 + a2
a+c b+c
X  ‹
= (a − b) −
2a2 + b2 + c 2 2b2 + c 2 + a2
X (a − b)2
= (a2 + b2 + c 2 − ab − bc − ca) ≥ 0.
(2a2 + b2 + c 2 )(2b2 + c 2 + a2 )

P 1.108. If a, b, c are nonnegative real numbers, then

3a2 − bc 3b2 − ca 3c 2 − ab 3
+ + ≤ .
2a2 + b2 + c 2 2b2 + c 2 + a2 2c 2 + a2 + b2 2
(Vasile Cîrtoaje, 2008)

First Solution. Write the inequality as


X 3 3a2 − bc
‹
− ≥ 3,
2 2a2 + b2 + c 2
X 8bc + 3(b − c)2
≥ 6.
2a2 + b2 + c 2
By the Cauchy-Schwarz inequality, we have

[4bc + (b − c)2 ]2 2(b + c)4


8bc + 3(b − c)2 ≥ = 2 .
1 b + c 2 + 4bc
2bc + (b − c) 2
3
Therefore, it suffices to prove that
X (b + c)4
≥ 2.
(2a2 + b2 + c 2 )(b2 + c 2 + 4bc)
Using again the Cauchy-Schwarz inequality, we get
P 2
X (b + c)4 (b + c)2
≥P = 2.
(2a2 + b2 + c 2 )(b2 + c 2 + 4bc) (2a2 + b2 + c 2 )(b2 + c 2 + 4bc)

The equality holds for a = b = c, for a = 0 and b = c (or any cyclic permutation),
and for b = c = 0 (or any cyclic permutation).
150 Vasile Cîrtoaje

Second Solution. Use the SOS method. Write the inequality as


X 1 3a2 − bc
‹
− ≥ 0,
2 2a2 + b2 + c 2
X (b + c + 2a)(b + c − 2a)
≥ 0,
2a2 + b2 + c 2
X (b + c + 2a)(b − a) + (b + c + 2a)(c − a)
≥ 0,
2a2 + b2 + c 2
X (b + c + 2a)(b − a) X (c + a + 2b)(a − b)
+ ≥ 0,
2a2 + b2 + c 2 2b2 + c 2 + a2
c + a + 2b b + c + 2a
X  ‹
(a − b) − ≥ 0,
2b2 + c 2 + a2 2a2 + b2 + c 2
X
(3ab + bc + ca − c 2 )(2c 2 + a2 + b2 )(a − b)2 ≥ 0.
Clearly, it suffices to show that
X
c(a + b − c)(2c 2 + a2 + b2 )(a − b)2 ≥ 0.

Assume that a ≥ b ≥ c. It is enough to prove that

a(b + c − a)(2a2 + b2 + c 2 )(b − c)2 + b(c + a − b)(2b2 + c 2 + a2 )(c − a)2 ≥ 0;

that is,

b(c + a − b)(2b2 + c 2 + a2 )(a − c)2 ≥ a(a − b − c)(2a2 + b2 + c 2 )(b − c)2 .

Since c + a − b ≥ a − b − c, it suffices to prove that

b(2b2 + c 2 + a2 )(a − c)2 ≥ a(2a2 + b2 + c 2 )(b − c)2 .

We can obtain this inequality by multiplying the inequalities

b2 (a − c)2 ≥ a2 (b − c)2

and
a(2b2 + c 2 + a2 ) ≥ b(2a2 + b2 + c 2 ).
The last inequality is equivalent to

(a − b)[(a − b)2 + ab + c 2 ] ≥ 0.
Symmetric Rational Inequalities 151

P 1.109. If a, b, c are nonnegative real numbers, then

(b + c)2 (c + a)2 (a + b)2


+ + ≥ 2.
4a2 + b2 + c 2 4b2 + c 2 + a2 4c 2 + a2 + b2
(Vasile Cîrtoaje, 2005)

Solution. By the Cauchy-Schwarz inequality, we have


P 2
X (b + c)2 (b + c)2
≥P
4a2 + b2 + c 2 (b + c)2 (4a2 + b2 + c 2 )

a4 + 3 a2 b2 + 4abc a + 2 ab(a2 + b2 )
P P P P
= 2ḑot P ≥2
a4 + 5 a2 b2 + 4abc a + ab(a2 + b2 )
P P P

because X X
ab(a2 + b2 ) ≥ 2 a2 b2 .
The equality holds for a = b = c, and for b = c = 0 (or any cyclic permutation).

P 1.110. If a, b, c are positive real numbers, then


P 1 3
(a) ≤ ;
11a2 + 2b + 2c
2 2 5(ab + bc + ca)

1 1 1
+
P
(b) ≤ .
4a2 +b +c
2 2 2(a + b + c ) ab + bc + ca
2 2 2

(Vasile Cîrtoaje, 2008)

Solution. We will prove that


X k+2 11 − 2k 2(k − 1)
≤ 2 +
ka2 +b +c
2 2 a +b +c
2 2 ab + bc + ca

for any k > 1. Due to homogeneity, we may assume that a2 + b2 + c 2 = 3. On this


hypothesis, we need to show that
X k+2 11 − 2k 2(k − 1)
≤ + .
(k − 1)a2 + 3 3 ab + bc + ca

Using the substitution m = 3/(k − 1), m > 0, the inequality can be written as
X 1 6
m(m + 1) ≤ 3m − 2 + .
a2 +m ab + bc + ca
152 Vasile Cîrtoaje

By the Cauchy-Schwarz inequality, we have


p p
(a2 + m)[m + (m + 1 − a)2 ] ≥ [a m + m(m + 1 − a)]2 = m(m + 1)2 ,

and hence
m(m + 1) a2 − 1
≤ + m + 2 − 2a,
a2 + m m+1
X 1 X
m(m + 1) ≤ 3(m + 2) − 2 a.
a2 + m
Thus, it suffices to show that
X 6
3(m + 2) − 2 a ≤ 3m − 2 + ;
ab + bc + ca
that is,
(4 − a − b − c)(ab + bc + ca) ≤ 3.
Let p = a + b + c. Since

2(ab + bc + ca) = (a + b + c)2 − (a2 + b2 + c 2 ) = p2 − 3,

we get

6 − 2(4 − a − b − c)(ab + bc + ca) = 6 − (4 − p)(p2 − 3)


= (p − 3)2 (p + 2) ≥ 0.

This completes the proof. The equality holds for a = b = c.

P 1.111. If a, b, c are nonnegative real numbers such that ab + bc + ca = 3, then


p p p
a b c 3
+ + ≥ .
b+c c+a a+b 2
(Vasile Cîrtoaje, 2006)

Solution. By the Cauchy-Schwarz inequality, we have

X pa
P 3/4 2
a 1 €X 3/4 Š2
≥P = a .
b+c a(b + c) 6

Thus, it suffices to show that

a3/4 + b3/4 + c 3/4 ≥ 3,

which follows immediately from Remark 1 from the proof of the inequality in P
3.33 in Volume 1. The equality occurs for a = b = c = 1.
Symmetric Rational Inequalities 153

Remark. Analogously, according to Remark 2 from the proof of P 3.33 in Volume


1, we can prove that
ak bk ck 3
+ + ≥
b+c c+a a+b 2
4 ln 2 4 ln 2
for all k ≥ 3 − ≈ 0.476. For k = 3 − , the equality occurs for a = b =
ln 3 p ln 3
c = 1, and also for a = 0 and b = c = 3 (or any cyclic permutation).

P 1.112. If a, b, c are nonnegative real numbers such that ab + bc + ca ≥ 3, then


1 1 1 1 1 1
+ + ≥ + + .
2+a 2+ b 2+c 1+ b+c 1+c+a 1+a+ b
(Vasile Cîrtoaje, 2014)

Solution. Consider c = min{a, b, c}, and denote


1 1 1 1 1 1
E(a, b, c) = + + − − − .
2+a 2+ b 2+c 1+ b+c 1+c+a 1+a+ b
If c ≥ 1, the desired inequality E(a, b, c) ≥ 0 follows by summing the obvious
inequalities
1 1
≥ ,
2+a 1+c+a
1 1
≥ ,
2+ b 1+a+ b
1 1
≥ .
2+c 1+ b+c
Consider further that c < 1. From
1−c 1 1 1 1
E(a, b, c) = − − + + −
(2 + a)(1 + c + a) 1 + a + b 2 + b 2 + c 1 + b + c
and
1−c 1 1 1 1
E(a, b, c) = − − + + − ,
(2 + b)(1 + b + c) 1 + a + b 2 + a 2 + c 1 + c + a

it follows that E(a, b, c) is increasing in a and b. Based on this result, it suffices to


prove the desired inequality only for

ab + bc + ca = 3.

Applying the AM-GM inequality, we get

3 = ab + bc + ca ≥ 3(abc)2/3 , abc ≤ 1,
154 Vasile Cîrtoaje

p
3
a + b + c ≥ 3 abc ≥ 3.
We will show that
1 1 1 1 1 1
+ + ≥1≥ + + .
2+a 2+ b 2+c 1+ b+c 1+c+a 1+a+ b
By direct calculation, we can show that the left inequality is equivalent to abc ≤ 1,
while the right inequality is equivalent to a + b + c ≥ 2 + abc. Clearly, these are
true and the proof is completed. The equality occurs for a = b = c = 1.

P 1.113. If a, b, c are the lengths of the sides of a triangle, then

a2 − bc b2 − ca c 2 − ab
(a) + + ≤ 0;
3a2 + b2 + c 2 3b2 + c 2 + a2 3c 2 + a2 + b2

a4 − b2 c 2 b4 − c 2 a2 c 4 − a2 b2
(b) + + ≤ 0.
3a4 + b4 + c 4 3b4 + c 4 + a4 3c 4 + a4 + b4
(Nguyen Anh Tuan and Vasile Cîrtoaje, 2006)

Solution. (a) Apply the SOS method. We have


X a2 − bc X (a − b)(a + c) + (a − c)(a + b)
2 =
3a2 + b2 + c 2 3a2 + b2 + c 2
X (a − b)(a + c) X (b − a)(b + c)
= +
3a2 + b2 + c 2 3b2 + c 2 + a2
a+c b+c
X  ‹
= (a − b) −
3a2 + b2 + c 2 3b2 + c 2 + a2
X (a − b)2
= (a + b + c − 2ab − 2bc − 2ca)
2 2 2
.
(3a2 + b2 + c 2 )(3b2 + c 2 + a2 )
Since

a2 + b2 + c 2 − 2ab − 2bc − 2ca = a(a − b − c) + b(b − c − a) + c(c − a − b) ≤ 0,

the conclusion follows. The equality holds for an equilateral triangle, and for a
degenerate triangle with a = 0 and b = c (or any cyclic permutation).
(b) Using the same way as above, we get
X a4 − b2 c 2 X (a2 − b2 )2
2 = A ,
3a4 + b4 + c 4 (3a4 + b4 + c 4 )(3b4 + c 4 + a4 )
Symmetric Rational Inequalities 155

where

A = a4 + b4 + c 4 − 2a2 b2 − 2b2 c 2 − 2c 2 a2
= −(a + b + c)(a + b − c)(b + c − a)(c + a − b) ≤ 0.

The equality holds for an equilateral triangle, and for a degenerate triangle with
a = b + c (or any cyclic permutation).

P 1.114. If a, b, c are the lengths of the sides of a triangle, then

bc ca ab 1
+ 2 + 2 ≥ .
4a2 +b +c
2 2 4b + c + a
2 2 4c + a + b
2 2 2
(Vasile Cîrtoaje and Vo Quoc Ba Can, 2010)

Solution. We apply the SOS method. Write the inequality as


X 2bc b2 c 2
X ‹
− ≥ 0,
4a2 + b2 + c 2 a2 b2 + b2 c 2 + c 2 a2
X bc(2a2 − bc)(b − c)2
≥ 0.
4a2 + b2 + c 2
Without loss of generality, assume that a ≥ b ≥ c. Then, it suffices to prove that

c(2b2 − ca)(c − a)2 b(2c 2 − ab)(a − b)2


+ ≥ 0.
4b2 + c 2 + a2 4c 2 + a2 + b2
Since
2b2 − ca ≥ c(b + c) − ca = c(b + c − a) ≥ 0
and

(2b2 − ca) + (2c 2 − ab) = 2(b2 + c 2 ) − a(b + c) ≥ (b + c)2 − a(b + c)


= (b + c)(b + c − a) ≥ 0,

it is enough to show that

c(a − c)2 b(a − b)2


≥ .
4b2 + c 2 + a2 4c 2 + a2 + b2
This follows by multiplying the inequalities

c 2 (a − c)2 ≥ b2 (a − b)2

and
b c
≥ 2 .
4b2 +c +a
2 2 4c + a2 + b2
156 Vasile Cîrtoaje

These inequalities are true, since

c(a − c) − b(a − b) = (b − c)(b + c − a) ≥ 0,

b(4c 2 + a2 + b2 ) − c(4b2 + c 2 + a2 ) = (b − c)[(b − c)2 + a2 − bc] ≥ 0.


The equality occurs for an equilateral triangle, and for a degenerate triangle with
a = b and c = 0 (or any cyclic permutation).

P 1.115. If a, b, c are the lengths of the sides of a triangle, then

1 1 1 9
+ 2 + 2 ≤ .
b2 +c 2 c +a 2 a +b 2 2(ab + bc + ca)

(Vo Quoc Ba Can, 2008)

Solution. Apply the SOS method. Write the inequality as


X • 3 ab + bc + ca ˜
− ≥ 0,
2 b2 + c 2
X 3(b2 + c 2 ) − 2(ab + bc + ca)
≥ 0,
b2 + c 2
X 3b(b − a) + 3c(c − a) + c(a − b) + b(a − c)
≥ 0,
b2 + c 2
X (a − b)(c − 3b) + (a − c)(b − 3c)
≥ 0,
b2 + c 2
X (a − b)(c − 3b) X (b − a)(c − 3a)
+ ≥ 0,
b2 + c 2 c 2 + a2
X
(a2 + b2 )(a − b)2 (ca + c b + 3c 2 − 3ab) ≥ 0.

Without loss of generality, assume that a ≥ b ≥ c. Since

ab + ac + 3a2 − 3bc > 0,

it suffices to prove that

(a2 + b2 )(a − b)2 (ca + c b + 3c 2 − 3ab) + (a2 + c 2 )(a − c)2 (ab + bc + 3b2 − 3ac) ≥ 0,

or, equivalently,

(a2 + c 2 )(a − c)2 (ab + bc + 3b2 − 3ac) ≥ (a2 + b2 )(a − b)2 (3ab − 3c 2 − ca − c b).
Symmetric Rational Inequalities 157

Since

bc + 3b2
 ‹
ab + bc + 3b − 3ac = a
2
+ b − 3c
a
bc + 3b2
 ‹
≥a + b − 3c
b+c
a(b − c)(4b + 3c)
= ≥0
b+c
and

(a b + bc + 3b2 − 3ac) − (3ab − 3c 2 − ca − c b) = 3(b2 + c 2 ) + 2bc − 2a(b + c)


≥ 3(b2 + c 2 ) + 2bc − 2(b + c)2
= (b − c)2 ≥ 0,

it suffices to show that

(a2 + c 2 )(a − c)2 ≥ (a2 + b2 )(a − b)2 .

This is equivalent to (b − c)A ≥ 0, where

A = 2a3 − 2a2 (b + c) + 2a(b2 + bc + c 2 ) − (b + c)(b2 + c 2 )


b + c 2 a(3b2 + 2bc + 3c 2 )
 ‹
= 2a a − + − (b + c)(b2 + c 2 )
2 2
b(3b2 + 2bc + 3c 2 )
≥ − (b + c)(b2 + c 2 )
2
(b − c)(b2 + bc + 2c 2 )
= ≥ 0.
2
The equality occurs for an equilateral triangle, and for a degenerate triangle with
a/2 = b = c (or any cyclic permutation).

P 1.116. If a, b, c are the lengths of the sides of a triangle, then

a+b b+c c+a


(a) + + > 5;
a−b b−c c−a

a2 + b2 b2 + c 2 c 2 + a2
(b) + + ≥ 3.
a2 − b2 b2 − c 2 c 2 − a2

(Vasile Cîrtoaje, 2003)


158 Vasile Cîrtoaje

Solution. Since the inequalities are symmetric, we consider

a > b > c.

(a) Let x = a − c and y = b − c. From a > b > c and a ≤ b + c, it follows

x > y > 0, c ≥ x − y.

We have
a + b b + c c + a 2c + x + y 2c + y 2c + x
+ + = + −
a−b b−c c−a x−y y x
1 1 1 x+y
 ‹
= 2c + − +
x−y y x x−y
2c x + y 2(x − y) x + y
> + ≥ +
y x−y y x−y
x−y
 ‹
y
=2 + + 1 ≥ 5.
y x−y
(b) We will show that

a2 + b2 b2 + c 2 c 2 + a2
+ + ≥ 3;
a2 − b2 b2 − c 2 c 2 − a2
that is,
b2 c2 a2
+ ≥ 2 .
a2 − b2 b2 − c 2 a − c2
Since
a2 (b + c)2
≤ ,
a2 − c 2 a2 − c 2
it suffices to prove that

b2 c2 (b + c)2
+ ≥ .
a2 − b2 b2 − c 2 a2 − c 2
This is equivalent to each of the following inequalities:
1 1 1 1 2bc
 ‹  ‹
b 2
− + c 2
− ≥ ,
a2 − b2 a2 − c 2 b2 − c 2 a2 − c 2 a2 − c 2
b2 (b2 − c 2 ) c 2 (a2 − b2 )
+ ≥ 2bc,
a2 − b2 b2 − c 2
[b(b2 − c 2 ) − c(a2 − b2 )]2 ≥ 0.
This completes the proof. If a > b > c, then the equality holds for a degenerate
triangle with a = b + c and b/c = x 1 , where x 1 ≈ 1.5321 is the positive root of the
equation x 3 − 3x − 1 = 0.
Symmetric Rational Inequalities 159

P 1.117. If a, b, c are the lengths of the sides of a triangle, then


b+c c+a a+b
 ‹
a b c
+ + +3≥6 + + .
a b c b+c c+a a+b

Solution. We apply the SOS method. Write the inequality as


X b+c X
2a
‹
−6≥3 −3 .
a b+c
Since
X b+c Xb c ‹ X (b − c)2
−6= + −6=
a c b bc
and
X 2a X 2a − b − c X a − b X a − c
−3= = +
b+c b+c b+c b+c
X a − b X b − a X (a − b)2
= + =
b+c c+a (b + c)(c + a)
X (b − c)2
= ,
(c + a)(a + b)
we can rewrite the inequality as
X
a(b + c)(b − c)2 Sa ≥ 0,
where
Sa = a(a + b + c) − 2bc.
Without loss of generality, assume that a ≥ b ≥ c. Since Sa > 0,
S b = b(a + b + c) − 2ca = (b − c)(a + b + c) + c(b + c − a) ≥ 0
and
X
a(b + c)(b − c)2 Sa ≥ b(c + a)(c − a)2 S b + c(a + b)(a − b)2 Sc
≥ (a − b)2 [b(c + a)S b + c(a + b)Sc ],
it suffices to prove that
b(c + a)S b + c(a + b)Sc ≥ 0.
This is equivalent to each of the following inequalities
(a + b + c)[a(b2 + c 2 ) + bc(b + c)] ≥ 2abc(2a + b + c),
a(a + b + c)(b − c)2 + (a + b + c)[2abc + bc(b + c)] ≥ 2abc(2a + b + c),
a(a + b + c)(b − c)2 + bc(2a + b + c)(b + c − a) ≥ 0.
Since the last inequality is true, the proof is completed. The equality occurs for an
equilateral triangle, and for a degenerate triangle with a/2 = b = c (or any cyclic
permutation).
160 Vasile Cîrtoaje

P 1.118. Let a, b, c be nonnegative real numbers, no two of which are zero. Prove
that
X 3a(b + c) − 2bc 3
≥ .
(b + c)(2a + b + c) 2
(Vasile Cîrtoaje, 2009)

Solution. Use the SOS method. Write the inequality as follows:


X • 3a(b + c) − 2bc 1
˜
− ≥ 0,
(b + c)(2a + b + c) 2
X 4a(b + c) − 6bc − b2 − c 2
≥ 0,
(b + c)(2a + b + c)
X b(a − b) + c(a − c) + 3b(a − c) + 3c(a − b)
≥ 0,
(b + c)(2a + b + c)
X (a − b)(b + 3c) + (a − c)(c + 3b)
≥ 0,
(b + c)(2a + b + c)
X (a − b)(b + 3c) X (b − a)(a + 3c)
+ ≥ 0,
(b + c)(2a + b + c) (c + a)(2b + c + a)
b + 3c a + 3c
X • ˜
(a − b) − ≥ 0,
(b + c)(2a + b + c) (c + a)(2b + c + a)
X
(a − b)(b − c)(c − a) (a2 − b2 )(a + b + 2c) ≥ 0.
Since X
(a2 − b2 )(a + b + 2c) = (a − b)(b − c)(c − a),
the conclusion follows. The equality holds for a = b, or b = c, or c = a.

P 1.119. Let a, b, c be nonnegative real numbers, no two of which are zero. Prove
that
X a(b + c) − 2bc
≥ 0.
(b + c)(3a + b + c)
(Vasile Cîrtoaje, 2009)

Solution. We apply the SOS method. Since


X a(b + c) − 2bc X b(a − c) + c(a − b)
=
(b + c)(3a + b + c) (b + c)(3a + b + c)
X c(b − a) X c(a − b)
= +
(c + a)(3b + c + a) (b + c)(3a + b + c)
Symmetric Rational Inequalities 161

X c(a + b − c)(a − b)2


= ,
(b + c)(c + a)(3a + b + c)(3b + c + a)
the inequality is equivalent to
X
c(a + b)(3c + a + b)(a + b − c)(a − b)2 ≥ 0.

Without loss of generality, assume that a ≥ b ≥ c. Since a + b − c ≥ 0, it suffices to


show that

b(c + a)(3b + c + a)(c + a − b)(a − c)2 ≥ a(b + c)(3a + b + c)(a − b − c)(b − c)2 .

This is true since


c + a − b ≥ a − b − c,
b2 (a − c)2 ≥ a2 (b − c)2 ,
c + a ≥ b + c,
a(3b + c + a) ≥ b(3a + b + c).
The equality holds for a = b = c, and for a = b and c = 0 (or any cyclic permuta-
tion).

P 1.120. Let a, b, c be positive real numbers such that a2 + b2 + c 2 ≥ 3. Prove that

a5 − a2 b5 − b2 c5 − c2
+ + ≥ 0.
a5 + b2 + c 2 b5 + c 2 + a2 c 5 + a2 + b2
(Vasile Cîrtoaje, 2005)

Solution. The inequality is equivalent to


1 1 1 3
+ + ≤ .
a5 + b2 + c 2 b5 + c 2 + a2 c 5 + a2 + b2 a2 + b2 + c 2
Setting a = t x, b = t y and c = tz, where

x, y, z > 0, x 2 + y 2 + z 2 = 3,

the condition a2 + b2 + c 2 ≥ 3 implies t ≥ 1, and the inequality becomes


1 1 1
+ + ≤ 1.
t 3 x 5 + y 2 + z2 t 3 y 5 + z2 + x 2 t 3z5 + x 2 + y 2

We see that it suffices to prove this inequality for t = 1, when it becomes


1 1 1
+ 5 + 5 ≤ 1.
x5 − x + 3 y − y + 3 z − z2 + 3
2 2
162 Vasile Cîrtoaje

Without loss of generality, assume that x ≥ y ≥ z. There are two cases to consider.
p
Case 1: z ≤ y ≤ x ≤ 2. The desired inequality follows by adding the inequalities

1 3 − x2 1 3 − y2 1 3 − z2
≤ , ≤ , ≤ .
x5 − x2 + 3 6 y5 − y2 + 3 6 z5 − z2 + 3 6
We have
1 3 − x2 (x − 1)2 (x 5 + 2x 4 − 3x 2 − 6x − 3)
− = ≤0
x5 − x2 + 3 6 6(x 5 − x 2 + 3)
since
6 3
 ‹
x + 2x − 3x − 6x − 3 = x x + 2x − 3 − − 2
5 4 2 2 3 2
x x
p p 3
 ‹
≤ x 2 2+4−3−3 2−
2
2
p 1
= −x 2 ( 2 + ) < 0.
2
p
Case 2: x > 2. From x 2 + y 2 + z 2 = 3, it follows that y 2 + z 2 < 1. Since
1 1 1 1
< p < p <
x5 − x + 3 (2 2 − 1)x + 3 2(2 2 − 1) + 3 6
2 2

and
1 1 1 1
+ 5 < + ,
y5 − y +3 z −z +3 3− y
2 2 2 3 − z2
it suffices to prove that
1 1 5
2
+ 2
≤ .
3− y 3−z 6
Indeed, we have
1 1 5 9( y 2 + z 2 − 1) − 5 y 2 z 2
+ − = < 0,
3 − y 2 3 − z2 6 6(3 − y 2 )(3 − z 2 )
which completes the proof. The equality occurs for a = b = c = 1.
p3
Remark. Since abc ≥ 1 involves a2 + b2 + c 2 ≥ 3 a2 b2 c 2 ≥ 3, the inequality is also
true under the condition abc ≥ 1. A proof of this inequality (which is a problem
from IMO-2005 - proposed by Hojoo Lee) is the following:
X a5 − a2 X a5 − a2 1 X 1
‹
≥ = 2
a − ,
a5 + b2 + c 2 a5 + a3 (b2 + c 2 ) a2 + b2 + c 2 a
X 1 1X
‹ X
a −2
≥ (a2 − bc) = (a − b)2 ≥ 0.
a 2
Symmetric Rational Inequalities 163

P 1.121. Let a, b, c be positive real numbers such that a2 + b2 + c 2 = a3 + b3 + c 3 .


Prove that
a2 b2 c2 3
+ + ≥ .
b+c c+a a+b 2
(Pham Huu Duc, 2008)

First Solution. By the Cauchy-Schwarz inequality, we have


P 3 2 P 3 P 2
X a2 a a a
≥P = P  P  P .
b+c a4 (b + c) a3 ab − abc a2

Therefore, it is enough to show that


€X Š €X Š X €X Š €X Š
2 a3 a2 + 3abc a2 ≥ 3 a3 ab .

Write this inequality as follows:


€X Š €X X Š €X Š €X Š
3 2 3
3 a a − ab − a − 3abc a2 ≥ 0,
€X Š €X X Š €X Š €X X Š €X Š
3 a3 a2 − ab − a a2 − ab a2 ≥ 0,
€X X Š” X €X Š €X Š—
a2 − ab 3 a3 − a a2 ≥ 0.
The last inequality is true since
€X X Š X
2 a2 − ab = (a − b)2 ≥ 0

and
X €X Š €X Š X X
3 a3 − a a2 = (a3 + b3 ) − ab(a + b)
X
= (a + b)(a − b)2 ≥ 0.

The equality occurs for a = b = c = 1.


Second Solution. Write the inequality in the homogeneous form A ≥ B, where
X a2 X 3(a3 + b3 + c 3 ) X
A=2 − a, B= − a.
b+c a2 + b2 + c 2
Since
X a(a − b) + a(a − c) X a(a − b) X b(b − a)
A= = +
b+c b+c c+a
X (a − b)2
= (a + b + c)
(b + c)(c + a)
164 Vasile Cîrtoaje

and
(a + b3 ) − ab(a + b) (a + b)(a − b)2
P 3 P P
B= = ,
a2 + b2 + c 2 a2 + b2 + c 2
we can write the inequality as
X• a + b + c a+b
˜
− (a − b)2 ≥ 0,
(b + c)(c + a) a2 + b2 + c 2
X (a − b)2
(a3 + b3 + c 3 − 2abc) ≥ 0.
(b + c)(c + a)
Since a3 + b3 + c 3 ≥ 3abc, the conclusion follows.

P 1.122. If a, b, c ∈ [0, 1], then

a b c
+ + ≤ 1.
bc + 2 ca + 2 ab + 2
(Vasile Cîrtoaje, 2010)

Solution. (a) First Solution. It suffices to show that


a b c
+ + ≤ 1,
abc + 2 abc + 2 abc + 2
which is equivalent to
abc + 2 ≥ a + b + c.
We have

abc + 2 − a − b − c = (1 − b)(1 − c) + (1 − a)(1 − bc) ≥ 0.

The equality holds for a = b = c = 1, and for a = 0 and b = c = 1 (or any cyclic
permutation).
Second Solution. Assume that a = max{a, b, c}. It suffices to show that

a b c
+ + ≤ 1.
bc + 2 bc + 2 bc + 2
that is,
a + b + c) ≤ 2 + bc.
We have
2 + bc − a − b − c) = 1 − a + (1 − b)(1 − c) ≥ 0.
Symmetric Rational Inequalities 165

P 1.123. Let a, b, c be positive real numbers such that a + b + c = 2. Prove that


1 1 1
 ‹
5(1 − ab − bc − ca) + + + 9 ≥ 0.
1 − ab 1 − bc 1 − ca
(Vasile Cîrtoaje, 2011)

Solution. Write the inequality as

5a(b + c) 5b(c + a) 5c(a + b)


24 − − − ≥ 0.
1 − bc 1 − ca 1 − ab
Since

4(1 − bc) ≥ 4 − (b + c)2 = (a + b + c)2 − (b + c)2 = a(a + 2b + 2c),

it suffices to show that


b+c c+a a+b
 ‹
6−5 − − ≥ 0,
a + 2b + 2c b + 2c + 2a c + 2a + 2b
which is equivalent to
X  b+c
‹
5 1− ≥ 9,
a + 2b + 2c
X 1
5(a + b + c) ≥ 9,
a + 2b + 2c
— X 1
”X ‹
(a + 2b + 2c) ≥ 9.
a + 2b + 2c
The last inequality follows immediately from the AM-HM inequality. The equality
holds for a = b = c = 2/3.

P 1.124. Let a, b, c be nonnegative real numbers such that a + b + c = 2. Prove that

2 − a2 2 − b2 2 − c2
+ + ≤ 3.
2 − bc 2 − ca 2 − ab
(Vasile Cîrtoaje, 2011)

First Solution. Write the inequality as follows:


X 2 − a2
‹
1− ≥ 0,
2 − bc
X a2 − bc
≥ 0,
2 − bc
X
(a2 − bc)(2 − ca)(2 − ab) ≥ 0,
166 Vasile Cîrtoaje
X
(a2 − bc)[4 − 2a(b + c) + a2 bc] ≥ 0,
X X X
4 (a2 − bc) − 2 a(b + c)(a2 − bc) + abc a(a2 − bc) ≥ 0.
By virtue of the AM-GM inequality,
X
a(a2 − bc) = a3 + b3 + c 3 − 3abc ≥ 0.

Then, it suffices to prove that


X X
2 (a2 − bc) ≥ a(b + c)(a2 − bc).

Indeed, we have
X X X
a(b + c)(a2 − bc) = a3 (b + c) − abc (b + c)
X X X
= a(b3 + c 3 ) − abc (b + c) = a(b + c)(b − c)2
X • a + (b + c) ˜2 X X
≤ (b − c)2 = (b − c)2 = 2 (a2 − bc).
2
The equality holds for a = b = c = 2/3, and for a = 0 and b = c = 1 (or any cyclic
permutation).
Second Solution. We apply the SOS method. Write the inequality as follows:
X a2 − bc
≥ 0,
2 − bc
X (a − b)(a + c) + (a − c)(a + b)
≥ 0,
2 − bc
X (a − b)(a + c) X (b − a)(b + c)
+ ≥ 0,
2 − bc 2 − ca
X (a − b)2 [2 − c(a + b) − c 2 ]
≥ 0,
(2 − bc)(2 − ca)
X
(a − b)2 (2 − ab)(1 − c) ≥ 0.
Assuming that a ≥ b ≥ c, it suffices to prove that

(b − c)2 (2 − bc)(1 − a) + (c − a)2 (2 − ca)(1 − b) ≥ 0.

Since
2(1 − b) = a − b + c ≥ 0, (c − a)2 ≥ (b − c)2 ,
it suffices to show that

(2 − bc)(1 − a) + (2 − ca)(1 − b) ≥ 0.
Symmetric Rational Inequalities 167

We have

(2 − bc)(1 − a) + (2 − ca)(1 − b) = 4 − 2(a + b) − c(a + b) + 2abc

(a + b) + (2 + c) 2
• ˜
≥ 4 − (a + b)(2 + c) ≥ 4 − = 0.
2

P 1.125. Let a, b, c be nonnegative real numbers such that a + b + c = 3. Prove that

3 + 5a2 3 + 5b2 3 + 5c 2
+ + ≥ 12.
3 − bc 3 − ca 3 − ab
(Vasile Cîrtoaje, 2010)

Solution. Use the SOS method. Write the inequality as follows:


X  3 + 5a2 ‹
− 4 ≥ 0,
3 − bc
X 5a2 + 4bc − 9
≥ 0,
3 − bc
X 5a2 + 4bc − (a + b + c)2
≥ 0,
3 − bc
X 4a2 − b2 − c 2 − 2ab + 2bc − 2ca
≥ 0,
3 − bc
X 2a2 − b2 − c 2 + 2(a − b)(a − c)
≥ 0,
3 − bc
X (a − b)(a + b) + (a − c)(a + c) + 2(a − b)(a − c)
≥ 0,
3 − bc
X [(a − b)(a + b) + (a − b)(a − c)] + [(a − c)(a + c) + (a − c)(a − b)]
≥ 0,
3 − bc
X (a − b)(2a + b − c) + (a − c)(2a + c − b)
≥ 0,
3 − bc
X (a − b)(2a + b − c) X (b − a)(2b + a − c)
+ ≥ 0,
3 − bc 3 − ca
X (a − b)2 [3 − 2c(a + b) + c 2 ]
≥ 0,
(3 − bc)(3 − ca)
X (a − b)2 (c − 1)2
≥ 0.
(3 − bc)(3 − ca)
The equality holds for a = b = c = 1.
168 Vasile Cîrtoaje

P 1.126. Let a, b, c be nonnegative real numbers such that a + b + c = 2. If

−1 7
≤m≤ ,
7 8
then
a2 + m b2 + m c2 + m 3(4 + 9m)
+ + ≥ .
3 − 2bc 3 − 2ca 3 − 2ab 19
(Vasile Cîrtoaje, 2010)

Solution. We apply the SOS method. Write the inequality as


X  a2 + m 4 + 9m ‹
− ≥ 0,
3 − 2bc 19
X 19a2 + 2(4 + 9m)bc − 12 − 8m
≥ 0.
3 − 2bc
Since
19a2 + 2(4 + 9m)bc − 12 − 8m =
= 19a2 + 2(4 + 9m)bc − (3 + 2m)(a + b + c)2
= (16 − 2m)a2 − (3 + 2m)(b2 + c 2 + 2ab + 2ac) + 2(1 + 7m)bc
= (3 + 2m)(2a2 − b2 − c 2 ) + 2(5 − 3m)(a2 + bc − ab − ac) + (4 − 10m)(ab + ac − 2bc)
= (3 + 2m)(a2 − b2 ) + (5 − 3m)(a − b)(a − c) + (4 − 10m)c(a − b)
+(3 + 2m)(a2 − c 2 ) + (5 − 3m)(a − c)(a − b) + (4 − 10m)b(a − c)
= (a − b)B + (a − c)C,
where
B = (8 − m)a + (3 + 2m)b − (1 + 7m)c,
C = (8 − m)a + (3 + 2m)c − (1 + 7m)b,
the inequality can be written as

B1 + C1 ≥ 0,

where
X (a − b)[(8 − m)a + (3 + 2m)b − (1 + 7m)c]
B1 = ,
3 − 2bc
X (b − a)[(8 − m)b + (3 + 2m)a − (1 + 7m)c]
C1 = .
3 − 2ca
We have
X (a − b)2 Sc
B1 + C1 = ,
(3 − 2bc)(3 − 2ca)
Symmetric Rational Inequalities 169

where

Sc =3(5 − 3m) − 2(8 − m)c(a + b) + 2(1 + 7m)c 2


=6(2m + 3)c 2 − 4(8 − m)c + 3(5 − 3m)
˜2
8−m (1 + 7m)(7 − 8m)
•
=6(2m + 3) c − + .
3(2m + 3) 3(2m + 3)

Since Sc ≥ 0 for −1/7 ≤ m ≤ 7/8, the proof is completed. The equality holds for
a = b = c = 2/3. If m = −1/7, then the equality holds also for a = 0 and b = c = 1
(or any cyclic permutation). If m = 7/8, then the equality holds also for a = 1 and
b = c = 1/2 (or any cyclic permutation).

Remark. The following more general statement holds:


• Let a, b, c be nonnegative real numbers such that a + b + c = 3. If

0 < k ≤ 3, m1 ≤ m ≤ m2 ,

where 
3
 −∞,

 0<k≤
2
m1 = (3 − k)(4 − k) 3 ,


 2(3 − 2k) , < k ≤ 3
2
p
36 − 4k − k2 + 4(9 − k) 3(3 − k)
m2 = ,
72 + k
then
a2 + mbc b2 + mca c 2 + mab 3(1 + m)
+ + ≥ ,
9 − kbc 9 − kca 9 − kab 9−k

with equality for a = b = c = 1. If 3/2 < k ≤ 3 and m = m1 , then the equality holds
also for
3
a = 0, b = c = .
2
If m = m2 , then the equality holds also for
p p
3k − 6 + 2 3(3 − k) 3− 3(3 − k)
a= , b=c= .
k k
The inequalities in P 1.124, P 1.125 and P 1.126 are particular cases of this
result (for k = 2 and m = m1 = −1, for k = 3 and m = m2 = 1/5, and for k = 8/3,
respectively).
170 Vasile Cîrtoaje

P 1.127. Let a, b, c be nonnegative real numbers such that a + b + c = 3. Prove that

47 − 7a2 47 − 7b2 47 − 7c 2
+ + ≥ 60.
1 + bc 1 + ca 1 + ab
(Vasile Cîrtoaje, 2011)

Solution. We apply the SOS method. Write the inequality as follows:


X  47 − 7a2 ‹
− 20 ≥ 0,
1 + bc
X 27 − 7a2 − 20bc
≥ 0,
1 + bc
X 3(a + b + c)2 − 7a2 − 20bc
≥ 0,
1 + bc
X −3(2a2 − b2 − c 2 ) + 2(a − b)(a − c) + 8(ab − 2bc + ca)
≥ 0,
1 + bc
X −3(a − b)(a + b) + (a − b)(a − c) + 8c(a − b)
+
1 + bc
X −3(a − c)(a + c) + (a − c)(a − b) + 8b(a − c)
+ ≥ 0,
1 + bc
X (a − b)(−2a − 3b + 7c) X (a − c)(−2a − 3c + 7b)
+ ≥ 0,
1 + bc 1 + bc
X (a − b)(−2a − 3b + 7c) X (b − a)(−2b − 3a + 7c)
+ ≥ 0,
1 + bc 1 + ca
X (a − b)2 [1 − 2c(a + b) + 7c 2 ]
≥ 0,
(1 + bc)(1 + ca)
X (a − b)2 (3c − 1)2
≥ 0,
(1 + bc)(1 + ca)
The equality holds for a = b = c = 1, and for a = 7/3 and b = c = 1/3 (or any
cyclic permutation).

Remark. The following more general statement holds:


• Let a, b, c be nonnegative real numbers such that a + b + c = 3. If

k > 0, m ≥ m1 ,

where p
+ + + 3(3 + k)
 2
36 4k − k 4(9 k)
, k 6= 72


72 − k

m1 = ,
238
, k = 72


5

Symmetric Rational Inequalities 171

then
a2 + mbc b2 + mca c 2 + mab 3(1 + m)
+ + ≤ ,
9 + kbc 9 + kca 9 + kab 9+k

with equality for a = b = c = 1. If m = m1 , then the equality holds also for


p p
3k + 6 − 2 3(3 + k) 3(3 + k) − 3
a= , b=c= .
k k
The inequality in P 1.127 is a particular case of this result (for k = 9 and m =
m1 = 47/7).

P 1.128. Let a, b, c be nonnegative real numbers such that a + b + c = 3. Prove that

26 − 7a2 26 − 7b2 26 − 7c 2 57
+ + ≤ .
1 + bc 1 + ca 1 + ab 2
(Vasile Cîrtoaje, 2011)

Solution. Use the SOS method. Write the inequality as follows:


X  19 26 − 7a2
‹
− ≥ 0,
2 1 + bc
X 14a2 + 19bc − 33
≥ 0,
1 + bc
X 42a2 + 57bc − 11(a + b + c)2
≥ 0,
1 + bc
X 11(2a2 − b2 − c 2 ) + 9(a − b)(a − c) − 13(ab − 2bc + ca)
≥ 0,
1 + bc
X 22(a − b)(a + b) + 9(a − b)(a − c) − 26c(a − b)
+
1 + bc
X 22(a − c)(a + c) + 9(a − c)(a − b) − 26b(a − c)
+ ≥ 0,
1 + bc
X (a − b)(31a + 22b − 35c) X (a − c)(31a + 22c − 35b)
+ ≥ 0,
1 + bc 1 + bc
X (a − b)(31a + 22b − 35c) X (b − a)(31b + 22a − 35c)
+ ≥ 0,
1 + bc 1 + ca
X (a − b)2 [9 + 31c(a + b) − 35c 2 ]
≥ 0,
(1 + bc)(1 + ca)
X
(a − b)2 (1 + ab)(1 + 11c)(3 − 2c) ≥ 0.
172 Vasile Cîrtoaje

Assume that a ≥ b ≥ c. Since 3 − 2c > 0, it suffices to show that

(b − c)2 (1 + bc)(1 + 11a)(3 − 2a) + (c − a)2 (1 + ab)(1 + 11b)(3 − 2b) ≥ 0;

that is,

(a − c)2 (1 + ab)(1 + 11b)(3 − 2b) ≥ (b − c)2 (1 + bc)(1 + 11a)(2a − 3).

Since 3 − 2b = a − b + c ≥ 0, we get this inequality by multiplying the inequalities

3 − 2b ≥ 2a − 3,

a(1 + ab) ≥ b(1 + bc),

a(1 + 11b) ≥ b(1 + 11a),

b2 (a − c)2 ≥ a2 (b − c)2 .
The equality holds for a = b = c = 1, and for a = b = 3/2 and c = 0 (or any cyclic
permutation).
Remark. The following more general statement holds:
• Let a, b, c be nonnegative real numbers such that a + b + c = 3. If

(3 + k)(4 + k)
k > 0, m ≤ m2 , m2 = ,
2(3 + 2k)

then
a2 + mbc b2 + mca c 2 + mab 3(1 + m)
+ + ≥ ,
9 + kbc 9 + kca 9 + kab 9+k

with equality for a = b = c = 1. When m = m2 , the equality holds also for a = 0 and
b = c = 3/2 (or any cyclic permutation).
The inequalities in P 1.128 is a particular cases of this result (for k = 9 and
m = m2 = 26/7).

P 1.129. If a, b, c are nonnegative real numbers, then


X 5a(b + c) − 6bc
≤ 3.
a2 + b2 + c 2 + bc

(Vasile Cîrtoaje, 2010)


Symmetric Rational Inequalities 173

First Solution. Apply the SOS method. If two of a, b, c are zero, then the inequality
is trivial. Consider further that

a2 + b2 + c 2 = 1, a ≥ b ≥ c, b > 0,

and write the inequality as follows:


X• 5a(b + c) − 6bc
˜
1− 2 ≥ 0,
a + b2 + c 2 + bc
X a2 + b2 + c 2 − 5a(b + c) + 7bc
≥ 0,
a2 + b2 + c 2 + bc
X (7b + 2c − a)(c − a) − (7c + 2b − a)(a − b)
≥ 0,
1 + bc
X (7c + 2a − b)(a − b) X (7c + 2b − a)(a − b)
− ≥ 0,
1 + ca 1 + bc
X
(a − b)2 (1 + ab)(3 + ac + bc − 7c 2 ) ≥ 0.
Since
3 + ac + bc − 7c 2 = 3a2 + 3b2 + ac + bc − 4c 2 > 0,
it suffices to prove that

(1 + bc)(3 + ab + ac − 7a2 )(b − c)2 + (1 + ac)(3 + ab + bc − 7b2 )(a − c)2 ≥ 0.

Since
3 + ab + ac − 7b2 = 3(a2 − b2 ) + 3c 2 + b(a − b) + bc ≥ 0
and 1 + ac ≥ 1 + bc, it is enough to show that

(3 + ab + ac − 7a2 )(b − c)2 + (3 + ab + bc − 7b2 )(a − c)2 ≥ 0.

From b(a − c) ≥ a(b − c) ≥ 0, we get b2 (a − c)2 ≥ a2 (b − c)2 , hence

b(a − c)2 ≥ a(b − c)2 .

Thus, it suffices to show that

b(3 + ab + ac − 7a2 ) + a(3 + ab + bc − 7b2 ) ≥ 0.

This is true if
b(3 + ab − 7a2 ) + a(3 + ab − 7b2 ) ≥ 0.
Indeed,

b(3 + ab − 7a2 ) + a(3 + ab − 7b2 ) = 3(a + b)(1 − 2ab) ≥ 0,

since
1 − 2ab = (a − b)2 + c 2 ≥ 0.
174 Vasile Cîrtoaje

The equality holds for a = b = c, and for a = b and c = 0 (or any cyclic permuta-
tion).
Second Solution. Without loss of generality, assume that a2 + b2 + c 2 = 1 and
a ≤ b ≤ c. Setting

p = a + b + c, q = ab + bc + ca, r = abc,

the inequality becomes


X 5q − 11bc
≤ 3,
1 + bc
Y X
3 (1 + bc) + (11bc − 5q)(1 + ca)(1 + ab) ≥ 0,

3(1 + q + pr + r 2 ) + 11(q + 2pr + 3r 2 ) − 5q(3 + 2q + pr) ≥ 0,

36r 2 + 5(5 − q)pr + 3 − q − 10q2 ≥ 0.

According to P 3.57-(a) in Volume 1, for fixed p and q, the product r = abc is


minimum when b = c or a = 0. Therefore, since 5 − q ≥ 4 > 0, it suffices to prove
the original homogeneous inequality for a = 0, and for b = c = 1. For a = 0, the
original inequality becomes

−6bc 10bc
+ 2 ≤ 3,
b2 + c + bc b + c 2
2

(b − c)2 (3b2 + 5bc + 3b2 ) ≥ 0,

while for b = c = 1, the original inequality becomes

10a − 6 5−a
+ 2 ≤ 3,
a2 + 3 a2 + a + 2

which is equivalent to
a(3a + 1)(a − 1)2 ≥ 0.

Remark. Similarly, we can prove the following generalization:


• Let a, b, c be nonnegative real numbers. If k > 0, then
X (2k + 3)a(b + c) + (k + 2)(k − 3)bc
≤ 3k,
a2 + b2 + c 2 + kbc

with equality for a = b = c, and for a = 0 and b = c (or any cyclic permutation).
Symmetric Rational Inequalities 175

P 1.130. Let a, b, c be nonnegative real numbers, no two of which are zero, and let

a2 + b2 + c 2
x= .
ab + bc + ca
Prove that

a b c 1 1
(a) + + + ≥x+ ;
b+c c+a a+b 2 x

4
 ‹
a b c
(b) 6 + + ≥ 5x + ;
b+c c+a a+b x

3 1 1
 ‹
a b c
(c) + + − ≥ x− .
b+c c+a a+b 2 3 x

(Vasile Cîrtoaje, 2011)

Solution. We will prove the more general inequality

2a 2b 2c 2(1 − k)
+ + + 1 − 3k ≥ (2 − k)x + ,
b+c c+a a+b x
where p
21 + 6 6
0 ≤ k ≤ k0 , k0 = ≈ 1.428.
25
For k = 0, k = 1/3 and k = 4/3, we get the inequalities in (a), (b) and (c),
respectively. Let p = a + b + c and q = ab + bc + ca. Since x = (p2 − 2q)/q, we can
write the inequality as follows:

a b c
+ + ≥ f (p, q),
b+c c+a a+b
X a 
+ 1 ≥ 3 + f (p, q),
b+c
p(p2 + q)
≥ 3 + f (p, q).
pq − abc
According to P 3.57-(a) in Volume 1, for fixed p and q, the product abc is minimum
when b = c or a = 0. Therefore, it suffices to prove the inequality for a = 0, and for
b = c = 1. For a = 0, using the substitution y = b/c + c/b, the desired inequality
becomes
2(1 − k)
2 y + 1 − 3k ≥ (2 − k) y + ,
y
( y − 2)[k( y − 1) + 1]
≥ 0.
y
176 Vasile Cîrtoaje

Since y ≥ 2, this inequality is clearly true. For b = c = 1, the desired inequality


becomes
4 (2 − k)(a2 + 2) 2(1 − k)(2a + 1)
a+ + 1 − 3k ≥ + ,
a+1 2a + 1 a2 + 2
which is equivalent to

a(a − 1)2 [ka2 + 3(1 − k)a + 6 − 4k] ≥ 0.


p
For 0 ≤ k ≤ 1, this is obvious, and for 1 < k ≤ (21 + 6 6)/25, we have
Æ
ka2 + 3(1 − k)a + 6 − 4k ≥ [2 k(6 − 4k) + 3(1 − k)]a ≥ 0.

The equality holds for a = b = c, and for a = 0 and p b = c (or any cyclic permuta-
tion). If k = k0 , then the equality holds also for (2 + 6)a = 2b = 2c (or any cyclic
permutation).

P 1.131. If a, b, c are real numbers, then


1 1 1 9
+ + ≤ .
a2 + 7(b2 + c 2 ) b2 + 7(c 2 + a2 ) c 2 + 7(a2 + b2 ) 5(a + b + c)2
(Vasile Cîrtoaje, 2008)
Solution. We use the highest coefficient method. Let

p = a + b + c, q = ab + bc + ca.

Write the inequality as f6 (a, b, c) ≥ 0, where


Y X
f6 (a, b, c) = 9 (a2 + 7b2 + 7c 2 ) − 5p2 (b2 + 7c 2 + 7a2 )(c 2 + 7a2 + 7b2 ).

Since Y Y
(a2 + 7b2 + 7c 2 ) = [7(p2 − 2q) − 6a2 ],
f6 (a, b, c) has the highest coefficient

A = 9(−6)3 < 0.

According to P 2.75 in Volume 1, it suffices to prove the original inequality for


b = c = 1, when the inequality reduces to
1 2 9
+ 2 ≤ ,
a2 + 14 7a + 8 5(a + 2)2
(a − 1)2 (a − 4)2 ≥ 0.
Thus, the proof is completed. The equality holds for a = b = c, and for a/4 = b = c
(or any cyclic permutation).
Symmetric Rational Inequalities 177

P 1.132. If a, b, c are real numbers, then

bc ca ab 3
+ + ≤ .
3a2 + b2 + c 2 3b2 + c 2 + a2 3c 2 + a2 + b2 5
(Vasile Cîrtoaje and Pham Kim Hung, 2005)

Solution. Use the highest coefficient method. Write the inequality as f6 (a, b, c) ≥ 0,
where
Y X
f6 (a, b, c) = 3 (3a2 + b2 + c 2 ) − 5 bc(3b2 + c 2 + a2 )(3c 2 + a2 + b2 ).

Let
p = a + b + c, q = ab + bc + ca.
From
Y X
f6 (a, b, c) = 3 (2a + p − 2q) − 5
2 2
bc(2b2 + p2 − 2q)(2c 2 + p2 − 2q),

it follows that f6 (a, b, c) has the same highest coefficient A as


X
24a2 b2 c 2 − 20 b3 c 3 ;

that is,
A = 24 − 60 < 0.
According to P 2.75 in Volume 1, it suffices to prove the original inequality for
b = c = 1, when the inequality is equivalent to

1 2a 3
+ 2 ≤ ,
3a2 +2 a +4 5

(a − 1)2 (3a − 2)2 ≥ 0.


Thus, the proof is completed. The equality holds for a = b = c, and for 3a/2 = b =
c (or any cyclic permutation).
Remark. The inequality in P 1.132 is a particular case (k = 3) of the following
more general result (Vasile Cîrtoaje, 2008):
• Let a, b, c be real numbers. If k > 1, then
X k(k − 3)a2 + 2(k − 1)bc 3(k + 1)(k − 2)
≤ ,
ka2 + b2 + c2 k+2
with equality for a = b = c, and for ka/2 = b = c (or any cyclic permutation).
178 Vasile Cîrtoaje

P 1.133. If a, b, c are real numbers such that a + b + c = 3, then


1 1 1 1
+ + ≤ .
8 + 5(b + c ) 8 + 5(c + a ) 8 + 5(a + b ) 6
2 2 2 2 2 2

(Vasile Cîrtoaje, 2006)

Solution. Use the highest coefficient method. Denote

p = a + b + c, q = ab + bc + ca,

and write the inequality in the homogeneous form


1 1 1 1
+ 2 + 2 ≤ 2,
8p2 + 45(b + c ) 8p + 45(c + a ) 8p + 45(a + b ) 6p
2 2 2 2 2 2

which is equivalent to f6 (a, b, c) ≥ 0, where


Y
f6 (a, b, c) = (53p2 − 90q − 45a2 )
X
− 6p2 (53p2 − 90q − 45b2 )(53p2 − 90q − 45c 2 ).

Clearly, f6 (a, b, c) has the highest coefficient

A = (−45)3 < 0.

According to P 2.75 in Volume 1, it suffices to prove the homogeneous inequality


for b = c = 1; that is,
1 2 1
+ ≤ .
8(a + 2) + 90 8(a + 2) + 45(1 + a ) 6(a + 2)2
2 2 2

Using the substitution


a + 2 = 3x,
the inequality becomes as follows:
1 2 1
+ ≤ ,
72x 2 + 90 72x + 45 + 45(3x − 2) ) 54x 2
2 2

1 2 1
+ ≤ ,
8x 2 + 10 53x 2 − 60x + 25 6x 2
x 4 − 12x 3 + 46x 2 − 60x + 25 ≥ 0,
(x − 1)2 (x − 5)2 ≥ 0,
(a − 1)2 (a − 13)2 ≥ 0.
The equality holds for a = b = c = 1, and for a = 13/5 and b = c = 1/5 (or any
cyclic permutation).
Symmetric Rational Inequalities 179

P 1.134. If a, b, c are real numbers, then

(a + b)(a + c) (b + c)(b + a) (c + a)(c + b) 4


+ 2 + 2 ≤ .
a + 4(b + c ) b + 4(c + a ) c + 4(a + b ) 3
2 2 2 2 2 2 2

(Vasile Cîrtoaje, 2008)

Solution. Use the highest coefficient method. Let

p = a + b + c, q = ab + bc + ca.

Write the inequality as f6 (a, b, c) ≥ 0, where


Y
f6 (a, b, c) = 4 (a2 + 4b2 + 4c 2 )
X
(a + b)(a + c)(b2 + 4c 2 + 4a2 )(c 2 + 4a2 + 4b2 )
−3
Y X
=4 (4p − 8q − 3a ) − 3
2 2
(a2 + q)(4p2 − 8q − 3b2 )(4p2 − 8q − 3c 2 ).
Thus, f6 (a, b, c) has the highest coefficient

A = 4(−3)3 − 34 < 0.

By P 2.75 in Volume 1, it suffices to prove the original inequality for b = c = 1,


when the inequality is equivalent to

(a + 1)2 4(a + 1) 4
+ ≤ ,
a2 + 8 4a2 + 5 3

(a − 1)2 (2a − 7)2 ≥ 0.


The equality holds for a = b = c, and for 2a/7 = b = c (or any cyclic permutation).

P 1.135. Let a, b, c be nonnegative real numbers, no two of which are zero. Prove
that X 1 1
≤ .
(b + c)(7a + b + c) 2(ab + bc + ca)
(Vasile Cîrtoaje, 2009)

First Solution. Write the inequality as


X• 4(ab + bc + ca)
˜
1− ≥ 1,
(b + c)(7a + b + c)
X (b − c)2 + 3a(b + c)
≥ 1.
(b + c)(7a + b + c)
180 Vasile Cîrtoaje

By the Cauchy-Schwarz inequality, we have


X (b − c)2 + 3a(b + c) 4(a + b + c)4
≥P .
(b + c)(7a + b + c) [(b − c)2 + 3a(b + c)](b + c)(7a + b + c)

Therefore, it suffices to show that


X
4(a + b + c)4 ≥ (b2 + c 2 − 2bc + 3ca + 3ab)(b + c)(7a + b + c).

Write this inequality as


X X X X
a + abc
4
a+3 ab(a + b ) − 8
2 2
a2 b2 ≥ 0,
X X X X
a4 + abc a− ab(a2 + b2 ) + 4 ab(a − b)2 ≥ 0.
Since X X X
a4 + abc a− ab(a2 + b2 ) ≥ 0
(Schur’s inequality of degree four), the conclusion follows. The equality holds for
a = b = c, and also for a = 0 and b = c (or any cyclic permutation).
Second Solution. Use the highest coefficient method. We need to prove that f6 (a, b, c) ≥
0, where Y
f6 (a, b, c) = (b + c)(7a + b + c)
X
−2(ab + bc + ca) (a + b)(a + c)(7b + c + a)(7c + a + b).
Let p = a + b + c. Clearly, f6 (a, b, c) has the same highest coefficient A as f (a, b, c),
where Y Y
f (a, b, c) = (b + c)(7a + b + c) = (p − a)(p + 6a);
that is,
A = (−6)3 < 0.
Thus, by P 3.76-(a) in Volume 1, it suffices to prove the original inequality for
b = c = 1, and for a = 0.
For b = c = 1, the inequality reduces to
1 2 1
+ ≤ ,
2(7a + 2) (a + 1)(a + 8) 2(2a + 1)

a(a − 1)2 ≥ 0.
For a = 0, the inequality can be written as
1 1 1 1
+ + ≤ ,
(b + c)2 c(7b + c) b(7c + b) 2bc

1 b2 + c 2 + 14bc 1
+ ≤ ,
(b + c)2 bc[7(b2 + c 2 ) + 50bc] 2bc
Symmetric Rational Inequalities 181

1 x + 14 1
+ ≤ ,
x + 2 7x + 50 2
where
b c
x= + , x ≥ 2.
c b
This reduces to the obvious inequality

(x − 2)(5x + 28) ≥ 0.

P 1.136. Let a, b, c be nonnegative real numbers, no two of which are zero. Prove
that X 1 9
≤ .
b + c + 4a(b + c) 10(ab + bc + ca)
2 2

(Vasile Cîrtoaje, 2009)

Solution. Use the highest coefficient method. Let

p = a + b + c, q = ab + bc + ca.

We need to prove that f6 (a, b, c) ≥ 0, where


Y
f6 (a, b, c) = 9 [b2 + c 2 + 4a(b + c)]
X
−10(ab + bc + ca) [a2 + b2 + 4c(a + b)][a2 + c 2 + 4b(a + c)]
Y X
=9 (p2 + 2q − a2 − 4bc) − 10q (p2 + 2q − c 2 − 4ab)(p2 + 2q − b2 − 4ca).
Clearly, f6 (a, b, c) has the same highest coefficient A as P3 (a, b, c), where
Y
P3 (a, b, c) = −9 (a2 + 4bc).

According to Remark 2 from the proof of P 2.75 in Volume 1,

A = P3 (1, 1, 1) = −9 · 125 < 0.

Thus, by P 3.76-(a) in Volume 1, it suffices to prove the original inequality for


b = c = 1, and for a = 0.
For b = c = 1, the inequality reduces to

1 2 9
+ 2 ≤ ,
2(4a + 1) a + 4a + 5 10(2a + 1)

a(a − 1)2 ≥ 0.
182 Vasile Cîrtoaje

For a = 0, the inequality becomes

1 1 1 9
+ 2 + 2 ≤ ,
b2 +c 2 b + 4bc c + 4bc 10bc

1 b2 + c 2 + 8bc 9
+ ≤ .
b +c
2 2 4bc(b + c ) + 17b c
2 2 2 2 10bc
1 x +8 9
+ ≤ ,
x 4x + 17 10
(x − 2)(26x + 85) ≥ 0,
where
b c
x= + , x ≥ 2.
c b
The equality holds for a = b = c, and also for a = 0 and b = c (or any cyclic
permutation).

P 1.137. Let a, b, c be nonnegative real numbers, no two of which are zero. If a + b +


c = 3, then
1 1 1 9
+ + ≤ .
3 − ab 3 − bc 3 − ca 2(ab + bc + ca)
(Vasile Cîrtoaje, 2011)

First Solution. We apply the SOS method. Write the inequality as


X  3 ab + bc + ca ‹
− ≥ 0.
2 3 − bc
X 9 − 2a(b + c) − 5bc
≥ 0,
3 − bc
X a2 + b2 + c 2 − 3bc
≥ 0.
3 − bc
Since

2(a2 + b2 + c 2 − 3bc) = 2(a2 − bc) + 2(b2 + c 2 − ab − ac) + 2(ab + ac − 2bc)

= (a − b)(a + c) + (a − c)(a + b) − 2b(a − b) − 2c(a − c) + 2c(a − b) + 2b(a − c)


= (a − b)(a − 2b + 3c) + (a − c)(a − 2c + 3b),
the required inequality is equivalent to
X (a − b)(a − 2b + 3c) + (a − c)(a − 2c + 3b)
≥ 0,
3 − bc
Symmetric Rational Inequalities 183

X (a − b)(a − 2b + 3c) X (b − a)(b − 2a + 3c)


+ ≥ 0,
3 − bc 3 − ca
X (a − b)2 [9 − c(a + b + 3c)]
≥ 0,
(3 − bc)(3 − ca)
X
(a − b)2 (3 − ab)(3 + c)(3 − 2c) ≥ 0.
Without loss of generality, assume that a ≥ b ≥ c. It suffices to prove that

(b − c)2 (3 − bc)(3 + a)(3 − 2a) + (c − a)2 (3 − ca)(3 + b)(3 − 2b) ≥ 0,

which is equivalent to

(a − c)2 (3 − ac)(3 + b)(3 − 2b) ≥ (b − c)2 (3 − bc)(a + 3)(2a − 3).

Since 3 − 2b = a − b + c ≥ 0, we can obtain this inequality by multiplying the


inequalities
b2 (a − c)2 ≥ a2 (b − c)2 ,
a(3 − ac) ≥ b(3 − bc),
a(3 + b)(3 − 2b) ≥ b(a + 3)(2a − 3) ≥ 0.
We have

a(3 − ac) − b(3 − bc) = (a − b)[3 − c(a + b)] = (a − b)(3 − 3c + c 2 )


≥ (a − b)(3 − 3c) ≥ 0.

Also, since a + b ≤ a + b + c = 3, we have

a(3 + b)(3 − 2b) − b(a + 3)(2a − 3) = 9(a + b) − 6ab − 2ab(a + b)

≥ 9(a + b) − 12ab ≥ 3(a + b)2 − 12ab = 3(a − b)2 ≥ 0.


The equality holds for a = b = c = 1, and for a = 0 and b = c = 3/2 (or any cyclic
permutation).
Second Solution. Write the inequality in the homogeneous form
1 1 1 3
+ 2 + 2 ≤ ,
p2 − 3ab p − 3bc p − 3ca 2q
where
p = a + b + c, q = ab + bc + ca.
We need to prove that f6 (a, b, c) ≥ 0, where
Y X
f6 (a, b, c) = 3 (p2 − 3bc) − 2q (p2 − 3ca)(p2 − 3ab).

Clearly, f6 (a, b, c) has the highest coefficient

A = 3(−3)3 < 0.
184 Vasile Cîrtoaje

Thus, by P 3.76-(a) in Volume 1, it suffices to prove the homogeneous inequality


for b = c = 1, and for a = 0.
For b = c = 1, the homogeneous inequality reduces to

2 1 3
+ ≤ ,
(a + 2)2 − 3a (a + 2)2 − 3 2(2a + 1)

a2 + 3a + 2 3
≤ ,
(a + a + 4)(a + 4a + 1) 2(2a + 1)
2 2

a(a + 3)(a − 1)2 ≥ 0.


For a = 0, the homogeneous inequality can be written as

2 1 3
+ ≤ ,
(b + c)2 (b + c) − 3bc
2 2bc

(b − c)2 (b2 + c 2 + bc)


≥ 0.
2bc(b + c)2 (b2 + c 2 − bc)

P 1.138. If a, b, c are nonnegative real numbers such that a + b + c = 3, then

bc ca ab 3
+ 2 + 2 ≤ .
a2 +a+6 b + b+6 c +c+6 8
(Vasile Cîrtoaje, 2009)

Solution. Write the inequality in the homogeneous form

bc ca ab 1
+ + ≤ , p = a + b + c.
3a2 + ap + 2p2 3b2 + bp + 2p2 3c 2 + cp + 2p2 8

We need to prove that f6 (a, b, c) ≥ 0, where


Y X
f6 (a, b, c) = (3a2 + ap + 2p2 ) − 8 bc(3b2 + bp + 2p2 )(3c 2 + cp + 2p2 ).

Clearly, f6 (a, b, c) has the same highest coefficient as


X
27a2 b2 c 2 − 72 b3 c 3 ;

that is,
A = 27 − 216 < 0.
By P 3.76-(a) in Volume 1, it suffices to prove the homogeneous inequality for
b = c = 1, and for a = 0.
Symmetric Rational Inequalities 185

For b = c = 1, the homogeneous inequality reduces to


1 2a 1
+ ≤ ,
2(3a2 + 5a + 4) 2a2 + 9a + 13 8

6a4 − 11a3 + 4a2 + a ≥ 0,


a(6a + 1)(a − 1)2 ≥ 0.
For a = 0, the homogeneous inequality can be written as
bc 1
≤ ,
2(b + c)2 8

(b − c)2 ≥ 0.
The equality holds for a = b = c = 1, and for a = 0 and b = c = 3/2 (or any cyclic
permutation).

P 1.139. If a, b, c are nonnegative real numbers such that ab + bc + ca = 3, then


1 1 1 1
+ 2 + 2 ≥ .
8a2 − 2bc + 21 8b − 2ca + 21 8c − 2ab + 21 9
(Michael Rozenberg, 2013)

Solution. Write the inequality in the homogeneous form


1 1 1 1
+ 2 + 2 ≥ , q = ab + bc + ca.
8a2 − 2bc + 7q 8b − 2ca + 7q 8c − 2ab + 7q 3q
We need to prove that f6 (a, b, c) ≥ 0, where
X Y
f6 (a, b, c) = 3q (8b2 − 2ca + 7q)(8c 2 − 2ab + 7q) − (8a2 − 2bc + 7q).

Clearly, f6 (a, b, c) has the same highest coefficient as P2 (a, b, c), where
Y
P2 (a, b, c) = − (8a2 − 2bc).

According to Remark 2 from the proof of P 2.75 in Volume 1, we have

A = P2 (1, 1, 1) = −63 < 0.

By P 3.76-(a) in Volume 1, it suffices to prove the homogeneous inequality for


b = c = 1, and for a = 0.
For b = c = 1, the homogeneous inequality reduces to
1 2 1
+ ≥ ,
8a2 + 14a + 5 12a + 15 3(2a + 1)
186 Vasile Cîrtoaje

1 2 1
+ ≥ ,
(4a + 5)(2a + 1) 3(4a + 5) 3(2a + 1)
which is an identity.
For a = 0, the homogeneous inequality can be written as

1 1 2
+ ≥ ,
b(8b + 7c) c(8c + 7b) 15bc

c b 2
+ ≥ ,
8b + 7c 8c + 7b 15
(b − c)2 ≥ 0.
The equality holds when two of a, b, c are equal.
Remark. The following identity holds for ab + bc + ca = 3:

8 (a − b)2
Q
X 9
−1= Q .
8a2 − 2bc + 21 (8a2 − 2bc + 21)

P 1.140. Let a, b, c be real numbers, no two of which are zero. Prove that

a2 + bc b2 + ca c 2 + ab (a + b + c)2
(a) + + ≥ ;
b2 + c 2 c 2 + a2 a2 + b2 a2 + b2 + c 2

a2 + 3bc b2 + 3ca c 2 + 3ab 6(ab + bc + ca)


(b) + 2 + 2 ≥ .
b2 + c 2 c + a2 a + b2 a2 + b2 + c 2
(Vasile Cîrtoaje, 2014)

Solution. (a) Using the known inequality


X a2 3

b +c
2 2 2
and the Cauchy-Schwarz inequality yields
X a2 + bc X a2 X bc X 1 bc
‹
= + ≥ +
b2 + c 2 b2 + c 2 b2 + c 2 2 b2 + c 2
P 2
X (b + c)2 (b + c) (a + b + c)2
= ≥ = .
2(b2 + c 2 ) 2(b2 + c 2 ) a2 + b2 + c 2
P

The equality holds for a = b = c.


Symmetric Rational Inequalities 187

(b) We have
X a2 + 3bc X a2 X 3bc 3 X 3bc
= + ≥ +
b2 + c 2 b2 + c 2 b2 + c 2 2 b2 + c 2
X 1 (b + c)2
‹
bc X
= −3 + 3 + 2 = −3 + 3
2 b + c2 2(b2 + c 2 )
P 2 P 2
3 (b + c) 3 a 6(ab + bc + ca)
≥ −3 + P = −3 + P = .
2(b2 + c 2 ) a2 a2 + b2 + c 2

The equality holds for a = b = c.

P 1.141. Let a, b, c be real numbers such that ab + bc + ca ≥ 0 and no two of which


are zero. Prove that
a(b + c) b(c + a) c(a + b) 3
+ + ≥ .
b2 + c 2 c 2 + a2 a2 + b2 10
(Vasile Cîrtoaje, 2014)

Solution. Since the problem remains unchanged by replacing a, b, c with −a, −b, −c,
it suffices to consider the cases a, b, c ≥ 0 and a < 0, b ≥ 0, c ≥ 0.
Case 1: a, b, c ≥ 0. We have
X a(b + c) X a(b + c)

b2 + c 2 (b + c)2
X a 3 3
= ≥ > .
b+c 2 10

Case 2: a < 0, b ≥ 0, c ≥ 0. Replacing a by −a, we need to show that

b(c − a) c(b − a) a(b + c) 3


+ 2 − 2 ≥ ,
a +c
2 2 a +b 2 b +c 2 10
where
bc
a, b, c ≥ 0, a≤ .
b+c
We show first that
b(c − a) b(c − x)
≥ 2 ,
a +c
2 2 x + c2
bc
where x = , x ≥ a. This is equivalent to
b+c

b(x − a)[(c − a)x + ac + c 2 ] ≥ 0,


188 Vasile Cîrtoaje

which is true because


c 2 (a + 2b + c)
(c − a)x + ac + c 2 = ≥ 0.
b+c
Similarly, we can show that
c(b − a) c(b − x)
≥ 2 .
a2 + b2 x + b2
In addition, since
a(b + c) x(b + c)
≤ 2 .
b +c
2 2 b + c2
it suffices to prove that
b(c − x) c(b − x) x(b + c) 3
+ 2 − 2 ≥ .
x +c
2 2 x +b 2 b +c 2 10
Denote
b c
p= , q= , p + q = 1.
b+c b+c
Since
b(c − x) p c(b − x) q
= , = ,
x 2 + c2 1 + p2 x 2 + b2 1 + q2
x(b + c) bc pq
= 2 = ,
b +c
2 2 b +c 2 1 − 2pq
we need to show that
p q pq 3
+ − ≥ .
1+p 2 1+q 2 1 − 2pq 10
This inequality is equivalent to
1 + pq pq 3
− ≥ ,
2 − 2pq + p q
2 2 1 − 2pq 10
(pq + 2)2 (1 − 4pq) ≥ 0.
Since
1 − 4pq = (p + q)2 − 4pq = (p − q)2 ≥ 0,
the proof is completed. The equality holds for −2a = b = c (or any cyclic permu-
tation).

P 1.142. If a, b, c are positive real numbers such that abc > 1, then
1 1 4
+ ≥ .
a + b + c − 3 abc − 1 ab + bc + ca − 3
(Vasile Cîrtoaje, 2011)
Symmetric Rational Inequalities 189

Solution (by Vo Quoc Ba Can). By the AM-GM inequality, we have


p
3
a + b + c ≥ 3 abc > 3,
p
3
ab + bc + ca ≥ a2 b2 c 2 > 3.
Without loss of generality, assume that a = min{a, b, c}. By the Cauchy-Schwarz
inequality, we have

1 1 abc − 1 p 1 2
 ‹• ˜  ‹
+ a(a + b + c − 3) + ≥ a+ p .
a + b + c − 3 abc − 1 a a
Therefore, it suffices to prove that
abc − 1
a(a + b + c − 3) +
(a + 1)2
a
≥ .
4a ab + bc + ca − 3
Since
abc − 1 (a − 1)3
a(a + b + c − 3) + = ab + bc + ca − 3 + ,
a a
this inequality can be written as follows:

(a + 1)2 (a − 1)3
−1≥ ,
4a a(ab + bc + ca − 3)

(a − 1)2 (a − 1)3
≥ ,
4a a(ab + bc + ca − 3)
(a − 1)2 (ab + bc + ca + 1 − 4a) ≥ 0.
This is true since Æ
3
bc ≥ (abc)2 > 1,
hence
ab + bc + ca + 1 − 4a > a2 + 1 + a2 + 1 − 4a = 2(a − 1)2 ≥ 0.
The equality holds for a = b = 1 and c > 1 (or any cyclic permutation).
Remark. Using this inequality, we can prove P 3.84 in Volume 1, which states that
1 1 1 1
 ‹
(a + b + c − 3) + + − 3 + abc + ≥2
a b c abc
for any positive real numbers a, b, c. This inequality is clearly true for abc = 1.
In addition, it remains unchanged by substituting a, b, c with 1/a, 1/b, 1/c, re-
spectively.
p Therefore, it suffices to consider the case abc > 1. Since a + b + c ≥
3
3 a bc > 3, we can write the required inequality as E ≥ 0, where

(abc − 1)2
E = ab + bc + ca − 3abc + .
a+ b+c−3
190 Vasile Cîrtoaje

According to the inequality in P 1.142, we have


4 1
 ‹
E ≥ ab + bc + ca − 3abc + (abc − 1) 2

ab + bc + ca − 3 abc − 1
4(abc − 1)2
= (ab + bc + ca − 3) + − 4(abc − 1)
ab + bc + ca − 3
v
t 4(abc − 1)2
≥ 2 (ab + bc + ca − 3) · − 4(abc − 1) = 0.
ab + bc + ca − 3

P 1.143. Let a, b, c be nonnegative real numbers, no two of which are zero. Prove
that
X (4b2 − ac)(4c 2 − ab) 27
≤ abc.
b+c 2
(Vasile Cîrtoaje, 2011)
Solution. Use the SOS method. Since
X (4b2 − ac)(4c 2 − ab) X bc(16bc + a2 ) X a(b3 + c 3 )
= −4
b+c b+c b+c
X bc(16bc + a2 ) X
= −4 a(b2 + c 2 ) + 12abc
b+c
X • a2 16bc
˜
= bc + − 4(b + c) + 12abc
b+c b+c
X • a2 (b − c)2
˜
= bc −4 + 12abc
b+c b+c
we can write the inequality as follows:
X •a a2 4(b − c)2
˜
bc − + ≥ 0,
2 b+c b+c
X bc(b − c)2 X 2a − b − c
8 ≥ abc .
b+c b+c
In addition, since
X 2a − b − c X (a − b) + (a − c) Xa−b X b−a
= = +
b+c b+c b+c c+a
X (a − b)2 X (b − c)2
= = ,
(b + c)(c + a) (c + a)(a + b)
the inequality can be restated as
X bc(b − c)2 X (b − c)2
8 ≥ abc ,
b+c (c + a)(a + b)
Symmetric Rational Inequalities 191

X bc(b − c)2 (8a2 + 8bc + 7ab + 7ac)


≥ 0.
(a + b)(b + c)(c + a)
Since the last form is obvious, the proof is completed. The equality holds for a =
b = c, and also for a = 0 and b = c (or any cyclic permutation).

P 1.144. Let a, b, c be nonnegative real numbers, no two of which are zero, such that

a + b + c = 3.

Prove that
a b c 2
+ + ≥ .
3a + bc 3b + ca 3c + ab 3

Solution. Since

3a + bc = a(a + b + c) + bc = (a + b)(a + c),

we can write the inequality as follows:

2
a(b + c) + b(c + a) + c(a + b) ≥ (a + b)(b + c)(c + a),
3

6(ab + bc + ca) ≥ 2[(a + b + c)(ab + bc + ca) − abc],

2abc ≥ 0.

The equality holds for a = 0, or b = 0, or c = 0.

P 1.145. Let a, b, c be positive real numbers such that

1 1 1
 ‹
(a + b + c) + + = 10.
a b c

Prove that
19 a b c 5
≤ + + ≤ .
12 b+c c+a a+b 3
(Vasile Cîrtoaje, 2012)
192 Vasile Cîrtoaje

First Solution. Write the hypothesis

1 1 1
 ‹
(a + b + c) + + = 10
a b c
as
b+c c+a a+b
+ + =7
a b c
and
(a + b)(b + c)(c + a) = 9abc.
Using the substitution

b+c c+a a+b


x= , y= , z= ,
a b c
we need to show that x + y + z = 7 and x yz = 9 involve

19 1 1 1 5
≤ + + ≤ ,
12 x y z 3

or, equivalently,
19 1 x(7 − x) 5
≤ + ≤ .
12 x 9 3
Clearly, x, y, z ∈ (0, 7). The left inequality is equivalent to

(x − 4)(2x − 3)2 ≤ 0,

while the right inequality is equivalent to

(x − 1)(x − 3)2 ≥ 0.

These inequalities are true if 1 ≤ x ≤ 4. To show that 1 ≤ x ≤ 4, from ( y + z)2 ≥


4 yz, we get
36
(7 − x)2 ≥ ,
x
(x − 1)(x − 4)(x − 9) ≥ 0,
1 ≤ x ≤ 4.
Thus, the proof is completed. The left inequality is an equality for 2a = b = c (or
any cyclic permutation), and the right inequality is an equality for a/2 = b = c (or
any cyclic permutation).
Second Solution. Due to homogeneity, assume that b + c = 2; this involves bc ≤ 1.
From the hypothesis
1 1 1
 ‹
(a + b + c) + + = 10,
a b c
Symmetric Rational Inequalities 193

we get
2a(a + 2)
bc = .
9a − 2
Since
(a − 2)(2a − 1)
bc − 1 = ,
9a − 2
from the condition bc ≤ 1, we get

1
≤ a ≤ 2.
2
We have
b c a(b + c) + b2 + c 2 2a + 4 − 2bc
+ = 2 = 2
c+a a+b a + (b + c)a + bc a + 2a + bc
2(7a + 12a − 4) 2(7a − 2)
2
= = ,
9a2 (a + 2) 9a2

hence
a b c a 2(7a − 2) 9a3 + 28a − 8
+ + = + = .
b+c c+a a+b 2 9a2 18a2
Thus, we need to show that

19 9a3 + 28a − 8 5
≤ ≤ .
12 18a2 3
These inequalities are true, since the left inequality is equivalent to

(2a − 1)(3a − 4)2 ≥ 0,

and the right inequality is equivalent to

(a − 2)(3a − 2)2 ≤ 0.

Remark. Similarly, we can prove the following generalization.

• Let a, b, c be positive real numbers such that

1 1 1 8k2
 ‹
(a + b + c) + + =9+ ,
a b c 1 − k2
where k ∈ (0, 1). Then,

k2 a b c 3 k2
≤ + + − ≤ .
1+k b+c c+a a+ b 2 1−k
194 Vasile Cîrtoaje

P 1.146. Let a, b, c be nonnegative real numbers, no two of which are zero, such that
a + b + c = 3. Prove that
9 a b c
< + + ≤ 1.
10 2a + bc 2b + ca 2c + ab
(Vasile Cîrtoaje, 2012)

Solution. (a) Since


a 1 −bc
− = ,
2a + bc 2 2(2a + bc)
we can write the right inequality as
X bc
≥ 1.
2a + bc
According to the Cauchy-Schwarz inequality, we have
P 2
b c + 2abc a
P 2 2 P
X bc bc
≥P = = 1.
2a + bc bc(2a + bc) 6abc + b2 c 2
P

The equality holds for a = b = c = 1, and also for a = 0, or b = 0, or c = 0.


(b) First Solution. For the nontrivial case a, b, c > 0, we can write the left
inequality as
X 1 9
> .
bc 10
2+
a
Using the substitution
v s v
t bc ca t ab
x= , y= , z= ,
a b c

we need to show that X 1 9


>
2+ x 2 10
for all positive real numbers x, y, z satisfying x y + yz + z x = 3. By expanding, the
inequality becomes
X X
4 x 2 + 48 > 9x 2 y 2 z 2 + 8 x 2 y 2.

Since X €X Š2 X X
x y =
2 2
xy − 2x yz x = 9 − 2x yz x,
we can write the desired inequality as
X X
4 x 2 + 16x yz x > 9x 2 y 2 z 2 + 24,
Symmetric Rational Inequalities 195

which is equivalent to

4(p2 − 12) + 16x yzp > 9x 2 y 2 z 2 ,

where p = x + y + z. Using Schur’s inequality

p3 + 9x yz ≥ 4p(x y + yz + z x),

which is equivalent to
p(p2 − 12) ≥ −9x yz,
it suffices to prove that

36x yz
− + 16x yzp > 9x 2 y 2 z 2 .
p

This is true if
36
− + 16p > 9x yz.
p
Since Æ
x + y +z ≥ 3(x y + yz + z x) = 3
and
x y + yz + z x p
1=
3
≥ x 2 y 2z2,
3
we have
36 36
− + 16p − 9x yz ≥ − + 48 − 9 > 0.
p 3
Second Solution. As it is shown at the first solution, it suffices to show that
X 1 9
>
2+ x 2 10
for all positive real numbers x, y, z satisfying x y + yz + z x = 3. Rewrite this in-
equality as
X x2 6
< .
2 + x2 5
Let p and q be two positive real numbers such that
p
p + q = 3.

By the Cauchy-Schwarz inequality, we have

x2 3x 2 (px + qx)2
= =
2 + x2 2(x y + yz + z x) + 3x 2 2x(x + y + z) + (x 2 + 2 yz)

p2 x q2 x 2
≤ + 2 .
2(x + y + z) x + 2 yz
196 Vasile Cîrtoaje

Therefore,
X x2 X p2 x X q2 x 2 p2 X x2
≤ + = + q 2
.
2 + x2 2(x + y + z) x 2 + 2 yz 2 x 2 + 2 yz

Thus, it suffices to prove that

p2 X x2 6
+ q2 < .
2 x + 2 yz
2 5

We claim that
X x2
< 2.
x 2 + 2 yz
Under this assumption, we only need to show that

p2 6
+ 2q2 ≤ .
2 5
p p
4 3 3 p p2 6
Indeed, choosing p = and q = , we have p + q = 3 and + 2q2 = . To
5 5 2 5
x2
< 2,
P
complete the proof, we need to prove the homogeneous inequality
x 2 + 2 yz
which is equivalent to
X yz 1
> .
x + 2 yz
2 2
By the Cauchy-Schwarz inequality, we get
P 2
y z + 2x yz x
P 2 2 P
X yz yz 1
≥P = > .
x + 2 yz yz(x + 2 yz) x yz x + 2 y z
P P
2 2 2 2 2

P 1.147. Let a, b, c be nonnegative real numbers, no two of which are zero. Prove
that
a3 b3 c3 a3 + b3 + c 3
+ + ≤ .
2a2 + bc 2b2 + ca 2c 2 + ab a2 + b2 + c 2
(Vasile Cîrtoaje, 2011)

Solution. Use the SOS method. Write the inequality as follows:


X• a3 a3
˜
− ≥ 0,
a2 + b2 + c 2 2a2 + bc
X a3 (a2 + bc − b2 − c 2 )
≥ 0,
2a2 + bc
Symmetric Rational Inequalities 197

X a3 [a2 (b + c) − b3 − c 3 ]
≥ 0,
(b + c)(2a2 + bc)
X a3 b(a2 − b2 ) + a3 c(a2 − c 2 )
≥ 0,
(b + c)(2a2 + bc)
X a3 b(a2 − b2 ) X b3 a(b2 − a2 )
+ ≥ 0,
(b + c)(2a2 + bc) (c + a)(2b2 + ca)
X ab(a + b)(a − b)2 [2a2 b2 + c(a3 + a2 b + ab2 + b3 ) + c 2 (a2 + ab + b2 )]
≥ 0.
(b + c)(c + a)(2a2 + bc)(2b2 + ca)
The equality holds for a = b = c, and also for a = 0 and b = c (or any cyclic
permutation).

P 1.148. If a, b, c are positive real numbers, then

a3 b3 c3 a+b+c
+ + ≥ .
4a + bc 4b + ca 4c + ab
2 2 2 5
(Vasile Cîrtoaje, 2011)
Solution. Use the SOS method. Write the inequality as follows:
X  a3 a
‹
− ≥ 0,
4a2 + bc 5
X a(a2 − bc)
≥ 0,
4a2 + bc
X a[(a − b)(a + c) + (a − c)(a + b)]
≥ 0,
4a2 + bc
X a(a − b)(a + c) X b(b − a)(b + c)
+ ≥ 0,
4a2 + bc 4b2 + ca
X c(a − b)2 [(a − b)2 + bc + ca − ab]
≥ 0.
(4a2 + bc)(4b2 + ca)
Clearly, it suffices to show that
X c(a − b)2 (bc + ca − ab)
≥ 0,
(4a2 + bc)(4b2 + ca)
which can be written as
X
(a − b)2 (bc + ca − ab)(4c 3 + abc) ≥ 0.

Assume that a ≥ b ≥ c. Since ca + ab − bc > 0, it is enough to prove that

(c − a)2 (ab + bc − ca)(4b3 + abc) + (a − b)2 (bc + ca − ab)(4c 3 + abc) ≥ 0,


198 Vasile Cîrtoaje

which is equivalent to
(a − c)2 (ab + bc − ca)(4b3 + abc) ≥ (a − b)2 (ab − bc − ca)(4c 3 + abc).
This inequality is true since ab + bc − ca > 0 and
(a − c)2 ≥ (a − b)2 , 4b3 + abc ≥ 4c 3 + abc, ab + bc − ca ≥ ab − bc − ca.
The equality holds for a = b = c.

P 1.149. If a, b, c are positive real numbers, then


1 1 1 3
+ + ≥ .
(2 + a)2 (2 + b)2 (2 + c)2 6 + ab + bc + ca
(Vasile Cîrtoaje, 2013)
Solution. By the Cauchy-Schwarz inequality, we have
X 1 4(a + b + c)2
≥ .
(2 + a)2 (2 + a)2 (b + c)2
P

Thus, it suffices to show that


X
4(a + b + c)2 (6 + ab + bc + ca) ≥ 3 (2 + a)2 (b + c)2 .
This inequality is equivalent to
2p2 q − 3q2 + 3pr + 12q ≥ 6(pq + 3r),
where
p = a + b + c, q = ab + bc + ca, r = abc.
According to AM-GM inequality, we have
Æ
(2p2 q − 3q2 + 3pr) + 12q ≥ 2 12q(2p2 q − 3q2 + 3pr).
Therefore, it is enough to prove the homogeneous inequality
4q(2p2 q − 3q2 + 3pr) ≥ 3(pq + 3r)2 ,
which can be written as
5p2 q2 ≥ 12q3 + 6pqr + 27r 2 .
Since pq ≥ 9r, we have
3(5p2 q2 − 12q3 − 6pqr − 27r 2 ) ≥ 15p2 q2 − 36q3 − 2p2 q2 − p2 q2
= 12q2 (p2 − 3q) ≥ 0.
The equality holds for a = b = c = 1.
Symmetric Rational Inequalities 199

P 1.150. If a, b, c are positive real numbers, then


1 1 1 3
+ + ≥ .
1 + 3a 1 + 3b 1 + 3c 3 + abc
(Vasile Cîrtoaje, 2013)
Solution. Set
p
3
p = a + b + c, q = ab + bc + ca, r= abc,

and write the inequality as follows:


X
(3 + r 3 ) (1 + 3b)(1 + 3c) ≥ 3(1 + 3a)(1 + 3b)(1 + 3c),

(3 + r 3 )(3 + 6p + 9q) ≥ 3(1 + 3p + 9q + 27r 3 ),


r 3 (2p + 3q) + 2 + 3p ≥ 26r 3 .
By virtue of the AM-GM inequality, we have

p ≥ 3r, q ≥ 3r 2 .

Therefore, it suffices to show that

r 3 (6r + 9r 2 ) + 2 + 9r ≥ 26r 3 ,

which is equivalent to the obvious inequality

(r − 1)2 (9r 3 + 24r 2 + 13r + 2) ≥ 0.

The equality holds for a = b = c = 1.

P 1.151. Let a, b, c be real numbers, no two of which are zero. If 1 < k ≤ 3, then
2ab 2bc 2ca
 ‹ ‹ ‹
k+ 2 k+ 2 k+ 2 ≥ (k − 1)(k2 − 1).
a +b 2 b +c 2 c +a 2

(Vasile Cîrtoaje and Vo Quoc Ba Can, 2011)


Solution. If a, b, c have the same sign, then
2ab 2bc 2ca
 ‹ ‹ ‹
k+ 2 k+ 2 k+ 2 > k3 > (k − 1)(k2 − 1).
a + b2 b + c2 c + a2
Since the inequality remains unchanged by replacing a, b, c with −a, −b, −c, it suf-
fices to consider further that a ≤ 0 and b, c ≥ 0. Setting −a for a, we need to show
that
2ab 2bc 2ca
 ‹ ‹ ‹
k− 2 k+ 2 k− 2 ≥ (k − 1)(k2 − 1)
a +b 2 b +c 2 c +a 2
200 Vasile Cîrtoaje

for a, b, c ≥ 0. Since

2ab 2ca (a − b)2 (a − c)


 ‹ ‹ • ˜• ˜
k− 2 k− 2 = k−1+ 2 k−1+ 2
a + b2 c + a2 a + b2 c + a2

(a − b)2 (a − c)2
• ˜
≥ (k − 1) + (k − 1)
2
+ 2 ,
a2 + b2 c + a2
it suffices to prove that

(a − b)2 (a − c)2 2bc


• ˜ ‹
k−1+ 2 + k + ≥ k2 − 1.
a +b 2 c +a
2 2 b +c
2 2

According to Lemma below, we have

(a − b)2 (a − c)2 (b − c)2


+ ≥ .
a2 + b2 c 2 + a2 (b + c)2
Thus, it suffices to show that

(b − c)2 2bc
• ˜ ‹
k−1+ k+ 2 ≥ k2 − 1,
(b + c)2 b + c2
which is equivalent to the obvious inequality

(b − c)4 + 2(3 − k)bc(b − c)2 ≥ 0.

The equality holds for a = b = c.


Lemma. If a, b, c ≥ 0, no two of which are zero, then

(a − b)2 (a − c)2 (b − c)2


+ ≥ .
a2 + b2 a2 + c 2 (b + c)2

Proof. Consider two cases: a2 ≤ bc and a2 ≥ bc.


Case 1: a2 ≤ bc. By the Cauchy-Schwarz inequality, we have

(a − b)2 (a − c)2 ) [(b − a) + (a − c)]2 (b − c)2


+ ≥ = .
a2 + b2 a2 + c 2 (a2 + b2 ) + (a2 + c 2 ) 2a2 + b2 + c 2
Thus, it suffices to show that
1 1
≥ ,
2a2 +b +c
2 2 (b + c)2

which is equivalent to a2 ≤ bc.


Case 2: a2 ≥ bc. By the Cauchy-Schwarz inequality, we have

(a − b)2 (a − c)2 [c(b − a) + b(a − c)]2 a2 (b − c)2


+ ≥ = .
a2 + b2 a2 + c 2 c 2 (a2 + b2 ) + b2 (a2 + c 2 ) a2 (b2 + c 2 ) + 2b2 c 2
Symmetric Rational Inequalities 201

Therefore, it suffices to prove that


a2 1
≥ ,
a2 (b2 + c 2 ) + 2b2 c 2 (b + c)2
which reduces to bc(a2 − bc) ≥ 0.

P 1.152. If a, b, c are non-zero and distinct real numbers, then


1 1 1 1 1 1 1 1 1
• ˜  ‹
+ + +3 + + ≥4 + + .
a2 b2 c 2 (a − b)2 (b − c)2 (c − a)2 ab bc ca

Solution. Write the inequality as


1 X 1 1 X 1
X ‹ X
− + 3 ≥ 3 .
a2 bc (b − c)2 bc
In virtue of the AM-GM inequality, it suffices to prove that
v 
t X 1 X 1 ‹ •X 1
˜ X 1
2 3 − ≥ 3 ,
a2 bc (b − c)2 bc
which is true if
X ‹2
1 X 1 1 1
X ‹ •X ˜
4 − ≥ 3 .
a2 bc (b − c)2 bc
Since P 2 P 2
‹2
1 1 a − ab
X X
= = ,
(b − c)2 b−c (a − b)2 (b − c)2 (c − a)2
we can rewrite this inequality as
€X X Š €X X Š2
4 a2 b2 − abc a a2 − ab ≥ 3(a + b + c)2 (a − b)2 (b − c)2 (c − a)2 .

Using the notations


p = a + b + c, q = ab + bc + ca, r = abc,
and the identity
(a − b)2 (b − c)2 (c − a)2 = −27r 2 − 2(2p2 − 9q)pr + p2 q2 − 4q3 ,
the inequality can be written as
4(q2 − 3pr)(p2 − 3q)2 ≥ 3p2 [−27r 2 − 2(2p2 − 9q)pr + p2 q2 − 4q3 ],
which is equivalent to
(9pr + p2 q − 6q2 )2 ≥ 0.
202 Vasile Cîrtoaje

P 1.153. Let a, b, c be positive real numbers, and let

a b b c c a
A= + + k, B= + + k, C= + + k,
b a c b a b
where −2 < k ≤ 4. Prove that
1 1 1 1 4
+ + ≤ + .
A B C k + 2 A+ B + C − k − 2
(Vasile Cîrtoaje, 2009)

Solution. Let us denote


a b c
x= , y= , z= .
b c a
We need to show that
X x 1 4
≤ +
x 2 + kx + 1 k + 2 x + x y + 2k − 2
P P

for all positive real numbers x, y, z satisfying x yz = 1. Write this inequality as


follows:
X 1 x
‹
2 4
− 2 ≥ −P ,
k + 2 x + kx + 1 k+2 x + x y + 2k − 2
P

P
X (x − 1)2 2 yz(x − 1)2
≥P ,
x 2 + kx + 1 x + x y + 2k − 2
P

X (x − 1)2 [−x + y + z + x( y + z) − yz − 2]
≥ 0.
x 2 + kx + 1
Since

−x + y + z + x( y + z) − yz − 2 = (x + 1)( y + z) − (x + yz + 2)

= (x + 1)( y + z) − (x + 1)( yz + 1) = −(x + 1)( y − 1)(z − 1),


the inequality is equivalent to
X x2 − 1
−(x − 1)( y − 1)(z − 1) ≥ 0;
x 2 + kx + 1
that is, E ≥ 0, where
X
E = −(x − 1)( y − 1)(z − 1) (x 2 − 1)( y 2 + k y + 1)(z 2 + kz + 1).

We have X
(x 2 − 1)( y 2 + k y + 1)(z 2 + kz + 1) =
€X X Š €X X Š
= k(k − 2) x− xy + x2 y2 − x2
Symmetric Rational Inequalities 203

= k(k − 2)(x − 1)( y − 1)(z − 1) − (x 2 − 1)( y 2 − 1)(z 2 − 1)


= −(x − 1)( y − 1)(z − 1)[(x + 1)( y + 1)(z + 1) − k(k − 2)],
hence

E = (x − 1)2 ( y − 1)2 (z − 1)2 [(x + 1)( y + 1)(z + 1) − k(k − 2)].

Since
p p p
(x + 1)( y + 1)(z + 1) − k(k − 2) ≥ (2 x)(2 y)(2 z) − k(k − 2)
= (2 + k)(4 − k) ≥ 0,

it follows that E ≥ 0. The equality holds for a = b, or b = c, or c = a.

P 1.154. If a, b, c are nonnegative real numbers, no two of which are zero, then
1 1 1 1 1 1
+ + ≥ + + .
b2 + bc + c 2 c 2 + ca + a2 a2 + ab + b2 2a2 + bc 2b2 + ca 2c 2 + ab
(Vasile Cîrtoaje, 2014)

Solution. Write the inequality as follows:


X 1 1
‹
− ≥ 0,
b2 + bc + c 2 2a2 + bc
X (a2 − b2 ) + (a2 − c 2 )
≥ 0,
(b2 + bc + c 2 )(2a2 + bc)
X a2 − b2 X b2 − a2
+ ≥ 0,
(b2 + bc + c 2 )(2a2 + bc) (c 2 + ca + a2 )(2b2 + ca)
X c(a2 − b2 )(a − b)
(a +b +c −ab−bc−ca)
2 2 2
≥ 0.
(b2 + bc + c 2 )(c 2 + ca + a2 )(2a2 + bc)(2b2 + ca)
Clearly, the last inequality is obvious. The equality holds for a = b = c.

P 1.155. If a, b, c are nonnegative real numbers such that a + b + c ≤ 3, then


1 1 1 1 1 1
(a) + + ≥ + + ;
2a + 1 2b + 1 2c + 1 a + 2 b + 2 c + 2
1 1 1 1 1 1
(b) + + ≥ 2 + 2 + 2 .
2ab + 1 2bc + 1 2ca + 1 a + 2 b + 2 c + 2
(Vasile Cîrtoaje, 2014)
204 Vasile Cîrtoaje

Solution. Denote p
p = a + b + c, 3q ≤ p ≤ 3,
q = ab + bc + ca, 0 ≤ q ≤ 3.
(a) Use the SOS method. Write the inequality as follows
X 1 1
‹
− ≥ 0,
2a + 1 a + 2
X 1−a
≥ 0,
(2a + 1)(a + 2)
X (a + b + c) − 3a
≥ 0,
(2a + 1)(a + 2)
X (b − a) + (c − a)
≥ 0,
(2a + 1)(a + 2)
X b−a X a−b
+ ≥ 0,
(2a + 1)(a + 2) (2b + 1)(b + 2)
1 1
X • ˜
(a − b) − ,
(2b + 1)(b + 2) (2a + 1)(a + 2)
X
(a − b)2 (2a + 2b + 5)(2c + 1)(c + 2) ≥ 0.
The equality holds for a = b = c = 1.

(b) Write the inequality as


1 X 1 1 3
X ‹
≥ − + ,
2ab + 1 a +2 2
2 2
2
X 2 X a
+ ≥ 3.
2ab + 1 a2 + 2
By the Cauchy-Schwarz inequality, we have
X 1 9 9
≥P =
2ab + 1 (2ab + 1) 2q + 3
and
P 2
X a2 a p2
≥ =
a2 + 2 (a2 + 2) p2 − 2q + 6
P

2(3 − q) 2(3 − q) 3q
=1− 2 ≥1− = .
p − 2q + 6 q+6 q+6
Therefore, it suffices to show that
18 3q
+ ≥ 3,
2q + 3 q + 6
which is equivalent to the obvious inequality q ≤ 3. The equality holds for a = b =
c = 1.
Symmetric Rational Inequalities 205

P 1.156. If a, b, c are nonnegative real numbers such that a + b + c = 4, then

1 1 1 1 1 1
+ + ≥ 2 + 2 + 2 .
ab + 2 bc + 2 ca + 2 a + 2 b + 2 c + 2
(Vasile Cîrtoaje, 2014)

First Solution (by Nguyen Van Quy). Use the SOS method. Rewrite the inequality
as follows:
X 2 1 1
‹
− − ≥ 0,
ab + 2 a2 + 2 b2 + 2
X• a(a − b) b(b − a)
˜
+ ≥ 0,
(ab + 2)(a2 + 2) (ab + 2)(b2 + 2)
X (2 − ab)(a − b)2 (c 2 + 2)
≥ 0.
ab + 2
Without loss of generality, assume that a ≥ b ≥ c ≥ 0. Then,

a(b + c) (a + b + c)2
bc ≤ ac ≤ ≤ =2
2 8
and
X (2 − ab)(a − b)2 (c 2 + 2) (2 − ab)(a − b)2 (c 2 + 2) (2 − ac)(a − c)2 (b2 + 2)
≥ +
ab + 2 ab + 2 ac + 2
(2 − ab)(a − b)2 (c 2 + 2) (2 − ac)(a − b)2 (c 2 + 2)
≥ +
ab + 2 ab + 2
(4 − ab − ac)(a − b) (c + 2)
2 2
=
ab + 2
(a − b − c)2 (a − b)2 (c 2 + 2)
=
4(ab + 2)

The equality holds for a = b = c = 4/3, and also for a = 2 and b = c = 1 (or any
cyclic permutation).
Second Solution. Write the inequality as

1 X 1 1 3
X ‹
≥ − + ,
bc + 2 a +2 2
2 2
X 1 X a2 3
+ ≥ .
bc + 2 2(a + 2) 2
2

Assume that a ≥ b ≥ c, and denote

b+c 4
s= , p = bc, 0 ≤ s ≤ , 0 ≤ p ≤ s2 .
2 3
206 Vasile Cîrtoaje

By the Cauchy-Schwarz inequality, we have

b2 c2 (b + c)2 s2
+ ≥ = .
2(b2 + 2) 2(c 2 + 2) 2(b2 + 2) + 2(c 2 + 2) + 4 2s2 − p + 2
In addition,
1 1 a(b + c) + 4 2as + 4
+ = = 2 .
ca + 2 ab + 2 (ab + 2)(ac + 2) a p + 4as + 4
Therefore, it suffices to show that E(a, b, c) ≥ 0, where

1 s2 2(as + 2) a2 3
E(a, b, c) = + 2 + 2 + − .
p + 2 2s − p + 2 a p + 4as + 4 2(a + 2) 2
2

Use the mixing variables method. We will prove that

E(a, b, c) ≥ E(a, s, s) ≥ 0.

We have
1 1 1 1
 ‹  ‹
E(a, b, c) − E(a, s, s) = − +s 2

p + 2 s2 + 2 2s2 − p + 2 s2 + 2
1 1
 ‹
+ 2(as + 2) 2 −
a p + 4as + 4 a2 s2 + 4as + 4
s2 − p s2 (s2 − p)
= −
(p + 2)(s2 + 2) (s2 + 2)(2s2 − p + 2)
2a2 (s2 − p)
+ 2 .
(a p + 4as + 4)(as + 2)

Since s2 − p ≥ 0, we need to show that

1 2a2 s2
+ ≥ ,
(p + 2)(s2 + 2) (a2 p + 4as + 4)(as + 2) (s2 + 2)(2s2 − p + 2)
which is equivalent to

2a2 p(s2 + 1) − 2
≥ .
(a2 p + 4as + 4)(as + 2) (p + 2)(s2 + 2)(2s2 − p + 2)
Since
a2 p + 4as + 4 ≤ a2 s2 + 4as + 4 = (as + 2)2
and
2s2 − p + 2 ≥ s2 + 2,
it is enough to prove that

2a2 p(s2 + 1) − 2
≥ .
(as + 2)3 (p + 2)(s2 + 2)2
Symmetric Rational Inequalities 207

In addition, since
as + 2 = (4 − 2s)s + 2 ≤ 4
and
p(s2 + 1) − 2 2(s2 + 2) 2(s2 + 2)
=s +1−
2
≤s +1− 2
2
= s2 − 1,
p+2 p+2 s +2
it suffices to show that
a2 s2 − 1
≥ 2 ,
32 (s + 2)2
which is equivalent to
(2 − s)2 (2 + s2 )2 ≥ 8(s2 − 1).
4
Indeed, for the nontrivial case 1 < s ≤ , we have
3
4 2
 ‹
(2 − s) (2 + s ) − 8(s − 1) ≥ 2 −
2 2 2 2
(2 + s2 )2 − 8(s2 − 1)
3
4 4 
= (s4 − 14s2 + 22) = (7 − s2 )2 − 27
9 9
‹2
4 16 88

≥ 7− − 27 = > 0.
9 9 729

To end the proof, we need to show that E(a, s, s) ≥ 0. We have

1 s2 2 a2 3
E(a, s, s) = + + + −
s2 + 2 s2 + 2 as + 2 2(a2 + 2) 2
(s − 1)2 (3s − 4)2
= ≥ 0.
2(s2 + 2)(1 + 2s − s2 )(2s2 − 8s + 9)

P 1.157. If a, b, c are nonnegative real numbers, no two of which are zero, then

ab + bc + ca (a − b)2 (b − c)2 (c − a)2


(a) + ≤ 1;
a2 + b2 + c 2 (a2 + b2 )(b2 + c 2 )(c 2 + a2 )

ab + bc + ca (a − b)2 (b − c)2 (c − a)2


(b) + 2 ≤ 1.
a2 + b2 + c 2 (a − ab + b2 )(b2 − bc + c 2 )(c 2 − ca + a2 )

(Vasile Cîrtoaje, 2014)

Solution. (a) First Solution. Consider the nontrivial case where a, b, c are distinct
and write the inequality as follows:

(a − b)2 (b − c)2 (c − a)2 (a − b)2 + (b − c)2 + (c − a)2


≤ ,
(a2 + b2 )(b2 + c 2 )(c 2 + a2 ) 2(a2 + b2 + c 2 )
208 Vasile Cîrtoaje

(a2 + b2 ) + (b2 + c 2 ) + (c 2 + a2 ) (a − b)2 + (b − c)2 + (c − a)2


≤ ,
(a2 + b2 )(b2 + c 2 )(c 2 + a2 ) (a − b)2 (b − c)2 (c − a)2
X 1 X 1
≤ .
(b + c )(c + a )
2 2 2 2 (b − c) (c − a)2
2

Since
a2 + b2 ≥ (a − b)2 , b2 + c 2 ≥ (b − c)2 , c 2 + a2 ≥ (c − a)2 ,
the conclusion follows. The equality holds for a = b = c.
Second Solution. Assume that a ≥ b ≥ c. We have

a b + bc + ca (a − b)2 (b − c)2 (c − a)2 ab + bc + ca (a − b)2 (a − c)2


+ ≤ +
a2 + b2 + c 2 (a2 + b2 )(b2 + c 2 )(c 2 + a2 ) a2 + b2 + c 2 (a2 + b2 )(a2 + c 2 )
2ab + c 2 (a − b)2 a2
≤ 2 +
a + b2 + c 2 a2 (a2 + b2 + c 2 )
2ab + c 2 + (a − b)2
= = 1.
a2 + b2 + c 2
(b) Consider the nontrivial case where a, b, c are distinct and write the inequal-
ity as follows:

(a − b)2 (b − c)2 (c − a)2 (a − b)2 + (b − c)2 + (c − a)2


≤ ,
(a2 − ab + b2 )(b2 − bc + c 2 )(c 2 − ca + a2 ) 2(a2 + b2 + c 2 )

2(a2 + b2 + c 2 ) (a − b)2 + (b − c)2 + (c − a)2


≤ ,
(a2 − ab + b2 )(b2 − bc + c 2 )(c 2 − ca + a2 ) (a − b)2 (b − c)2 (c − a)2
X 1 2(a2 + b2 + c 2 )
≥ 2 .
(a − b)2 (a − c)2 (a − ab + b2 )(b2 − bc + c 2 )(c 2 − ca + a2 )
Assume that a = min{a, b, c}, and use the substitution

b = a + x, c = a + y, x, y ≥ 0.

The inequality can be written as

1 1 1
+ + 2 ≥ 2 f (a),
x2 y2 x 2 (x − y)2 y (x − y)2

where

3a2 + 2(x + y)a + x 2 + y 2


f (a) = 2 .
(a + x a + x 2 )(a2 + y a + y 2 )[a2 + (x + y)a + x 2 − x y + y 2 ]

We will show that


1 1 1
+ + 2 ≥ 2 f (0) ≥ 2 f (a).
x2 y2 x 2 (x − y)2 y (x − y)2
Symmetric Rational Inequalities 209

The left inequality is equivalent to

x2 + y2 − x y x2 + y2
≥ .
x 2 y 2 (x − y)2 x 2 y 2 (x 2 − x y + y 2 )

Indeed,

x2 + y2 − x y x2 + y2 1
− 2 2 2 = ≥ 0.
x y (x − y)
2 2 2 x y (x − x y + y ) (x − y) (x − x y + y 2 )
2 2 2

Also, since

(a2 + x a + x 2 )(a2 + y a + y 2 ) ≥ (x 2 + y 2 )a2 + x y(x + y)a + x 2 y 2

and
a2 + (x + y)a + x 2 − x y + y 2 ≥ x 2 − x y + y 2 ,
we get f (a) ≤ g(a), where

3a2 + 2(x + y)a + x 2 + y 2


g(a) = .
[(x 2 + y 2 )a2 + x y(x + y)a + x 2 y 2 ](x 2 − x y + y 2 )

Therefore,

x2 + y2
f (0) − f (a) ≥ − g(a)
x 2 y 2 (x 2 − x y + y 2 )
(x 4 − x 2 y 2 + y 4 )a2 + x y(x + y)(x − y)2 a
= 2 2 2 ≥ 0.
x y (x − x y + y 2 )[(x 2 + y 2 )a2 + x y(x + y)a + x 2 y 2 ]

Thus, the proof is completed. The equality holds for a = b = c.

P 1.158. If a, b, c are nonnegative real numbers, no two of which are zero, then

a2 + b2 + c 2 9(a − b)2 (b − c)2 (c − a)2


≥1+ .
ab + bc + ca (a + b)2 (b + c)2 (c + a)2

(Vasile Cîrtoaje, 2014)

Solution. Consider the nontrivial case where

0 ≤ a < b < c,

and write the inequality as follows:

(a − b)2 + (b − c)2 + (c − a)2 9(a − b)2 (b − c)2 (c − a)2


≥ ,
2(ab + bc + ca) (a + b)2 (b + c)2 (c + a)2
210 Vasile Cîrtoaje

(a − b)2 + (b − c)2 + (c − a)2 18(ab + bc + ca)


≥ ,
(a − b)2 (b − c)2 (c − a)2 (a + b)2 (b + c)2 (c + a)2
X 1 18(ab + bc + ca)
≥ .
(b − a)2 (c − a)2 (a + b)2 (a + c)2 (b + c)2
Since
X 1 1 1 1 2(b2 + c 2 − bc)
≥ 2 2+ 2 + =
(b − a)2 (c − a)2 b c b (b − c)2 c 2 (b − c)2 b2 c 2 (b − c)2

and
ab + bc + ca ab + bc + ca 1
≤ ≤ ,
(a + b) (a + c) (b + c)
2 2 2 (ab + bc + ca) (b + c)
2 2 bc(b + c)2
it suffices to show that
b2 + c 2 − bc 9
≥ .
b2 c 2 (b − c)2 bc(b + c)2
Write this inequality as follows:

(b + c)2 − 3bc 9(b + c)2 − 36bc


≥ ,
bc (b + c)2

(b + c)2 36bc
− 12 + ≥ 0,
bc (b + c)2
(b + c)4 − 12bc(b + c)2 + 36b2 c 2 ≥ 0,
[(b + c)2 − 6bc]2 ≥ 0.
Thus, the proof is completed. The equality holds for a = b = c, and also for a = 0
and b/c + c/b = 4 (or any cyclic permutation).

P 1.159. If a, b, c are nonnegative real numbers, no two of which are zero, then

a2 + b2 + c 2 p (a − b)2 (b − c)2 (c − a)2


≥ 1 + (1 + 2)2 2 .
ab + bc + ca (a + b2 )(b2 + c 2 )(c 2 + a2 )

(Vasile Cîrtoaje, 2014)

Solution.
p Consider the nontrivial case where a, b, c are distinct and denote k =
1 + 2. Write the inequality as follows:

(a − b)2 + (b − c)2 + (c − a)2 k2 (a − b)2 (b − c)2 (c − a)2


≥ 2 ,
2(ab + bc + ca) (a + b2 )(b2 + c 2 )(c 2 + a2 )

(a − b)2 + (b − c)2 + (c − a)2 2k2 (ab + bc + ca)


≥ ,
(a − b)2 (b − c)2 (c − a)2 (a2 + b2 )(b2 + c 2 )(c 2 + a2 )
Symmetric Rational Inequalities 211

X 1 2k2 (ab + bc + ca)


≥ .
(b − a)2 (c − a)2 (a2 + b2 )(b2 + c 2 )(c 2 + a2 )
Assume that a = min{a, b, c}, and use the substitution

b = a + x, c = a + y, x, y ≥ 0.

The inequality becomes


1 1 1
+ + 2 ≥ 2k2 f (a),
x2 y2 x 2 (x − y)2 y (x − y)2
where
3a2 + 2(x + y)a + x y
f (a) = .
(2a2 + 2x a + x 2 )(2a2 + 2 y a + y 2 )[2a2 + 2(x + y)a + x 2 + y 2 ]
We will show that
1 1 1
+ + ≥ 2k2 f (0) ≥ 2k2 f (a).
x 2 y 2 x 2 (x − y)2 y 2 (x − y)2
We have
1 1 1 2(x 2 + y 2 − x y) 2k2 x y
+ + − 2k 2
f (0) = −
x 2 y 2 x 2 (x − y)2 y 2 (x − y)2 x 2 y 2 (x − y)2 x 2 y 2 (x 2 + y 2 )
p
2[x + y − (2 + 2 )x y]2
2 2
= 2 2 ≥ 0.
x y (x − y)2 (x 2 − x y + y 2 )
Also, since

(2a2 + 2x a + x 2 )(2a2 + 2 y a + y 2 ) ≥ 2(x 2 + y 2 )a2 + 2x y(x + y)a + x 2 y 2

and
2a2 + 2(x + y)a + x 2 + y 2 ≥ x 2 + y 2 ,
we get f (a) ≤ g(a), where

3a2 + 2(x + y)a + x y


g(a) = .
[2(x 2 + y 2 )a2 + 2x y(x + y)a + x 2 y 2 ](x 2 + y 2 )
Therefore,
1
f (0) − f (a) ≥ − g(a)
x y(x 2 + y 2 )
(2x 2 + 2 y 2 − 3x y)a2
= ≥ 0.
x y(x 2 + y 2 )[2(x 2 + y 2 )a2 + 2x y(x + y)a + x 2 y 2 ]
Thus, the proof is completed.
p The equality holds for a = b = c, and also for a = 0
and b/c + c/b = 2 + 2 (or any cyclic permutation).
212 Vasile Cîrtoaje

P 1.160. If a, b, c are nonnegative real numbers, no two of which are zero, then

2 2 2 5 5 5
+ + ≥ + + .
a + b b + c c + a 3a + b + c 3b + c + a 3c + a + b

Solution. Use the SOS method. Write the inequality as follows:


X 2 5
‹
− ≥ 0,
b + c 3a + b + c
X 2a − b − c
≥ 0,
(b + c)(3a + b + c)
X a−b X a−c
+ ≥ 0,
(b + c)(3a + b + c) (b + c)(3a + b + c)
X a−b X b−a
+ ≥ 0,
(b + c)(3a + b + c) (c + a)(3b + c + a)
X (a − b)2 (a + b − c)
,
(b + c)(c + a)(3a + b + c)(3b + c + a)
X
(b − c)2 Sa ≥ 0,
where
Sa = (b + c − a)(b + c)(3a + b + c).
Assume that a ≥ b ≥ c. Since Sc > 0, it suffices to show that

(b − c)2 Sa + (a − c)2 S b ≥ 0.

Since S b ≥ 0, we have

(b − c)2 Sa + (a − c)2 S b ≥ (b − c)2 Sa + (b − c)2 S b = (b − c)2 (Sa + S b ).

Thus, it is enough to prove that Sa + S b ≥ 0, which is equivalent to

(c + a − b)(c + a)(3b + c + a) ≥ (b + c − a)(b + c)(3a + b + c).

For the nontrivial case b + c − a > 0, since c + a − b ≥ b + c − a, we only need to


show that
(c + a)(3b + c + a) ≥ (b + c)(3a + b + c).
Indeed,

(c + a)(3b + c + a) − (b + c)(3a + b + c) = (a − b)(a + b − c) ≥ 0.

The equality holds for a = b = c, and also for a = 0 and b = c (or any cyclic
permutation).
Symmetric Rational Inequalities 213

P 1.161. If a, b, c are real numbers, no two of which are zero, then


8a2 + 3bc 8b2 + 3ca 8c 2 + 3ab
(a) + + ≥ 11;
b2 + bc + c 2 c 2 + ca + a2 a2 + ab + b2

8a2 − 5bc 8b2 − 5ca 8c 2 − 5ab


(b) + + ≥ 9.
b2 − bc + c 2 c 2 − ca + a2 a2 − ab + b2

(Vasile Cîrtoaje, 2011)

Solution. Consider the more general inequality

a2 + mbc b2 + mca c 2 + mab 3(m + 1)


+ + ≥ ,
b2 + kbc + c 2 c 2 + kca + a2 a2 + kab + b2 k+2
which can be written as f6 (a, b, c) ≥ 0, where
X
f6 (a, b, c) = (k + 2) (a2 + mbc)(a2 + kab + b2 )(a2 + kac + c 2 )
Y
−3(m + 1) (b2 + kbc + c 2 ).
Let
p = a + b + c, q = ab + bc + ca.
From
X
f6 (a, b, c) = (k + 2) (a2 + mbc)(kab − c 2 + p2 − 2q)(kac − b2 + p2 − 2q)
Y
−3(m + 1) (kbc − a2 + p2 − 2q).

it follows that f6 (a, b, c) has the same highest coefficient A as

(k + 2)P2 (a, b, c) − 3(m + 1)P3 (a, b, c),

where X
P2 (a, b, c) = (a2 + mbc)(kab − c 2 )(kac − b2 ),
Y
P3 (a, b, c) = (kbc − a2 ).

According to Remark 2 from the proof of P 2.75 in Volume 1,

A = (k + 2)P2 (1, 1, 1) − 3(m + 1)P3 (1, 1, 1)


= 3(k + 2)(m + 1)(k − 1)2 − 3(m + 1)(k − 1)3 = 9(m + 1)(k − 1)2 .

Also, we have

f6 (a, 1, 1) = (k + 2)(a2 + ka + 1)(a − 1)2 [a2 + (k + 2)a + 1 + 2k − 2m].


214 Vasile Cîrtoaje

(a) For our particular case m = 3/8 and k = 1, we have A = 0. Therefore,


according to P 2.75 in Volume 1, it suffices to prove that f6 (a, 1, 1) ≥ 0 for all real
a. Indeed,
3 2
 ‹
f6 (a, 1, 1) = 3(a + a + 1)(a − 1) a +
2 2
≥ 0.
2
Thus, the proof is completed. The equality holds for a = b = c, and also for
−2a/3 = b = c (or any cyclic permutation).
(b) For m = −5/8 and k = −1, we have
27
A=
2
and
1
f6 (a, 1, 1) = (a2 − a + 1)(a − 1)2 (2a + 1)2 .
4
Since A > 0, we will use the highest coefficient cancellation method. Consider the
homogeneous polynomial

P(a, b, c) = abc + Bp3 + C pq,

where B and C are real constants. Since the desired inequality becomes an equality
for a = b = c = 1, and also for a = −1/2 and b = c = 1, we will determine B and
C such that P(1, 1, 1) = P(−1/2, 1, 1) = 0; that is,
4 −5
B= , C= ,
27 9
when
4p3 5pq
P(a, b, c) = abc + − ,
27 9
2
P(a, 1, 1) = (a − 1)2 (2a + 1).
27
We will show that
27 2
f6 (a, b, c) ≥ P (a, b, c).
2
Let us denote
27 2
g6 (a, b, c) = f6 (a, b, c) −P (a, b, c).
2
Since g6 (a, b, c) has the highest coefficient equal to zero, it suffices to prove that
g6 (a, 1, 1) ≥ 0 for all real a (see P 2.75 in Volume 1). Indeed,
27 2 1
g6 (a, 1, 1) = f6 (a, 1, 1)− P (a, 1, 1) = (a−1)2 (2a+1)2 (19a2 −11a+19) ≥ 0.
2 108
The equality holds for a = b = c, and also for −2a = b = c (or any cyclic permuta-
tion).
Symmetric Rational Inequalities 215

P 1.162. If a, b, c are real numbers, no two of which are zero, then

4a2 + bc 4b2 + ca 4c 2 + ab
+ + ≥ 1.
4b2 + 7bc + 4c 2 4c 2 + 7ca + 4a2 4a2 + 7ab + 4b2
(Vasile Cîrtoaje, 2011)

Solution. Write the inequality as f6 (a, b, c) ≥ 0, where


X Y
f6 (a, b, c) = (4a2 + bc)(4a2 +7ab +4b2 )(4a2 +7ac +4c 2 )− (4b2 +7bc +4c 2 ).

Let
p = a + b + c, q = ab + bc + ca.
From
X
f6 (a, b, c) = (4a2 + bc)(7ab − 4c 2 + 4p2 − 8q)(7ac − 4b2 + 4p2 − 8q)
Y
− (7bc − 4a2 + 4p2 − 8q),
it follows that f6 (a, b, c) has the same highest coefficient A as

P2 (a, b, c) − P3 (a, b, c),

where X
P2 (a, b, c) = (4a2 + bc)(7ab − 4c 2 )(7ac − 4b2 ),
Y
P3 (a, b, c) = (7bc − 4a2 ).
According to Remark 2 from the proof of P 2.75 in Volume 1,

A = P2 (1, 1, 1) − P3 (1, 1, 1) = 135 − 27 = 108.

Since A > 0, we will apply the highest coefficient cancellation method. Consider the
homogeneous polynomial

P(a, b, c) = abc + Bp3 + C pq,

where B and C are real constants. We will show that there are two real numbers B
and C such that the following sharper inequality holds

f6 (a, b, c) ≥ 108P 2 (a, b, c).

Let us denote
g6 (a, b, c) = f6 (a, b, c) − 108P 2 (a, b, c).
Clearly, g6 (a, b, c) has the highest coefficient equal to zero. Then, by P 2.75 in
Volume 1, it suffices to prove that there exist B and C such that g6 (a, 1, 1) ≥ 0 for
all real a.
216 Vasile Cîrtoaje

We have
g6 (a, 1, 1) = f6 (a, 1, 1) − 108P 2 (a, 1, 1),
where
f6 (a, 1, 1) = 4(4a2 + 7a + 4)(a − 1)2 (4a2 + 15a + 16),
P(a, 1, 1) = a + B(a + 2)3 + C(a + 2)(2a + 1).
Let us denote g(a) = f6 (a, 1, 1). Since

g(−2) = 0,

the condition
g 0 (−2) = 0,
which involves C = −5/9, is necessary to have g(a) ≥ 0 in the vicinity of a = −2.
On the other hand, from g(1) = 0, we get B = 4/27. For these values of B and C,
we get
2(a − 1)2 (2a + 1)
P(a, 1, 1) = ,
27
4
g6 (a, 1, 1) = (a − 1)2 (a + 2)2 (416a2 + 728a + 431) ≥ 0.
27
The proof is completed. The equality holds for a = b = c, and for a = 0 and
b + c = 0 (or any cyclic permutation).

P 1.163. If a, b, c are real numbers, no two of which are equal, then

1 1 1 27
+ + ≥ .
(a − b)2 (b − c)2 (c − a)2 4(a + b + c − ab − bc − ca)
2 2 2

First Solution. Write the inequality as follows:


• 1 1 1 27
 ˜
(a − b) + (b − c) + (a − c)
2 2 2
+ + ≥ ,
(a − b)2 (b − c)2 (a − c)2 2

(a − b)2 (b − c)2 (a − c)2 (a − c)2 27


• ˜• ˜
+ +1 + +1 ≥ ,
(a − c)2 (a − c) 2 (a − b) 2 (b − c) 2 2
1 1 27
 ‹
(x + y + 1)
2 2
+ + 1 ≥ ,
x2 y2 2
where
a−b b−c
x= , y= , x + y = 1.
a−c a−c
Symmetric Rational Inequalities 217

We have
1 1 27 (x + 1)2 (x − 2)2 (2x − 1)2
 ‹
(x + y + 1)
2 2
+ + 1 − = ≥ 0.
x2 y2 2 2x 2 (1 − x)2
The proof is completed. The equality holds for 2a = b + c (or any cyclic permuta-
tion).
Second Solution. Assume that a > b > c. We have
1 1 2 8 8
+ ≥ ≥ = .
(a − b)2 (b − c)2 (a − b)(b − c) [(a − b) + (b − c)] 2 (a − c)2
Therefore, it suffices to show that
9 27
≥ ,
(a − c)2 4(a + b + c − ab − bc − ca)
2 2 2

which is equivalent to
(a − 2b + c)2 ≥ 0.

Third Solution. Write the inequality as f6 (a, b, c) ≥ 0, where


X
f6 (a, b, c) = 4(a +b +c −ab−bc−ca)
2 2 2
(a−b)2 (a−c)2 −27(a−b)2 (b−c)2 (c−a)2 .

Clearly, f6 (a, b, c) has the same highest coefficient A as

−27(a − b)2 (b − c)2 (c − a)2 ;

that is,
A = −27(−27) = 729.
Since A > 0, we will use the highest coefficient cancellation method. Define the
homogeneous polynomial
1
 ‹
P(a, b, c) = abc + B(a + b + c) − 3B +
3
(a + b + c)(ab + bc + ca)
9
which satisfies P(1, 1, 1) = 0. We will show that there is a real value of B such that
the following sharper inequality holds

f6 (a, b, c) ≥ 729P 2 (a, b, c).

Let us denote
g6 (a, b, c) = f6 (a, b, c) − 729P 2 (a, b, c).
Clearly, g6 (a, b, c) has the highest coefficient equal to zero. Then, by P 2.75 in
Volume 1, it suffices to prove that g6 (a, 1, 1) ≥ 0 for all real a. We have

f6 (a, 1, 1) = 4(a − 1)6


218 Vasile Cîrtoaje

and
1
P(a, 1, 1) = (a − 1)2 [9B(a + 2) + 2],
9
hence

g6 (a, 1, 1) = f6 (a, 1, 1) − 729P 2 (a, 1, 1)


= (27B + 2)(a + 2)(a − 1)4 [(2 − 27B)a − 54B − 8].

Choosing B = −2/27, we get g6 (a, 1, 1) = 0 for all real a.


Remark. The inequality is equivalent to

(a − 2b + c)2 (b − 2c + a)2 (c − 2a + b)2 ≥ 0.

P 1.164. If a, b, c are real numbers, no two of which are zero, then


1 1 1 14
+ 2 + 2 ≥ .
a2 − ab + b 2 b − bc + c 2 c − ca + a 2 3(a + b2 + c 2 )
2

(Vasile Cîrtoaje and BJSL, 2014)

Solution. Write the inequality as f6 (a, b, c) ≥ 0, where


X
f6 (a, b, c) =3(a2 + b2 + c 2 ) (a2 − ab + b2 )(a2 − ac + c 2 )
− 14(a2 − ab + b2 )(b2 − bc + c 2 )(c 2 − ca + a2 ).

Clearly, f6 (a, b, c) has the same highest coefficient A as

−14(a2 − ab + b2 )(b2 − bc + c 2 )(c 2 − ca + a2 );

that is, according to Remark 2 from the proof of P 2.75 in Volume 1,

A = −14(−1 − 1)3 = 112.

Since A > 0, we apply the highest coefficient cancellation method. Consider the
homogeneous polynomial

P(a, b, c) = abc + B(a + b + c)3 + C(a + b + c)(ab + bc + ca).

We will show that there are two real numbers B and C such that the following
sharper inequality holds

f6 (a, b, c) ≥ 112P 2 (a, b, c).

Let us denote
g6 (a, b, c) = f6 (a, b, c) − 112P 2 (a, b, c).
Symmetric Rational Inequalities 219

Clearly, g6 (a, b, c) has the highest coefficient equal to zero. By P 2.75 in Volume 1,
it suffices to prove that g6 (a, 1, 1) ≥ 0 for all real a. We have

g6 (a, 1, 1) = f6 (a, 1, 1) − 112P 2 (a, 1, 1),

where
f6 (a, 1, 1) = (a2 − a + 1)(3a4 − 3a3 + a2 + 8a + 4),
P(a, 1, 1) = a + B(a + 2)3 + C(a + 2)(2a + 1).
Let us denote g(a) = g6 (a, 1, 1). Since

g(−2) = 0,

the condition
g 0 (−2) = 0,
which involves C = −4/7, is necessary to have g(a) ≥ 0 in the vicinity of a = −2.
In addition, setting B = 9/56, we get

1
P(a, 1, 1) = (9a3 − 10a2 + 4a + 8),
56
3 6
g6 (a, 1, 1) = (a + 4a5 + 8a4 + 16a3 + 20a2 + 16a + 16)
28
3(a + 2)2 (a2 + 2)2
= ≥ 0.
28
The proof is completed. The equality holds for a = 0 and b + c = 0 (or any cyclic
permutation).

P 1.165. If a, b, c are real numbers, then

a2 + bc b2 + ca c 2 + ab 1
+ + ≥ .
2a + b + c
2 2 2 a + 2b + c
2 2 2 a + b + 2c
2 2 2 6
(Vasile Cîrtoaje, 2010)

Solution. Write the inequality as f6 (a, b, c) ≥ 0, where


X
f6 (a, b, c) =6 (a2 + bc)(a2 + 2b2 + c 2 )(a2 + b2 + 2c 2 )
− (2a2 + b2 + c 2 )(a2 + 2b2 + c 2 )(a2 + b2 + 2c 2 ).

Clearly, f6 (a, b, c) has the same highest coefficient A as f (a, b, c), where
X
f (a, b, c) = 6 (a2 + bc)b2 c 2 − a2 b2 c 2 = 17a2 b2 c 2 + 6(a3 b3 + b3 c 3 + c 3 a3 );
220 Vasile Cîrtoaje

that is,
A = 17 + 6 · 3 = 35.
Since A > 0, we apply the highest coefficient cancellation method. Consider the
homogeneous polynomial

P(a, b, c) = abc + B(a + b + c)3 + C(a + b + c)(ab + bc + ca)

and show that there are two real numbers B and C such that the following sharper
inequality holds
f6 (a, b, c) ≥ 35P 2 (a, b, c).
Let us denote
g6 (a, b, c) = f6 (a, b, c) − 35P 2 (a, b, c).
Clearly, g6 (a, b, c) has the highest coefficient equal to zero. By P 2.75 in Volume 1,
it suffices to prove that g6 (a, 1, 1) ≥ 0 for all real a. We have

g6 (a, 1, 1) = f6 (a, 1, 1) − 35P 2 (a, 1, 1),

where
f6 (a, 1, 1) = 4(a2 + 1)(a2 + 3)(a + 3)2 ,
P(a, 1, 1) = a + B(a + 2)3 + C(a + 2)(2a + 1).
Let
g(a) = g6 (a, 1, 1).
Since g(−2) = 0, we can have g(a) ≥ 0 in the vicinity of a = −2 only if g 0 (−2) = 0,
which involves C = 19/35. Since f6 (−3, 1, 1) = 0, we enforce P(−3, 1, 1) = 0,
which provides B = −2/7. Thus,
2 19 −2(a + 3)(5a2 − 4a + 7)
P(a, 1, 1) = a − (a + 1)3 + (a + 2)(2a + 1) =
7 35 35
and
4
g6 (a, 1, 1) = 4(a2 + 1)(a2 + 3)(a + 3)2 − (a + 3)2 (5a2 − 4a + 7)2
35
8
= (a + 3)2 (a + 2)2 (5a2 + 7) ≥ 0.
35
The proof is completed. The equality holds for a = 0 and b + c = 0 (or any cyclic
permutation), and also for −a/3 = b = c (or any cyclic permutation).

P 1.166. If a, b, c are real numbers, then


2b2 + 2c 2 + 3bc 2c 2 + 2a2 + 3ca 2a2 + 2b2 + 3ab 3
+ + ≥ .
(a + 3b + 3c)2 (b + 3c + 3a)2 (c + 3a + 3b)2 7
(Vasile Cîrtoaje, 2010)
Symmetric Rational Inequalities 221

Solution. Write the inequality as f6 (a, b, c) ≥ 0, where


X Y
f6 (a, b, c) = 7 (2b2 + 2c 2 + 3bc)(b + 3c + 3a)2 (c + 3a + 3b)2 − 3 (a + 3b + 3c)2 .

We have
f6 (a, 1, 1) = (a − 1)2 (a − 8)2 (3a + 4)2 .
Let
p = a + b + c, q = ab + bc + ca, r = abc.
From
X Y
f6 (a, b, c) = 7 (2p2 − 4q + 3bc − 2a2 )(3p − 2b)2 (3p − 2c)2 − 3 (3p − 2a)2 ,

it follows that f (a, b, c) has the same highest coefficient A as g(a, b, c), where
X Y € X Š
g(a, b, c) = 7 (3bc−2a2 )(−2b)2 (−2c)2 −3 (−2a)2 = 48 7 b3 c 3 − 18a2 b2 c 2 ;

that is,
A = 48(21 − 18) = 144.
Since the highest coefficient A is positive, we will use the highest coefficient cancel-
lation method. There are two cases to consider: p2 + q ≥ 0 and p2 + q < 0.
Case 1: p2 + q ≥ 0. Since

f6 (1, 1, 1) = f6 (8, 1, 1) = 0,

define the homogeneous function

P(a, b, c) = r + Bp3 + C pq

such that P(1, 1, 1) = P(8, 1, 1) = 0; that is,

1 3 8
P(a, b, c) = r + p − pq,
45 45
which leads to

45a + (a + 2)3 − 8(a + 2)(2a + 1) (a − 1)2 (a − 8)


P(a, 1, 1) = = .
45 45

We will show that the following sharper inequality holds for p2 + q ≥ 0:

f6 (a, b, c) ≥ 144P 2 (a, b, c).

Let us denote
g6 (a, b, c) = f6 (a, b, c) − 144P 2 (a, b, c).
222 Vasile Cîrtoaje

Since the highest coefficient of g6 (a, b, c) is zero, it suffices to prove that g6 (a, 1, 1) ≥
0 for all real a such that (a + 2)2 + 2a + 1 ≥ 0, that is
a ∈ (−∞, −5] ∪ [−1, ∞)
(see Remark 3 from the proof of P 2.75 in Volume 1). We have
g6 (a, 1, 1) = f6 (a, 1, 1) − 144P 2 (a, 1, 1)
1
= (a − 1)2 (a − 8)2 [225(3a + 4)2 − 16(a − 1)2 ]
225
7
= (a − 1)2 (a − 8)2 (41a + 64)(7a + 8) ≥ 0.
225
Case 2: p2 + q < 0. Since
f6 (1, 1, 1) = f6 (−4/3, 1, 1) = 0,
define the homogeneous function
P(a, b, c) = r + Bp3 + C pq
such that P(1, 1, 1) = P(−4/3, 1, 1) = 0; that is,
1 3 10
P(a, b, c) = r + p − pq,
3 9
which leads to
9a + 3(a + 2)3 − 10(a + 2)(2a + 1) (a − 1)2 (3a + 4)
P(a, 1, 1) = = .
9 9
We will show that the following sharper inequality holds for p2 + q < 0:
f6 (a, b, c) ≥ 144P 2 (a, b, c).
Let us denote
g6 (a, b, c) = f6 (a, b, c) − 144P 2 (a, b, c).
Since the highest coefficient of g6 (a, b, c) is zero, it suffices to prove that g6 (a, 1, 1) ≥
0 for all real a such that (a + 2)2 + 2a + 1 < 0, that is
a ∈ (−5, −1)
(see Remark 3 from the proof of P 2.75 in Volume 1). We have
g6 (a, 1, 1) = f6 (a, 1, 1) − 144P 2 (a, 1, 1)
1
= (a − 1)2 (3a + 4)2 [9(a − 8)2 − 16(a − 1)2 ]
9
7
= (a − 1)2 (3a + 4)2 (20 + a)(4 − a) ≥ 0.
9
The proof is completed. The equality holds for a = b = c, for a/8 = b = c (or any
cyclic permutation), and also for −3a/4 = b = c (or any cyclic permutation).
Symmetric Rational Inequalities 223

P 1.167. If a, b, c are nonnegative real numbers, then

6b2 + 6c 2 + 13bc 6c 2 + 6a2 + 13ca 6a2 + 6b2 + 13ab


+ + ≤ 3.
(a + 2b + 2c)2 (b + 2c + 2a)2 (c + 2a + 2b)2

(Vasile Cîrtoaje, 2010)

Solution. Write the inequality as f6 (a, b, c) ≥ 0, where


Y X
f6 (a, b, c) = 3 (a + 2b + 2c)2 − (6b2 + 6c 2 + 13bc)(b + 2c + 2a)2 (c + 2a + 2b)2 .

Let
p = a + b + c, q = ab + bc + ca.
From
Y X
f6 (a, b, c) = 3 (2p − a)2 − (6p2 − 12q + 13bc − 6a2 )(2p − b)2 (2p − c)2 ,

it follows that f (a, b, c) has the same highest coefficient A as g(a, b, c), where
Y X X
g(a, b, c) = 3 (−a)2 − (13bc − 6a2 )(−b)2 (−c)2 = 21a2 b2 c 2 − 13 b3 c 3 ;

that is,
A = 21 − 39 = −18.
Since the highest coefficient A is negative, it suffices to prove the desired inequality
for b = c = 1, and for a = 0 (see P 3.76-(a) in Volume 1).
For b = c = 1, the inequality becomes

25 2(6a2 + 13a + 6)
+ ≤ 3,
(a + 4)2 (2a + 3)2

2(6a2 + 13a + 6) 3a2 + 24a + 23


≤ ,
(2a + 3)2 (a + 4)2
5(2a + 3)(a − 1)2
≥ 0.
(2a + 3)2 (a + 4)2
For a = 0, the inequality turns into

6b2 + 6c 2 + 13bc 6c 2 6b2


+ + ≤ 3,
4(b + c)2 (b + 2c)2 (2b + c)2

6b2 + 6c 2 + 13bc 6[(b2 + c 2 )2 + 4bc(b2 + c 2 ) + 6b2 c 2 ]


+ ≤ 3.
4(b + c)2 (2b2 + 2c 2 + 5bc)2
If bc = 0, then the inequality is an identity. For bc 6= 0, we may consider bc = 1
(due to homogeneity). Denoting

x = b2 + c 2 , x ≥ 2,
224 Vasile Cîrtoaje

the inequality becomes

6x + 13 6(x 2 + 4x + 6)
+ ≤ 3,
4(x + 2) (2x + 5)2
which reduces to the obvious inequality

20x 2 + 34x − 13 ≥ 0.

The equality holds for a = b = c, and also for a = b = 0 (or any cyclic permutation).

P 1.168. If a, b, c are nonnegative real numbers such that a + b + c = 3, then

3a2 + 8bc 3b2 + 8ca 3c 2 + 8ab


+ + ≤ 3.
9 + b2 + c 2 9 + c 2 + a2 9 + a2 + b2
(Vasile Cîrtoaje, 2010)

Solution. Let
p = a + b + c, q = ab + bc + ca.
Write the inequality in the homogeneous form

3a2 + 8bc 3b2 + 8ca 3c 2 + 8ab


+ + ≤ 3,
p2 + b2 + c 2 p2 + c 2 + a2 p2 + a2 + b2

which is equivalent to f6 (a, b, c) ≥ 0, where


Y X
f6 (a, b, c) = 3 (p2 + b2 + c 2 ) − (3a2 + 8bc)(p2 + c 2 + a2 )(p2 + a2 + b2 ).

From
Y X
f6 (a, b, c) = 3 (2p2 − 2q − a2 ) − (3a2 + 8bc)(2p2 − 2q − b2 )(2p2 − 2q − c 2 ),

it follows that f (a, b, c) has the same highest coefficient A as g(a, b, c), where
Y X X
g(a, b, c) = 3 (−a)2 − (3a2 + 8bc)(−b2 )(−c 2 ) = −12a2 b2 c 2 − 8 b3 c 3 ;

that is,
A = −12 − 24 = −36.
Since the highest coefficient A is negative, it suffices to prove the homogeneous
inequality for b = c = 1 and for a = 0 (see P 3.76-(a) in Volume 1).
For b = c = 1, we need to show that

3a2 + 8 2(3 + 8a)


+ ≤ 3,
(a + 2)2 + 2 (a + 2)2 + a2 + 1
Symmetric Rational Inequalities 225

which is equivalent to

3a2 + 8 2(8a + 3)
+ 2 ≤ 3,
a + 4a + 6 2a + 4a + 5
2

8a + 3 6a + 5
≤ ,
2a2 + 4a + 5 a2 + 4a + 6
4a3 − a2 − 10a + 7 ≥ 0,
(a − 1)2 (4a + 7) ≥ 0.
For a = 0, we need to show that

8bc 3b2 3c 2
+ + ≤ 3.
(b + c)2 + b2 + c 2 (b + c)2 + c 2 (b + c)2 + b2
Clearly, it suffices to show that

8bc 3(b2 + c 2 )
+ ≤ 3,
(b + c)2 + b2 + c 2 (b + c)2
which is equivalent to
4bc 6bc
≤ ,
b2 + c + bc
2 (b + c)2
bc(b − c)2 ≥ 0.
The equality holds for a = b = c = 1, and also for a = b = 0 and c = 3 (or any
cyclic permutation).

P 1.169. If a, b, c are nonnegative real numbers such that a + b + c = 3, then

5a2 + 6bc 5b2 + 6ca 5c 2 + 6ab


+ + ≥ 3.
9 + b2 + c 2 9 + c 2 + a2 9 + a2 + b2
(Vasile Cîrtoaje, 2010)

Solution. We use the highest coefficient method. Let

p = a + b + c, q = ab + bc + ca.

Write the inequality in the homogeneous form f6 (a, b, c) ≥ 0, where


X Y
f6 (a, b, c) = (5a + 6bc)(p + c + a )(p + a + b ) − 3
2 2 2 2 2 2 2
(p2 + b2 + c 2 ).

From
X Y
f6 (a, b, c) = (5a2 + 6bc)(2p2 − 2q − b2 )(2p2 − 2q − c 2 ) − 3 (2p2 − 2q − a2 ),
226 Vasile Cîrtoaje

it follows that f6 (a, b, c) has the same highest coefficient A as


X X
f (a, b, c) = (5a2 +6bc)(−b2 )(−c 2 )−3(−a2 )(−b2 )(−c 2 ) = 18a2 b2 c 2 +6 b3 c 3 ;

therefore,
A = 18 + 18 = 36.
On the other hand,

f6 (a, 1, 1) = 4a(2a2 + 4a + 5)(a + 1)(a − 1)2 ≥ 0

and

f6 (0, b, c) = 6bcBC + 5b2 AB + 5c 2 AC − 3ABC


= −3(A − 2bc)BC + 5A(b2 B + c 2 C),

where

A = (b + c)2 + b2 + c 2 , B = (b + c)2 + b2 , C = (b + c)2 + c 2 .

Substituting
(b + c)2 = 4x, bc = y, x ≥ y,
we have
A = 2(4x − y), B = 4x + b2 , C = 4x + c 2 ,
A − 2bc = 4(2x − y),
BC = 16x + 4x(b + c ) + b c = 16x 2 + 4x(4x − 2 y) + y 2 = 32x 2 − 8x y + y 2 ,
2 2 2 2 2

b2 B + c 2 C = 4x(b2 + c 2 ) + b4 + c 4 = 2(16x 2 − 12x y + y 2 ),


therefore

f6 (0, b, c) = −12(2x − y)(32x 2 − 8x y + y 2 ) + 20(4x − y)(16x 2 − 12x y + y 2 )


= 8(64x 3 − 88x 2 y + 25x y 2 − y 3 ) = 8(x − y)(64x 2 − 24x y + y 2 ).

Since
f6 (1, 1, 1) = f6 (0, 1, 1) = 0,
define the homogeneous function

P(a, b, c) = abc + B(a + b + c)3 + C(a + b + c)(ab + bc + ca)

such that P(1, 1, 1) = P(0, 1, 1) = 0; that is,


1 4
P(a, b, c) = abc + (a + b + c)3 − (a + b + c)(ab + bc + ca).
9 9
We have
a(a − 1)2 a2 (a − 1)4
P(a, 1, 1) = , P (a, 1, 1) =
2
,
9 81
Symmetric Rational Inequalities 227

(b + c)(b − c)2 64x(x − y)2


P(0, b, c) = , P (0, b, c) =
2
.
9 81
We will prove the sharper inequality g6 (a, b, c) ≥ 0, where

g6 (a, b, c) = f6 (a, b, c) − 36P 2 (a, b, c).

Clearly, g6 (a, b, c) has the highest coefficient A = 0. Then, according to P 3.76-


(a) in Volume 1, it suffices to prove that g6 (a, 1, 1) ≥ 0 and g6 (0, b, c) ≥ 0 for
a, b, c ≥ 0.
We have
4a(a − 1)2 h(a)
g6 (a, 1, 1) = f6 (a, 1, 1) − 36P 2 (a, 1, 1) = ,
9
where

h(a) = 9(2a2 + 4a + 5)(a + 1) − a(a − 1)2


> (a − 1)2 (a + 1) − a(a − 1)2 = (a − 1)2 ≥ 0.

Also, we have

8(x − y)g(x, y)
g6 (0, b, c) = f6 (0, b, c) − 36P 2 (0, b, c) = ,
9
where

g(x, y) = 9(64x 2 − 24x y + y 2 ) − 32x(x − y)


> (64x 2 − 24x y + y 2 ) − 32x(x − y) = 32x 2 + 8x y + y 2 > 0.

The equality holds for a = b = c = 1, and also for a = 0 and b = c = 3/2 (or any
cyclic permutation).

P 1.170. If a, b, c are nonnegative real numbers such that a + b + c = 3, then


1 1 1 3
+ 2 + 2 ≤ .
a2 + bc + 12 b + ca + 12 c + ab + 12 14
(Vasile Cîrtoaje, 2010)

Solution. Write the inequality in the homogeneous form


1 1 1 9
+ + ≤ ,
3(a2 + bc) + 4p 2 3(b + ca) + 4p
2 2 3(c + ab) + 4p
2 2 14p2

where
p = a + b + c.
228 Vasile Cîrtoaje

The inequality is equivalent to f6 (a, b, c) ≥ 0, where


Y X
f6 (a, b, c) = 9 (3a + 3bc + 4p ) − 14p
2 2 2
(3b2 + 3ca + 4p2 )(3c 2 + 3ab + 4p2 ).

Clearly, f6 (a, b, c) has the same highest coefficient A as


Y
g(a, b, c) = 243 (a2 + bc).

According to Remark 2 from the proof of P 2.75 in Volume 1, we have

A = g(1, 1, 1) = 243 · 8 = 1944.

Since the highest coefficient A is positive, we will apply the highest coefficient can-
cellation method. We have

f6 (a, 1, 1) =9[3a2 + 3 + 4(a + 2)2 ][3a + 3 + 4(a + 2)2 ]2


− 14(a + 2)2 [3a + 3 + 4(a + 2)2 ]2
− 28(a + 2)2 [3a + 3 + 4(a + 2)2 ][3a2 + 3 + 4(a + 2)2 ]
=9(7a2 + 16a + 19)(4a2 + 19a + 19)2 − 14(a + 2)2 (4a2 + 19a + 19)2
− 28(a + 2)2 (4a2 + 19a + 19)(7a2 + 16a + 19)
=3(4a2 + 19a + 19) f (a),

where

f (a) = 3(7a2 + 16a + 19)(4a2 + 19a + 19) − 14(a + 2)2 (6a2 + 17a + 19)
= 17a3 − 15a2 − 21a + 19 = (a − 1)2 (17a + 19);

therefore,
f6 (a, 1, 1) = 3(4a2 + 19a + 19)(a − 1)2 (17a + 19).
Since
f6 (1, 1, 1) = f6 (1, 0, 0) = 0,
define the homogeneous function

P(a, b, c) = abc + B(a + b + c)3 + C(a + b + c)(ab + bc + ca)

such that P(1, 1, 1) = P(1, 0, 0) = 0; that is,


1
P(a, b, c) = abc − (a + b + c)(ab + bc + ca).
9
We will prove the sharper inequality g6 (a, b, c) ≥ 0, where

g6 (a, b, c) = f6 (a, b, c) − 1944P 2 (a, b, c).

Clearly, g6 (a, b, c) has the highest coefficient A = 0. Then, it suffices to prove that
g6 (a, 1, 1) ≥ 0 and g6 (0, b, c) ≥ 0 for a, b, c ≥ 0 (see P 3.76-(a) in Volume 1).
Symmetric Rational Inequalities 229

To show that g6 (a, 1, 1) ≥ 0, which can be written as

f6 (a, 1, 1) − 1944P 2 (a, 1, 1) ≥ 0,

we see that
(a + 2)(2a + 1) −2(a − 1)2
P(a, 1, 1) = a − = ,
9 9
4(a − 1)4
P 2 (a, 1, 1) = ,
81
hence

g6 (a, 1, 1) = 3(4a2 + 19a + 19)(a − 1)2 (17a + 19) − 96(a − 1)4


= 3(a − 1)2 h(a),

where
h(a) = (4a2 + 19a + 19)(17a + 19) − 32(a − 1)2 .
We need to show that h(a) ≥ 0 for a ≥ 0. Indeed, since

(4a2 + 19a + 19)(17a + 19) > (19a + 19)(17a + 17) > 32(a + 1)2 ,

we get
h(a) > 32[(a + 1)2 − (a − 1)2 ] = 128a ≥ 0.
To show that g6 (0, b, c) ≥ 0, denote

x = (b + c)2 , y = bc.

We have

f6 (0, b, c) = 9ABC − 14x[BC + A(B + C)] = (9A − 14x)BC − 14xA(B + C),

where
A = 4x + 3 y, B = 4x + 3b2 , C = 4x + 3c 2 .
Since

9A − 14x = 22x + 27 y, B + C = 8x + 3(x − 2 y) = 11x − 6 y,

BC = 16x 2 + 12x(x − 2 y) + 9 y 2 = 28x 2 − 24x y + 9 y 2 ,


we get

f6 (0, b, c) = (22x + 27 y)(28x 2 − 24x y + 9 y 2 ) − 14x(4x + 3 y)(11x − 6 y)


= 3 y(34x 2 − 66x y + 81 y 2 ).

Also,
−bc(b + c) x y2
P(0, b, c) = , P (0, b, c) =
2
.
9 81
230 Vasile Cîrtoaje

Hence

g6 (0, b, c) = f6 (0, b, c) − 1944P 2 (0, b, c) = 3 y(34x 2 − 74x y + 81 y 2 )


≥ 3 y(25x 2 − 90x y + 81 y 2 ) = 3 y(5x − 9 y)2 ≥ 0.

The equality holds for a = b = c, and also for a = b = 0 (or any cyclic permutation).

P 1.171. If a, b, c are nonnegative real numbers, no two of which are zero, then
1 1 1 45
+ 2 + 2 ≥ .
a2 +b 2 b +c 2 c +a 2 8(a + b + c ) + 2(ab + bc + ca)
2 2 2

(Vasile Cîrtoaje, 2014)

First Solution (by Nguyen Van Quy). Multiplying by a2 + b2 + c 2 , the inequality be-
comes
X a2 45(a2 + b2 + c 2 )
+3≥ .
b2 + c 2 8(a2 + b2 + c 2 ) + 2(ab + bc + ca)
Applying the Cauchy-Schwarz inequality, we have
P 2
X a 2 a2 (a2 + b2 + c 2 )2
≥ = .
b2 + c 2 a2 (b2 + c 2 ) 2(a2 b2 + b2 c 2 + c 2 a2 )
P

Therefore, it suffices to show that

(a2 + b2 + c 2 )2 45(a2 + b2 + c 2 )
+ 3 ≥ ,
2(a2 b2 + b2 c 2 + c 2 a2 ) 8(a2 + b2 + c 2 ) + 2(ab + bc + ca)
which is equivalent to

(a2 + b2 + c 2 )2 45(a2 + b2 + c 2 )
−3≥ − 9,
a2 b2 + b2 c 2 + c 2 a2 4(a2 + b2 + c 2 ) + ab + bc + ca

a4 + b4 + c 4 − a2 b2 − b2 c 2 − c 2 a2 9(a2 + b2 + c 2 − ab − bc − ca)
≥ .
a2 b2 + b2 c 2 + c 2 a2 4(a2 + b2 + c 2 ) + ab + bc + ca
By Schur’s inequality of degree four, we have

a4 + b4 + c 4 − a2 b2 − b2 c 2 − c 2 a2 ≥ (a2 + b2 + c 2 − ab − bc − ca)(ab + bc + ca).

Therefore, it suffices to show that

[4(a2 + b2 + c 2 ) + ab + bc + ca](ab + bc + ca) ≥ 9(a2 b2 + b2 c 2 + c 2 a2 ).

Since
(ab + bc + ca)2 ≥ a2 b2 + b2 c 2 + c 2 a2 ,
Symmetric Rational Inequalities 231

this inequality is true if

4(a2 + b2 + c 2 )(ab + bc + ca) ≥ 8(a2 b2 + b2 c 2 + c 2 a2 ),

which is equivalent to the obvious inequality

ab(a − b)2 + bc(b − c)2 + ca(c − a)2 + abc(a + b + c) ≥ 0.

The equality holds for a = b = c, and also for a = 0 and b = c (or any cyclic
permutation).
Second Solution. Write the inequality as f6 (a, b, c) ≥ 0, where
 X 2 2 2 2 Y
f6 (a, b, c) = 8(a2 + b2 + c 2 ) + 2(ab + bc + ca) (a +b )(a +c )−45 (b2 +c 2 ).

Clearly, f6 (a, b, c) has the same highest coefficient A as


Y Y
f (a, b, c) = −45 (b2 + c 2 ) = −45 (p2 − 2q − a2 ),

where p = a + b + c and q = ab + bc + ca; that is,

A = 45.

Since A > 0, we will apply the highest coefficient cancellation method. We have

f6 (a, 1, 1) = 4a(2a + 5)(a2 + 1)(a − 1)2 ,

f6 (0, b, c) = (b − c)2 [8(b4 + c 4 ) + 18bc(b2 + c 2 ) + 15b2 c 2 ].


Since
f6 (1, 1, 1) = f6 (0, 1, 1) = 0,
define the homogeneous function

P(a, b, c) = abc + B(a + b + c)3 + C(a + b + c)(ab + bc + ca)

such that P(1, 1, 1) = P(0, 1, 1) = 0; that is,


1 4
P(a, b, c) = abc + (a + b + c)3 − (a + b + c)(ab + bc + ca).
9 9
We will show that the following sharper inequality holds

f6 (a, b, c) ≥ 45P 2 (a, b, c).

Let us denote
g6 (a, b, c) = f6 (a, b, c) − 45P 2 (a, b, c).
Clearly, g6 (a, b, c) has the highest coefficient equal to zero. By P 3.76-(a) in Volume
1, it suffices to prove that g6 (a, 1, 1) ≥ 0 and g6 (0, b, c) ≥ 0 for all a, b, c ≥ 0. We
have
a(a − 1)2
P(a, 1, 1) = ,
9
232 Vasile Cîrtoaje

hence

a(a − 1)2 (67a3 + 190a2 + 67a + 180)


g6 (a, 1, 1) = f6 (a, 1, 1)−45P 2 (a, 1, 1) = ≥ 0.
9
Also, we have
(b + c)(b − c)2
P(0, b, c) = ,
9
hence

g6 (0, b, c) = f6 (0, b, c) − 45P 2 (0, b, c)


(b − c)2 [67(b4 + c 4 ) + 162bc(b2 + c 2 ) + 145b2 c 2 ]
= ≥ 0.
9

P 1.172. If a, b, c are real numbers, no two of which are zero, then

a2 − 7bc b2 − 7ca c 2 − 7ab 9(ab + bc + ca)


+ 2 + 2 + ≥ 0.
b2 + c 2 a + b2 a + b2 a2 + b2 + c 2
(Vasile Cîrtoaje, 2014)

Solution. Let
p = a + b + c, q = ab + bc + ca, r = abc.
Write the inequality as f8 (a, b, c) ≥ 0, where
X
f8 (a, b, c) =(a + b + c )
2 2 2
(a2 − 7bc)(a2 + b2 )(a2 + c 2 )
Y
+ 9(ab + bc + ca) (b2 + c 2 )

is a symmetric homogeneous polynomial of degree eight. Notice that any symmet-


ric homogeneous polynomial of degree eight f8 (a, b, c) can be written in the form

f8 (a, b, c) = A(p, q)r 2 + B(p, q)r + C(p, q),

where the highest polynomial A(p, q) has the form

A(p, q) = αp2 + βq.

Since
X
f8 (a, b, c) =(p2 − 2q) (a2 − 7bc)(p2 − 2q − c 2 )(p2 − 2q − b2 )
Y
+ 9q (p2 − 2q − a2 ),
Symmetric Rational Inequalities 233

f8 (a, b, c) has the same highest polynomial as


X
g8 (a, b, c) =(p2 − 2q) (a2 − 7bc)(−c 2 )(−b2 ) + 9q(−a2 )(−b2 )(−c 2 )
€ X Š
=(p − 2q) 3r − 7
2 2
b c − 9qr 2 ;
3 3

that is,
A(p, q) = (p2 − 2q)(3 − 21) − 9q = −9(p2 − 3q).
Since A(p, q) ≤ 0 for all real a, b, c, it suffices to prove the original inequality for
b = c = 1 (see Lemma below). We need to show that
a2 − 7 2(7a − 1) 9(2a + 1)
− + ≥ 0,
2 a2 + 1 a2 + 2
which is equivalent to
(a − 1)2 (a + 2)2 (a2 − 2a + 3) ≥ 0.
The equality holds for a = b = c, and also for −a/2 = b = c (or any cyclic permu-
tation).
Lemma. Let
p = a + b + c, q = ab + bc + ca, r = abc,
and let f8 (a, b, c) be a symmetric homogeneous polynomial of degree eight written in
the form
f8 (a, b, c) = A(p, q)r 2 + B(p, q)r + C(p, q),
where A(p, q) ≤ 0 for all real a, b, c. The inequality f8 (a, b, c) ≥ 0 holds for all real
numbers a, b, c if and only if f8 (a, 1, 1) ≥ 0 for all real a.
Proof. For fixed p and q,
h8 (r) = A(p, q)r 2 + B(p, q)r + C(p, q)
is a concave quadratic function of r which is minimum when r is minimum or
maximum; that is, according to P 2.53 in Volume 1, when two of a, b, c are equal.
Thus, the inequality f8 (a, b, c) ≥ 0 holds for all real numbers a, b, c if and only if
f8 (a, 1, 1) ≥ 0 and f8 (a, 0, 0) ≥ 0 for all real a. The last condition is not necessary
because it follows from the first condition as follows:
f8 (a, 0, 0) = lim f8 (a, t, t) = lim t 8 f8 (a/t, 1, 1) ≥ 0.
t→0 t→0

Notice that A(p, q) is called the highest polynomial of f8 (a, b, c).


Remark. This Lemma can be extended for the case where the highest polynomial
A(p, q is not nonnegative for all real a, b, c.
• The inequality f8 (a, b, c) ≥ 0 in the preceding Lemma holds for all real numbers
a, b, c satisfying
A(p, q) ≤ 0
if and only if f8 (a, 1, 1) ≥ 0 for all real a satisfying A(a + 2, 2a + 1) ≤ 0.
234 Vasile Cîrtoaje

P 1.173. If a, b, c are nonnegative real numbers, no two of which are zero, then

a2 − 4bc b2 − 4ca c 2 − 4ab 9(ab + bc + ca) 9


+ 2 + 2 + ≥ .
b2 + c 2 c + a2 a + b2 a2 + b2 + c 2 2
(Vasile Cîrtoaje, 2014)

Solution. Let
p = a + b + c, q = ab + bc + ca, r = abc.
Write the inequality as f8 (a, b, c) ≥ 0, where
X
f8 (a, b, c) =2(a + b + c )
2 2 2
(a2 − 4bc)(a2 + b2 )(a2 + c 2 )
Y
+ 9(2ab + 2bc + 2ca − a2 − b2 − c 2 ) (b2 + c 2 )

is a symmetric homogeneous polynomial of degree eight. Any symmetric homoge-


neous polynomial of degree eight can be written in the form

f8 (a, b, c) = A(p, q)r 2 + B(p, q)r + C(p, q),

where A(p, q) = αp2 + βq is called the highest polynomial of f8 (a, b, c). From
X
f8 (a, b, c) =2(p2 − 2q) (a2 − 4bc)(p2 − 2q − c 2 )(p2 − 2q − b2 )
Y
+ 9(4q − p2 (p2 − 2q − a2 ),

it follows that f8 (a, b, c) has the same highest polynomial as


X
g8 (a, b, c) =2(p2 − 2q) (a2 − 4bc)b2 c 2 + 9(4q − p2 )(−a2 b2 c 2 )
€ X Š
=2(p2 − 2q) 3r 2 − 4 b3 c 3 − 9(4q − p2 )r 2 ;

that is,
A(p, q) = 2(p2 − 2q)(3 − 12) − 9(4q − p2 ) = −9p2 .
Since A(p, q) ≤ 0 for all a, b, c ≥ 0, it suffices to prove the original inequality for
b = c = 1, and for a = 0 (see Lemma below).
For b = c = 1, the original inequality becomes

a2 − 4 2(4a − 1) 9(2a + 1) 9
− + ≥ ,
2 a2 + 1 a2 + 2 2
which is equivalent to
a(a + 4)(a − 1)4 ≥ 0.
For a = 0, the original inequality turns into

b2 c 2 5bc 9
+ 2+ 2 ≥ .
c 2 b b +c 2 2
Symmetric Rational Inequalities 235

Substituting
b c
x= + , x ≥ 2,
c b
the inequality becomes
5 9
(x 2 − 2) + ≥ ,
x 2
(x − 2)(2x 2 + 4x − 5) ≥ 0.
Thus, the proof is completed. The equality holds for a = b = c, and also for a = 0
and b = c (or any cyclic permutation).
Lemma. Let
p = a + b + c, q = ab + bc + ca, r = abc,
and let f8 (a, b, c) be a symmetric homogeneous polynomial of degree eight written in
the form
f8 (a, b, c) = A(p, q)r 2 + B(p, q)r + C(p, q),
where A(p, q) ≤ 0 for all a, b, c ≥ 0. The inequality f8 (a, b, c) ≥ 0 holds for all
a, b, c ≥ 0 if and only if the inequalities f8 (a, 1, 1) ≥ 0 and f8 (0, b, c) ≥ 0 hold for all
a, b, c ≥ 0.
Proof. For fixed p and q,

h8 (r) = A(p, q)r 2 + B(p, q)r + C(p, q)

is a concave quadratic function of r. Therefore, h8 (r) is minimum when r is mini-


mum or maximum; that is, according to P 3.57 in Volume 1, when b = c or a = 0.
Thus, the conclusion follows. Notice that A(p, q) is called the highest polynomial of
f8 (a, b, c).
Remark. This Lemma can be extended for the case where the highest polynomial
A(p, q is not nonnegative for all a, b, c ≥ 0.
• The inequality f8 (a, b, c) ≥ 0 in the preceding Lemma holds for all a, b, c ≥ 0
satisfying A(p, q) ≤ 0 if and only if the inequalities f8 (a, 1, 1) ≥ 0 and f8 (0, b, c) ≥ 0
hold for all a, b, c ≥ 0 satisfying A(a + 2, 2a + 1) ≤ 0 and A(b + c, bc) ≤ 0.

P 1.174. If a, b, c are real numbers such that abc 6= 0, then

(b + c)2 (c + a)2 (a + b)2 10(a + b + c)2


+ + ≥ 2 + .
a2 b2 c2 3(a2 + b2 + c 2 )

(Vasile Cîrtoaje and Michael Rozenberg, 2014)


236 Vasile Cîrtoaje

Solution. Let
p = a + b + c, q = ab + bc + ca, r = abc.
By the Cauchy-Schwarz inequality, we have
P 2 P 2 P 2
X (b + c)2 (b + c)2 2 a + ab 2(p2 − q)2
≥ P =P P = .
a2 a2 (b + c)2 a2 b2 + abc a q2 − pr

Therefore, it suffices to show that

2(p2 − q)2 10p2


≥ 2 + ,
q2 − pr 3(p2 − 2q)
which is equivalent to
3(p2 − q)2 8p2 − 6q
≥ 2 .
q2 − pr p − 2q
Using Schur’s inequality
p3 + 9r ≥ 4pq,
we get
4pq − p3 p4 − 4p2 q + 9q2
q2 − pr ≤ q2 − p · = .
9 9
Thus, it suffices to prove that

27(p2 − q)2 8p2 − 6q


≥ 2 ,
p4 − 4p2 q + 9q2 p − 2q
which is equivalent to the obvious inequality

p2 (p2 − 3q)(19p2 − 13q) ≥ 0.

The equality holds for a = b = c.

P 1.175. Let a, b, c be real numbers such that ab + bc + ca ≥ 0 and no two of which


are zero. Prove that
a b c 3
(a) + + ≥ ;
b+c c+a a+b 2
(b) i f ab ≤ 0, then
a b c
+ + ≥ 2.
b+c c+a a+b
(Vasile Cîrtoaje, 2014)
Symmetric Rational Inequalities 237

Solution. Let as show first that b + c 6= 0, c + a 6= 0 and a + b 6= 0. Indeed, if


b + c = 0, then ab + bc + ca ≥ 0 yields bc ≥ 0, hence b = c = 0, which is not
possible (because, by hypothesis, at most one of a, b, c can be zero).
(a) Use the SOS method. Write the inequality as follows:
X a  9
+1 ≥ ,
b+c 2
”X — X 1 ‹

(b + c) ≥ 9,
b+c
Xa + b a + c ‹
+ − 2 ≥ 0,
a+c a+b
X (b − c)2
≥ 0,
(a + b)(a + c)
X (b − c)2
≥ 0.
a2 + (ab + bc + ca)
Clearly, the last inequality is true. The equality holds for a = b = c 6= 0.
(b) From ab + bc + ca ≥ 0, it follows that if one of a, b, c is zero, then the
others are the same sign. In this case, the desired inequality is trivial. Consider
further that abc 6= 0. Since the problem remains unchanged by replacing a, b, c
with −a, −b, −c, it suffices to consider

a < 0 < b ≤ c.

First Solution. We will show that

F (a, b, c) > F (0, b, c) ≥ 2,

where
a b c
F (a, b, c) = + + .
b+c c+a a+b
The right inequality is true because

b c
F (0, b, c) = + ≥ 2.
c b
Since
1
• ˜
b c
F (a, b, c) − F (0, b, c) = a − − ,
b + c c(c + a) b(a + b)
the left inequality is true if

b c 1
+ > .
c(c + a) b(a + b) b+c
238 Vasile Cîrtoaje

From ab + bc + ca ≥ 0, we get
−ca −ab
c+a≥ > 0, a+b≥ > 0,
b c
hence
b b c c
> 2, > 2.
c(c + a) c b(a + b) b
Therefore, it suffices to prove that
b c 1
+ ≥ .
c 2 b2 b+c
Indeed, by virtue of the AM-GM inequality, we have
b c 1 2 1
+ 2− ≥ p − p > 0.
c 2 b b+c bc 2 bc
This completes the proof. The equality holds for a = 0 and b = c, or b = 0 and
a = c.
Second Solution. From b + c > 0 and

(b + c)(a + b) = b2 + (ab + bc + ca) > 0,

(b + c)(c + a) = c 2 + (ab + bc + ca) > 0,


it follows that
a + b > 0, c + a > 0.
By virtue of the Cauchy-Schwarz inequality and AM-GM inequality, we have
a b c a (b + c)2
+ + ≥ +
b+c c+a a+b b + c b(c + a) + c(a + b)
a (b + c)2
= +
b + c 2bc + a(b + c)
a (b + c)2
> +
2a + b + c (b + c)2
+ a(b + c)
2
4a 2(b + c)
> + = 2.
2a + b + c 2a + b + c

P 1.176. If a, b, c are nonnegative real numbers, then


a b c ab + bc + ca
+ + ≥ .
7a + b + c 7b + c + a 7c + a + b (a + b + c)2
(Vasile Cîrtoaje, 2014)
Symmetric Rational Inequalities 239

First Solution. Use the SOS method. Write the inequality as follows:
X• 2a a(b + c)
˜
− ≥ 0,
7a + b + c (a + b + c)2
X a[(a − b) + (a − c)](a − b − c)
≥ 0,
7a + b + c
X a(a − b)(a − b − c) X a(a − c)(a − b − c)
+ ≥ 0,
7a + b + c 7a + b + c
X a(a − b)(a − b − c) X b(b − a)(b − c − a)
+ ≥ 0,
7a + b + c 7b + c + a
a(a − b − c) b(b − c − a)
X • ˜
(a − b) − ≥ 0,
7a + b + c 7b + c + a
X
(a − b)2 (a2 + b2 − c 2 + 14ab)(a + b + 7c) ≥ 0.
Since
a2 + b2 − c 2 + 14ab ≥ (a + b)2 − c 2 = (a + b + c)(a + b − c),
it suffices to show that
X
(a − b)2 (a + b − c)(a + b + 7c) ≥ 0.

Assume that a ≥ b ≥ c. It is enough to show that

(a − c)2 (a − b + c)(a + 7b + c) + (b − c)2 (−a + b + c)(7a + b + c) ≥ 0.

For the nontrivial case b > 0, we have

a2 a
(a − c) ≥ 2 (b − c)2 ≥ (b − c)2 .
2
b b
Thus, it suffices to prove that

a(a − b + c)(a + 7b + c) + b(−a + b + c)(7a + b + c) ≥ 0.

Since
a(a + 7b + c) ≥ b(7a + b + c),
we have
a(a − b + c)(a + 7b + c) + b(−a + b + c)(7a + b + c) ≥
≥ b(a − b + c)(7a + b + c) + b(−a + b + c)(7a + b + c)
= 2bc(7a + b + c) ≥ 0.
This completes the proof. The equality holds for a = b = c, and also for a = 0 and
b = c (or any cyclic permutation).
240 Vasile Cîrtoaje

Second Solution. Assume that

a ≤ b ≤ c, a + b + c = 3,

and use the substitution


2a + 1 2b + 1 2c + 1
x= , y= , z= .
3 3 3
We have b + c ≥ 2 and
1
≤ x ≤ y ≤ z, x + y + z = 3, x ≤ 1, y + z ≥ 2.
3
The inequality can be written as follows:

a b c 9 − a2 − b2 − c 2
+ + ≥ ,
2a + 1 2b + 1 2c + 1 6
a2 + b2 + c 2 1 1 1
≥ + + ,
3 2a + 1 2b + 1 2c + 1
(2a + 1)2 + (2b + 1)2 + (2c + 1)2 − 15 1 1 1
≥ + + ,
12 2a + 1 2b + 1 2c + 1
1 1 1
 ‹
9(x + y + z ) ≥ 4
2 2 2
+ + + 15.
x y z
We will use the mixing variables method. More precisely, we will show that

E(x, y, z) ≥ E(x, t, t) ≥ 0,

where
t = ( y + z)/2 = (3 − x)/2,
1 1 1
 ‹
E(x, y, z) = 9(x + y + z ) − 4
2 2 2
+ + − 15.
x y z
We have
1 1 2
 ‹
E(x, y, z) − E(x, t, t) = 9( y + z − 2t ) − 4
2 2 2
+ −
y z t
( y − z)2 [9 yz( y + z) − 8]
= ≥0
2 yz( y + z)

since
9 yz = (2b + 1)(2c + 1) ≥ 2(b + c) + 1 ≥ 5, y + z ≥ 2.
Also,

4 8 (x − 1)2 (3x − 1)(8 − 3x)


E(x, t, t) = 9x 2 + 2t 2 − 15 − − = ≥ 0.
x t 2x(3 − x)
Symmetric Rational Inequalities 241

Third Solution. Write the inequality as f5 (a, b, c) ≥ 0, where f5 (a, b, c) is a sym-


metric homogeneous inequality of degree five. According to P 3.68-(a) in Volume
1, it suffices to prove the inequality for a = 0 and for b = c = 1, when the inequality
is equivalent to
(b − c)2 (b2 + c 2 + 11bc) ≥ 0

and
a(a − 1)2 (a + 14) ≥ 0,

respectively.

P 1.177. If a, b, c are positive real numbers such that abc = 1, then

a+b+c 1 1 1 8
+ + + ≥ .
30 a+1 b+1 c+1 5

(Vasile Cîrtoaje, 2018)

p
Solution. Assume that a ≥ b ≥ c, which involves ab ≥ 1. Since a + b ≥ 2 ab and
p p p
1 1 2 ( a − b)2 ( ab − 1)
+ −p = p ≥ 0,
a+1 b+1 ab + 1 (a + 1)(b + 1)( ab + 1)

it suffices to show that


p
2 ab + c 2 1 8
+p + ≥ .
30 ab + 1 c + 1 5
p
Substituting ab = 1/t, which implies c = t 2 , the inequality becomes

t3 + 2 2t 1 8
+ + 2 ≥ ,
30t t +1 t +1 5

t 6 + t 5 + 13t 4 − 45t 3 + 44t 2 − 16t + 2 ≥ 0,

(t − 1)2 [t 4 + 3t 3 + 2(3t − 1)2 ] ≥ 0.

The equality holds for a = b = c = 1.


242 Vasile Cîrtoaje

P 1.178. Let f be a real function defined on an interval I, and let x, y, s ∈ I such that
x + m y = (1 + m)s, where m > 0. Prove that the inequality

f (x) + m f ( y) ≥ (1 + m) f (s)

holds if and only if


h(x, y) ≥ 0,
where
g(x) − g( y) f (u) − f (s)
h(x, y) = , g(u) = .
x−y u−s
(Vasile Cîrtoaje, 2006)

Solution. From

f (x) + m f ( y) − (1 + m) f (s) = [ f (x) − f (s)] + m[ f ( y) − f (s)]


= (x − s)g(x) + m( y − s)g( y)
m
= (x − y)[g(x) − g( y)]
1+m
m
= (x − y)2 h(x, y),
1+m
the conclusion follows.
Remark. From the proof above, it follows that P 1.178 is also valid for the case
where f is defined on I \ {u0 } and x, y, s 6= u0 .

P 1.179. Let a, b, c ≤ 8 be real numbers such that a + b + c = 3. Prove that

13a − 1 13b − 1 13c − 1 3


+ 2 + ≤ .
a2 + 23 b + 23 c 2 + 23 2
(Vasile Cîrtoaje, 2008)

Solution. Write the inequality as

−3
f (a) + f (b) + f (c) ≥ ,
2
where
1 − 13u
f (u) = .
u2 + 23
Assume that a ≤ b ≤ c, and denote

b+c
s= .
2
Symmetric Rational Inequalities 243

We have
3−a
s= , 1 ≤ s ≤ 8.
2
We claim that
f (b) + f (c) ≥ 2 f (s).
To show this, according to P 1.178, it suffices to show that

h(b, c) ≥ 0,

where
g(b) − g(c) f (u) − f (s)
h(b, c) = , g(u) = .
b−c u−s
We have
(13s − 1)u − s − 299
g(u) = ,
(s2 + 23)(u2 + 23)
(1 − 13s)bc + (s + 299)(b + c) + 23(13s − 1)
h(b, c) = .
(s2 + 23)(b2 + 23)(c 2 + 23)
Since 1 − 13s < 0 and bc ≤ s2 , we get

(1 − 13s)s2 + (s + 299)(2s) + 23(13s − 1)


h(b, c) ≥
(s2 + 23)(b2 + 23)(c 2 + 23)
−13s3 + 3s2 + 897s − 23
=
(s2 + 23)(b2 + 23)(c 2 + 23)
−13s3 + 3s2 + 897s − 712
>
(s2 + 23)(b2 + 23)(c 2 + 23)
(8 − s)(13s2 + 101s − 89)
= ≥ 0.
(s2 + 23)(b2 + 23)(c 2 + 23)

Therefore,

3 3 3−a 3
 ‹
f (a) + f (b) + f (c) + ≥ f (a) + 2 f (s) + = f (a) + 2 f +
2 2 2 2
1 − 13a 4(13a − 37) 3
= 2 + +
a + 23 a2 − 6a + 101 2
3(a − 1)2 (a + 11)2
= ≥ 0.
2(a2 + 23)(a2 − 6a + 101)

The equality holds for a = b = c = 1, and also for a = −11 and b = c = 7 (or any
cyclic permutation).
244 Vasile Cîrtoaje

3
P 1.180. Let a, b, c 6= be nonnegative real numbers such that a + b + c = 3. Prove
4
that
1−a 1− b 1−c
+ + ≥ 0.
(4a − 3)2 (4b − 3)2 (4c − 3)2
(Vasile Cîrtoaje, 2006)

Solution. Write the inequality as

f (a) + f (b) + f (c) ≥ 0,

where
1−u
f (u) = .
(4u − 3)2
Assume that a ≤ b ≤ c, and denote

b+c
s= .
2
We have
3−a 3
s= , 1≤s≤ .
2 2
We claim that
f (b) + f (c) ≥ 2 f (s).
According to Remark from P 1.178, it suffices to show that

h(b, c) ≥ 0,

where
g(b) − g(c) f (u) − f (s)
h(b, c) = , g(u) = .
b−c u−s
We have
16(s − 1)u − 16s + 15
g(u) = ,
(4s − 3)2 (4u − 3)2
1 −32(s − 1)bc + 64s2 − 90s + 27
h(b, c) = .
8 (4s − 3)2 (4b − 3)2 (4c − 3)2
Since s − 1 ≥ 0 and bc ≤ s2 , we get

1 −32(s − 1)s2 + 64s2 − 90s + 27


h(b, c) ≥
8 (4s − 3)2 (4b − 3)2 (4c − 3)2
−32s3 + 96s2 − 90s + 27
=
(4s − 3)2 (4b − 3)2 (4c − 3)2
(3 − 2s)(3 − 4s)2
= ≥ 0.
(4s − 3)2 (4b − 3)2 (4c − 3)2
Symmetric Rational Inequalities 245

Therefore,

3−a
 ‹
f (a) + f (b) + f (c) ≥ f (a) + 2 f (s) = f (a) + 2 f
2
1−a a−1 12a(a − 1)2
= + = ≥ 0.
(4a − 3)2 (3 − 2a)2 (4a − 3)2 (3 − 2a)2

The equality holds for a = b = c = 1, and also for a = 0 and b = c = 3/2 (or any
cyclic permutation).

P 1.181. If a, b, c are the lengths of the sides of a triangle, then

a2 b2 c2 1
+ 2 + 2 ≥ .
4a + 5bc 4b + 5ca 4c + 5ab 3
2

(Vasile Cîrtoaje, 2009)

Solution. Use the highest coefficient method. Write the inequality as f6 (a, b, c) ≥
0, where
X Y
f6 (a, b, c) = 3 a (4b + 5ca)(4c + 5ab) −
2 2 2
(4a2 + 5bc)
X X
= −45a2 b2 c 2 − 25abc a3 + 40 a3 b3 .

Since f6 (a, b, c) has the highest coefficient

A = −45 − 75 + 120 = 0,

according to P 3.76-(b) in Volume 1, it suffices to prove the original inequality for


b = c = 1 and 0 ≤ a ≤ 2, and for a = b + c.
Case 1: b = c = 1, 0 ≤ a ≤ 2. The original inequality becomes

a2 2 1
+ ≥ ,
4a + 5 5a + 4 3
2

(2 − a)(a − 1)2 ≥ 0.
Case 2: a = b + c. Using the Cauchy-Schwarz inequality

b2 c2 (b + c)2
+ ≥ ,
4b2 + 5ca 4c 2 + 5ab 4(b2 + c 2 ) + 5a(b + c)

it suffices to show that


a2 (b + c)2 1
+ ≥ ,
4a2 + 5bc 4(b2 + c 2 ) + 5a(b + c) 3
246 Vasile Cîrtoaje

which is equivalent to
1 1 1
+ ≥ .
4(b2 + c ) + 13bc 9(b + c ) + 10bc
2 2 2 3(b + c 2 + 2bc)
2

Using the substitution


b c
x= + , x ≥ 2,
c b
the inequality becomes
1 1 1
+ ≥ ,
4x + 13 9x + 10 3(x + 2)
(x − 2)(3x − 4) ≥ 0.
The equality holds for an equilateral triangle, and for a degenerate triangle with
a/2 = b = c (or any cyclic permutation).

P 1.182. If a, b, c are the lengths of the sides of a triangle, then


1 1 1 3
+ 2 + 2 ≥ .
7a2 +b +c
2 2 7b + c + a
2 2 7c + a + b
2 2 (a + b + c)2
(Vo Quoc Ba Can, 2010)

Solution. Use the highest coefficient method. Denote

p = a + b + c, q = ab + bc + ca,

and write the inequality as f6 (a, b, c) ≥ 0, where


X Y
f6 (a, b, c) = p2 (7b2 + c 2 + a2 )(7c 2 + a2 + b2 ) − 3 (7a2 + b2 + c 2 )
X Y
= p2 (6b2 + p2 − 2q)(6c 2 + p2 − 2q) − 3 (6a2 + p2 − 2q).

Since f6 (a, b, c) has the highest coefficient

A = −3 · 63 < 0,

according to P 3.76-(b) in Volume 1, it suffices to prove the original inequality for


b = c = 1 and 0 ≤ a ≤ 2, and for a = b + c.
Case 1: b = c = 1, 0 ≤ a ≤ 2. The original inequality reduces to
1 2 3
+ ≥ ,
7a2 + 2 a2 + 8 (a + 2)2

a(8 − a)(a − 1)2 ≥ 0.


Symmetric Rational Inequalities 247

Case 2: a = b + c. Write the inequality as

1 1 1 3
+ 2 + 2 ≥ .
4(b2 + c ) + 7bc 4b + c + bc 4c + b + bc
2 2 2 2(b + c)2

Since
3 1 3 1 5
− ≤ − = ,
2(b + c)2 4(b2 + c 2 ) + 7bc 2(b + c)2 4(b2 + c 2 ) + 8bc 4(b + c)2

it suffices to show that


1 1 5
+ 2 ≥ ,
4b2 + c + bc 4c + b + bc
2 2 4(b + c)2

which is equivalent to

4[5(b2 + c 2 ) + 2bc][(b2 + c 2 ) + 2bc] ≥ 5(4b2 + c 2 + bc)(4c 2 + b2 + bc),

4[5(b2 + c 2 )2 + 12bc(b2 + c 2 ) + 4b2 c 2 ] ≥ 5[4(b2 + c 2 )2 + 5bc(b2 + c 2 ) + 10b2 c 2 ],


bc[23(b − c)2 + 12bc] ≥ 0.
The equality holds for an equilateral triangle, and for a degenerate triangle with
a = 0 and b = c (or any cyclic permutation).

P 1.183. Let a, b, c be the lengths of the sides of a triangle. If k > −2, then
X a(b + c) + (k + 1)bc 3(k + 3)
≤ .
b2 + kbc + c2 k+2
(Vasile Cîrtoaje, 2009)

Solution. Use the highest coefficient method. Let

p = a + b + c, q = ab + bc + ca.

Write the inequality as f6 (a, b, c) ≥ 0, where


Y
f6 (a, b, c) = 3(k + 3) (b2 + kbc + c 2 )
X
−(k + 2) [a(b + c) + (k + 1)bc](c 2 + kca + a2 )(a2 + kab + b2 ).
From Y
f6 (a, b, c) = 3(k + 3) (p2 − 2q + kbc − a2 )
X
−(k + 2) (q + kbc)(p2 − 2q + kca − b2 )(p2 − 2q + kab − c 2 ),
248 Vasile Cîrtoaje

it follows that f6 (a, b, c) has the same highest coefficient A as f (a, b, c), where

f (a, b, c) = 3(k + 3)P3 (a, b, c) − k(k + 2)P2 (a, b, c),


Y X
P3 (a, b, c) = (kbc − a ),
2
P2 (a, b, c) = bc(kca − b2 )(kab − c 2 ).
According to Remark 2 from the proof of P 2.75 in Volume 1, we have

A = 3(k + 3)P3 (1, 1, 1) − k(k + 2)P2 (1, 1, 1)


= 3(k + 3)(k − 1)3 − 3k(k + 2)(k − 1)2 = −9(k − 1)2 ≤ 0.

Taking into account P 3.76-(b) in Volume 1, it suffices to prove the original inequal-
ity for b = c = 1 and 0 ≤ a ≤ 2, and for a = b + c.
Case 1: b = c = 1, 0 ≤ a ≤ 2. The original inequality reduces to

2a + k + 1 2(k + 2)a + 2 3(k + 3)


+ 2 ≤ ,
k+2 a + ka + 1 k+2
a − k − 4 (k + 2)a + 1
+ 2 ≤ 0,
k+2 a + ka + 1
(2 − a)(a − 1)2 ≥ 0.
Case 2: a = b + c. Write the inequality as follows:
X • a(b + c) + (k + 1)bc ˜
3
−1 ≤ ,
b + kbc + c
2 2 k+2
X ab + bc + ca − b2 − c 2 3
≤ ,
b2 + kbc + c2 k+2
3bc bc − c 2 bc − b2 3
+ 2 + 2 ≤ .
b + kbc + c
2 2 b + (k + 2)(bc + c ) c + (k + 2)(bc + b ) k + 2
2 2

Since
3bc 3
≤ ,
b2 + kbc + c 2 k+2
it suffices to prove that

bc − c 2 bc − b2
+ ≤ 0.
b2 + (k + 2)(bc + c 2 ) c 2 + (k + 2)(bc + b2 )

This reduces to the obvious inequality

(b − c)2 (b2 + bc + c 2 ) ≥ 0.

The equality holds for an equilateral triangle, and for a degenerate triangle with
a/2 = b = c (or any cyclic permutation).
Symmetric Rational Inequalities 249

P 1.184. Let a, b, c be the lengths of the sides of a triangle. If k > −2, then
X 2a2 + (4k + 9)bc 3(4k + 11)
≤ .
b2 + kbc + c 2 k+2
(Vasile Cîrtoaje, 2009)

Solution. Use the highest coefficient method. Let

p = a + b + c, q = ab + bc + ca.

Write the inequality as f6 (a, b, c) ≥ 0, where


Y
f6 (a, b, c) = 3(4k + 11) (b2 + kbc + c 2 )
X
−(k + 2) [2a2 + (4k + 9)bc](c 2 + kca + a2 )(a2 + kab + b2 ).
From Y
f6 (a, b, c) = 3(4k + 11) (p2 − 2q + kbc − a2 )
X
−(k + 2) [2a2 + (4k + 9)bc](p2 − 2q + kca − b2 )(p2 − 2q + kab − c 2 ),
it follows that f6 (a, b, c) has the same highest coefficient A as f (a, b, c), where

f (a, b, c) = 3(4k + 11)P3 (a, b, c) − (k + 2)P2 (a, b, c),


Y
P3 (a, b, c) = (kbc − a2 ),
X
P2 (a, b, c) = [2a2 + (4k + 9)bc](kca − b2 )(kab − c 2 ).
According to Remark 2 from the proof of P 2.75 in Volume 1, we have

A = 3(4k + 11)P3 (1, 1, 1) − (k + 2)P2 (1, 1, 1)


= 3(4k + 11)(k − 1)3 − 3(k + 2)(4k + 11)(k − 1)2
= −9(4k + 11)(k − 1)2 ≤ 0.

Taking into account P 3.76-(b) in Volume 1, it suffices to prove the original inequal-
ity for b = c = 1 and 0 ≤ a ≤ 2, and for a = b + c.
Case 1: b = c = 1, 0 ≤ a ≤ 2. The original inequality reduces to

2a2 + 4k + 9 2(4k + 9)a + 4 3(4k + 11)


+ ≤ ,
k+2 a2 + ka + 1 k+2

a2 − 4k − 12 (4k + 9)a + 2
+ 2 ≤ 0,
k+2 a + ka + 1
(2 − a)(a − 1)2 ≥ 0,
250 Vasile Cîrtoaje

Case 2: a = b + c. Write the inequality as follows:


X • 2a2 + (4k + 9)bc ˜
9
− 4 ≤ ,
b2 + kbc + c 2 k+2
X 2a2 − 4b2 − 4c 2 + 9bc 9
≤ ,
b2 + kbc + c 2 k+2
13bc − 2b2 − 2c 2 bc − 2b2 + c 2 bc − 2c 2 + b2 9
+ + ≤ .
b + kbc + c
2 2 b + (k + 2)(bc + c ) c + (k + 2)(bc + b ) k + 2
2 2 2 2

Since
9 13bc − 2b2 − 2c 2 (2k + 13)(b − c)2
− =
k+2 b2 + kbc + c 2 (k + 2)(b2 + kbc + c 2 )
and
bc − 2b2 + c 2 bc − 2c 2 + b2
+ =
b2 + (k + 2)(bc + c 2 ) c 2 + (k + 2)(bc + b2 )
(b − c)2 (b2 + c 2 + 3bc) − 2(k + 2)(b2 − c 2 )2
= ,
[b2 + (k + 2)(bc + c 2 )][c 2 + (k + 2)(bc + b2 ]
we only need to show that

2k + 13 2(k + 2)(b + c)2 − b2 − c 2 − 3bc


+ ≥ 0.
(k + 2)(b2 + kbc + c 2 ) [b2 + (k + 2)(bc + c 2 )][c 2 + (k + 2)(bc + b2 ]
Using the substitution
b c
x= + , x ≥ 2,
c b
the inequality can be written as

2k + 13 (2k + 3)x + 4k + 5
+ ≥ 0,
(k + 2)(x + k) (k + 2)x + (k + 2)(k + 3)x + 2k2 + 6k + 5
2

which is equivalent to

4(k + 2)(k + 4)x 2 + 2(k + 2)B x + C ≥ 0,

where
B = 2k2 + 13k + 22, C = 8k3 + 51k2 + 98k + 65.
Since
B = 2(k + 2)2 + 5(k + 2) + 4 > 0,
C = 8(k + 2)3 + 2k2 + (k + 1)2 > 0,
the conclusion follows. The equality holds for an equilateral triangle, and for a
degenerate triangle with a/2 = b = c (or any cyclic permutation).
Symmetric Rational Inequalities 251

P 1.185. If a, b, c are nnonnegative numbers such that abc = 1, then


1 1 1 1
+ + + ≥ 1.
(a + 1)2 (b + 1)2 (c + 1)2 2(a + b + c − 1)

(Vasile Cîrtoaje, 2018)

Solution. Let
p = a + b + c, q = ab + bc + ca.
By the Cauchy-Schwarz inequality, we have

( bc)2
P
X 1 X b2 c 2
= ≥P
(a + 1)2 (1 + bc)2 (1 + bc)2

q2
= .
q2 + 2q − 2p + 3
Thus we only need to show that

q2 1
+ ≥ 1,
q + 2q − 2p + 3 2(p − 1)
2

which is equivalent to
(q − 2p + 3)2 ≥ 0.
The equality occurs for a = b = c = 1.

P 1.186. If a, b, c are positive real numbers such that

a ≤ b ≤ c, a2 bc ≥ 1,

then
1 1 1 3
+ + ≥ .
1 + a3 1 + b3 1 + c 3 1 + abc
(Vasile Cîrtoaje, 2008)

Solution. Since
1 1 2 (x − y)2 (x y − 1)
+ − = ,
1 + x2 1 + y2 1 + x y (1 + x 2 )(1 + y 2 )(1 + x y)

we have
1 1 2
+ ≥ ,
1+ b 3 1+c 3 1 + t3
where p
t= bc, at ≥ 1, t ≥ 1, t ≥ a.
252 Vasile Cîrtoaje

So, we only need to show that

1 2 3
+ ≥ ,
1+a 3 1+ t 3 1 + at 2
which is equivalent to
a(t 2 − a2 ) 2t 2 (t − a)
≥ ,
1 + a3 1 + t3
(t − a)2 [at 2 (2a + t) − a − 2t] ≥ 0.
This is true since

at 2 (2a + t) − a − 2t ≥ t(2a + t) − a − 2t = (t − 1)2 + (at − 1) + a(t − 1) ≥ 0.

The equality occurs for a = b = c ≥ 1.

Remark 1. The inequality is true for the weaker condition

a8/5 bc ≥ 1,

that is a4 t 5 ≥ 1. Since bc ≥ 1, it suffices to show that at 2 (2a + t) − a − 2t ≥ 0.


This is true if the following homogeneous inequality is true:

at 2
(2a + t) ≥ a + 2t,
(a4 t 5 )1/3

that is
t 1/3 (2a + t) ≥ a1/3 (a + 2t).
Setting a = 1 and t = z 3 ≥ 1, the inequality becomes as follows:

z(2 + z 3 ) ≥ 1 + 2z 3 ,

z 4 − 1 ≥ 2z(z 2 − 1),
(z 2 − 1)(z − 1)2 ≥ 0.

Remark 2. The inequality is also true for the condition

a4 b5 ≥ 1.

Indeed, if a4 b5 ≥ 1, then b ≥ 1, bc ≥ b2 ≥ 1 and

a4 (bc)5/2 ≥ 1,

which is equivalent to to the condition a8/5 bc ≥ 1 from Remark 1.

Remark 3. From P 1.186, the following statement follows (V. Cirtoaje and V. Vor-
nicu):
Symmetric Rational Inequalities 253

• If a, b, c, d are positive real numbers such that

a ≥ b ≥ c ≥ d, abcd ≥ 1,

then
1 1 1 3
+ + ≥ .
1+a 3 1+ b 3 1+c 3 1 + abc
This is valid because c ≤ b ≤ a and c 2 ba ≥ 1.

P 1.187. If a, b, c are positive real numbers such that

a ≤ b ≤ c, a2 c ≥ 1,

then
1 1 1 3
+ + ≥ .
1+a 3 1+ b 3 1+c 3 1 + abc
(Vasile Cîrtoaje, 2021)
Solution. Denote p
d= ac, d ≥ 1.
If d = 1, then ac = 1 and a2 c ≥ 1 yield a = b = c = 1, and the required inequality
is an equality. Consider next that d > 1. For fixed a and c, write the inequality as
f (b) ≥ 0, where
1 1 1 3
f (b) = + + − , b ∈ [a, c],
1+a 3 1+ b 3 1+c 3 1 + abc
and calculate the derivative
1 0 d2 b2
f (b) = −
3 (1 + d 2 b)2 (1 + b3 )2

(d b2 − 1)(b − d)[d(1 + b3 ) + b(d 2 b + 1)]


= .
(1 + d 2 b)2 (1 + b3 )2
1 p p
If a ≤ p , then f 0 (b) ≤ 0 for b ∈ [1/ d, d] and f 0 (b) ≥ 0 for b ∈ [a, 1/ d] ∪
d p p
[d, c], hence f (b) is decreasing on [1/ d, d] and increasing on [a, 1/ d] ∪ [d, c].
1
Thus, it suffices to show that f (a) ≥ 0 and f (d) ≥ 0. If a ≥ p , then f 0 (b) ≤ 0 for
d
b ∈ [a, d] and f 0 (b) ≥ 0 for b ∈ [d, c], f (b) is decreasing on [a, d] and increasing
on [d, c], hence it suffices to show that f (d) ≥ 0. In conclusion, we only need to
show that f (a) ≥ 0 and f (d) ≥ 0. Write the inequality f (a) ≥ 0 as follows:
2 1 3
+ ≥ ,
1+a 3 1+c 3 1 + a2 c
254 Vasile Cîrtoaje

2a2 (c − a) c(c 2 − a2 )
≥ ,
1 + a3 1 + c3
(c − a)2 [a2 c(a + 2c) − 2a − c] ≥ 0.
This is true because

a2 c(a + 2c) − 2a − c ≥ (a + 2c) − 2a − c = c − a ≥ 0.

Write now the inequality f (d) ≥ 0 as


1 1 2
+ ≥ .
1 + a3 1 + c 3 1 + (ac)3/2
Since
1 1 2 (x − y)2 (x y − 1)
+ − = ,
1 + x2 1 + y2 1 + x y (1 + x 2 )(1 + y 2 )(1 + x y)
the inequality is equivalent to
2  
a3/2 − c 3/2 (ac)3/2 − 1 ≥ 0.

This is true because


(ac)3 ≥ (a2 c)2 ≥ 1.
The equality occurs for a = b = c ≥ 1.

P 1.188. If a, b, c are positive real numbers such that

a ≤ b ≤ c, 2a + c ≥ 3,

then
1 1 1 3
+ + ≥ 2 .
3 + a2 3 + b2 3 + c 2 3 + a+b+c
3

(Vasile Cîrtoaje, 2021)

Solution. Denote
a+b+c
s= , s ≥ 1.
3
For fixed a and c, write the inequality as f (b) ≥ 0, where
1 1 1 3
f (b) = + + − , b ∈ [a, c],
3+a 2 3+ b 2 3+c 2 3 + s2
and calculate the derivative
1 0 s b (b − s)g(b)
f (b) = − = ,
2 (3 + s2 )2 (3 + b2 )2 (3 + s2 )2 (3 + b2 )2
Symmetric Rational Inequalities 255

where
g(b) = bs(b2 + bs + s2 + 6) − 9.
Denote
a+c
d= , d ≥ 1.
2
If d = 1, then a + c = 2 and 2a + c ≥ 3 yield a = b = c = 1, and the required
inequality is an equality. Consider next that d > 1. Since

b + 2d
s= ,
3
we have
2(b − d)
b−s = ,
3
b(b + 2d) 2 b(b + 2d) (b + 2d)2
• ˜
g(b) = b + + + 6 − 9.
3 3 9
Since g(b) is strictly increasing, g(0) = −9 and

g(d) = 3(d 4 + 2d 2 − 3) > 0,

there is an unique d1 ∈ (0, d) such that g(d1 ) = 0, g(b) ≤ 0 for b ≤ d1 and g(b) ≥ 0
for b ≥ d1 . If a ≤ d1 , then f 0 (b) ≤ 0 for b ∈ [d1 , d] and f 0 (b) ≥ 0 for b ∈
[a, d1 ]∪[d, c], hence f (b) is decreasing on [d1 , d] and increasing on [a, d1 ]∪[d, c].
Thus, it suffices to show that f (a) ≥ 0 and f (d) ≥ 0. If a ≥ d1 , then f 0 (b) ≤ 0 for
b ∈ [a, d] and f 0 (b) ≥ 0 for b ∈ [d, c], f (b) is decreasing on [a, d] and increasing
on [d, c], hence it suffices to show that f (d) ≥ 0. In conclusion, we only need to
show that f (a) ≥ 0 and f (d) ≥ 0. Denoting

2a + c
p= ,
3
we may write the inequality f (a) ≥ 0 as follows:

2 1 3
+ ≥ ,
3 + a2 3 + c 2 3 + p2

2(p2 − a2 ) c 2 − p2
≥ ,
3 + a2 3 + c2
(a − c)2 [(a + c)p + ac − 3] ≥ 0,
(a − c)2 (2a2 + 6ac + c 2 − 9) ≥ 0.
This is true because

2a2 + 6ac + c 2 − 9 = (2a + c)2 − 9 + 2a(c − a) ≥ 0.


256 Vasile Cîrtoaje

Write now the inequality f (d) ≥ 0 as follows:


1 1 2
+ ≥ ,
3+a 2 3+c 2 3 + d2
d 2 − a2 c2 − d 2
≥ ,
3 + a2 3 + c2
(a − c)2 [(a + c)d + ac) − 3] ≥ 0,
(a − c)2 (a2 + 4ac + c 2 − 6) ≥ 0.
This is true because

3(a2 + 4ac + c 2 ) − 18 ≥ 3(a2 + 4ac + c 2 ) − 2(2a + c)2 = (c − a)(c + 5a) ≥ 0.

The equality occurs for a = b = c ≥ 1, and also for a = b = 0 and c = 3.

P 1.189. If a, b, c are positive real numbers such that

a ≤ b ≤ c, 9a + 8b ≥ 17,

then
1 1 1 3
+ + ≥ 2 .
3+a 2 3+ b 2 3+c 2
3 + a+b+c
3

(Vasile Cîrtoaje, 2021)

Solution. From a ≤ b ≤ c and 9a + 8b ≥ 17, it follows that

1 ≤ b ≤ c, a + b + c ≥ 3.

As in the preceding P 1.188, denote

a+b+c
s= , 1 ≤ s ≤ c,
3
and, for fixed a and b, write the inequality as f (c) ≥ 0, where
1 1 1 3
f (c) = + + − , c ≥ b.
3 + a2 3 + b2 3 + c 2 3 + s2
We show that
f (c) ≥ f (b) ≥ 0.
Since
1 0 s c (c − s)[cs(c 2 + cs + s2 + 6) − 9]
f (c) = − = ≥ 0,
2 (3 + s2 )2 (3 + c 2 )2 (3 + s2 )2 (3 + c 2 )2
Symmetric Rational Inequalities 257

f (c) is increasing, therefore f (c) ≥ f (b). Denote

a + 2b
p= ,
3

Write now the inequality f (b) ≥ 0 as follows:

1 2 3
+ ≥ ,
3+a 2 3+ b 2 3 + p2

p2 − a2 2(b2 − p2 )
≥ ,
3 + a2 3 + b2
(a − b)2 [(a + b)p + ab − 3] ≥ 0 ,

(a − b)2 (a2 + 6ab + 2b2 − 9) ≥ 0 .


This is true if
16(a2 + 6ab + 2b2 ) ≥ (7a + 5b)2 ,
which is equivalent to
(b − a)(b + 220a) ≥ 0.
The equality occurs for a = b = c ≥ 1.

Remark. Actually, the inequality is valid for the weaker condition

3
ka + b ≥ k + 1, k = p − 1,
2

when the inequality

(k + 1)2 (a2 + 6ab + 2b2 ) ≥ 9(ka + b)2 ,

reduces to the form


a(b − a) ≥ 0.
3
The equality occurs for a = b = c ≥ 1, and also for a = 0 and b = c = p .
2

P 1.190. Let a, b, c, d be positive real numbers such that abcd = 1. Prove that
X 1
≤ 1.
1 + ab + bc + ca
258 Vasile Cîrtoaje

Solution. From
p p p
1 1 1 1 1 1 a+ b+ c
+ + ≥p +p +p = p ,
a b c bc ca ab abc
we get p
p p
p €p p p Š a+ b+ c
ab + bc + ca ≥ abc a+ b+ c = p .
d
Therefore,
p
X 1 X d
≤ p p p p = 1,
1 + ab + bc + ca a+ b+ c+ d
which is just the required inequality. The equality occurs for a = b = c = d = 1.

P 1.191. Let a, b, c, d be positive real numbers such that abcd = 1. Prove that
1 1 1 1
+ + + ≥ 1.
(1 + a)2 (1 + b)2 (1 + c)2 (1 + d)2
(Vasile Cîrtoaje, 1995)
First Solution. The inequality follows by summing the following inequalities (see
P 1.1):
1 1 1
+ ≥ ,
(1 + a)2 (1 + b)2 1 + ab
1 1 1 ab
+ ≥ = .
(1 + c)2 (1 + d)2 1 + cd 1 + ab
The equality occurs for a = b = c = d = 1.
Second Solution. Using the substitution
1 1 1 1
a= , b= , c= , d= ,
x4 y4 z4 t4
where x, y, z, t are positive real numbers such that x yz t = 1, the inequality be-
comes
x6 y6 z6 t6
‹2 +  ‹2 +  ‹2 +  ‹ ≥ 1.
1 1 1 1 2

x +
3 y +
3 z +
3 t +
3
x y z t
By the Cauchy-Schwarz inequality, we get
P 3 2 P 3 2
X x6 x x
‹2 ≥  ‹2 = P 6 .
+ 2+
P P
x 2 x x 2 y 2z2
1 1

x3 + x3 +
P
x x
Symmetric Rational Inequalities 259

Thus, it suffices to prove the homogeneous inequality


X X
2(x 3 y 3 + x 3 z 3 + x 3 t 3 + y 3 z 3 + y 3 t 3 + z 3 t 3 ) ≥ 2x yz t x2 + x 2 y 2z2.

We can get it by summing the inequalities


X
4(x y + x z + x t + y z + y t + z t ) ≥ 6x yz t
3 3 3 3 3 3 3 3 3 3 3 3
x2

and X
2(x 3 y 3 + x 3 z 3 + x 3 t 3 + y 3 z 3 + y 3 t 3 + z 3 t 3 ) ≥ 3 x 2 y 2z2,
Write these inequalities as
X
x 3 ( y 3 + z 3 + t 3 − 3 yz t) ≥ 0

and X
(x 3 y 3 + y 3 z 3 + z 3 x 3 − 3x 2 y 2 z 2 ) ≥ 0,
respectively. By the AM-GM inequality, we have

y 3 + z 3 + t 3 ≥ 3 yz t, x 3 y 3 + y 3 z 3 + z 3 x 3 ≥ 3x 2 y 2 z 2 .

Thus the conclusion follows.


Third Solution. Using the substitution
yz zt tx xy
a= , b= , c= , d= ,
x2 y2 z2 t2

where x, y, z, t are positive real numbers, the inequality becomes

x4 y4 z4 t4
+ + + ≥ 1.
(x 2 + yz)2 ( y 2 + z t)2 (z 2 + t x)2 (t 2 + x y)2

Using the Cauchy-Schwarz inequality two times, we deduce

x4 z4 x4 z4
+ ≥ +
(x 2 + yz)2 (z 2 + t x)2 (x 2 + y 2 )(x 2 + z 2 ) (z 2 + t 2 )(z 2 + x 2 )

1 x4 z4 x 2 + z2
 ‹
= 2 + ≥ 2 ,
x + z2 x 2 + y 2 z2 + t 2 x + y 2 + z2 + t 2
hence
x4 z4 x 2 + z2
+ ≥ .
(x 2 + yz)2 (z 2 + t x)2 x 2 + y 2 + z2 + t 2
Adding this to the similar inequality

y4 t4 y2 + t2
+ ≥ ,
( y 2 + z t)2 (t 2 + x y)2 x 2 + y 2 + z2 + t 2
260 Vasile Cîrtoaje

we get the required inequality.


Fourth Solution. Using the substitution
x y z t
a= , b= , c= , d= ,
y z t x

where x, y, z, t are positive real numbers, the inequality can be written as

y2 z2 t2 x2
+ + + ≥ 1.
(x + y)2 ( y + z)2 (z + t)2 (t + x)2

By the Cauchy-Schwarz inequality and the AM-GM inequality, we get

[ y( y + z)]2
P
X y2
≥P
(x + y)2 (x + y)2 ( y + z)2

[(x + y)2 + ( y + z)2 + (z + t)2 + (t + x)2 ]2


= ≥ 1.
4[(x + y)2 + (z + t)2 ][( y + z)2 + (t + x)2 ]

Remark. The following generalization holds true (Vasile Cîrtoaje, 2005):


p
• Let a1 , a2 , . . . , an be positive real numbers such that a1 a2 · · · an = 1. If k ≥ n−1,
then
1 1 1 n
+ + · · · + ≥ .
(1 + ka1 )2 (1 + ka2 )2 (1 + kan )2 (1 + k)2

1
P 1.192. Let a, b, c, d 6= be positive real numbers such that abcd = 1. Prove that
3
1 1 1 1
+ + + ≥ 1.
(3a − 1)2 (3b − 1)2 (3c − 1)2 (3d − 1)2

(Vasile Cîrtoaje, 2006)

First Solution. It suffices to show that


1 a−3
≥ −3 .
(3a − 1)2 a + b−3 + c −3 + d −3

This inequality is equivalent to

6a−2 + b−3 + c −3 + d −3 ≥ 9a−1 ,

which follows by the AM-GM inequality, as follows:


p
9
6a−2 + b−3 + c −3 + d −3 ≥ 9 a−12 b−3 c −3 d −3 = 9a−1 .
Symmetric Rational Inequalities 261

The equality occurs for a = b = c = d = 1.


Second Solution. Let a ≤ b ≤ c ≤ d. If a ≤ 2/3, then
1
≥ 1,
(3a − 1)2
and the desired inequality is clearly true. Otherwise, if 2/3 < a ≤ b ≤ c ≤ d, we
have
4a3 − (3a − 1)2 = (a − 1)2 (4a − 1) ≥ 0.
Using this result and the AM-GM inequality, we get
v
X 1 1X 1 t4 1
≥ ≥ = 1.
(3a − 1)2 4 a 3 a b c3 d 3
3 3

Third Solution. We have


1 1 a(a − 1)2 (a + 2)(a2 + 3)
− = ≥ 0;
(3a − 1)2 (a3 + 1)2 (3a − 1)2 (a3 + 1)2
therefore,
X 1 X 1
≥ .
(3a − 1)2 (a + 1)2
3

Thus, it suffices to prove that


X 1
≥ 1,
(a3 + 1)2
which is an immediate consequence of the inequality in P 1.191.

P 1.193. Let a, b, c, d be positive real numbers such that abcd = 1. Prove that
1 1 1 1
+ + + ≥ 1.
1+a+a +a
2 3 1+ b+ b + b
2 3 1+c+c +c
2 3 1 + d + d2 + d3
(Vasile Cîrtoaje, 1999)
First Solution. We get the desired inequality by summing the inequalities
1 1 1
+ ≥ ,
1+a+a +a
2 3 1+ b+ b + b
2 3 1 + (ab)3/2
1 1 1
+ ≥ .
1 + c + c2 + c3 1 + d + d 2 + d 3 1 + (cd)3/2
Thus, it suffices to show that
1 1 1
+ ≥ ,
1+ x2 +x +x
4 6 1+ y + y + y
2 4 6 1 + x3 y3
262 Vasile Cîrtoaje

where x and y are positive real numbers. Putting p = x y and s = x 2 + x y + y 2 ,


this inequality becomes

p3 (x 6 + y 6 )+ p2 (p −1)(x 4 + y 4 )− p2 (p2 − p +1)(x 2 + y 2 )− p6 − p4 +2p3 − p2 +1 ≥ 0,

p3 (x 3 − y 3 )2 + p2 (p − 1)(x 2 − y 2 )2 − p2 (p2 − p + 1)(x − y)2 + p6 − p4 − p2 + 1 ≥ 0,


p3 s2 (x − y)2 + p2 (p −1)(s + p)2 (x − y)2 − p2 (p2 − p +1)(x − y)2 + p6 − p4 − p2 +1 ≥ 0,
p2 (s + 1)(ps − 1)(x − y)2 + (p2 − 1)(p4 − 1) ≥ 0.
If ps − 1 ≥ 0, then the inequality is clearly true. Consider further that ps < 1. From
ps < 1 and s ≥ 3p, we get p2 < 1/3. Write the desired inequality in the form

p2 (1 + s)(1 − ps)(x − y)2 ≤ (1 − p2 )(1 − p4 ).

Since
p(x − y)2 = p(s − 3p) < 1 − 3p2 < 1 − p2 ,
it suffices to show that
p(1 + s)(1 − ps) ≤ 1 − p4 .
Indeed,

4p(1 + s)(1 − ps) ≤ [p(1 + s) + (1 − ps)]2 = (1 + p)2 < 2(1 + p2 ) < 4(1 − p4 ).

The equality occurs for a = b = c = d = 1.


Second Solution. Assume that a ≥ b ≥ c ≥ d, and write the inequality as
X 1
≥ 1.
(1 + a)(1 + a2 )

Since
1 1 1 1 1 1
≤ ≤ , ≤ ≤ ,
1+a 1+ b 1+c 1 + a2 1 + b2 1 + c2
by Chebyshev’s inequality, it suffices to prove that

1 1 1 1 1 1 1 1
 ‹ ‹
+ + + + + ≥ 1.
3 1+a 1+ b 1+c 1+a 2 1+ b 2 1+c 2 (1 + d)(1 + d 2 )

On the other hand, from Remark 3 of P 1.186, we have


p3
1 1 1 3 3 d
+ + ≥ p =p
1+a 1+ b 1+c 3
1 + abc
3
d +1
and p3
1 1 1 3 3 d2
+ + ≥ p =p .
1 + a2 1 + b2 1 + c 2 3
1 + a2 b2 c 2
3
d2 + 1
Symmetric Rational Inequalities 263

Thus, it suffices to prove that

3d 1
p p + ≥ 1.
(1 +
3
d)(1 +
3
d2 ) (1 + d)(1 + d 2 )
p
3
Putting x = d, this inequality becomes as follows:

3x 3 1
+ ≥ 1,
(1 + x)(1 + x 2 ) (1 + x 3 )(1 + x 6 )

3x 3 (1 − x + x 2 )(1 − x 2 + x 4 ) + 1 ≥ (1 + x 3 )(1 + x 6 ),
x 3 (2 − 3x + 2x 3 − 3x 5 + 2x 6 ) ≥ 0,
x 3 (1 − x)2 (2 + x + x 3 + 2x 4 ) ≥ 0.
Remark. The following generalization holds true (Vasile Cîrtoaje, 2004):
• If a1 , a2 , . . . , an are positive real numbers such that a1 a2 · · · an = 1, then

1 1 1
+ + ··· + ≥ 1.
1 + a1 + · · · + a1
n−1
1 + a2 + · · · + a2
n−1
1 + an + · · · + ann−1

P 1.194. Let a, b, c, d be positive real numbers such that abcd = 1. Prove that

1 1 1 1
+ + + ≥ 1.
1 + a + 2a2 1 + b + 2b2 1 + c + 2c 2 1 + d + 2d 2
(Vasile Cîrtoaje, 2006)

Solution. We will show that


1 1
≥ ,
1 + a + 2a 2 1 + a + a2k + a3k
k

where k = 5/6. Then, it suffices to show that


X 1
≥ 1,
1+ ak + a2k + a3k
which immediately follows from the inequality in P 1.193. Setting a = x 6 , x > 0,
the claimed inequality can be written as

1 1
≥ ,
1 + x 6 + 2x 12 1 + x 5 + x 10 + x 15
which is equivalent to
x 10 + x 5 + 1 ≥ 2x 7 + x.
264 Vasile Cîrtoaje

We can prove it by summing the AM-GM inequalities

x 5 + 4 ≥ 5x

and
5x 10 + 4x 5 + 1 ≥ 10x 7 .
This completes the proof. The equality occurs for a = b = c = d = 1.
Remark. The inequalities in P 1.191, P 1.193 and P 1.194 are particular cases of
the following more general inequality (Vasile Cîrtoaje, 2009):
• Let a1 , a2 , . . . , an (n ≥ 4) be positive real numbers such that a1 a2 · · · an = 1. If
p, q, r are nonnegative real numbers satisfying p + q + r = n − 1, then
i=n
X 1
≥ 1.
i=1
1 + pai + qai2 + r ai3

P 1.195. Let a, b, c, d be positive real numbers such that abcd = 1. Prove that

1 1 1 1 9 25
+ + + + ≥ .
a b c d a+b+c+d 4

Solution (by Vo Quoc Ba Can). Replacing a, b, c, d by a4 , b4 , c 4 , d 4 , respectively, the


inequality becomes as follows:

1 1 1 1 9 25
+ 4+ 4+ 4+ 4 ≥ ,
a 4 b c d a +b +c +d
4 4 4 4abcd
1 1 1 1 4 9 9
+ + + − ≥ − ,
a4 b4 c 4 d 4 abcd 4abcd a4 + b4 + c 4 + d 4
1 1 1 1 4 9(a4 + b4 + c 4 + d 4 − 4abcd)
+ + + − ≥ .
a4 b4 c 4 d 4 abcd 4abcd(a4 + b4 + c 4 + d 4 )
Using the identities

a4 + b4 + c 4 + d 4 − 4abcd = (a2 − b2 )2 + (c 2 − d 2 )2 + 2(ab − cd)2 ,

1 1 1 1 4 (a2 − b2 )2 (c 2 − d 2 )2 2(ab − cd)2


+ + + − = + + 2 2 2 2 ,
a4 b4 c 4 d 4 abcd a4 b4 c4 d 4 a b c d
the inequality can be written as

(a2 − b2 )2 (c 2 − d 2 )2 2(ab − cd)2 9[(a2 − b2 )2 + (c 2 − d 2 )2 + 2(ab − cd)2 ]


+ + ≥ ,
a4 b4 c4 d 4 a2 b2 c 2 d 2 4abcd(a4 + b4 + c 4 + d 4 )
Symmetric Rational Inequalities 265

4cd(a4 + b4 + c 4 + d 4 ) 2 2 4ab(a + b + c + d )
4 4 4 4
• ˜ • ˜
(a − b )
2 2 2
− 9 + (c − d )
2
−9
a3 b3 c3 d 3

2 4(a + b + c + d )
• 4 4 4 4 ˜
+2(ab − cd) − 9 ≥ 0.
abcd
By the AM-GM inequality, we have

a4 + b4 + c 4 + d 4 ≥ 4abcd.

Therefore, it suffices to show that

4cd(a4 + b4 + c 4 + d 4 ) 2 2 4ab(a + b + c + d )
4 4 4 4
• ˜ • ˜
(a −b )
2 2 2
− 9 +(c −d )
2
− 9 ≥ 0.
a3 b3 c3 d 3

Without loss of generality, assume that a ≥ c ≥ d ≥ b. Since

(a2 − b2 )2 ≥ (c 2 − d 2 )2

and

4cd(a4 + b4 + c 4 + d 4 ) 4(a4 + b4 + c 4 + d 4 ) 4(a4 + 3b4 )


≥ ≥ > 9,
a3 b3 a3 b a3 b
it is enough to prove that

4cd(a4 + b4 + c 4 + d 4 ) 4ab(a4 + b4 + c 4 + d 4 )
• ˜ • ˜
−9 + − 9 ≥ 0,
a3 b3 c3 d 3

which is equivalent to
 ‹
cd ab
2(a + b + c + d ) 3 3 + 3 3
4 4 4 4
≥ 9.
a b c d

Indeed, by the AM-GM inequality,

2
 ‹  ‹
cd ab
2(a + b + c + d ) 3 3 + 3 3 ≥ 8abcd
4 4 4 4
= 16 > 9.
a b c d abcd

The equality occurs for a = b = c = d = 1.

P 1.196. If a, b, c, d are real numbers such that a + b + c + d = 0, then

(a − 1)2 (b − 1)2 (c − 1)2 (d − 1)2


+ + 2 + ≤ 4.
3a2 + 1 3b2 + 1 3c + 1 3d 2 + 1
266 Vasile Cîrtoaje

Solution. Since
3(a − 1)2 (3a + 1)2
4− = ,
3a2 + 1 3a2 + 1
we can write the inequality as
X (3a + 1)2
≥ 4.
3a2 + 1
On the other hand, since

4a2 = 3a2 + (b + c + d)2 ≤ 3a2 + 3(b2 + c 2 + d 2 ) = 3(a2 + b2 + c 2 + d 2 ),

9 9(a2 + b2 + c 2 + d 2 ) + 4
3a2 + 1 ≤ (a2 + b2 + c 2 + d 2 ) + 1 = ,
4 4
we have
(3a + 1)2
P
X (3a + 1)2 4
≥ = 4.
3a2 + 1 9(a2 + b2 + c 2 + d 2 ) + 4
The equality holds for a = b = c = d = 0, and also for a = 1 and b = c = d = −1/3
(or any cyclic permutation).
Remark. The following generalization is also true.
• If a1 , a2 , . . . , an are real numbers such that a1 + a2 + · · · + an = 0, then

(a1 − 1)2 (a2 − 1)2 (an − 1)2


+ + · · · + ≤ n,
(n − 1)a12 + 1 (n − 1)a22 + 1 (n − 1)an2 + 1

with equality for a1 = a2 = · · · = an = 0, and also for a1 = 1 and a2 = a3 = · · · =


an = −1/(n − 1) (or any cyclic permutation).

P 1.197. If a, b, c, d ≥ −5 such that a + b + c + d = 4, then

1−a 1− b 1−c 1−d


+ + + ≥ 0.
(1 + a)2 (1 + b)2 (1 + c)2 (1 + d)2

Solution. Assume that a ≤ b ≤ c ≤ d. We show first that x ∈ R \ {−1} involves

1− x −1
≥ ,
(1 + x)2 8

and x ∈ [−5, 1/3] \ {−1} involves

1− x 3
≥ .
(1 + x)2 8
Symmetric Rational Inequalities 267

Indeed, we have
1− x 1 (x − 3)2
+ = ≥0
(1 + x)2 8 8(1 + x)2
and
1− x 3 (5 + x)(1 − 3x)
− = ≥ 0.
(1 + x)2 8 8(1 + x)2
Therefore, if a ≤ 1/3, then

1−a 1− b 1−c 1−d 3 1 1 1


+ + + ≥ − − − = 0.
(1 + a)2 (1 + b)2 (1 + c)2 (1 + d)2 8 8 8 8

Assume now that 1/3 ≤ a ≤ b ≤ c ≤ d. Since

1−a ≥1− b ≥1−c ≥1−d

and
1 1 1 1
≥ ≥ ≥ ,
(1 + a)2 (1 + b)2 (1 + c)2 (1 + d)2
by Chebyshev’s inequality, we have

1−a 1− b 1−c 1−d


+ + + ≥
(1 + a)2 (1 + b)2 (1 + c)2 (1 + d)2

1 ”X — •X 1
˜
≥ (1 − a) = 0.
4 (1 + a)2
The equality holds for a = b = c = d = 1, and also for a = −5 and b = c = d = 3
(or any cyclic permutation).

P 1.198. Let a1 , a2 , . . . , an be positive real numbers such that a1 + a2 + · · · + an = n.


Prove that X 1 1
≤ .
(n + 1)a1 + a2 + · · · + an
2 2 2 2
(Vasile Cîrtoaje, 2008)

First Solution. By the Cauchy-Schwarz inequality, we have


X n2 X (a1 + a2 + · · · + an )2
=
(n + 1)a12 + a22 + · · · + an2 2a12 + (a12 + a22 ) + · · · + (a12 + an2 )
a22 an2

X 1 
≤ + + ··· + 2
2 a12 + a22 a1 + an2
n n(n − 1) n2
= + = ,
2 2 2
268 Vasile Cîrtoaje

from which the conclusion follows. The equality holds for a1 = a2 = · · · = an = 1.


Second Solution. Write the inequality as
X a12 + a22 + · · · + an2 a12 + a22 + · · · + an2
≤ .
(n + 1)a12 + a22 + · · · + an2 2

Since
a12 + a22 + · · · + an2 na12
=1− ,
(n + 1)a12 + a22 + · · · + an2 (n + 1)a12 + a22 + · · · + an2
we need to prove that
X a12 a12 + a22 + · · · + an2
+ ≥ 1.
(n + 1)a12 + a22 + · · · + an2 2n

By the Cauchy-Schwarz inequality, we have


X a12 (a1 + a2 + · · · + an )2

(n + 1)a12 + a22 + · · · + an2 [(n + 1)a12 + a22 + · · · + an2 ]
P

n
= .
2(a1 + a2 + · · · + an2 )
2 2

Then, it suffices to prove that

n a12 + a22 + · · · + an2


+ ≥ 2,
a12 + a22 + · · · + an2 n

which follows immediately from the AM-GM inequality.

P 1.199. Let a1 , a2 , . . . , an be real numbers such that a1 + a2 + · · · + an = 0. Prove


that
(a1 + 1)2 (a2 + 1)2 (an + 1)2 n
+ + · · · + ≥ .
a1 + n − 1 a2 + n − 1
2 2
an + n − 1 n − 1
2

(Vasile Cîrtoaje, 2010)

Solution. Without loss of generality, assume that an2 = max{a12 , a22 , · · · , an2 }. Since

(an + 1)2 n (n − 1 − an )2
= − ,
an2 + n − 1 n − 1 (n − 1)(an2 + n − 1)

we can write the inequality as


n−1
X (ai + 1)2 (n − 1 − an )2
≥ .
i=1
a 2
i + n − 1 (n − 1)(a 2 + n − 1)
n
Symmetric Rational Inequalities 269

From the Cauchy-Schwarz inequality


– n−1 ™ – n−1 ™ – n−1 ™2
X X (a + 1)2 X
i
(ai + n − 1)
2
≥ (ai + 1) ,
i=1
a2 + n − 1
i=1 i i=1

we get
n−1
X (ai + 1)2 (n − 1 − an )2
≥ .
2
+
Pn−1 2
i=1
a i n − 1 a + (n − 1)2
i=1 i

Thus, it suffices to show that


n−1
X
ai2 + (n − 1)2 ≤ (n − 1)(an2 + n − 1),
i=1

−a1
which is clearly true. The proof is completed. The equality holds for = a2 =
n−1
a3 = · · · = an (or any cyclic permutation).

P 1.200. Let a1 , a2 , . . . , an be positive real numbers such that a1 a2 · · · an = 1. Prove


that
1 1 1
(a) + + ··· + ≥ 1;
1 + (n − 1)a1 1 + (n − 1)a2 1 + (n − 1)an

1 1 1
(b) + + ··· + ≤ 1.
a1 + n − 1 a2 + n − 1 an + n − 1
(Vasile Cîrtoaje, 1991)

Solution. (a) First Solution. Let k = (n−1)/n. We can get the required inequality
by summing the inequalities
−k
1 ai
≥ −k
1 + (n − 1)ai a1 + a2−k + · · · + an−k

for i = 1, 2, · · · , n. The inequality is equivalent to

a1−k + · · · + ai−1
−k
+ ai+1
−k
+ · · · + an−k ≥ (n − 1)ai1−k ,

which follows from the AM-GM inequality. The equality holds for a1 = a2 = · · · =
an = 1.
Second Solution. Replacing all ai by 1/ai , the inequality becomes
a1 a2 an
+ + ··· + ≥ 1.
a1 + n − 1 a2 + n − 1 an + n − 1
270 Vasile Cîrtoaje

By the Cauchy-Schwarz inequality, we have


P p 2
X ai a1
≥P .
ai + n − 1 (a1 + n − 1)

Thus, we still have to prove that


€X p Š2 X
a1 ≥ a1 + n(n − 1),

which is equivalent to X p
2 ai a j ≥ n(n − 1).
1≤i< j≤n

Since a1 a2 · · · an = 1, this inequality follows from the AM-GM inequality.


Third Solution. Use the contradiction method. Assume that
1 1 1
+ + ··· + <1
1 + (n − 1)a1 1 + (n − 1)a2 1 + (n − 1)an

and show that a1 a2 · · · an > 1 (which contradicts the hypothesis a1 a2 · · · an = 1).


Let
1
xi = , 0 < x i < 1, i = 1, 2, · · · , n.
1 + (n − 1)ai
Since
1 − xi
ai = , i = 1, 2, · · · , n,
(n − 1)x i
we need to show that
x1 + x2 + · · · + x n < 1
implies
(1 − x 1 )(1 − x 2 ) · · · (1 − x n ) > (n − 1)n x 1 x 2 · · · x n .
Using the AM-GM inequality, we have
‚ Œ1/(n−1)
X Y
1 − xi > x k ≥ (n − 1) xk .
k6=i k6=i

Multiplying the inequalities


‚ Œ1/(n−1)
Y
1 − x i > (n − 1) xk , i = 1, 2, · · · , n,
k6=i

the conclusion follows.


(b) This inequality follows from the inequality in (a) by replacing all ai with
1/ai . The equality holds for a1 = a2 = · · · = an = 1.
Symmetric Rational Inequalities 271

Remark. The inequalities in P 1.200 are particular cases of the following more
general results (Vasile Cîrtoaje, 2005):
• Let a1 , a2 , . . . , an be positive real numbers such that a1 a2 · · · an = 1. If

0 < k ≤ n − 1, p ≥ n1/k − 1,

then
1 1 1 n
+ + · · · + ≥ .
(1 + pa1 )k (1 + pa2 )k (1 + pan )k (1 + p)k

• Let a1 , a2 , . . . , an be positive real numbers such that a1 a2 · · · an = 1. If

1  n 1/k
k≥ , 0<p≤ − 1,
n−1 n−1
then
1 1 1 n
+ + · · · + ≤ .
(1 + pa1 )k (1 + pa2 )k (1 + pan )k (1 + p)k

P 1.201. Let a1 , a2 , . . . , an be positive real numbers such that a1 a2 · · · an = 1. Prove


that
1 1 1
+ + ··· + ≥ 1.
1 − a1 + na1 1 − a2 + na2
2 2
1 − an + nan2
(Vasile Cîrtoaje, 2009)

Solution. First, we show that


1 1
≥ ,
1 − x + nx 2 1 + (n − 1)x k
1
where x > 0 and k = 2 + . Write the inequality as
n−1
(n − 1)x k + x ≥ nx 2 .

We can get this inequality using the AM-GM inequality as follows:


p
n
(n − 1)x k + x ≥ n x (n−1)k x = nx 2 .

Thus, it suffices to show that


1 1 1
+ + ··· + ≥ 1,
1 + (n − 1)a1k 1 + (n − 1)a2k 1 + (n − 1)ank

which follows immediately from the inequality (a) in the preceding P 1.200. The
equality holds for a1 = a2 = · · · = an = 1.
272 Vasile Cîrtoaje

Remark 1. Similarly, we can prove the following more general statement.


• Let a1 , a2 , . . . , an be positive real numbers such that a1 a2 · · · an = 1. If p and q
p
are real numbers such that p + q = n − 1 and n − 1 ≤ q ≤ ( n + 1)2 , then

1 1 1
+ + · · · + ≥ 1.
1 + pa1 + qa12 1 + pa2 + qa22 1 + pan + qan2

Remark 2. We can extend the inequality in Remark 1 as follows (Vasile Cîrtoaje,


2009).
• Let a1 , a2 , . . . , an be positive real numbers such that a1 a2 · · · an = 1. If p and q
p
are real numbers such that p + q = n − 1 and 0 ≤ q ≤ ( n + 1)2 , then

1 1 1
+ + ··· + ≥ 1.
1 + pa1 + qa1 1 + pa2 + qa2
2 2
1 + pan + qan2

P 1.202. Let a1 , a2 , . . . , an be positive real numbers such that


k(n − k − 1)
a1 , a2 , . . . , an ≥ , k>1
kn − k − 1
and
a1 a2 · · · an = 1.
Prove that
1 1 1 n
+ + ··· + ≤ .
a1 + k a2 + k an + k 1 + k
(Vasile Cîrtoaje, 2005)

Solution. We use the induction method. Let


1 1 1 n
En (a1 , a2 , . . . , an ) = + + ··· + − .
a1 + k a2 + k an + k 1 + k
For n = 2, we have
p p
(1 − k)( a1 − a2 )2
E2 (a1 , a2 ) = ≤ 0.
(1 + k)(a1 + k)(a2 + k)
Assume that the inequality is true for n − 1 numbers (n ≥ 3), and prove that
En (a1 , a2 , . . . , an ) ≥ 0 for a1 a2 · · · an = 1 and a1 , a2 , . . . , an ≥ pn , where
k(n − k − 1)
pn = .
kn − k − 1
Due to symmetry, we may assume that a1 ≥ 1 and a2 ≤ 1. There are two cases to
consider.
Symmetric Rational Inequalities 273

Case 1: a1 a2 ≤ k2 . From a1 a2 ≥ a2 , pn−1 < pn and a1 , a2 , . . . , an ≥ pn , it follows that

a1 a2 , a3 , · · · , an > pn−1 .

Then, by the induction hypothesis, we have En−1 (a1 a2 , a2 , . . . , an ) ≤ 0; thus, it suf-


fices to show that

En (a1 , a2 , . . . , an ) ≤ En−1 (a1 a2 , a2 , . . . , an ).

This is equivalent to

1 1 1 1
+ − − ≤ 0,
a1 + k a2 + k a1 a2 + k 1 + k

which reduces to the obvious inequality

(a1 − 1)(1 − a2 )(a1 a2 − k2 ) ≤ 0.

Case 2: a1 a2 ≥ k2 . Since

1 1 a1 + a2 + 2k a1 + a2 + 2k 1
+ = ≤ =
a1 + k a2 + k a1 a2 + k(a1 + a2 ) + k2 k2 + k(a1 + a2 ) + k2 k

and
1 1 n−2 kn − k − 1
+ ··· + ≤ = ,
a3 + k an + k pn + k k(k + 1)
we have
1 kn − k − 1 n
En (a1 , a2 , . . . , an ) ≤ + − = 0.
k k(k + 1) 1+k
Thus, the proof is completed. The equality holds for a1 = a2 = · · · = an = 1.
Remark. For k = n − 1, we get the inequality (b) in P 1.200.

P 1.203. If a1 , a2 , . . . , an ≥ 0, then

1 1 1 n
+ + ··· + ≥ .
1 + na1 1 + na2 1 + nan n + a1 a2 · · · an

(Vasile Cîrtoaje, 2013)

Solution. If one of a1 , a2 , . . . , an is zero, the inequality is obvious. Consider further


that a1 , a2 , . . . , an > 0 and let

r=
p
n
a1 a2 · · · an .
274 Vasile Cîrtoaje

By the Cauchy-Schwarz inequality, we have


Pp 2 Pp 2
X 1 a2 a3 · · · an a2 a3 · · · an
≥P =P .
1 + na1 (1 + na1 )a2 a3 · · · an a2 a3 · · · an + n2 r n

Therefore, it suffices to show that


€X p Š2 X
(n + r )
n
a2 a3 · · · an ≥n a2 a3 · · · an + n3 r n .

By the AM-GM inequality, we have


€X p Š2 X
a2 a3 · · · an ≥ a2 a3 · · · an + n(n − 1)r n−1 .

Thus, it is enough to prove that


”X — X
(n + r n ) a2 a3 · · · an + n(n − 1)r n−1 ≥ n a2 a3 · · · an + n3 r n ,

which is equivalent to
X
rn a2 a3 · · · an + n(n − 1)r 2n−1 + n2 (n − 1)r n−1 ≥ n3 r n .

Also, by the AM-GM inequality,


X
a2 a3 · · · an ≥ nr n−1 ,

and it suffices to show the inequality

nr 2n−1 + n(n − 1)r 2n−1 + n2 (n − 1)r n−1 ≥ n3 r n ,

which can be rewritten as

n2 r n−1 (r n − nr + n − 1) ≥ 0.

Indeed, by the AM-GM inequality, we get


p
n
rn + n − 1 = rn + 1 + · · · + 1 ≥ n r n · 1 · · · 1 = nr.

The equality holds for a1 = a2 = · · · = an = 1.


Chapter 2

Symmetric Nonrational Inequalities

2.1 Applications

2
2.1. If a, b are nonnegative real numbers such that a2 + b2 ≤ 1 + p , then
3
p
a b 2(a2 + b2 )
+ ≤ .
2a2 + 1 2b2 + 1 a2 + b2 + 1

2.2. If a, b, c are real numbers, then


Xp Æ
a2 − ab + b2 ≤ 6(a2 + b2 + c 2 ) − 3(ab + bc + ca).

2.3. If a, b, c are positive real numbers, then


p p p 2bc 2ca 2ab
a b+c+b c+a+c a+b≥ p +p +p .
b+c c+a a+b

2.4. If a, b, c are nonnegative real numbers, then


v
p p p t a2 + b2 + c 2
a2 − ab + b2 + b2 − bc + c 2 + c 2 − ca + a2 ≤ 3 .
2

2.5. If a, b, c are nonnegative real numbers, then


v v v
t 2 t 2 t 2 p
a2 + b2 − ab + b2 + c 2 − bc + c 2 + a2 − ca ≥ 2 a2 + b2 + c 2 .
3 3 3

275
276 Vasile Cîrtoaje

2.6. If a, b, c are nonnegative real numbers, then


Xp Æ
a2 + ab + b2 ≥ 4(a2 + b2 + c 2 ) + 5(ab + bc + ca).

2.7. If a, b, c are positive real numbers, then


Xp Æ
a2 + ab + b2 ≤ 5(a2 + b2 + c 2 ) + 4(ab + bc + ca).

2.8. If a, b, c are nonnegative real numbers, then


Xp p p
a2 + ab + b2 ≤ 2 a2 + b2 + c 2 + ab + bc + ca.

2.9. If a, b, c are nonnegative real numbers, then


p p p p p
a2 + 2bc + b2 + 2ca + c 2 + 2ab ≤ a2 + b2 + c 2 + 2 ab + bc + ca.

2.10. If a, b, c are nonnegative real numbers, then

1 1 1 1 2
p +p +p ≥p +p .
a2 + 2bc b2 + 2ca c 2 + 2ab a2 + b2 + c 2 ab + bc + ca

2.11. If a, b, c are positive real numbers, then


p p p p p
2a2 + bc + 2b2 + ca + 2c 2 + ab ≤ 2 a2 + b2 + c 2 + ab + bc + ca.

p
2.12. Let a, b, c be nonnegative real numbers such that a + b + c = 3. If k = 3−1,
then XÆ p
a(a + kb)(a + kc) ≤ 3 3.

2.13. If a, b, c are nonnegative real numbers such that a + b + c = 3, then



a(2a + b)(2a + c) ≥ 9.
Symmetric Nonrational Inequalities 277

2.14. Let a, b, c be nonnegative real numbers such that a + b + c = 3. Prove that


Æ Æ Æ
b2 + c 2 + a(b + c) + c 2 + a2 + b(c + a) + a2 + b2 + c(a + b) ≥ 6.

2.15. Let a, b, c be nonnegative real numbers such that a + b + c = 3. Prove that


p p p
(a) a(3a2 + abc) + b(3b2 + abc) + c(3c 2 + abc) ≥ 6;
p p p p
(b) 3a2 + abc + 3b2 + abc + 3c 2 + abc ≥ 3 3 + abc.

2.16. Let a, b, c be positive real numbers such that ab + bc + ca = 3. Prove that


Æ Æ Æ
a (a + 2b)(a + 2c) + b (b + 2c)(b + 2a) + c (c + 2a)(c + 2b) ≥ 9.

2.17. Let a, b, c be nonnegative real numbers such that a + b + c = 1. Prove that


Æ Æ Æ p
a + (b − c)2 + b + (c − a)2 + c + (a − b)2 ≥ 3.

2.18. Let a, b, c be nonnegative real numbers, no two of which are zero. Prove that
v v v
t a(b + c) t b(c + a) t c(a + b)
+ + ≥ 2.
a2 + bc b2 + ca c 2 + ab

2.19. Let a, b, c be positive real numbers such that abc = 1. Prove that
1 1 1
p
3
+p
3
+p
3
≥ 1.
a2 + 25a + 1 b2 + 25b + 1 c 2 + 25c + 1

2.20. If a, b, c are nonnegative real numbers, then


p p p 3
a2 + bc + b2 + ca + c 2 + ab ≤ (a + b + c).
2

2.21. If a, b, c are nonnegative real numbers, then


p p p p
a2 + 9bc + b2 + 9ca + c 2 + 9ab ≥ 5 ab + bc + ca.
278 Vasile Cîrtoaje

2.22. If a, b, c are nonnegative real numbers, then



(a2 + 4bc)(b2 + 4ca) ≥ 5(ab + ac + bc).

2.23. If a, b, c are nonnegative real numbers, then



(a2 + 9bc)(b2 + 9ca) ≥ 7(ab + ac + bc).

2.24. If a, b, c are nonnegative real numbers, then



(a2 + b2 )(b2 + c 2 ) ≤ (a + b + c)2 .

2.25. If a, b, c are nonnegative real numbers, then



(a2 + ab + b2 )(b2 + bc + c 2 ) ≥ (a + b + c)2 .

2.26. If a, b, c are nonnegative real numbers, then



(a2 + 7ab + b2 )(b2 + 7bc + c 2 ) ≥ 7(ab + ac + bc).

2.27. If a, b, c are nonnegative real numbers, then


v
X t 7 7 13
‹ ‹
a + ab + b
2 2 b + bc + c ≤
2 2 (a + b + c)2 .
9 9 12

2.28. If a, b, c are nonnegative real numbers, then


v
X t 1 1 61
‹ ‹
a + ab + b
2 2 b + bc + c ≤
2 2 (a + b + c)2 .
3 3 60

2.29. If a, b, c are nonnegative real numbers, then

a b c
p +p +p ≥ 1.
4b2 + bc + 4c 2 4c 2 + ca + 4a2 4a2 + ab + 4b2
Symmetric Nonrational Inequalities 279

2.30. If a, b, c are nonnegative real numbers, then

a b c a+b+c
p +p +p ≥p .
b2 + bc + c 2 c 2 + ca + a2 a2 + ab + b2 ab + bc + ca

2.31. If a, b, c are nonnegative real numbers, then

a b c a+b+c
p +p +p ≤p .
a2 + 2bc b2 + 2ca c 2 + 2ab ab + bc + ca

2.32. If a, b, c are nonnegative real numbers, then


p p p
a3 + b3 + c 3 + 3abc ≥ a2 a2 + 3bc + b2 b2 + 3ca + c 2 c 2 + 3ab.

2.33. Let a, b, c be nonnegative real numbers, no two of which are zero. Prove that

a b c
p +p +p ≤ 1.
4a2 + 5bc 4b2 + 5ca 4c 2 + 5ab

2.34. Let a, b, c be nonnegative real numbers. Prove that


p p p
a 4a2 + 5bc + b 4b2 + 5ca + c 4c 2 + 5ab ≥ (a + b + c)2 .

2.35. Let a, b, c be nonnegative real numbers. Prove that


p p p
a a2 + 3bc + b b2 + 3ca + c c 2 + 3ab ≥ 2(ab + bc + ca).

2.36. Let a, b, c be nonnegative real numbers. Prove that


p p p
a a2 + 8bc + b b2 + 8ca + c c 2 + 8ab ≤ (a + b + c)2 .

2.37. Let a, b, c be nonnegative real numbers, no two of which are zero. Prove that

a2 + 2bc b2 + 2ca c 2 + 2ab p


p +p +p ≥ 3 ab + bc + ca.
b2 + bc + c 2 c 2 + ca + a2 a2 + ab + b2
280 Vasile Cîrtoaje

2.38. Let a, b, c be nonnegative real numbers, no two of which are zero. If k ≥ 1,


then
a k+1 b k+1 c k+1 ak + bk + c k
+ + ≤ .
2a2 + bc 2b2 + ca 2c 2 + ab a+b+c

2.39. If a, b, c are positive real numbers, then


a2 − bc b2 − ca c 2 − ab
(a) p +p +p ≥ 0;
3a2 + 2bc 3b2 + 2ca 3c 2 + 2ab
a2 − bc b2 − ca c 2 − ab
(b) p +p +p ≥ 0.
8a2 + (b + c)2 8b2 + (c + a)2 8c 2 + (a + b)2

p
2.40. Let a, b, c be positive real numbers. If 0 ≤ k ≤ 1 + 2 2, then

a2 − bc b2 − ca c 2 − ab
p +p +p ≥ 0.
ka2 + b2 + c 2 kb2 + c 2 + a2 kc 2 + a2 + b2

2.41. If a, b, c are nonnegative real numbers, then


p p p
(a2 − bc) b + c + (b2 − ca) c + a + (c 2 − ab) a + b ≥ 0.

2.42. If a, b, c are nonnegative real numbers, then


p p p
(a2 − bc) a2 + 4bc + (b2 − ca) b2 + 4ca + (c 2 − ab) c 2 + 4ab ≥ 0.

2.43. If a, b, c are nonnegative real numbers, then


v v v
t a3 t b3 t c3
+ + ≥ 1.
a3 + (b + c)3 b3 + (c + a)3 c 3 + (a + b)3

2.44. If a, b, c are positive real numbers, then


v v v
u
1 1 1 1 1 1
t  ‹ t  ‹
(a + b + c) + + ≥ 1 + 1 + (a + b + c ) 2 + 2 + 2 .
t
2 2 2
a b c a b c
Symmetric Nonrational Inequalities 281

2.45. If a, b, c are positive real numbers, then


v
1 1 1 1 1 1
t  ‹  ‹
5 + 2(a + b + c ) 2 + 2 + 2 − 2 ≥ (a + b + c)
2 2 2 + + .
a b c a b c

2.46. If a, b, c are real numbers, then


Æ
2(1 + abc) + 2(1 + a2 )(1 + b2 )(1 + c 2 ) ≥ (1 + a)(1 + b)(1 + c).

2.47. Let a, b, c be nonnegative real numbers, no two of which are zero. Prove that
v v v
t a2 + bc t b2 + ca t c 2 + ab 1
+ + ≥ 2 + p .
b2 + c 2 c 2 + a2 a2 + b2 2

2.48. If a, b, c are nonnegative real numbers, then


Æ Æ Æ Æ
a(2a + b + c) + b(2b + c + a) + c(2c + a + b) ≥ 12(ab + bc + ca).

2.49. Let a, b, c be nonnegative real numbers such that a + b + c = 3. Prove that


Æ Æ Æ
a (4a + 5b)(4a + 5c) + b (4b + 5c)(4b + 5a) + c (4c + 5a)(4c + 5b) ≥ 27.

2.50. Let a, b, c be nonnegative real numbers such that ab + bc + ca = 3. Prove


that
Æ Æ Æ
a (a + 3b)(a + 3c) + b (b + 3c)(b + 3a) + c (c + 3a)(c + 3b) ≥ 12.

2.51. Let a, b, c be nonnegative real numbers such that a2 + b2 + c 2 = 3. Prove that


p p p Æ
2 + 7ab + 2 + 7bc + 2 + 7ca ≥ 3 3(ab + bc + ca).

2.52. Let a, b, c be nonnegative real numbers such that a2 + b2 + c 2 = 3. Prove that

a b c
+ 2 + 2 ≤ 1.
2a2 + 1 2b + 1 2c + 1
282 Vasile Cîrtoaje

2.53. Let a, b, c be nonnegative real numbers such that ab + bc + ca = 3. Prove


that
Pp
(a) a(b + c)(a2 + bc) ≥ 6;
p p
a(b + c) a2 + 2bc ≥ 6 3;
P
(b)
p
a(b + c) (a + 2b)(a + 2c) ≥ 18.
P
(c)

2.54. Let a, b, c be nonnegative real numbers such that ab + bc + ca = 3. Prove


that p p p
a bc + 3 + b ca + 3 + c ab + 3 ≥ 6.

2.55. Let a, b, c be nonnegative real numbers such that a + b + c = 3. Prove that


p
(b + c) b2 + c 2 + 7bc ≥ 18;
P
(a)
p p
(b + c) b2 + c 2 + 10bc ≤ 12 3.
P
(b)

2.56. Let a, b, c be nonnegative real numbers such then a + b + c = 2. Prove that


p p p p
a + 4bc + b + 4ca + c + 4ab ≥ 4 ab + bc + ca.

2.57. If a, b, c are nonnegative real numbers, then


p p p p
a2 + b2 + 7ab + b2 + c 2 + 7bc + c 2 + a2 + 7ca ≥ 5 ab + bc + ca.

2.58. If a, b, c are nonnegative real numbers, then


p p p Æ
a2 + b2 + 5ab + b2 + c 2 + 5bc + c 2 + a2 + 5ca ≥ 21(ab + bc + ca).

2.59. Let a, b, c be nonnegative real numbers such that ab + bc + ca = 3. Prove


that v
p p p t2
a a2 + 5 + b b2 + 5 + c c 2 + 5 ≥ (a + b + c)2 .
3
Symmetric Nonrational Inequalities 283

2.60. Let a, b, c be nonnegative real numbers such that a2 + b2 + c 2 = 1. Prove that


p p p
a 2 + 3bc + b 2 + 3ca + c 2 + 3ab ≥ (a + b + c)2 .

2.61. Let a, b, c be nonnegative real numbers such that a + b + c = 3. Prove that


v v v
t 2a + bc t 2b + ca t 2c + ab
(a) a +b +c ≥ 3;
3 3 3
v v v
t a(1 + b + c) t b(1 + c + a) t c(1 + a + b)
(b) a +b +c ≥ 3.
3 3 3

2.62. If a, b, c are nonnegative real numbers such that a + b + c = 3, then


Æ Æ Æ
8(a2 + bc) + 9 + 8(b2 + ca) + 9 + 8(c 2 + ab) + 9 ≥ 15.

9
2.63. Let a, b, c be nonnegative real numbers such that a + b + c = 3. If k ≥ ,
8
then p p p p
a2 + bc + k + b2 + ca + k + c 2 + ab + k ≥ 3 2 + k.

2.64. If a, b, c are nonnegative real numbers such that a + b + c = 3, then


p p p p
a3 + 2bc + b3 + 2ca + c 3 + 2ab ≥ 3 3.

2.65. If a, b, c are positive real numbers, then


p p p p
a2 + bc b2 + ca c 2 + ab 3 2
+ + ≥ .
b+c c+a a+b 2

2.66. If a, b, c are nonnegative real numbers, no two of which are zero,then


p p p
bc + 4a(b + c) ca + 4b(c + a) ab + 4c(a + b) 9
+ + ≥ .
b+c c+a a+b 2
284 Vasile Cîrtoaje

2.67. If a, b, c are nonnegative real numbers, no two of which are zero,then


p p p
a a2 + 3bc b b2 + 3ca c c 2 + 3ab
+ + ≥ a + b + c.
b+c c+a a+b

2.68. If a, b, c are nonnegative real numbers, no two of which are zero,then


v v v
t 2a(b + c) t 2b(c + a) t 2c(a + b)
+ + ≥ 2.
(2b + c)(b + 2c) (2c + a)(c + 2a) (2a + b)(a + 2b)

2.69. If a, b, c are nonnegative real numbers such that ab + bc + ca = 3, then


v s v v s v
t bc ca t ab t bc ca t ab
+ + ≤1≤ + + .
3a + 6
2 3b2 + 6 3c 2 + 6 6a2 + 3 6b2 + 3 6c 2 + 3

2.70. Let a, b, c be nonnegative real numbers such that ab + bc + ca = 3. If k > 1,


than
a k (b + c) + b k (c + a) + c k (a + b) ≥ 6.

2.71. Let a, b, c be nonnegative real numbers such that a + b + c = 2. If

2 ≤ k ≤ 3,

than
a k (b + c) + b k (c + a) + c k (a + b) ≤ 2.

2.72. Let a, b, c be nonnegative real numbers, no two of which are zero. If m >
n ≥ 0, than

bm + c m c m + am am + bm
(b + c − 2a) + (c + a − 2b) + (a + b − 2c) ≥ 0.
bn + c n c n + an an + bn

2.73. Let a, b, c be positive real numbers such that abc = 1. Prove that
p p p
a2 − a + 1 + a2 − a + 1 + a2 − a + 1 ≥ a + b + c.
Symmetric Nonrational Inequalities 285

2.74. Let a, b, c be positive real numbers such that abc = 1. Prove that
p p p
16a2 + 9 + 16b2 + 9 + 16b2 + 9 ≥ 4(a + b + c) + 3.

2.75. Let a, b, c be positive real numbers such that abc = 1. Prove that
p p p
25a2 + 144 + 25b2 + 144 + 25c 2 + 144 ≤ 5(a + b + c) + 24.

2.76. If a, b are positive real numbers such that ab + bc + ca = 3, then


p p p
(a) a2 + 3 + b2 + 3 + b2 + 3 ≥ a + b + c + 3;
p p p p
(b) a + b + b + c + c + a ≥ 4(a + b + c) + 6.

2.77. If a, b, c are nonnegative real numbers such that a + b + c = 3, then


Æ Æ Æ
(5a2 + 3)(5b2 + 3) + (5b2 + 3)(5c 2 + 3) + (5c 2 + 3)(5a2 + 3) ≥ 24.

2.78. If a, b, c are nonnegative real numbers such that a + b + c = 3, then


v
p p p t 4(a2 + b2 + c 2 ) + 42
a2 + 1 + b2 + 1 + c 2 + 1 ≥ .
3

2.79. If a, b, c are nonnegative real numbers such that a + b + c = 3, then


p p p p
(a) a2 + 3 + b2 + 3 + c 2 + 3 ≥ 2(a2 + b2 + c 2 ) + 30;
p p p p
(b) 3a2 + 1 + 3b2 + 1 + 3c 2 + 1 ≥ 2(a2 + b2 + c 2 ) + 30.

2.80. If a, b, c are nonnegative real numbers such that a + b + c = 3, then


Æ Æ Æ
(32a2 + 3)(32b2 + 3) + (32b2 + 3)(32c 2 + 3) + (32c 2 + 3)(32a2 + 3) ≤ 105.

2.81. If a, b, c are positive real numbers, then

b+c c+a a+b


−3 + −3 + − 3 ≥ 2.
a b c
286 Vasile Cîrtoaje

2.82. If a, b, c are real numbers such that abc 6= 0, then

b+c c+a a+b


+ + ≥ 2.
a b c

2.83. Let a, b, c be nonnegative real numbers, no two of which are zero, and let
2a 2b 2c
x= , y= , z= .
b+c c+a a+b
Prove that
p p p
(a) xy+ yz + z x ≥ x yz + 2;
p p p
(b) x + y +z+ xy+
yz + z x ≥ 6;
p p p
x + y + z ≥ 8 + x yz;
p
(c)
p p p
yz zx xy
(d) + + ≥ 1.
x +2 y +2 z+2

2.84. Let a, b, c be nonnegative real numbers, no two of which are zero, and let
2a 2b 2c
x= , y= , z= .
b+c c+a a+b
Prove that p p p
1 + 24x + 1 + 24 y + 1 + 24z ≥ 15.

2.85. If a, b, c are positive real numbers, then


v v v
t 7a t 7b t 7c
+ + ≤ 3.
a + 3b + 3c b + 3c + 3a c + 3a + 3b

2.86. If a, b, c are positive real numbers such that a + b + c = 3, then


Æ
3
Æ
3
Æ
3
p
3
a2 (b2 + c 2 ) + b2 (c 2 + a2 ) + c 2 (a2 + b2 ) ≤ 3 2.

2.87. If a, b, c are nonnegative real numbers, no two of which are zero, then
1 1 1 1 2
+ + ≥ +p .
a+b b+c c+a a+b+c ab + bc + ca
Symmetric Nonrational Inequalities 287

2.88. If a, b ≥ 1, then

1 1 1 1
p + ≥p +p .
3ab + 1 2 3a + 1 3b + 1

1
2.89. Let a, b, c be positive real numbers such that a + b + c = 3. If k ≥ p , then
2

(abc)k (a2 + b2 + c 2 ) ≤ 3.

2.90. If a, b, c ∈ [0, 4] and ab + bc + ca = 4, then


p p p p
a+b+ b+c+ c+a ≤3+ 5.

2.91. Let v
t a2 + b2 + c 2 a+b+c
F (a, b, c) = − ,
3 3
where a, b, c are positive real numbers such that

a4 bc ≥ 1, a ≤ b ≤ c.

Then,
1 1 1
 ‹
F (a, b, c) ≥ F , , .
a b c

2.92. Let v
t a2 + b2 + c 2 a+b+c
F (a, b, c) = − ,
3 3
where a, b, c are positive real numbers such that

a2 (b + c) ≥ 1, a ≤ b ≤ c.

Then,
1 1 1
 ‹
F (a, b, c) ≥ F , , .
a b c
288 Vasile Cîrtoaje

2.93. Let v
t a2 + b2 + c 2 a+b+c
F (a, b, c) = − ,
3 3
where a, b, c are positive real numbers such that

a4 (b2 + c 2 ) ≥ 2, a ≤ b ≤ c.

Then,
1 1 1
 ‹
F (a, b, c) ≥ F , , .
a b c

2.94. Let
p
3 3
F (a, b, c) = abc − 1
,
a + 1b + 1c
where a, b, c are positive real numbers such that

a4 b7 c 7 ≥ 1, a ≥ b ≥ c.

Then,
1 1 1
 ‹
F (a, b, c) ≥ F , , .
a b c

2.95. Let p
4 4
F (a, b, c, d) = abcd − 1
,
a + + 1c +
1
b
1
d
where a, b, c, d are positive real numbers. If ab ≥ 1 and cd ≥ 1, then then
1 1 1 1
 ‹
F (a, b, c, d) ≥ F , , , .
a b c d

2.96. Let a, b, c, d be nonnegative real numbers such that a2 + b2 + c 2 + d 2 = 1.


Prove that
p p p p p p p p
1 − a + 1 − b + 1 − c + 1 − d ≥ a + b + c + d.

2.97. Let a, b, c, d be positive real numbers. Prove that


p
A + 2 ≥ B + 4,

where
1 1 1 1
 ‹
A = (a + b + c + d) + + + − 16,
a b c d
1 1 1 1
 ‹
B = (a + b + c + d ) 2 + 2 + 2 + 2 − 16.
2 2 2 2
a b c d
Symmetric Nonrational Inequalities 289

2.98. Let a1 , a2 , . . . , an be nonnegative real numbers such that a1 + a2 +· · ·+ an = 1.


Prove that
3a1 + 1 + 3a2 + 1 + · · · + 3an + 1 ≥ n + 1.
p p p

2.99. Let a1 , a2 , . . . , an be positive real numbers such that a1 a2 · · · an = 1. Prove


that
1 1 1
+p + ··· + p ≥ 1.
1 + (n2 − 1)a1 1 + (n2 − 1)a2 1 + (n2 − 1)an
p

2.100. Let a1 , a2 , . . . , an be positive real numbers such that a1 a2 · · · an = 1. Prove


that n
X 1 1
≥ .
i=1 1 + 1 + 4n(n − 1)ai
p
2

2.101. If a1 , a2 , . . . , an are positive real numbers such that a1 a2 · · · an = 1, then


v
t a2 + a2 + · · · + a2
1 2 n
a1 + a2 + · · · + an ≥ n − 1 + .
n

2.102. If a1 , a2 , . . . , an are positive real numbers such that a1 a2 · · · an = 1, then


q Æ
(n − 1)(a12 + a22 + · · · + an2 ) + n − n(n − 1) ≥ a1 + a2 + · · · + an .

2.103. Let a1 , a2 , . . . , an be positive real numbers such that a1 a2 · · · an ≥ 1. If k > 1,


then
X a1k
≥ 1.
a1k + a2 + · · · + an

2.104. Let a1 , a2 , . . . , an be positive real numbers such that a1 a2 · · · an ≥ 1. If


−2
≤ k < 1,
n−2
then
X a1k
≤ 1.
a1k + a2 + · · · + an
290 Vasile Cîrtoaje

2.105. Let a1 , a2 , . . . , an be nonnegative real numbers such that a1 +a2 +· · ·+an ≥ n.


If 1 < k ≤ n + 1, then X a1
≤ 1.
a1 + a2 + · · · + an
k

2.106. Let a1 , a2 , . . . , an be positive real numbers such that a1 a2 · · · an ≥ 1. If k > 1,


then X a1
≤ 1.
a1k + a2 + · · · + an

2.107. Let a1 , a2 , . . . , an be positive real numbers such that a1 a2 · · · an ≥ 1. If


2
−1 − ≤ k < 1,
n−2
then X a1
≥ 1.
a1k + a2 + · · · + an

2.108. Let a1 , a2 , . . . , an be positive real numbers such that a1 a2 · · · an = 1. If k ≥ 0,


then X 1
≤ 1.
a1 + a2 + · · · + an
k

2.109. Let a1 , a2 , . . . , an be nonnegative real numbers such that a1 +a2 +· · ·+an ≤ n.


If 0 ≤ k < 1, then
1 1 1
+ + ··· + ≥ 1.
a1k + a2 + · · · + an a1 + a2k + · · · + an a1 + a2 + · · · + ank

2.110. Let a1 , a2 , . . . , an be positive real numbers. If k > 1, then


X ak + ak + · · · + ak n(a1k + a2k + · · · + ank )
2 3 n
≤ .
a2 + a3 + · · · + a n a1 + a2 + · · · + an

2.111. Let f be a convex function on the closed interval [a, b], and let a1 , a2 , . . . , an ∈
[a, b] such that
a1 + a2 + · · · + an = pa + qb,
where p, q ≥ 0 such that p + q = n. Prove that

f (a1 ) + f (a2 ) + · · · + f (an ) ≤ p f (a) + q f (b).


Symmetric Nonrational Inequalities 291

2.2 Solutions
2
P 2.1. If a, b are nonnegative real numbers such that a2 + b2 ≤ 1 + p , then
3
p
a b 2(a2 + b2 )
+ ≤ .
2a2 + 1 2b2 + 1 a2 + b2 + 1
(Vasile Cîrtoaje, 2012)
Solution. With
a2 + b2 1 1
s= , p = ab, 0 ≤ p ≤ s ≤ +p ,
2 2 3
the inequality becomes as follows:
p
(2p + 1) 2(s + p)
p
2 s
≤ ,
4p2 + 4s + 1 2s + 1
v
t 2s (2p + 1)(2s + 1)
−1≥ − 1,
s+p 4p2 + 4s + 1
s−p 2(s − p)(2p − 1)
≥ .
4p2 + 4s + 1
v
t 2s
(s + p) +1
s+p
Thus, we need to show that
1 2(2p − 1)
≥ 2 .
4p + 4s + 1
v
t 2s
(s + p) +1
s+p
v
t 2s
Since ≥ 1, it suffices to show that
s+p

1 2(2p − 1)
≥ 2 ,
4p + 4s + 1
v v
t 2s t 2s
(s + p) +
s+p s+p

which is equivalent to
Æ
4p2 + 4s + 1 ≥ 4(2p − 1) 2s(s + p).

For the nontrivial case 2p − 1 > 0, which involves 2s − 1 > 0, since 2 2s(s + p) ≤
p

2s + (s + p), it suffices to show that

4p2 + 4s + 1 ≥ 2(2p − 1)(3s + p),


292 Vasile Cîrtoaje

that is
10s + 1 ≥ 2p(6s − 1).
We have

10s + 1 − 2p(6s − 1) ≥ 10s + 1 − 2s(6s − 1) = 1 + 12s − 12s2 ≥ 0.

The equality holds for a = b.

P 2.2. If a, b, c are real numbers, then


Xp Æ
a2 − ab + b2 ≤ 6(a2 + b2 + c 2 ) − 3(ab + bc + ca).

Solution. By squaring, the inequality becomes as follows:



2(ab + bc + ca) + 2 (a2 − ab + b2 )(a2 − ac + c 2 ) ≤ 4(a2 + b2 + c 2 ),
X €p p Š2
a2 − ab + b2 − a2 − ac + c 2 ≥ 0.
The equality holds for a = b = c, and also for a = 0 and b = c (or any cyclic
permutation).

P 2.3. If a, b, c are positive real numbers, then


p p p 2bc 2ca 2ab
a b+c+b c+a+c a+b≥ p +p +p .
b+c c+a a+b
(Lorian Saceanu, 2015)

Solution. Use the SOS method. Write the inequality as follows:


X p X 2bc
a b+c− p ≥ 0,
b+c
X a(b + c) − 2bc
p ≥ 0,
b+c
X b(a − c) X c(a − b)
p + p ≥ 0,
b+c b+c
X c(b − a) X c(a − b)
p + p ≥ 0,
c+a b+c
Symmetric Nonrational Inequalities 293

1 1
X  ‹
c(a − b) p −p ≥ 0,
b+c c+a
X c(a − b)2
p p p  ≥ 0.
(b + c)(c + a) b + c + c + a
The equality holds for a = b = c.

P 2.4. If a, b, c are nonnegative real numbers, then


v
p p p t a2 + b2 + c 2
a2 − ab + b2 + b2 − bc + c 2 + c − ca + a ≤ 3
2 2 .
2

Solution (by Nguyen Van Quy). Assume that c = min{a, b, c}. Since

b2 − bc + c 2 ≤ b2

and
c 2 − ca + a2 ≤ a2 ,
it suffices to show that
v
p t a2 + b2 + c 2
a2 − ab + b2 + b + a ≤ 3 .
2
Using the Cauchy-Schwarz inequality, we have

(a + 2
˜
p t b)
a2 − ab + b2 + a + b ≤ (a2 − ab + b2 ) + (1 + k)
k
v
t (1 + k)[(1 + k)(a2 + b2 ) + (2 − k)ab]
= , k > 0.
k
Choosing k = 2, we get
v v
p t a2 + b2 t a2 + b2 + c 2
a2 − ab + b2 + a + b ≤ 3 ≤3 = 3.
2 2
The equality holds for a = b and c = 0 (or any cyclic permutation).

P 2.5. If a, b, c are nonnegative real numbers, then


v v v
t 2 t 2 t 2 p
a2 + b2 − ab + b2 + c 2 − bc + c 2 + a2 − ca ≥ 2 a2 + b2 + c 2 .
3 3 3
(Vasile Cîrtoaje, 2012)
294 Vasile Cîrtoaje

First Solution. By squaring, the inequality becomes



2 (3a2 + 3b2 − 2ab)(3a2 + 3c 2 − 2ac) ≥ 6(a2 + b2 + c 2 ) + 2(ab + bc + ca),
X €p p Š2
6(a2 + b2 + c 2 − ab − bc − ca) ≥ 3a2 + 3b2 − 2ab − 3a2 + 3c 2 − 2ac ,
X X (b − c)2 (3b + 3c − 2a)2
3 (b − c)2 ≥ p p 2 ,
3a2 + 3b2 − 2ab + 3a2 + 3c 2 − 2ac
 
X (3b + 3c − 2a)2
(b − c)2 1 − p p 2  .
9a + 9b − 6ab + 9a + 9c − 6ac
2 2 2 2

Since p Æ
9a2 + 9b2 − 6ab = (3b − a)2 + 8a2 ≥ |3b − a|,
p Æ
9a2 + 9c 2 − 6ac = (3c − a)2 + 8a2 ≥ |3c − a|,
it suffices to show that
‹2 
|3b + 3c − 2a|
X  
(b − c) 1 −
2
≥ 0.
|3b − a| + |3c − a|

This is true since

|3b + 3c − 2a| = |(3b − a) + (3c − a)| ≤ |3b − a| + |3c − a|.

The equality holds for a = b = c, and also for b = c = 0 (or any cyclic permutation).

Second Solution. Assume that a ≥ b ≥ c. Write the inequality as


Æ Æ Æ
(a + b)2 + 2(a − b)2 + (b + c)2 + 2(b − c)2 + (a + c)2 + 2(a − c)2 ≥
Æ
≥ 2 3(a2 + b2 + c 2 ).
By Minkowski’s inequality, it suffices to show that
Æ Æ
[(a + b) + (b + c) + (a + c)]2 + 2[(a − b) + (b − c) + (a − c)]2 ≥ 2 3(a2 + b2 + c 2 ),

which is equivalent to
Æ Æ
(a + b + c)2 + 2(a − c)2 ≥ 3(a2 + b2 + c 2 ).

By squaring, the inequality turns into

(a − b)(b − c) ≥ 0.
Symmetric Nonrational Inequalities 295

P 2.6. If a, b, c are nonnegative real numbers, then


Xp Æ
a2 + ab + b2 ≥ 4(a2 + b2 + c 2 ) + 5(ab + bc + ca).

(Vasile Cîrtoaje, 2009)


First Solution. By squaring, the inequality becomes

(a2 + ab + b2 )(a2 + ac + c 2 ) ≥ (a + b + c)2 .

Using the Cauchy-Schwarz inequality, we get


v
b 2 3b2 

c 2 3c 2
XÆ Xu  ‹ ˜
(a + ab + b )(a + ac + c ) = a+ + a+ +
2 2 2 2
t
2 4 2 4
X • c  3bc
‹ ˜
b 
≥ a+ a+ + = (a + b + c)2 .
2 2 4
The equality holds for a = b = c, and also for b = c = 0 (or any cyclic permutation).

Second Solution. Assume that a ≥ b ≥ c. By Minkowski’s inequality, we get


Xp XÆ
2 a2 + ab + b2 = 3(a + b)2 + (a − b)2
Æ
≥ 3[(a + b) + (b + c) + (c + a)]2 + [(a − b) + (b − c) + (a − c)]2
Æ
= 2 3(a + b + c)2 + (a − c)2 .
Therefore, it suffices to show that

3(a + b + c)2 + (a − c)2 ≥ 4(a2 + b2 + c 2 ) + 5(ab + bc + ca),

which is equivalent to the obvious inequality

(a − b)(b − c) ≥ 0.

Remark. Similarly, we can prove the following generalization.


• Let a, b, c be nonnegative real numbers. If |k| ≤ 2, then
Xp Æ
a2 + kab + b2 ≥ 4(a2 + b2 + c 2 ) + (3k + 2)(ab + bc + ca),

with equality for a = b = c, and also for b = c = 0 (or any cyclic permutation).
For k = −2/3 and k = 1, we get the inequalities in P 2.5 and P 2.6, respectively.
For k = −1 and k = 0, we get the inequalities
Xp Æ
a2 − ab + b2 ≥ 4(a2 + b2 + c 2 ) − ab − bc − ca,
Xp Æ
a2 + b2 ≥ 4(a2 + b2 + c 2 ) + 2(ab + bc + ca).
296 Vasile Cîrtoaje

P 2.7. If a, b, c are positive real numbers, then


Xp Æ
a2 + ab + b2 ≤ 5(a2 + b2 + c 2 ) + 4(ab + bc + ca).

(Michael Rozenberg, 2008)

First Solution (by Vo Quoc Ba Can). Using the Cauchy-Schwarz inequality, we have
€X p Š2 ”X — X b2 + bc + c 2 ‹
b + bc + c
2 2 ≤ (b + c)
b+c
b + bc + c 2
X 2 ‹ X a  2
= 2(a + b + c) =2 1+ (b + bc + c 2 )
b+c b+c
X 2a(b2 + bc + c 2 )
= 4(a + b + c ) + 2(ab + bc + ca) +
2 2 2
b+c
X  bc
‹
= 4(a + b + c ) + 2(ab + bc + ca) +
2 2 2
2a b + c −
b+c
X 1
= 4(a2 + b2 + c 2 ) + 6(ab + bc + ca) − 2abc .
b+c
Thus, it suffices to prove that
X 1
4(a2 + b2 + c 2 )+6(ab + bc + ca)−2abc ≤ 5(a2 + b2 + c 2 )+4(ab + bc + ca),
b+c
which is equivalent to Schur’s inequality
X 1
2(ab + bc + ca) ≤ a2 + b2 + c 2 + 2abc .
b+c
We can prove this inequality by writing it as follows:
X  bc
‹
(a + b + c) ≤ 2
2
a a+ ,
b+c
X a
(a + b + c)2 ≤ 2(ab + bc + ca) ,
b+c
”X —X a
(a + b + c)2 ≤ a(b + c) .
b+c
Clearly, the last inequality follows from the Cauchy-Schwarz inequality. The equal-
ity holds for a = b = c.
Second Solution. Use the SOS method. Let us denote
p p p
A = b2 + bc + c 2 , B = c 2 + ca + a2 , C = a2 + ab + b2 .

Without loss of generality, assume that a ≥ b ≥ c. By squaring, the inequality


becomes X X X
2 BC ≤ 3 a2 + 3 ab,
Symmetric Nonrational Inequalities 297
X X X
a2 − ab ≤ (B − C)2 ,
X X (b − c)2
(b − c) ≤ 2(a + b + c)
2 2
.
(B + C)2
Since
(B + C)2 ≤ 2(B 2 + C 2 ) = 2(2a2 + b2 + c 2 + ca + ab),
it suffices to show that
X X (b − c)2
(b − c)2 ≤ (a + b + c)2 ,
2a2 + b2 + c 2 + ca + ab
which is equivalent to X
(b − c)2 Sa ≥ 0,
where
−a2 + ab + 2bc + ca
Sa = ,
2a2 + b2 + c 2 + ca + ab
−b2 + bc + 2ca + ab
Sb = 2 ≥ 0,
2b + c 2 + a2 + ab + bc
−c 2 + ca + 2ab + bc
Sc = 2 ≥ 0.
2c + a2 + b2 + bc + ca
Since
X a2
(b − c)2 Sa ≥ (b − c)2 Sa + (a − c)2 S b ≥ (b − c)2 Sa + 2 (b − c)2 S b
b
2 Sa Sb
‹
a
≥ (b − c) Sa + (b − c) S b = a(b − c)
2 2
+ ,
b a b
we only need to prove that
Sa S b
+ ≥ 0,
a b
which is equivalent to
−b2 + bc + 2ca + ab a2 − ab − 2bc − ca
≥ .
b(2b2 + c 2 + a2 + ab + bc) a(2a2 + b2 + c 2 + ca + ab)
Consider the nontrivial case where a2 − ab − 2bc − ca ≥ 0. Since

(2a2 + b2 + c 2 + ca + ab) − (2b2 + c 2 + a2 + ab + bc) = (a − b)(a + b + c) ≥ 0,

it suffices to show that


−b2 + bc + 2ca + ab a2 − ab − 2bc − ca
≥ .
b a
Indeed,

a(−b2 + bc + 2ca + ab) − b(a2 − ab − 2bc − ca) = 2c(a2 + ab + b2 ) > 0.


298 Vasile Cîrtoaje

P 2.8. If a, b, c are nonnegative real numbers, then


Xp p p
a2 + ab + b2 ≤ 2 a2 + b2 + c 2 + ab + bc + ca.

(Vasile Cîrtoaje, 2010)

First Solution (by Nguyen Van Quy). Assume that a = max{a, b, c}. Since
p p Æ
a2 + ab + b2 + c 2 + ca + a2 ≤ 2[(a2 + ab + b2 ) + (c 2 + ca + a2 )],

it suffices to show that


p p p p
2 A + b2 + bc + c 2 ≤ 2 X + Y ,

where
1
A = a2 + (b2 + c 2 + ab + ac), X = a2 + b2 + c 2 , Y = ab + bc + ca.
2
Write the desired inequality as follows:
p p p p
2( A − X ) ≤ Y − b2 + bc + c 2 ,

2(A − X ) Y − (b2 + bc + c 2 )
p p ≤p p ,
A+ X Y + b2 + bc + c 2
b(a − b) + c(a − c) b(a − b) + c(a − c)
p p ≤p p .
A+ X Y + b2 + bc + c 2
Since b(a − b) + c(a − c) ≥ 0, we only need to show that
p p p p
A + X ≥ Y + b2 + bc + c 2 .

This inequality is true because X ≥ Y and


p p
A ≥ b2 + bc + c 2 .

Indeed,

2(A − b2 − bc − c 2 ) = 2a2 + (b + c)a − (b + c)2 = (2a − b − c)(a + b + c) ≥ 0.

The equality holds for a = b = c, and also for b = c = 0 (or any cyclic permutation).
Second Solution. In the first solution of P 2.7, we have shown that
€X p Š2 X 1
b2 + bc + c 2 ≤ 4(a2 + b2 + c 2 ) + 6(ab + bc + ca) − 2abc .
b+c
Thus, it suffices to prove that
X 1 € p p Š2
4(a2 +b2 +c 2 )+6(ab+bc+ca)−2abc ≤ 2 a2 + b2 + c 2 + ab + bc + ca ,
b+c
Symmetric Nonrational Inequalities 299

which is equivalent to
X 1 Æ
2abc + 4 (a2 + b2 + c 2 )(ab + bc + ca) ≥ 5(ab + bc + ca).
b+c
Since X 1 9 9
≥P = ,
b+c (b + c) 2(a + b + c)
it is enough to prove that
9abc Æ
+ 4 (a2 + b2 + c 2 )(ab + bc + ca) ≥ 5(ab + bc + ca),
a+b+c
which can be written as
9abc Æ
+ 4 q(p2 − 2q) ≥ 5q,
p
where
p = a + b + c, q = ab + bc + ca.
p
For p2 ≥ 4q, this inequality is true because 4 q(p2 − 2q) ≥ 5q. Consider further

3q ≤ p2 ≤ 4q.

By Schur’s inequality of third degree, we have


9abc
≥ 4q − p2 .
p
Therefore, it suffices to show that
Æ
(4q − p2 ) + 4 q(p2 − 2q) ≥ 5q,

which is Æ
4 q(p2 − 2q) ≥ p2 + q.
Indeed,
16q(p2 − 2q) − (p2 + q)2 = (p2 − 3q)(11q − p2 ) ≥ 0.
Third Solution. Let us denote
p p p
A = b2 + bc + c 2 , B = c 2 + ca + a2 , C = a2 + ab + b2 ,
p p
X = a2 + b2 + c 2 , Y = ab + bc + ca.
By squaring, the inequality becomes
X X
2 BC ≤ 2 a2 + 4X Y,
X
(B − C)2 ≥ 2(X − Y )2 ,
300 Vasile Cîrtoaje

P 2
X (b − c)2 (b − c)2
2(a + b + c)2 ≥ .
(B + C)2 (X + Y )2
Since
B + C ≤ (c + a) + (a + b) = 2a + b + c,
it suffices to show that
P 2
X (b − c)2 (b − c)2
2(a + b + c)2 ≥ .
(2a + b + c)2 (X + Y )2

According to the Cauchy-Schwarz inequality, we have


P 2
X (b − c)2 (b − c)2
≥P .
(2a + b + c)2 (b − c)2 (2a + b + c)2

Therefore, it is enough to prove that

2(a + b + c)2 1
≥ ,
(b − c)2 (2a + b + c)2 (X + Y )2
P

which is
1X
(a + b + c)2 (X + Y )2 ≥ (b − c)2 (2a + b + c)2 .
2
We see that
€X X Š €X X Š
(a + b + c)2 (X + Y )2 ≥ a2 + 2 ab a2 + ab
€X Š2 €X Š €X Š €X Š2
= a2 + 3 ab a2 + 2 ab
X X X
≥ a4 + 3 ab(a2 + b2 ) + 4 a2 b2
and X X
(b − c)2 (2a + b + c)2 = (b − c)2 [4a2 + 4a(b + c) + (b + c)2 ]
X X X
=4 a (b − c) + 4
2 2
a(b − c)(b − c ) +
2 2
(b2 − c 2 )2
X X X
≤8 a2 b2 + 4 a(b3 + c 3 ) + 2 a4 .
Thus, it suffices to show that
X X X X X X
a4 + 3 ab(a2 + b2 ) + 4 a2 b2 ≥ 4 a2 b2 + 2 a(b3 + c 3 ) + a4 ,

which is equivalent to the obvious inequality


X
ab(a2 + b2 ) ≥ 0.
Symmetric Nonrational Inequalities 301

P 2.9. If a, b, c are nonnegative real numbers, then


p p p p p
a2 + 2bc + b2 + 2ca + c 2 + 2ab ≤ a2 + b2 + c 2 + 2 ab + bc + ca.

(Vasile Cîrtoaje and Nguyen Van Quy, 1989)

Solution (by Nguyen Van Quy). Let


p p
X = a2 + b2 + c 2 , Y= ab + bc + ca.

Consider the nontrivial case when no two of a, b, c are zero and write the inequality
as X€ p Š
X − a2 + 2bc ≥ 2(X − Y ),

(b − c)2
P
X (b − c)2
p ≥ .
X + a2 + 2bc X +Y
By the Cauchy-Schwarz inequality, we have

2 2
P 
X (b − c) 2 (b − c)
p ≥P p .
X + a2 + 2bc (b − c)2 X + a2 + 2bc

Therefore, it suffices to show that

(b − c)2
P
1
p ≥ ,
(b − c)2 X + a2 + 2bc X +Y
P

which is equivalent to
X € p Š
(b − c)2 Y − a2 + 2bc ≥ 0.

From € p Š2
Y− a2 + 2bc ≥ 0.
we get
p Y 2 − (a2 + 2bc) (a − b)(c − a)
Y− a2 + 2bc ≥ = .
2Y 2Y
Thus,
X € p Š X (b − c)2 (a − b)(c − a)
(b − c)2 Y − a2 + 2bc ≥
2Y
(a − b)(b − c)(c − a) X
= (b − c) = 0.
2Y
The equality holds for a = b, or b = c, or c = a.
302 Vasile Cîrtoaje

P 2.10. If a, b, c are nonnegative real numbers, then

1 1 1 1 2
p +p +p ≥p +p .
a2 + 2bc b2 + 2ca c 2 + 2ab a2 + b2 + c 2 ab + bc + ca
(Vasile Cîrtoaje, 1989)

Solution . Let p p
X= a2 + b2 + c 2 , Y= ab + bc + ca.
Consider the nontrivial case when Y > 0 and write the inequality as
X 1 1 1 1
‹  ‹
p − ≥ 2 − ,
a2 + 2bc X Y X

(b − c)2
P
X (b − c)2
p p ≥ .
a2 + 2bc X + a2 + 2bc Y (X + Y )
By the Cauchy-Schwarz inequality, we have
P 2
X (b − c)2 (b − c)2
p p ≥P p p .
a2 + 2bc X + a2 + 2bc (b − c)2 a2 + 2bc X + a2 + 2bc

Therefore, it suffices to show that

(b − c)2
P
1
p p ≥ ,
(b − c)2 a2 + 2bc X + a2 + 2bc Y (X + Y )
P

which is equivalent to
X p
(b − c)2 [X Y − X a2 + 2bc + (a − b)(c − a)] ≥ 0.

Since
X X
(b − c)2 (a − b)(c − a) = (a − b)(b − c)(c − a) (b − c) = 0,

we can write the inequality as


X € p Š
(b − c)2 X Y − a2 + 2bc ≥ 0,
X € p Š
(b − c)2 Y − a2 + 2bc ≥ 0.
We have proved this inequality at the preceding problem P 2.9. The equality holds
for a = b, or b = c, or c = a.
Symmetric Nonrational Inequalities 303

P 2.11. If a, b, c are positive real numbers, then


p p p p p
2a2 + bc + 2b2 + ca + 2c 2 + ab ≤ 2 a2 + b2 + c 2 + ab + bc + ca.

Solution. We will apply Lemma below for


X = 2a2 + bc, Y = 2b2 + ca, Z = 2c 2 + ab
and
A = B = a2 + b2 + c 2 , C = ab + bc + ca.
We have
X + Y + Z = A + B + C, A = B ≥ C.
Without loss of generality, assume that
a ≥ b ≥ c,
which involves
X ≥ Y ≥ Z.
By Lemma below, it suffices to show that
max{X , Y, Z} ≥ A, min{X , Y, Z} ≤ C.
Indeed, we have
max{X , Y, Z} − A = X − A = (a2 − b2 ) + c(b − c) ≥ 0,
min{X , Y, Z} − C = Z − C = c(2c − a − b) ≤ 0.
Equality holds for a = b = c.
Lemma. If X , Y, Z and A, B, C are positive real numbers such that
X + Y + Z = A + B + C,
max{X , Y, Z} ≥ max{A, B, C}, min{X , Y, Z} ≤ min{A, B, C},
then p p p p p p
X+ Y+ Z≤ A + B + C.
Proof. On the assumption that X ≥ Y ≥ Z and A ≥ B ≥ C, we have
X ≥ A, Z ≤ C,
hence
p p p p p p p p p p p p
X + Y + Z − A− B − C = ( X − A ) + ( Y − B ) + ( Z − C )
X −A Y −B Z −C X −A Y −B Z −C
≤ p + p + p ≤ p + p + p
2 A 2 B 2 C 2 B 2 B 2 C
C −Z Z −C 1 1
 ‹
= p + p = (C − Z) p − p ≤ 0.
2 B 2 C 2 B 2 C

Remark. This Lemma is a particular case of Karamata’s inequality.


304 Vasile Cîrtoaje

p
P 2.12. Let a, b, c be nonnegative real numbers such that a + b + c = 3. If k = 3−1,
then XÆ p
a(a + kb)(a + kc) ≤ 3 3.

Solution. By the Cauchy-Schwarz inequality, we have


XÆ r€X Š ”X —
a(a + kb)(a + kc) ≤ a (a + kb)(a + kc) .

Thus, it suffices to show that


rX
(a + kb)(a + kc) ≤ a + b + c,

which is an identity. The equality holds for a = b = c = 1, and also for a = 3 and
b = c = 0 (or any cyclic permutation).

P 2.13. If a, b, c are nonnegative real numbers such that a + b + c = 3, then



a(2a + b)(2a + c) ≥ 9.

Solution. Write the inequality as follows:


X ”Æ Æ —
a(2a + b)(2a + c) − a 3(a + b + c) ≥ 0,
X
(a − b)(a − c)Ea ≥ 0,
where p
a
Ea = p p .
(2a + b)(2a + c) + 3a(a + b + c)
Assume that a ≥ b ≥ c. Since (c − a)(c − b)Ec ≥ 0, it suffices to show that

(a − c)Ea ≥ (b − c)E b ,

which is equivalent to
Æ Æ Æ
(a − b) 3ab(a + b + c)+(a − c) a(2b + c)(2b + a) ≥ (b − c) b(2a + b)(2a + c).

This is true if
Æ Æ
(a − c) a(2b + c)(2b + a) ≥ (b − c) b(2a + b)(2a + c).

For the nontrivial case b > c, we have


p
a−c a a
≥ ≥p .
b−c b b
Symmetric Nonrational Inequalities 305

Therefore, it is enough to show that

a2 (2b + c)(2b + a) ≥ b2 (2a + b)(2a + c).

Write this inequality as

a2 (2ab + 2bc + ca) ≥ b2 (2ab + bc + 2ca).

It is true if
a(2ab + 2bc + ca) ≥ b(2ab + bc + 2ca).
Indeed,

a(2ab + 2bc + ca) − b(2ab + bc + 2ca) = (a − b)(2ab + bc + ca) ≥ 0.

The equality holds for a = b = c = 1, and also for a = b = 3/2 and c = 0 (or any
cyclic permutation).

P 2.14. Let a, b, c be nonnegative real numbers such that a + b + c = 3. Prove that


Æ Æ Æ
b2 + c 2 + a(b + c) + c 2 + a2 + b(c + a) + a2 + b2 + c(a + b) ≥ 6.

Solution. Denote

A = b2 + c 2 + a(b + c), B = c 2 + a2 + b(c + a), C = a2 + b2 + c(a + b),

and write the inequality in the homogeneous form


p p p
A + B + C ≥ 2(a + b + c).

Further, we use the SOS method.


First Solution. By squaring, the inequality becomes
Xp X X
2 BC ≥ 2 a2 + 6 bc,
X X p p 2
(b − c)2 ≥ B− C ,
X
(b − c)2 Sa ≥ 0,
where
(b + c − a)2
Sa = 1 − p p .
( B + C)2
Since
(b + c − a)2 a(a + 3b + 3c)
Sa ≥ 1 − = ≥ 0, S b ≥ 0, Sc ≥ 0,
B+C B+C
306 Vasile Cîrtoaje

the conclusion follows. The equality holds for a = b = c = 1, and also for a = 3
and b = c = 0 (or any cyclic permutation).
Second Solution. Write the desired inequality as follows:
Xp
( A − b − c) ≥ 0,

X c(a − b) + b(a − c)
p ≥ 0,
A+ b + c
X c(a − b) X c(b − a)
p + p ≥ 0,
A+ b + c B+c+a
X c(a − b)[a − b − (pA − pB)]
p p ≥ 0.
( A + b + c)( B + c + a)
It suffices to show that
p p
(a − b)[a − b + ( B − A)] ≥ 0.

Indeed,
p p a+b−c
 ‹
(a − b)[a − b + ( B − A)] = (a − b) 1 + p
2
p ≥ 0,
B+ A
because, for the nontrivial case a + b − c < 0, we have

a+b−c a+b−c
1+ p p >1+ > 0.
B+ A c+c

16
Generalization. Let a, b, c be nonnegative real numbers. If 0 < k ≤ , then
9

(b + c)2 + k(ab − 2bc + ca) ≥ 2(a + b + c).

16
Notice that if k = , then the equality holds for a = b = c = 1, for a = 0 and
9
b = c (or any cyclic permutation), and for b = c = 0 (or any cyclic permutation).

P 2.15. Let a, b, c be nonnegative real numbers such that a + b + c = 3. Prove that


p p p
(a) a(3a2 + abc) + b(3b2 + abc) + c(3c 2 + abc) ≥ 6;
p p p p
(b) 3a2 + abc + 3b2 + abc + 3c 2 + abc ≥ 3 3 + abc.
(Lorian Saceanu, 2015)
Symmetric Nonrational Inequalities 307

Solution. (a) Write the inequality in the homogeneous form


X Æ
3 a (a + b)(a + c) ≥ 2(a + b + c)2 .

First Solution. Use the SOS method. Write the inequality as


X X 3 X €p p Š2
a2 − ab ≥ a a+b− a+c ,
2
X X a(b − c)2
(b − c)2 ≥ 3 p p 2 ,
a+b+ a+c
X
(b − c)2 Sa ≥ 0,
where
3a
Sa = 1 − p p 2 .
a+b+ a+c
Since
3a
Sa ≥ 1 − p p 2 > 0, S b > 0, Sc > 0,
a+ a
the inequality is true. The equality holds for a = b = c = 1.
Second Solution. By Hölder’s inequality, we have

( a)3
P
”X Æ —2 27
a (a + b)(a + c) ≥ P a =P a .
(a + b)(a + c) (a + b)(a + c)
Therefore, it suffices to show that
X a 3
≤ .
(a + b)(a + c) 4
This inequality has the homogeneous form
X a 9
≤ ,
(a + b)(a + c) 4(a + b + c)
which is equivalent to the obvious inequality
X
a(b − c)2 ≥ 0.

(b) By squaring, the inequality becomes


X XÆ
3 a2 + 2 (3b2 + abc)(3c 2 + abc) ≥ 27 + 6abc.

According to the Cauchy-Schwarz inequality, we have


Æ
(3b2 + abc)(3c 2 + abc) ≥ 3bc + abc.
308 Vasile Cîrtoaje

Therefore, it suffices to show that


X X
3 a2 + 6 bc + 6abc ≥ 27 + 6abc,

which is an identity. The equality holds for a = b = c = 1, and also for a = 0, or


b = 0, or c = 0.

P 2.16. Let a, b, c be positive real numbers such that ab + bc + ca = 3. Prove that


Æ Æ Æ
a (a + 2b)(a + 2c) + b (b + 2c)(b + 2a) + c (c + 2a)(c + 2b) ≥ 9.

First Solution. Use the SOS method. Write the inequality as follows:
X Æ
a (a + 2b)(a + 2c) ≥ 3(ab + bc + ca),
X X 1 X €p p Š2
a2 − ab ≥ a a + 2b − a + 2c ,
2
X X a(b − c)2
(b − c) ≥ 4
2
p p 2 ,
a + 2b + a + 2c
X
(b − c)2 Sa ≥ 0,
where
4a
Sa = 1 − p p 2 .
a + 2b + a + 2c
Since
4a
Sa > 1 − p p 2 = 0, S b > 0, Sc > 0,
a+ a
the inequality is true. The equality holds for a = b = c = 1.
Second Solution. We use the AM-GM inequality to get
X Æ X 2a(a + 2b)(a + 2c) X 2a(a + 2b)(a + 2c)
a (a + 2b)(a + 2c) = ≥
(a + 2b) + (a + 2c)
p
2 (a + 2b)(a + 2c)
1 X
= a(a + 2b)(a + 2c).
a+b+c
Thus, it suffices to show that
X
a(a + 2b)(a + 2c) ≥ 9(a + b + c).

Write this inequality in the homogeneous form


X
a(a + 2b)(a + 2c) ≥ 3(a + b + c)(ab + bc + ca),
Symmetric Nonrational Inequalities 309

which is equivalent to Schur’s inequality of degree three

a3 + b3 + c 3 + 3abc ≥ ab(a + b) + bc(b + c) + ca(c + a).

P 2.17. Let a, b, c be nonnegative real numbers such that a + b + c = 1. Prove that


Æ Æ Æ p
a + (b − c)2 + b + (c − a)2 + c + (a − b)2 ≥ 3.

(Phan Thanh Nam, 2007)

Solution. By squaring, the inequality becomes



[a + (b − c)2 ][b + (c − a)2 ] ≥ 3(ab + bc + ca).

Applying the Cauchy-Schwarz inequality, it suffices to show that


Xp X
ab + (b − c)(a − c) ≥ 3(ab + bc + ca).

This is equivalent to the homogeneous inequality


€X Š €X p Š X
a ab + a2 ≥ 4(ab + bc + ca).
p p p
Making the substitution x = a, y = b, z = c, the inequality turns into
€X Š €X Š X X
x2 xy + x4 ≥ 4 x 2 y 2,

which is equivalent to
X X X X
x4 + x y(x 2 + y 2 ) + x yz x ≥4 x 2 y 2.

Since X X
4 2 2
x y ≤2 x y(x 2 + y 2 ),
it suffices to show that
X X X
x 4 + x yz x≥ x y(x 2 + y 2 ),

1
which is just Schur’s inequality of degree four. The equality holds for a = b = c = ,
3
1
and for a = 0 and b = c = (or any cyclic permutation).
2
310 Vasile Cîrtoaje

P 2.18. Let a, b, c be nonnegative real numbers, no two of which are zero. Prove that
v v v
t a(b + c) t b(c + a) t c(a + b)
+ + ≥ 2.
a2 + bc b2 + ca c 2 + ab

(Vasile Cîrtoaje, 2006)

Solution. Using the AM-GM inequality gives


v
t a(b + c) a(b + c) 2a(b + c) 2a(b + c)
=p ≥ = .
a2 + bc (a2 + bc)(ab + ac) (a2 + bc) + (ab + ac) (a + b)(a + c)

Therefore, it suffices to show that

a(b + c) b(c + a) c(a + b)


+ + ≥ 1,
(a + b)(a + c) (b + c)(b + a) (c + a)(c + b)

which is equivalent to

a(b + c)2 + b(c + a)2 + c(a + b)2 ≥ (a + b)(b + c)(c + a),

4abc ≥ 0.
The equality holds for a = 0 and b = c (or any cyclic permutation).

P 2.19. Let a, b, c be positive real numbers such that abc = 1. Prove that

1 1 1
p
3
+p
3
+p
3
≥ 1.
a2 + 25a + 1 b2 + 25b + 1 c 2 + 25c + 1

Solution. Replacing a, b, c by a3 , b3 , c 3 , respectively, we need to show that abc = 1


yields
1 1 1
p
3
+p 3
+p 3
≥ 1.
a6 + 25a3 + 1 b6 + 25b3 + 1 c 6 + 25c 3 + 1
We first show that
1 1
p ≥ 2 .
a6 + 25a3 + 1 a + a + 1
3

This is equivalent to
(a2 + a + 1)3 ≥ a6 + 25a3 + 1,
which is true since

(a2 + a + 1)3 − (a6 + 25a3 + 1) = 3a(a − 1)2 (a2 + 4a + 1) ≥ 0.


Symmetric Nonrational Inequalities 311

Therefore, it suffices to prove that

1 1 1
+ 2 + 2 ≥ 1.
a2 +a+1 b + b+1 b + b+1
Putting
yz zx xy
a= , b= , c= , x, y, z > 0
x2 y2 z2
we need to show that
X x4
≥ 1.
x 4 + x 2 yz + y 2 z 2
Indeed, the Cauchy-Schwarz inequality gives
P 2 2
x + 2 y 2z2
P 4 P
X x4 x
≥P =P ≥ 1.
x 4 + x 2 yz + y 2 z 2 (x 4 + x 2 yz + y 2 z 2 ) x 4 + x yz x + y 2 z 2
P P

The equality holds for a = b = c = 1.

P 2.20. If a, b, c are nonnegative real numbers, then


p p p 3
a2 + bc + b2 + ca + c 2 + ab ≤ (a + b + c).
2
(Pham Kim Hung, 2005)

Solution. Without loss of generality, assume that a ≥ b ≥ c. Since the equality


occurs for a = b and c = 0, we use the inequalities
p c
a2 + bc ≤ a +
2
and p p Æ
b2 + ca + c 2 + ab ≤ 2(b2 + ca) + 2(c 2 + ab).
Thus, it suffices to prove that
Æ a + 3b + 2c
2(b2 + ca) + 2(c 2 + ab) ≤ .
2
By squaring, this inequality becomes

a2 + b2 − 4c 2 − 2ab + 12bc − 4ca ≥ 0,

(a − b − 2c)2 + 8c(b − c) ≥ 0.
The equality holds for a = b and c = 0 (or any cyclic permutation).
312 Vasile Cîrtoaje

P 2.21. If a, b, c are nonnegative real numbers, then


p p p p
a2 + 9bc + b2 + 9ca + c 2 + 9ab ≥ 5 ab + bc + ca.

(Vasile Cîrtoaje, 2012)

Solution (by Nguyen Van Quy). Assume that

c = min{a, b, c}.

Since the equality occurs for a = b and c = 0, we use the inequality


p p
c 2 + 9ab ≥ 3 ab.

On the other hand, by Minkowski’s inequality, we have


s
p p €p p Š2
a2 + 9bc + b2 + 9ca ≥ (a + b)2 + 9c a+ b .

Therefore, it suffices to show that


s
€p p Š2 p p
(a + b)2 + 9c a + b ≥ 5 ab + bc + ca − 3 ab.

By squaring, this inequality becomes


p Æ
(a + b)2 + 18c ab + 30 ab(ab + bc + ca) ≥ 34ab + 16c(a + b).

Since
˜2
c(a + b) c(a + b)(3ab − ac − bc)
•
ab(ab + bc + ca) − ab + = ≥ 0,
3 9
p
it suffices to show that f (c) ≥ 0 for 0 ≤ c ≤ ab, where
p
f (c) = (a + b)2 + 18c ab + [30ab + 10c(a + b)] − 34ab − 16c(a + b)
p
= (a + b)2 − 4ab + 6c(3 ab − a − b).
p
Since f (c) is a linear function, we only need to prove that f (0) ≥ 0 and f ( ab) ≥
0. We have
f (0) = (a − b)2 ≥ 0,
p p p
f ( ab) = (a + b)2 + 14ab − 6(a + b) ab ≥ (a + b)2 + 9ab − 6(a + b) ab
€ p Š2
= a + b − 3 ab ≥ 0.
The equality holds for a = b and c = 0 (or any cyclic permutation).
Symmetric Nonrational Inequalities 313

P 2.22. If a, b, c are nonnegative real numbers, then



(a2 + 4bc)(b2 + 4ca) ≥ 5(ab + ac + bc).

(Vasile Cîrtoaje, 2012)

Solution. Assume that


a ≥ b ≥ c.
First Solution (by Michael Rozenberg). Use the SOS method. For b = c = 0, the
inequality is trivial. Consider further that b > 0 and write the inequality as follows:
X ”Æ —
(b2 + 4ca)(c 2 + 4ab) − (bc + 2ab + 2ac) ≥ 0,

X (b2 + 4ca)(c 2 + 4ab) − (bc + 2ab + 2ac)2


p ≥ 0,
(b2 + 4ca)(c 2 + 4ab) + bc + 2a(b + c)
X
(b − c)2 Sa ≥ 0,
where
a(b + c − a) Æ
Sa = , A = (b2 + 4ca)(c 2 + 4ab) + bc + 2a(b + c),
A
b(c + a − b) Æ
Sb = , B = (c 2 + 4ab)(a2 + 4bc) + ca + 2b(c + a),
B
c(a + b − c) Æ
Sc = , C = (a2 + 4bc)(b2 + 4ac) + ab + 2c(a + b).
C
Since S b ≥ 0 and Sc ≥ 0, we have
X a2
(b − c)2 Sa ≥ (b − c)2 Sa + (a − c)2 S b ≥ (b − c)2 Sa + 2 (b − c)2 S b
b
bSa aS b
 ‹
a
= (b − c)2 + .
b a b
Thus, it suffices to prove that

bSa aS b
+ ≥ 0,
a b
which is equivalent to

b(b + c − a) a(c + a − b)
+ ≥ 0.
A B
Since
b(b + c − a) a(c + a − b) b(b − a) a(a − b) (a − b)(aA − bB)
+ ≥ + = ,
A B A B AB
314 Vasile Cîrtoaje

it is enough to show that


aA − bB ≥ 0.
Indeed,
p ” p p —
aA − bB = c 2 + 4ab a b2 + 4ca − b a2 + 4bc + 2(a − b)(ab + bc + ca)
p
4c(a3 − b3 ) c 2 + 4ab
= p p + 2(a − b)(ab + bc + ca) ≥ 0.
a b2 + 4ca + b a2 + 4bc
The equality holds for a = b = c, and also for a = b and c = 0 (or any cyclic
permutation).
Second Solution (by Nguyen Van Quy). Write the inequality as
€p p p Š2
a2 + 4bc + b2 + 4ca + c 2 + 4ab ≥ a2 + b2 + c 2 + 14(ab + bc + ca),
p p p Æ
a2 + 4bc + b2 + 4ca + c 2 + 4ab ≥ a2 + b2 + c 2 + 14(ab + bc + ca).
For t = 2c, the inequality (b) in Lemma below becomes
p p Æ
a2 + 4bc + b2 + 4ca ≥ (a + b)2 + 8(a + b)c.

Thus, it suffices to show that


Æ p Æ
(a + b)2 + 8(a + b)c + c 2 + 4ab ≥ a2 + b2 + c 2 + 14(ab + bc + ca).

By squaring, this inequality becomes


Æ
[(a + b)2 + 8(a + b)c] (c 2 + 4ab) ≥ 4ab + 3(a + b)c,

2(a + b)c 3 − 2(a + b)2 c 2 + 2ab(a + b)c + ab(a + b)2 − 4a2 b2 ≥ 0,


2(a + b)(a − c)(b − c)c + ab(a − b)2 ≥ 0.
Lemma. Let a,b and t be nonnegative numbers such that

t ≤ 2(a + b).

Then,
p
(a) (a2 + 2bt)(b2 + 2at) ≥ ab + (a + b)t;
p p p
(b) a2 + 2bt + b2 + 2at ≥ (a + b)2 + 4(a + b)t.

Proof. (a) By squaring, the inequality becomes

(a − b)2 t[2(a + b) − t] ≥ 0,

which is clearly true.


(b) By squaring, this inequality turns into the inequality in (a).
Symmetric Nonrational Inequalities 315

P 2.23. If a, b, c are nonnegative real numbers, then



(a2 + 9bc)(b2 + 9ca) ≥ 7(ab + ac + bc).

(Vasile Cîrtoaje, 2012)

Solution (by Nguyen Van Quy). We see that the equality holds for a = b and c = 0.
Without loss of generality, assume that

c = min{a, b, c}.

For t = 4c, the inequality (a) in Lemma from the preceding P 2.22 becomes
Æ
(a2 + 8bc)(b2 + 8ca) ≥ ab + 4(a + b)c.

Thus, we have Æ
(a2 + 9bc)(b2 + 9ca) ≥ ab + 4(a + b)c
and
p €p p Š p Æ
4
c 2 + 9ab a2 + 9bc + b2 + 9ca ≥ 3 ab · 2 (a2 + 9bc)(b2 + 9ca)
p Æ Æ
≥ 6 ab · ab + 4(a + b)c = 3 4a2 b2 + 16abc(a + b)
Æ
≥ 3 4a2 b2 + 4abc(a + b) + c 2 (a + b)2 = 3(2ab + bc + ca).
Therefore,

(a2 + 9bc)(b2 + 9ca) ≥ (ab + 4bc + 4ca) + 3(2ab + bc + ca)
= 7(ab + bc + ca).

The equality holds for a = b and c = 0 (or any cyclic permutation).

P 2.24. If a, b, c are nonnegative real numbers, then


Æ Æ Æ
(a2 + b2 )(b2 + c 2 ) + (b2 + c 2 )(c 2 + a2 ) + (c 2 + a2 )(a2 + b2 ) ≤ (a + b + c)2 .

(Vasile Cîrtoaje, 2007)

Solution. Without loss of generality, assume that

a = min{a, b, c}.

Let us denote
a a
y= + b, z= + c.
2 2
316 Vasile Cîrtoaje

Since
a2 + b2 ≤ y 2 , b2 + c 2 ≤ y 2 + z 2 , c 2 + a2 ≤ z 2 ,
it suffices to prove that
p
yz + ( y + z) y 2 + z 2 ≤ ( y + z)2 .

This is true since


p y 2z2
y 2 + yz + z 2 − ( y + z) y 2 + z 2 = ≥ 0.
y 2 + yz + z 2 + ( y + z) y 2 + z 2
p

The equality holds for a = b = 0 (or any cyclic permutation).

P 2.25. If a, b, c are nonnegative real numbers, then



(a2 + ab + b2 )(b2 + bc + c 2 ) ≥ (a + b + c)2 .

Solution. By the Cauchy-Schwarz inequality, we have


 ‹2  •
3b2 c 2 3c 2
˜
b
(a + ab + b )(a + ac + c ) =
2 2 2 2
a+ + a+ +
2 4 2 4

c  3bc a(b + c)
 ‹
b 
≥ a+ a+ + = a2 + + bc.
2 2 4 2
Then,
X• a(b + c)
XÆ ˜
(a2 + ab + b2 )(a2 + ac + c2) ≥ a +
2
+ bc = (a + b + c)2 .
2

The equality holds for a = b = c, and also for b = c = 0 (or any cyclic permutation).

P 2.26. If a, b, c are nonnegative real numbers, then



(a2 + 7ab + b2 )(b2 + 7bc + c 2 ) ≥ 7(ab + ac + bc).

(Vasile Cîrtoaje, 2012)


Symmetric Nonrational Inequalities 317

First Solution. Without loss of generality, assume that

c = min{a, b, c}.

We see that the equality holds for a = b and c = 0. Since


Æ
(a2 + 7ac + c 2 )(b2 + 7bc + c 2 ) ≥ (a + 2c)(b + 2c) ≥ ab + 2c(a + b),

it suffices to show that


p €p p Š
a2 + 7ab + b2 a2 + 7ac + b2 + 7bc ≥ 6ab + 5c(a + b).

By Minkowski’s inequality, we have


s
p p €p p Š2
a2 + 7ac + b2 + 7bc ≥(a + b)2 + 7c a+ b
v
t 28abc
≥ (a + b)2 + 7c(a + b) + .
a+b
Therefore, it suffices to show that
28abc
• ˜
(a + 7ab + b ) (a + b) + 7c(a + b) +
2 2 2
≥ (6ab + 5bc + 5ca)2 .
a+b
1
Due to homogeneity, we may assume that a + b = 1. Let us denote d = ab, d ≤ .
4
Since
2ab
c≤ = 2d,
a+b
1
we need to show that f (c) ≥ 0 for 0 ≤ c ≤ 2d ≤ , where
2
f (c) = (1 + 5d)(1 + 7c + 28cd) − (6d + 5c)2 .

Since f (c) is concave, it suffices to show that f (0) ≥ 0 and f (2d) ≥ 0. Indeed,

f (0) = 1 + 5d − 36d 2 = (1 − 4d)(1 + 9d) ≥ 0

and

f (2d) = (1 + 5d)(1 + 14d + 56d 2 ) − 256d 2 ≥ (1 + 4d)(1 + 14d + 56d 2 ) − 256d 2

= (1 − 4d)(1 + 22d − 56d 2 ) ≥ d(1 − 4d)(22 − 56d) ≥ 0.


The equality holds for a = b and c = 0 (or any cyclic permutation).
Second Solution. We will use the inequality
p 2x y
x 2 + 7x y + y 2 ≥ x + y + , x, y ≥ 0,
x+y
318 Vasile Cîrtoaje

which, by squaring, reduces to

x y(x − y)2 ≥ 0.

We have
X 2ab 2ac
XÆ ‹ ‹
(a2 + 7ab + b2 )(a2 + 7ac + c2) ≥ a+b+ a+c+
a+b a+c
X X X 2a2 b X 2a2 c X 2abc
≥ a2 + 3 ab + + + .
a+b a+c a+b
Since
X 2a2 b X 2a2 c X 2a2 b X 2b2 a X
+ = + =2 ab
a+b a+c a+b b+a
and X 2abc 18abc 9abc
≥P = ,
a+b (a + b) a + b + c
it suffices to show that
X 9abc X
a2 + ≥2 ab,
a+b+c
which is just Schur’s inequality of degree three.

P 2.27. If a, b, c are nonnegative real numbers, then


v
X t 7 7 13
‹ ‹
a2 + ab + b2 b2 + bc + c ≤
2 (a + b + c)2 .
9 9 12

(Vasile Cîrtoaje, 2012)

Solution (by Nguyen Van Quy). Without loss of generality, assume that

c = min{a, b, c}.

It is easy to see that the equality holds for a = b = 1 and c = 0. By the AM-GM
inequality, the following inequality holds for any k > 0:
v ‚v v Œ
t 7 t 7 t 7
12 a2 + ab + b2 a2 + ac + c 2 + b2 + bc + c 2 ≤
9 9 9
‚v v Œ2
36 2 7 7 7
 ‹ t t
≤ a + ab + b2 + k a2 + ac + c 2 + b2 + bc + c 2 .
k 9 9 9
Symmetric Nonrational Inequalities 319

We can use this inequality to prove the original inequality only if


‚v v Œ2
36 2 7 7 7
 ‹ t t
a + ab + b2 = k a2 + ac + c 2 + b2 + bc + c 2
k 9 9 9

for a = b = 1 and c = 0. This condition if satisfied for k = 5. Therefore, it suffices


to show that

7 7 36 2 7
t ‹ ‹  ‹
12 a + ac + c
2 2 b + bc + c +
2 2 a + ab + b +2
9 9 5 9
‚v v Œ2
t 7 t 7
+5 a2 + ac + c 2 + b2 + bc + c 2 ≤ 13(a + b + c)2 .
9 9
which is equivalent to

7 7 4(a + b)2 + 94ab 199c(a + b)
t ‹ ‹
22 a2 + ac + c 2 b2 + bc + c ≤
2 + 3c 2 + .
9 9 5 9

Since
v v
7 7 16 16
t ‹ ‹ t ‹ ‹
2 a2 + ac + c 2 b + bc + c ≤ 2
2 2 a +
2 ac b +
2 bc
9 9 9 9
v 
16 16
t ‹  ‹
=2 a b+ c ·b a+ c
9 9
16 16
 ‹  ‹
≤a b+ c +b a+ c
9 9
16c(a + b)
= 2ab + ,
9
we only need to prove that

8c(a + b) 4(a2 + b2 ) + 102ab 199c(a + b)


• ˜
22 ab + ≤ + 3c 2 + .
9 5 9

This reduces to the obvious inequality

4(a − b)2 23c(a + b)


+ + 3c 2 ≥ 0.
5 9
Thus, the proof is completed. The equality holds for a = b and c = 0 (or any cyclic
permutation).
320 Vasile Cîrtoaje

P 2.28. If a, b, c are nonnegative real numbers, then


v
X t 1 1 61
‹ ‹
a + ab + b
2 2 b + bc + c ≤
2 2 (a + b + c)2 .
3 3 60
(Vasile Cîrtoaje, 2012)
Solution (by Nguyen Van Quy). Without loss of generality, assume that
c = min{a, b, c}.
It is easy to see that the equality holds for c = 0 and 11(a2 + b2 ) = 38ab. By the
AM-GM inequality, the following inequality holds for any k > 0:
v ‚v v Œ
t 1 t 1 t 1
60 a2 + ab + b2 a2 + ac + c 2 + b2 + bc + c 2 ≤
3 3 3
‚v v Œ2
36 2 1 1 1
 ‹ t t
≤ a + ab + b2 + 25k a2 + ac + c 2 + b2 + bc + c 2 .
k 3 3 3
We can use this inequality to prove the original inequality only if the equality
‚v v Œ2
36 2 1 1 1
 ‹ t t
a + ab + b = 25k
2
a2 + ac + c 2 + b2 + bc + c 2
k 3 3 3

holds for c = 0 and 11(a2 + b2 ) = 38ab. This necessary condition if satisfied for
k = 1. Therefore, it suffices to show that

1 1 1
t ‹ ‹  ‹
60 a + ab + b
2 2 b + bc + c + 36 a + ab + b +
2 2 2 2
3 3 3
‚v v Œ2
t 1 t 1
+25 a2 + ac + c 2 + b2 + bc + c 2 ≤ 61(a + b + c)2 ,
3 3
which is equivalent to

1 1 31c(a + b)
t ‹ ‹
10 a + ac + c
2 2 b + bc + c ≤ 10ab + c 2 +
2 2 .
3 3 3
Since
v v
1 1 4 4
t ‹ ‹ t ‹ ‹
2 a + ac + c
2 2 b + bc + c ≤ 2
2 2 a + ac
2 b + bc
2
3 3 3 3
v 
4 4
t ‹  ‹
=2 a b+ c ·b a+ c
3 3
4 4
 ‹  ‹
≤a b+ c +b a+ c
3 3
4c(a + b)
= 2ab + ,
3
Symmetric Nonrational Inequalities 321

we only need to prove that


2c(a + b) 31c(a + b)
• ˜
10 ab + ≤ 10ab + c 2 + .
3 3
This reduces to the obvious inequality

3c 2 + 11c(a + b) ≥ 0.

Thus, the proof is completed. The equality holds for 11(a2 + b2 ) = 38ab and c = 0
(or any cyclic permutation).

P 2.29. If a, b, c are nonnegative real numbers, then


a b c
p +p +p ≥ 1.
4b2 + bc + 4c 2 4c 2 + ca + 4a2 4a2 + ab + 4b2
(Pham Kim Hung, 2006)
Solution. By Hölder’s inequality, we have
P 3
a + 3 ab(a + b) + 6abc
‹2 P 3 P
a
X
a
p ≥P = .
a(4b2 + bc + 4c 2 ) 4 ab(a + b) + 3abc
P
4b2 + bc + 4c 2
Thus, it suffices to show that
X X
a3 + 3abc ≥ ab(a + b),

which is Schur’s inequality of degree three. The equality holds for a = b = c, and
also for a = 0 and b = c (or any cyclic permutation).

P 2.30. If a, b, c are nonnegative real numbers, then


a b c a+b+c
p +p +p ≥p .
b2 + bc + c2 c2 + ca + a2 a2 + ab + b2 ab + bc + ca

Solution. By Hölder’s inequality, we have


‹2 P 3 P 2
a a
X
a
p ≥P = P ,
b2 + bc + c 2 a(b2 + bc + c 2 ) ab
from which the desired inequality follows. The equality holds for a = b = c, and
also for a = 0 and b = c (or any cyclic permutation).
322 Vasile Cîrtoaje

P 2.31. If a, b, c are nonnegative real numbers, then

a b c a+b+c
p +p +p ≤p .
a2 + 2bc b2 + 2ca c 2 + 2ab ab + bc + ca
(Ho Phu Thai, 2007)

Solution. Without loss of generality, assume that

a ≥ b ≥ c.

First Solution. Since


c c
p ≤p ,
c 2 + 2ab ab + bc + ca
it suffices to show that
a b a+b
p +p ≤p ,
a2 + 2bc b2 + 2ca ab + bc + ca
which is equivalent to
p p p p
a( a2 + 2bc − ab + bc + ca) b( ab + bc + ca − b2 + 2ca)
p ≥ p .
a2 + 2bc b2 + 2ca
Since p p
a2 + 2bc − ab + bc + ca ≥ 0
and
a b
p ≥p ,
a2 + 2bc b2 + 2ca
it suffices to show that
p p p p
a2 + 2bc − ab + bc + ca ≥ ab + bc + ca − b2 + 2ca,

which is equivalent to
p p p
a2 + 2bc + b2 + 2ca ≥ 2 ab + bc + ca.

Using the AM-GM inequality, it suffices to show that

(a2 + 2bc)(b2 + 2ca) ≥ (ab + bc + ca)2 ,

which is equivalent to the obvious inequality

c(a − b)2 (2a + 2b − c) ≥ 0.

The equality holds for a = b = c, and also for a = b and c = 0 (or any cyclic
permutation).
Symmetric Nonrational Inequalities 323

Second Solution. By the Cauchy-Schwarz inequality, we have


X ‹2
a €X Š X a 
p ≤ a .
a2 + 2bc a2 + 2bc

Thus, it suffices to prove that


X a a+b+c
≤ .
a2 + 2bc ab + bc + ca
This is equivalent to
X  1 1
‹
a − 2 ≥ 0,
ab + bc + ca a + 2bc
X a(a − b)(a − c)
≥ 0.
a2 + 2bc
We have
X a(a − b)(a − c) a(a − b)(a − c) b(b − c)(b − a)
≥ +
a2 + 2bc a2 + 2bc b2 + 2ca
c(a − b)2 [2a(a − c) + 2b(b − c) + 3ab]
= ≥ 0.
(a2 + 2bc)(b2 + 2ca)

P 2.32. If a, b, c are nonnegative real numbers, then


p p p
a3 + b3 + c 3 + 3abc ≥ a2 a2 + 3bc + b2 b2 + 3ca + c 2 c 2 + 3ab.

(Vo Quoc Ba Can, 2008)

Solution. For a = 0, the inequality is an identity. Consider further that a, b, c > 0,


and write the inequality as follows:
X p
a2 ( a2 + 3bc − a) ≤ 3abc,

X 3a2 bc
p ≤ 3abc,
a2 + 3bc + a
X 1
p ≤ 1.
1 + 3bc/a2 + 1
Using the notation

1 1 1
x=p , y=p , z=p ,
1 + 3bc/a2 +1 1 + 3ca/b2 +1 1 + 3ab/c 2 + 1
324 Vasile Cîrtoaje

implies

bc 1 − 2x ca 1 − 2 y ab 1 − 2z 1
2
= , = , = , 0 < x, y, z < ,
a 3x 2 b2 3 y2 c 2 3z 2 2

(1 − 2x)(1 − 2 y)(1 − 2z) = 27x 2 y 2 z 2 .


We need to prove that
x + y +z ≤1
1
for 0 < x, y, z < such that (1 − 2x)(1 − 2 y)(1 − 2z) = 27x 2 y 2 z 2 . To do it, we will
2
use the contradiction method. Thus, assume that

1
x + y + z > 1, 0 < x, y, z < ,
2
and show that
(1 − 2x)(1 − 2 y)(1 − 2z) < 27x 2 y 2 z 2 .
We have

(1 − 2x)(1 − 2 y)(1 − 2z) < (x + y + z − 2x)(x + y + z − 2 y)(x + y + z − 2z)


< ( y + z − x)(z + x − y)(x + y − z)(x + y + z)3
≤ 3( y + z − x)(z + x − y)(x + y − z)(x + y + z)(x 2 + y 2 + z 2 )
= 3(2x 2 y 2 + 2 y 2 z 2 + 2z 2 x 2 − x 4 − y 4 − z 4 )(x 2 + y 2 + z 2 ).

Therefore, it suffices to show that

(2x 2 y 2 + 2 y 2 z 2 + 2z 2 x 2 − x 4 − y 4 − z 4 )(x 2 + y 2 + z 2 ) ≤ 9x 2 y 2 z 2 ,

which is equivalent to
X
x 6 + y 6 + z 5 + 3x 2 y 2 z 2 ≥ y 2 z 2 ( y 2 + z 2 ).

Clearly, this is just Schur’s inequality of degree three applied to x 2 , y 2 , z 2 . So, the
proof is completed. The equality holds for a = b = c, and also for a = 0 or b = 0
or c = 0.

P 2.33. Let a, b, c be nonnegative real numbers, no two of which are zero. Prove that

a b c
p +p +p ≤ 1.
4a + 5bc
2 4b + 5ca
2 4c + 5ab
2

(Vasile Cîrtoaje, 2004)


Symmetric Nonrational Inequalities 325

First Solution (by Vo Quoc Ba Can). If one of a, b, c is zero, then the desired in-
equality is an equality. Consider next that a, b, c > 0 and denote

1
 ‹
a b c
x=p , y=p , z=p , x, y, z ∈ 0, .
4a2 + 5bc 4b2 + 5ca 4c 2 + 5ab 2

We have
bc 1 − 4x 2 ca 1 − 4 y 2 ab 1 − 4z 2
= , = , = ,
a2 5x 2 b2 5 y2 c2 5z 2
and
(1 − 4x 2 )(1 − 4 y 2 )(1 − 4z 2 ) = 125x 2 y 2 z 2 .
We use the contradiction method. For the sake of contradiction, assume that x +
y + z > 1. Using the AM-GM inequality and the Cauchy-Schwarz inequality, we
have
1 Y 1 Y
x 2 y 2z2 = (1 − 4x 2 ) < [(x + y + z)2 − 4x 2 ]
125 125
1 Y Y
= (3x + y + z) · ( y + z − x)
125
 x + y + z 3 Y
≤ ( y + z − x)
3
1 Y
≤ (x 2 + y 2 + z 2 )(x + y + z) ( y + z − x)
9
1
= (x 2 + y 2 + z 2 )[2(x 2 y 2 + y 2 z 2 + z 2 x 2 ) − x 4 − y 4 − z 4 ],
9
hence

9x 2 y 2 z 2 < (x 2 + y 2 + z 2 )[2(x 2 y 2 + y 2 z 2 + z 2 x 2 ) − x 4 − y 4 − z 4 ],
X
x 6 + y 6 + z 6 + 3x 2 y 2 z 2 < x 2 y 2 (x 2 + y 2 ).
The last inequality contradicts Schur’s inequality
X
x 6 + y 6 + z 6 + 3x 2 y 2 z 2 ≥ x 2 y 2 (x 2 + y 2 ).

Thus, the proof is completed. The equality holds for a = b = c, and also for a = 0
or b = 0 or c = 0.
Second Solution. Use the mixing variables method. In the nontrivial case when
bc ca ab
a, b, c > 0, setting x = 2 , y = 2 and z = 2 (that implies x yz = 1), the desired
a b c
inequality becomes E(x, y, z) ≤ 1, where

1 1 1
E(x, y, z) = p +p +p .
4 + 5x 4 + 5y 4 + 5z
326 Vasile Cîrtoaje

Without loss of generality, we may assume that

x ≥ y ≥ z, x ≥ 1, yz ≤ 1.

We will prove that


p p
E(x, y, z) ≤ E(x, yz, yz) ≤ 1.
The left inequality has the form

1 1 1
+p ≤p p .
4 + 5y 4 + 5z 4 + 5 yz
p

For the nontrivial case y 6= z, consider y > z and denote


y +z p
s= , p = yz,
2
Æ
q = (4 + 5 y)(4 + 5z).
We have s > p, p ≤ 1 and
p p
q = 16 + 40s + 25p2 > 16 + 40p + 25p2 = 4 + 5p.

By squaring, the desired inequality becomes in succession as follows:

1 1 2 4
+ + ≤ ,
4 + 5 y 4 + 5z q 4 + 5p
1 1 2 2 2
+ − ≤ − ,
4 + 5 y 4 + 5z 4 + 5p 4 + 5p q
8 + 10s 2 2(q − 4 − 5p)
− ≤ ,
q 2 4 + 5p q(4 + 5p)
(s − p)(5p − 4) 8(s − p)
≤ ,
q (4 + 5p)
2 q(4 + 5p)(q + 4 + 5p)
5p − 4 8
≤ ,
q q + 4 + 5p
25p2 − 16 ≤ (12 − 5p)q.
The last inequality is true since

(12 − 5p)q − 25p2 + 16 > (12 − 5p)(4 + 5p) − 25p2 + 16


= 2(8 − 5p)(4 + 5p) > 0.

In order to prove the right inequality, namely

1 2
p +p p ≤ 1,
4 + 5x 4 + 5 yz
Symmetric Nonrational Inequalities 327

let us denote Æ p
4 + 5 yz = 3t, t ∈ (2/3, 1].
Since
1 25
x= = ,
yz (9t − 4)2
2

the inequality becomes

9t 2 − 4 2
p + ≤ 1,
3 36t 4 − 32t 2 + 21 3t
€p Š
(2 − 3t) 36t 4 − 32t 2 + 21 − 3t 2 − 2t ≤ 0.
Since 2 − 3t < 0, we still have to show that
p
36t 4 − 32t 2 + 21 ≥ 3t 2 + 2t.

Indeed, we have

36t 4 − 32t 2 + 21 − (3t 2 + 2t)2 = 3(t − 1)2 (9t 2 + 14t + 7) ≥ 0.

P 2.34. Let a, b, c be nonnegative real numbers. Prove that


p p p
a 4a2 + 5bc + b 4b2 + 5ca + c 4c 2 + 5ab ≥ (a + b + c)2 .

(Vasile Cîrtoaje, 2004)

First Solution. Write the inequality as


X €p Š
a 4a2 + 5bc − 2a ≥ 2(ab + bc + ca) − a2 − b2 − c 2 ,
X 1
5abc p ≥ 2(ab + bc + ca) − a2 − b2 − c 2 .
4a + 5bc + 2a
2

Writing Schur’s inequality


X
a3 + b3 + c 3 + 3abc ≥ ab(a2 + b2 )

in the form
9abc
≥ 2(ab + bc + ca) − a2 − b2 − c 2 ,
a+b+c
it suffices to prove that
X 5 9
p ≥ .
4a2 + 5bc + 2a a+b+c
328 Vasile Cîrtoaje

Let p = a + b + c and q = ab + bc + ca. By the AM-GM inequality, we have


p
p 2 (16a2 + 20bc)(3b + 3c)2 (16a2 + 20bc) + (3b + 3c)2
4a + 5bc =
2 ≤
12(b + c) 12(b + c)
16a2 + 16bc + 10(b + c)2 8a2 + 5b2 + 5c 2 + 18bc
≤ = ,
12(b + c) 6(b + c)
hence X 5 X 5
p ≥
4a2 + 5bc + 2a 8a + 5b + 5c 2 + 18bc
2 2
+ 2a
6(b + c)
X 30(b + c) X 30(b + c)
= = .
8a2 + 5b2 + 5c 2 + 12ab + 18bc + 12ac 5p2 + 2q + 3a2 + 6bc
Thus, it suffices to show that
X 30(b + c) 9
≥ .
5p2 + 2q + 3a + 6bc
2 p
By the Cauchy-Schwarz inequality, we get
P 2
X 30(b + c) 30 (b + c)
≥P
5p2 + 2q + 3a2 + 6bc (b + c)(5p2 + 2q + 3a2 + 6bc)

120p2 120p2
= = .
10p3 + 4pq + 9 bc(b + c) 10p3 + 13pq − 27abc
P

Therefore, it is enough to show that


120p2 9
≥ ,
10p + 13pq − 27abc
3 p
which is equivalent to
10p3 + 81abc ≥ 39pq.
From Schur’s inequality p3 + 9abc ≥ 4pq and the known inequality pq ≥ 9abc, we
have
10p3 + 81abc − 39pq = 10(p3 + 9abc − 4pq) + pq − 9abc ≥ 0.
This completes the proof. The equality holds for a = b = c, and also for a = 0 and
b = c (or any cyclic permutation).
Second Solution. By the Cauchy-Schwarz inequality, we have
€X p Š X a
‹
a 4a + 5bc
2 p ≥ (a + b + c)2 .
4a + 5bc
2

From this inequality and the inequality in P 2.33, namely


X a
p ≤ 1,
4a2 + 5bc
Symmetric Nonrational Inequalities 329

the desired inequality follows.

Remark. Using the same way as in the second solution, we can prove the following
inequalities for a, b, c > 0 satisfying abc = 1:
p p p
a 4a2 + 5 + b 4b2 + 5 + c 4c 2 + 5 ≥ (a + b + c)2 ;
p p p
4a4 + 5 + 4b4 + 5 + 4c 4 + 5 ≥ (a + b + c)2 .
The first inequality is a consequence of the the Cauchy-Schwarz inequality
€X p Š X a
‹
a 4a + 5
2 p ≥ (a + b + c)2
4a2 + 5
and the inequality X a
p ≤ 1, abc = 1,
4a2 + 5
which follows from the inequality in P 2.33 by replacing bc/a2 , ca/b2 , ab/c 2 with
1/a2 , 1/b2 , 1/c 2 , respectively.
The second inequality is a consequence of the the Cauchy-Schwarz inequality
Š X 2 ‹
€X p a
4a4 + 5 p ≥ (a + b + c)2
4a + 5
4

and the inequality


X a2
p ≤ 1, abc = 1,
4a4 + 5
which follows from the inequality in P 2.33 by replacing bc/a2 , ca/b2 , ab/c 2 with
1/a4 , 1/b4 , 1/c 4 , respectively.

P 2.35. Let a, b, c be nonnegative real numbers. Prove that


p p p
a a2 + 3bc + b b2 + 3ca + c c 2 + 3ab ≥ 2(ab + bc + ca).

(Vasile Cîrtoaje, 2005)

First Solution (by Vo Quoc Ba Can). Using the AM-GM inequality yields
X p X a(b + c)(a2 + 3bc)
a a2 + 3bc = p
(b + c)2 (a2 + 3bc)
X 2a(b + c)(a2 + 3bc)
≥ .
(b + c)2 + (a2 + 3bc)
330 Vasile Cîrtoaje

Thus, it suffices to prove that


X 2a(b + c)(a2 + 3bc) X
≥ a(b + c).
a2 + b2 + c 2 + 5bc
We will use the SOS method. Write the inequality as follows:
X a(b + c)(a2 − b2 − c 2 + bc)
≥ 0,
a2 + b2 + c 2 + 5bc
X a3 (b + c) − a(b3 + c 3 )
≥ 0,
a2 + b2 + c 2 + 5bc
X ab(a2 − b2 ) − ac(c 2 − a2 )
≥ 0,
a2 + b2 + c 2 + 5bc
X ab(a2 − b2 ) X ba(a2 − b2 )
− ≥ 0,
a2 + b2 + c 2 + 5bc b2 + c 2 + a2 + 5ca
X 5abc(a + b)(a − b)2
≥ 0.
(a2 + b2 + c 2 + 5bc)(a2 + b2 + c 2 + 5ac)
The equality holds a = b = c, and also for a = 0 and b = c (or any cyclic permuta-
tion).
Second Solution. Write the inequality as
X€ p Š
a a2 + 3bc − a2 ≥ 2(ab + bc + ca) − a2 − b2 − c 2 .

Due to homogeneity, we may assume that a + b + c = 3. By the AM-GM inequality,


we have
p 3abc 12abc
a a2 + 3bc − a2 = p = p
a2 + 3bc + a 2 4(a2 + 3bc) + 4a
12abc
≥ .
4 + a2 + 3bc + 4a
Thus, it suffices to show that
X 1
12abc ≥ 2(ab + bc + ca) − a2 − b2 − c 2 .
4 + a + 3bc + 4a
2

On the other hand, by Schur’s inequality of degree three, we have

9abc
≥ 2(ab + bc + ca) − a2 − b2 − c 2 .
a+b+c
Therefore, it is enough to prove that
X 1 3
≥ .
4 + a2 + 3bc + 4a 4(a + b + c)
Symmetric Nonrational Inequalities 331

By the Cauchy-Schwarz inequality, we have


X 1 9 9
≥P =
4+ + 3bc + 4a (4 + a + 3bc + 4a) 24 + a + 3 ab
P P
a2 2 2

27
= P 2
a + 3 a2 + 9 ab
P P
8
P
9 a 3
= P 2 ≥ P .
a + 3 ab 4 a
P
11

P 2.36. Let a, b, c be nonnegative real numbers. Prove that


p p p
a a2 + 8bc + b b2 + 8ca + c c 2 + 8ab ≤ (a + b + c)2 .

Solution. Multiplying by a + b + c, the inequality becomes


X Æ
a (a + b + c)2 (a2 + 8bc) ≤ (a + b + c)3 .

Since Æ
2 (a + b + c)2 (a2 + 8bc) ≤ (a + b + c)2 + (a2 + 8bc),
it suffices to show that
X
a[(a + b + c)2 + (a2 + 8bc)] ≤ 2(a + b + c)3 ,

which can be written as

a3 + b3 + c 3 + 24abc ≤ (a + b + c)3 .

This inequality is equivalent to

a(b − c)2 + b(c − a)2 + c(a − b)2 ≥ 0.

The equality holds for a = b = c, and also for b = c = 0 (or any cyclic permutation).

P 2.37. Let a, b, c be nonnegative real numbers, no two of which are zero. Prove that

a2 + 2bc b2 + 2ca c 2 + 2ab p


p +p +p ≥ 3 ab + bc + ca.
b2 + bc + c 2 c 2 + ca + a2 a2 + ab + b2
(Michael Rozenberg and Marius Stanean, 2011)
332 Vasile Cîrtoaje

Solution. By the AM-GM inequality, we have


X a2 + 2bc X 2(a2 + 2bc)pab + bc + ca
p =
b2 + bc + c 2
p
2 (b2 + bc + c 2 )(ab + bc + ca)
p X 2(a2 + 2bc)
≥ ab + bc + ca
(b2 + bc + c 2 ) + (ab + bc + ca)
p X 2(a2 + 2bc)
= ab + bc + ca .
(b + c)(a + b + c)
Thus, it suffices to show that
a2 + 2bc b2 + 2ca c 2 + 2ab 3
+ + ≥ (a + b + c).
b+c c+a a+b 2
This inequality is equivalent to
1X
a + b + c + abc(a + b + c) ≥
4 4 4
ab(a + b)2 .
2
We can prove this inequality by summing Schur’s inequality of fourth degree
X
a4 + b4 + c 4 + abc(a + b + c) ≥ ab(a2 + b2 )

and the obvious inequality


X 1X
ab(a2 + b2 ) ≥ ab(a + b)2 .
2
The equality holds for a = b = c.

P 2.38. Let a, b, c be nonnegative real numbers, no two of which are zero. If k ≥ 1,


then
a k+1 b k+1 c k+1 ak + bk + c k
+ + ≤ .
2a2 + bc 2b2 + ca 2c 2 + ab a+b+c
(Vasile Cîrtoaje and Vo Quoc Ba Can, 2011)
Solution. Write the inequality as follows:
X ak a k+1

− ≥ 0,
a + b + c 2a2 + bc
X a k (a − b)(a − c)
≥ 0.
2a2 + bc
Assume that a ≥ b ≥ c. Since (c − a)(c − b) ≥ 0, it suffices to show that

a k (a − b)(a − c) b k (b − a)(b − c)
+ ≥ 0.
2a2 + bc 2b2 + ca
Symmetric Nonrational Inequalities 333

This is true if
a k (a − c) b k (b − c)
− ≥ 0,
2a2 + bc 2b2 + ca
which is equivalent to

a k (a − c)(2b2 + ca) ≥ b k (b − c)(2a2 + bc).

Since a k /b k ≥ a/b, it remains to show that

a(a − c)(2b2 + ca) ≥ b(b − c)(2a2 + bc),

which is equivalent to the obvious inequality

(a − b)c[a2 + 3ab + b2 − c(a + b)] ≥ 0.

The equality holds for a = b = c, and also for a = b and c = 0 (or any cyclic
permutation).

P 2.39. If a, b, c are positive real numbers, then


a2 − bc b2 − ca c 2 − ab
(a) p + p + p ≥ 0;
3a2 + 2bc 3b2 + 2ca 3c 2 + 2ab
a2 − bc b2 − ca c 2 − ab
(b) p +p +p ≥ 0.
8a2 + (b + c)2 8b2 + (c + a)2 8c 2 + (a + b)2
(Vasile Cîrtoaje, 2006)

Solution. (a) Use the SOS technique. Let


p p p
A = 3a2 + 2bc, B = 3b2 + 2ca, C= 3c 2 + 2ab.

We have
X a2 − bc X (a − b)(a + c) + (a − c)(a + b)
2 =
A A
X (a − b)(a + c) X (b − a)(b + c)
= +
A B
a+c b+c
X ‹
= (a − b) −
A B
X a − b (a + c)2 B 2 − (b + c)2 A2
= · ,
AB (a + c)B + (b + c)A

hence
X a2 − bc X c(a − b)2 2(a − b)2 + c(a + b + 2c)
2 = · ≥ 0.
A AB (a + c)B + (b + c)A
334 Vasile Cîrtoaje

The equality holds for a = b = c.


(b) Let
Æ Æ Æ
A = 8a2 + (b + c)2 , B= 8b2 + (c + a)2 , C= 8c 2 + (a + b)2 b.

As we have shown before,


X a2 − bc X a − b (a + c)2 B 2 − (b + c)2 A2
2 = · ,
A AB (a + c)B + (b + c)A

hence
X a2 − bc X (a − b)2 C1
2 = · ≥ 0,
A AB (a + c)B + (b + c)A
since

C1 = [(a + c) + (b + c)][(a + c)2 + (b + c)2 ] − 8ac(b + c) − 8bc(a + c)


≥ [(a + c) + (b + c)](4ac + 4bc) − 8ac(b + c) − 8bc(a + c)
= 4c(a − b)2 ≥ 0.

The equality holds for a = b = c.

p
P 2.40. Let a, b, c be positive real numbers. If 0 ≤ k ≤ 1 + 2 2, then

a2 − bc b2 − ca c 2 − ab
p +p +p ≥ 0.
ka2 + b2 + c 2 kb2 + c 2 + a2 kc 2 + a2 + b2

Solution. Use the SOS method. Let


p p p
A = ka2 + b2 + c 2 , B = kb2 + c 2 + a2 , C= kc 2 + a2 + b2 .

As we have shown at the preceding problem,


X a2 − bc X a − b (a + c)2 B 2 − (b + c)2 A2
2 = · ;
A AB (a + c)B + (b + c)A

therefore
X a2 − bc X (a − b)2 C1
2 = · ,
A AB (a + c)B + (b + c)A
where
C1 = (a2 + b2 + c 2 )(a + b + 2c) − (k − 1)c(2ab + bc + ca).
Symmetric Nonrational Inequalities 335

It suffices to show that C1 ≥ 0. Putting a + b = 2x, we have a2 + b2 ≥ 2x 2 , ab ≤ x 2 ,


hence
p
C1 ≥ (a2 + b2 + c 2 )(a + b + 2c) − 2 2 c(2ab + bc + ca)
p
≥ (2x 2 + c 2 )(2x + 2c) − 2 2 c(2x 2 + 2c x)
p
= 2(x + c)(x 2 − c)2 ≥ 0.
The equality holds for a = b = c.

P 2.41. If a, b, c are nonnegative real numbers, then


p p p
(a2 − bc) b + c + (b2 − ca) c + a + (c 2 − ab) a + b ≥ 0.

First Solution. Let us denote


v v
tb+c s
c+a ta + b
x= , y= , z= ,
2 2 2
hence
a = y 2 + z2 − x 2, b = z2 + x 2 − y 2, c = x 2 + y 2 − z2.
The inequality turns into
x y(x 3 + y 3 ) + yz( y 3 + z 3 ) + z x(z 3 + x 3 ) ≥ x 2 y 2 (x + y) + y 2 z 2 ( y + z) + z 2 x 2 (z + x),
which is equivalent to the obvious inequality
x y(x + y)(x − y)2 + yz( y + z)( y − z)2 + z x(z + x)(z − x)2 ≥ 0.
The equality holds for a = b = c, and also for b = c = 0 (or any cyclic permutation).

Second Solution. Use the SOS technique. Write the inequality as


A(a2 − bc) + B(b2 − ca) + C(c 2 − ab) ≥ 0,
where p p p
A= b + c, B= c + a, C= a + b.
We have
X X
2 A(a2 − bc) = A[(a − b)(a + c) + (a − c)(a + b)]
X X
= A(a − b)(a + c) + B(b − a)(b + c)
X
= (a − b)[A(a + c) − B(b + c)]
X A2 (a + c)2 − B 2 (b + c)2
= (a − b) ·
A(a + c) + B(b + c)
X (a − b)2 (a + c)(b + c)
= ≥ 0.
A(a + c) + B(b + c)
336 Vasile Cîrtoaje

P 2.42. If a, b, c are nonnegative real numbers, then


p p p
(a2 − bc) a2 + 4bc + (b2 − ca) b2 + 4ca + (c 2 − ab) c 2 + 4ab ≥ 0.

(Vasile Cîrtoaje, 2005)

Solution. If two of a, b, c are zero, then the inequality is clearly true. Otherwise,
write the inequality as
AX + BY + C Z ≥ 0,
where p p p
a2 + 4bc b2 + 4ca c 2 + 4ab
A= , B= , C= ,
b+c c+a a+b
X = (a2 − bc)(b + c), Y = (b2 − bc)(b + c), Z = (c 2 − ab)(a + b).
Without loss of generality, assume that

a ≥ b ≥ c.

We have
X ≥ 0, Z ≤ 0, X + Y + Z = 0.
In addition,
X − Y = ab(a − b) + 2(a2 − b2 )c + (a − b)c 2 ≥ 0
and

a4 − b4 + 2(a3 − c 3 )c + (a2 − c 2 )c 2 + 4abc(a − b) − 4(a − b)c 3


A2 − B 2 =
(b + c)2 (c + a)2

4abc(a − b) − 4(a − b)c 3 4c(a − b)(ab − c 2 )


≥ = ≥ 0.
(b + c)2 (c + a)2 (b + c)2 (c + a)2
Since

2(AX + BY + C Z) = (A − B)(X − Y ) + (A + B)(X + Y ) + 2C Z


= (A − B)(X − Y ) − (A + B − 2C)Z,

it suffices to show that


A + B − 2C ≥ 0.
This is true if AB ≥ C 2 . Using the Cauchy-Schwarz inequality gives
p p
ab + 4c ab ab + 2c ab + 2c 2
AB ≥ ≥ .
(b + c)(c + a) (b + c)(c + a)

Thus, it is enough to show that


p
(a + b)2 (ab + 2c ab + 2c 2 ) ≥ (b + c)(c + a)(c 2 + 4ab).
Symmetric Nonrational Inequalities 337

Write this inequality as


p €p p Š2
a b(a − b) + 2c
2
ab(a + b) a − b + c 2 [2(a + b)2 − 5ab − c(a + b) − c 2 ] ≥ 0.

It is true since

2(a + b)2 − 5ab − c(a + b) − c 2 = a(2a − b − c) + b(b − c) + b2 − c 2 ≥ 0.

The equality holds for a = b = c, and also for a = b and c = 0 (or any cyclic
permutation).

P 2.43. If a, b, c are nonnegative real numbers, then


v v v
t a3 t b3 t c3
+ + ≥ 1.
a3 + (b + c)3 b3 + (c + a)3 c 3 + (a + b)3

Solution. For a = 0, the inequality reduces to the obvious inequality


p p p
b3 + c 3 ≥ b3 + c 3 .

For a, b, c > 0, write the inequality as


v
X u 1
‹ ≥ 1.
u
b+c 3
u 
1+
t
a

For any x ≥ 0, we have


p Æ (1 + x) + (1 − x + x 2 ) 1
1 + x3 = (1 + x)(1 − x + x 2 ) ≤ = 1 + x 2.
2 2
Therefore, we get
v
X u 1 X 1
‹3 ≥
u
b+c 1 b+c 2
u   ‹
1+ 1+
t
a 2 a
X1 X a2
≥ = = 1.
b2 + c 2 a2 + b2 + c 2
1+
a2
The equality holds for a = b = c, and also for b = c = 0 (or any cyclic permutation).
338 Vasile Cîrtoaje

P 2.44. If a, b, c are positive real numbers, then

v v v
u
1 1 1 1 1 1
t  ‹ t  ‹
(a + b + c) + + ≥ 1 + 1 + (a + b + c ) 2 + 2 + 2 .
t
2 2 2
a b c a b c

(Vasile Cîrtoaje, 2002)

Solution. Using the Cauchy-Schwarz inequality, we have


v
€X Š X 1 ‹ t€X X Š X 1 X 1‹
a = a +2
2 bc +2
a a2 bc
v v
t€X Š X 1
 ‹ t€X Š X 1 ‹

≥ a 2 +2 bc
a2 bc
v v
t€X Š X 1
 ‹ t€X Š X 1 ‹

= a 2 +2 a ,
a2 a

hence
–v ™2 v
t€X Š X 1 ‹ t€X Š X 1 ‹
a −1 ≥1+ a2 ,
a a2

v v v
t€X Š X 1 ‹ u t€X Š X 1 ‹
−1≥ 1+
t
a a2 .
a a2
The equality holds if and only if
€X Š X 1 ‹ X 1 ‹ €X Š
2
a = bc ,
bc a2

which is equivalent to

(a2 − bc)(b2 − ca)(c 2 − ab) = 0.

Consequently, the equality occurs for a2 = bc or b2 = ca or c 2 = ab.

P 2.45. If a, b, c are positive real numbers, then


v
1 1 1 1 1 1
t  ‹  ‹
5+ 2(a2 + b2 + c 2 ) 2 + 2 + 2 − 2 ≥ (a + b + c) + + .
a b c a b c

(Vasile Cîrtoaje, 2004)


Symmetric Nonrational Inequalities 339

Solution. Let us denote


a b c b c a
x= + + , y= + + .
b c a a b c
From
1 1 1
 ‹
2(a + b + c ) 2 + 2 + 2 − 2 =
2 2 2
a b c
 2 2 2‹  2
c 2 a2
‹
a b c b
=2 2 + 2 + 2 +2 2 + 2 + 2 +4
b c a a b c
= 2(x − 2 y) + 2( y − 2x) + 4
2 2

= (x + y − 2)2 + (x − y)2
≥ (x + y − 2)2

and
1 1 1
 ‹
(a + b + c) + + = x + y + 3,
a b c
we get
v
1 1 1
t  ‹
2(a2 + b2 + c 2 ) 2 + 2 + 2 − 2 ≥ x + y − 2
a b c
1 1 1
 ‹
= (a + b + c) + + − 5.
a b c

The equality occurs for a = b or b = c or c = a.

P 2.46. If a, b, c are real numbers, then


Æ
2(1 + abc) + 2(1 + a2 )(1 + b2 )(1 + c 2 ) ≥ (1 + a)(1 + b)(1 + c).

(Wolfgang Berndt, 2006)

First Solution. Denoting

p = a + b + c, q = ab + bc + ca, r = abc,

the inequality becomes


Æ
2(p2 + q2 + r 2 − 2pr − 2q + 1) ≥ p + q − r − 1.

It suffices to show that

2(p2 + q2 + r 2 − 2pr − 2q + 1) ≥ (p + q − r − 1)2 ,


340 Vasile Cîrtoaje

which is equivalent to

p2 + q2 + r 2 − 2pq + 2qr − 2pr + 2p − 2q − 2r + 1 ≥ 0,

(p − q − r + 1)2 ≥ 0.
The equality holds for p + 1 = q + r and q ≥ 1. The last condition follows from
p + q − r − 1 ≥ 0.
Second Solution. Since

2(1 + a2 ) = (1 + a)2 + (1 − a)2

and
(1 + b2 )(1 + c 2 ) = (b + c)2 + (bc − 1)2 ,
by the Cauchy-Schwarz inequality, we get
Æ
2(1 + a2 )(1 + b2 )(1 + c 2 ) ≥ (1 + a)(b + c) + (1 − a)(bc − 1)
= (1 + a)(1 + b)(1 + c) − 2(1 + abc).

P 2.47. Let a, b, c be nonnegative real numbers, no two of which are zero. Prove that
v v v
t a2 + bc t b2 + ca t c 2 + ab 1
+ + ≥ 2 + p .
b2 + c 2 c 2 + a2 a2 + b2 2
(Vo Quoc Ba Can, 2006)

Solution. Assume that


a ≥ b ≥ c.
It suffices to show that
v v v
t a2 + c 2 t b2 + c 2 t ab 1
+ + ≥2+ p .
b +c
2 2 c +a
2 2 a +b
2 2
2
Let us denote v
t a2 + c 2 s
a
x= , y= .
b2 + c2 b
From
(a − b)(ab − c 2 )
x2 − y2 = ≥ 0,
b(b2 + c 2 )
it follows that
x ≥ y ≥ 1.
Symmetric Nonrational Inequalities 341

Also, from
1 1 (x − y)(x y − 1)
 ‹
x+ − y+ = ≥ 0,
x y xy
we have v v v
t a2 + c 2 t b2 + c 2 s
a tb
+ ≥ + .
b2 + c 2 c 2 + a2 b a
Therefore, it is enough to show that
s v v
a t b t ab 1
+ + ≥2+ p ,
b a a +b
2 2
2
which is equivalent to
s v v
a tb 1 t ab
+ −2≥ p − ,
b a 2 a2 + b2
p p
( a − b)2 (a − b)2
p ≥p p p .
ab 2(a2 + v 2 a2 + b2 + 2ab
p p
Since 2 ab ≤ 2(a2 + b2 ), it suffices to show that
p p
( a + b )2
2≥ p p .
a2 + b2 + 2ab
Indeed,
€p p Š Æ p p €p p Š2
2 a2 + b2 + 2ab > 2(a2 + b2 ) + 2 ab ≥ a + b + 2 ab = a+ b .

The equality holds for a = b and c = 0 (or any cyclic permutation).

P 2.48. If a, b, c are nonnegative real numbers, then


Æ Æ Æ Æ
a(2a + b + c) + b(2b + c + a) + c(2c + a + b) ≥ 12(ab + bc + ca).

(Vasile Cîrtoaje, 2012)

Solution. By squaring, the inequality becomes



a2 + b2 + c 2 + bc(2b + c + a)(2c + a + b) ≥ 5(ab + bc + ca).

Using the Cauchy-Schwarz inequality yields


XÆ XÆ
bc(2b + c + a)(2c + a + b) = (2b2 + bc + ab)(2c 2 + bc + ac)
342 Vasile Cîrtoaje
X€ p Š X p
≥ 2bc + bc + a bc = 3(ab + bc + ca) + a bc.
Therefore, it suffices to show that
X p
a2 + b2 + c 2 + a bc ≥ 2(ab + bc + ca).

We can get this inequality by summing Schur’s inequality


X p Xp
a2 + b2 + c 2 + a bc ≥ ab(a + b)

and Xp
ab (a + b) ≥ 2(ab + bc + ca).
The last inequality is equivalent to the obvious inequality
Xp €p p Š2
ab a− b ≥ 0.

The equality holds for a = b = c, and also for a = 0 and b = c (or any cyclic
permutation).

P 2.49. Let a, b, c be nonnegative real numbers such that a + b + c = 3. Prove that


Æ Æ Æ
a (4a + 5b)(4a + 5c) + b (4b + 5c)(4b + 5a) + c (4c + 5a)(4c + 5b) ≥ 27.

(Vasile Cîrtoaje, 2010)

Solution. Use the SOS technique. Assume that

a ≥ b ≥ c,

consider the nontrivial case b > 0, and write the inequality in the following equiv-
alent homogeneous forms:
X Æ
a (4a + 5b)(4a + 5c) ≥ 3(a + b + c)2 ,
€X X Š X €p p Š2
2 a2 − ab ≥ a 4a + 5b − 4a + 5c ,
X X 25a(b − c)2
(b − c)2 ≥ p p 2 ,
4a + 5b + 4a + 5c
X
(b − c)2 Sa ≥ 0,
where
25a
Sa = 1 − p p 2 .
4a + 5b + 4a + 5c
Symmetric Nonrational Inequalities 343

Since
25b 25b
Sb = 1 − p p 2 ≥ 1 − p p 2 = 0
4b + 5c + 4b + 5a 4b + 9b
and
25c 25c 25
Sc = 1 − p p 2 ≥ 1 − p p 2 = 1 − 36 > 0,
4c + 5a + 4c + 5b 9c + 9c

we have
X a2
(b − c)2 Sa ≥ (b − c)2 Sa + (a − c)2 S b ≥ (b − c)2 Sa + 2 (b − c)2 S b
b
 ‹
a b a
= (b − c)2 Sa + S b .
b a b
Thus, it suffices to prove that
b a
Sa + S b ≥ 0.
a b
We have
p p 2
25a a 4a + 5b − 4a
Sa ≥ 1 − p p 2 = 1 − b2
,
4a + 5b + 4a
p p 2
25b b 4b + 5a − 4b
Sb ≥ 1 − p p 2 = 1 − ,
4b + 4b + 5a a2
hence
p p 2 p p 2
b a b 4a + 5b − 4a a 4b + 5a − 4b
Sa + S b ≥ − + −
a b a b v b a
‚v Œ
t 4a2 5a t 4b2 5b 
a b
‹
=4 + + + −7 + − 10
b2 b a2 a b a
q p
= 4 4x 2 + 5x − 8 + 2 20x + 41 − 7x − 10,

where
a b
x=
+ ≥ 2.
b a
To end the proof, we only need to show that x ≥ 2 yields
q p
4 4x 2 + 5x − 8 + 2 20x + 41 ≥ 7x + 10.

By squaring, this inequality becomes


p
15x 2 − 60x − 228 + 32 20x + 41 ≥ 0.
344 Vasile Cîrtoaje

Indeed,
p p
15x 2 − 60x − 228 + 32 20x + 41 ≥ 15x 2 − 60x − 228 + 32 81 = 15(x − 2)2 ≥ 0.

3
The equality holds for a = b = c = 1, and also for a = b = and c = 0 (or any
2
cyclic permutation).

P 2.50. Let a, b, c be nonnegative real numbers such that ab + bc + ca = 3. Prove


that
Æ Æ Æ
a (a + 3b)(a + 3c) + b (b + 3c)(b + 3a) + c (c + 3a)(c + 3b) ≥ 12.

(Vasile Cîrtoaje, 2010)

Solution. Use the SOS method. Assume that a ≥ b ≥ c (b > 0), and write the
inequality as X Æ
a (a + 3b)(a + 3c) ≥ 4(ab + bc + ca),
X X X €p p Š2
2( 2
a − ab) = a a + 3b − a + 3c ,
X X 9a(b − c)2
(b − c) ≥
2
p p 2 ,
a + 3b + a + 3c
X
(b − c)2 Sa ≥ 0,
where
9a
Sa = 1 − p p 2 .
a + 3b + a + 3c
Since
9b 9b
Sb = 1 − p p 2 ≥ 1 − p p 2 = 0
b + 3c + b + 3a b + 4b
and
9c 9c 9
Sc = 1 − p p 2 ≥ 1 − p p 2 = 1 − 16 > 0,
c + 3a + c + 3b 4c + 4c

we have
X a2
(b − c)2 Sa ≥ (b − c)2 Sa + (a − c)2 S b ≥ (b − c)2 Sa + 2 (b − c)2 S b
b
 ‹
a b a
= (b − c)2 Sa + S b .
b a b
Symmetric Nonrational Inequalities 345

Thus, it suffices to prove that

b a
Sa + S b ≥ 0.
a b
We have p p 2
9a a a + 3b − a
Sa ≥ 1 − p p 2 = 1 − b2
,
a + 3b + a
p p 2
9b b b + 3a − b
Sb ≥ 1 − p p 2 = 1 − ,
b + b + 3a a2

hence
p p 2 p p 2
b a b a + 3b − a a b + 3a − b
Sa + S b ≥ − + −
a b a b v b a
‚v Œ 
t a2 3a t b2 3b a b
‹
=2 + + + − + −6
b2 b a2 a b a
Æ p
= 2 x 2 + 3x − 2 + 2 3x + 10 − x − 6,

where
a b
x=
+ ≥ 2.
b a
To end the proof, it remains to show that
Æ p
2 x 2 + 35x − 2 + 2 3x + 10 ≥ x + 6

for x ≥ 2. By squaring, this inequality becomes


p
3x 2 − 44 + 8 3x + 10 ≥ 0.

Indeed, p
3x 2 − 44 + 8 3x + 10 ≥ 12 − 44 + 32 = 0.
p
The equality holds for a = b = c = 1, and also for a = b = 3 and c = 0 (or any
cyclic permutation).

P 2.51. Let a, b, c be nonnegative real numbers such that a2 + b2 + c 2 = 3. Prove that


p p p Æ
2 + 7ab + 2 + 7bc + 2 + 7ca ≥ 3 3(ab + bc + ca).

(Vasile Cîrtoaje, 2010)


346 Vasile Cîrtoaje

Solution. Use the SOS method. Consider a ≥ b ≥ c. Since the inequality is trivial
for b = c = 0, we may assume that b > 0. By squaring, the desired inequality
becomes XÆ
6+2 (2 + 7ab)(2 + 7ac) ≥ 20(ab + bc + ca),
X €p p Š2
6(a2 + b2 + c 2 − ab − bc − ca) ≥ 2 + 7ab − 2 + 7ac ,
X X 49a2 (b − c)2
3 (b − c)2 ≥ p p 2 ,
2 + 7ab + 2 + 7ac
X
(b − c)2 Sa ≥ 0,
where
49a2
Sa = 1 − p p 2 ,
6 + 21ab + 6 + 21ac
49b2
Sb = 1 − p p 2 ,
6 + 21ab + 6 + 21bc
49c 2
Sc = 1 − p p 2 .
6 + 21ac + 6 + 21bc
Since 6 ≥ 2(a2 + b2 ) ≥ 4ab, we have

49a2 49a2 a
Sa ≥ 1 − p p 2 ≥ 1 − p p 2 = 1 − ,
4ab + 21ab + 6 5 ab + 2 ab b

49b2 49b2 b
Sb ≥ 1 − p p 2 ≥ 1 − p p 2 = 1 − ,
4ab + 21ab + 6 5 ab + 2 ab a

49c 2 49c 2 49
Sc ≥ 1 − p p ≥ 1 − = 1 − > 0.
(5c + 5c)2
2
4ab + 21ac + 4ab + 21bc 100

Therefore,
X
(b − c)2 Sa ≥ (b − c)2 Sa + (c − a)2 S b
 ‹
 a b
≥ (b − c) 1 −
2
+ (c − a) 1 −
2
b a
(a − b) (ab − c )
2 2
= ≥ 0.
ab
p
The equality holds for a = b = c = 1, and also for a = b = 3 and c = 0 (or any
cyclic permutation).
Symmetric Nonrational Inequalities 347

P 2.52. Let a, b, c be nonnegative real numbers such that a2 + b2 + c 2 = 3. Prove that

a b c
+ 2 + 2 ≤ 1.
2a2 + 1 2b + 1 2c + 1
(Vasile Cîrtoaje, 2006)

Solution. Assume that a ≤ b ≤ c and denote


a b c
f (a, b, c) = + 2 + 2 .
2a2+ 1 2b + 1 2c + 1
We will show that
f (a, b, c) ≤ f (s, s, c) ≤ 1,
where v
t a2 + b2
s= , s ≤ 1.
2
The inequality f (a, b, c) ≤ f (s, s, c) follows from P 2.1. The inequality f (s, s, c) ≤ 1
is equivalent to
2s c
+ 2 ≤ 1,
2s + 1 2c + 1
2

where
2s2 + c 2 = 3, 0 ≤ s ≤ 1 ≤ c.
Write the requested inequality as follows:
1 c 2s 2
− 2 ≥ 2 − ,
3 2c + 1 2s + 1 3
(c − 1)(2c − 1) 2(1 − s)(2s − 1)
≥ ,
2c 2 + 1 2s2 + 1
(c 2 − 1)(2c − 1) 2(1 − s2 )(2s − 1)
≥ .
(c + 1)(2c 2 + 1) (1 + s)(2s2 + 1)
Since
c 2 − 1 = 2(1 − s2 ) ≥ 0,
we only need to show that
2c − 1 2s − 1
≥ ,
(c + 1)(2c 2 + 1) (s + 1)(2s2 + 1)

which is equivalent to (c − s)A ≥ 0, where

A = 2(s + c)2 + 2(s + c) + 3 − 6sc − 4sc(s + c).

Substituting
s+c p
x= , y= sc, x ≥ y,
2
348 Vasile Cîrtoaje

we need to show that A(x, y) ≥ 0, where

A(x, y) = 8x 2 + 4x + 3 − 6 y 2 − 8x y 2 .

From p p
3 = 2s2 + c 2 ≥ 2 2sc = 2 2 y 2 ,
we get v
t 3
y≤ p .
2 2
We will show that
A(x, y) ≥ A( y, y) ≥ 0.
We have

A(x, y) − A( y, y) = 4(x − y)(2x + 2 y + 1 − 2 y 2 ) ≥ 4(x − y)[2 y(2 − y) + 1] ≥ 0

and
A( y, y) = 3 + 4 y + 2 y 2 − 8 y 3 .
From
3 4 2
 ‹
A( y, y) = y
3
+ + −8 ,
y3 y2 y
v
t 3
it follows that it suffices to show that A( y, y) ≥ 0 for y = p . Indeed, we have
2 2

3 3
 ‹
A( y, y) = 3 + 2 y − 4(2 y − 1) y = 3 + p − 4 p − 1 y
2 2
2 2
p p
3 2 + 3 − 4(3 − 2) y B
= p =p p p ,
2 2[3 2 + 3 + 4(3 − 2) y]
where
p p p p p
B = (3 2 + 3)2 − 16(3 − 2)2 y 2 = 9( 2 + 1)2 − 12 2(3 − 2)2
p
= 57(3 − 2 2) > 0.
The equality holds for a = b = c = 1.
Remark. The following more general statement is also valid.
• If a, b, c, d are nonnegative real numbers such that a2 + b2 + c 2 + d 2 = 4, then

a b c d 4
+ 2 + 2 + 2 ≤ .
2a2 + 1 2b + 1 2c + 1 2d + 1 3
Symmetric Nonrational Inequalities 349

P 2.53. Let a, b, c be nonnegative real numbers such that ab + bc + ca = 3. Prove


that
Pp
(a) a(b + c)(a2 + bc) ≥ 6;
p p
a(b + c) a2 + 2bc ≥ 6 3;
P
(b)
p
a(b + c) (a + 2b)(a + 2c) ≥ 18.
P
(c)
(Vasile Cîrtoaje, 2010)

Solution. Assume that


a ≥ b ≥ c, b > 0.
(a) Write the inequality in the homogeneous form

a(b + c)(a2 + bc) ≥ 2(ab + bc + ca).

First Solution. Write the homogeneous inequality as


XÆ ”p Æ —
a(b + c) a2 + bc − a(b + c) ≥ 0,
p
X (a − b)(a − c) a(b + c)
p p ≥ 0.
a2 + bc + a(b + c)
Since (c − a)(c − b) ≥ 0, it suffices to show that
p p
(a − b)(a − c) a(b + c) (b − c)(b − a) b(c + a)
p p + p p ≥ 0.
a2 + bc + a(b + c) b2 + ca + b(c + a)
This is true if
p p
(a − c) a(b + c) (b − c) b(c + a)
p p ≥p p .
a2 + bc + a(b + c) b2 + ca + b(c + a)
Since Æ Æ
a(b + c) ≥ b(c + a),
it suffices to show that
a−c b−c
p p ≥p p .
a2 + bc + a(b + c) b2 + ca + b(c + a)
Moreover, since
p Æ p Æ
a2 + bc ≥ a(b + c), b2 + ca ≤ b(c + a),

it is enough to show that


a−c b−c
p ≥p .
a2 + bc b2 + ca
350 Vasile Cîrtoaje

Indeed, we have

(a − c)2 (b2 + ca) − (b − c)2 (a2 + bc) = (a − b)(a2 + b2 + c 2 + 3ab − 3bc − 3ca) ≥ 0,

because

a2 + b2 + c 2 + 3ab − 3bc − 3ca = (a2 − bc) + (b − c)2 + 3a(b − c) ≥ 0.


p
The equality holds for a = b = c = 1, and also for a = b = 3 and c = 0 (or any
cyclic permutation).
Second Solution. By squaring, the homogeneous inequality becomes
X XÆ
a(b+c)(a2 + bc)+2 bc(a + b)(a + c)(b2 + ca)(c 2 + ab) ≥ 4(ab+ bc+ca)2 .

Since
(b2 + ca)(c 2 + ab) − bc(a + b)(a + c) = a(b + c)(b − c)2 ≥ 0,
it suffices to show that
X X
a(b + c)(a2 + bc) + 2 bc(a + b)(a + c) ≥ 4(ab + bc + ca)2 ,

which is equivalent to X
bc(b − c)2 ≥ 0.
(b) Write the inequality as
X p p
a(b + c) a2 + 2bc ≥ 2(ab + bc + ca) ab + bc + ca,
X ”p p —
a(b + c) a2 + 2bc − ab + bc + ca ≥ 0,
X a(b + c)(a − b)(a − c)
p p ≥ 0.
a2 + 2bc + ab + bc + ca
Since (c − a)(c − b) ≥ 0, it suffices to show that

a(b + c)(a − b)(a − c) b(c + a)(b − c)(b − a)


p p +p p ≥ 0.
a2 + 2bc + ab + bc + ca b2 + 2ca + ab + bc + ca
This is true if
a(b + c)(a − c) b(c + a)(b − c)
p p ≥p p .
a2 + 2bc + ab + bc + ca b2 + 2ca + ab + bc + ca
Since
(b + c)(a − c) ≥ (c + a)(b − c),
it suffices to show that
a b
p p ≥p p .
a2 + 2bc + ab + bc + ca b2 + 2ca + ab + bc + ca
Symmetric Nonrational Inequalities 351

Moreover, since
p p p p
a2 + 2bc ≥ ab + bc + ca, b2 + 2ca ≤ ab + bc + ca,

it is enough to show that


a b
p ≥p .
a2 + 2bc b2 + 2ca
Indeed, we have

a2 (b2 + 2ca) − b2 (a2 + 2bc) = 2c(a3 − b3 ) ≥ 0.


p
The equality holds for a = b = c = 1, and also for a = b = 3 and c = 0 (or any
cyclic permutation).
(c) Write the inequality as follows:
X Æ Æ
a(b + c) (a + 2b)(a + 2c) ≥ 2(ab + bc + ca) 3(ab + bc + ca),
X ”Æ Æ —
a(b + c) (a + 2b)(a + 2c) − 3(ab + bc + ca) ≥ 0,
X a(b + c)(a − b)(a − c)
p p ≥ 0.
(a + 2b)(a + 2c) + 3(ab + bc + ca)
Since (c − a)(c − b) ≥ 0, it suffices to show that
a(b + c)(a − c) b(c + a)(b − c)
p p ≥p p .
(a + 2b)(a + 2c) + 3(ab + bc + ca) (b + 2c)(b + 2a) + 3(ab + bc + ca)
Since
(b + c)(a − c) ≥ (c + a)(b − c),
it suffices to show that
a b
p p ≥p p .
(a + 2b)(a + 2c) + 3(ab + bc + ca) (b + 2c)(b + 2a) + 3(ab + bc + ca)
Moreover, since
Æ Æ Æ Æ
(a + 2b)(a + 2c) ≥ 3(ab + bc + ca), (b + 2c)(b + 2a) ≤ 3(ab + bc + ca),

it is enough to show that


a b
p ≥p .
(a + 2b)(a + 2c) (b + 2c)(b + 2a)

This is true if p p
a b
p ≥p .
(a + 2b)(a + 2c) (b + 2c)(b + 2a)
352 Vasile Cîrtoaje

Indeed, we have

a(b + 2c)(b + 2a) − b(a + 2b)(a + 2c) = (a − b)(ab + 4bc + 4ca) ≥ 0.


p
The equality holds for a = b = c = 1, and also for a = b = 3 and c = 0 (or any
cyclic permutation).

P 2.54. Let a, b, c be nonnegative real numbers such that ab + bc + ca = 3. Prove


that p p p
a bc + 3 + b ca + 3 + c ab + 3 ≥ 6.
(Vasile Cîrtoaje, 2010)

Solution. Use the SOS method. Denote


p p p
A = ab + 2bc + ca, B = bc + 2ca + ab, C= ca + 2ab + bc,

and write the inequality as follows:


X
aA ≥ 2(ab + bc + ca),
X
a (A − b − c) ≥ 0,
X a(ab + ac − b2 − c 2 )
≥ 0,
A+ b + c
X a b(a − b) + ac(a − c)
≥ 0,
A+ b + c
X ab(a − b) X ba(b − a)
+ ≥ 0,
A+ b + c B+c+a
1 1
X  ‹
ab(a − b) − ≥ 0,
A+ b + c B + c + a
X
ab(a + b + C)(a − b)(a − b + B − A) ≥ 0,
X  c 
ab(a + b + C)(a − b) 1 +
2
≥ 0.
A+ B
p
The equality holds for a = b = c = 1, and for a = 0 and b = c = 3 (or any cyclic
permutation).
Symmetric Nonrational Inequalities 353

P 2.55. Let a, b, c be nonnegative real numbers such that a + b + c = 3. Prove that


p
(b + c) b2 + c 2 + 7bc ≥ 18;
P
(a)
p p
(b + c) b2 + c 2 + 10bc ≤ 12 3.
P
(b)
(Vasile Cîrtoaje, 2010)

Solution. Use the SOS technique.


(a) Write the inequality in the equivalent homogeneous forms
X p
(b + c) b2 + c 2 + 7bc ≥ 2(a + b + c)2 ,
X” p —
(b + c) b2 + c 2 + 7bc − b2 − c 2 − 4bc ≥ 0,
X (b + c)2 (b2 + c 2 + 7bc) − (b2 + c 2 + 4bc)2
p ≥ 0,
(b + c) b2 + c 2 + 7bc + b2 + c 2 + 4bc
X bc(b − c)2
p ≥ 0.
(b + c) b2 + c 2 + 7bc + b2 + c 2 + 4bc
3
The equality holds for a = b = c = 1, for a = 0 and b = c = (or any cyclic
2
permutation), and for a = 3 and b = c = 0 (or any cyclic permutation).
(b) Write the inequality as follows:
X Æ
(b + c) 3(b2 + c 2 + 10bc) ≤ 4(a + b + c)2 ,
X” Æ —
2b2 + 2c 2 + 8bc − (b + c) 3(b2 + c 2 + 10bc) ≥ 0,
X 4(b2 + c 2 + 4bc)2 − 3(b + c)2 (b2 + c 2 + 10bc)
p ≥ 0,
2b2 + 2c 2 + 8bc + (b + c) 3(b2 + c 2 + 10bc)
X (b − c)4
p ≥ 0.
2b2 + 2c 2 + 8bc + (b + c) 3(b2 + c 2 + 10bc)
The equality holds for a = b = c = 1.

P 2.56. Let a, b, c be nonnegative real numbers such then a + b + c = 2. Prove that


p p p p
a + 4bc + b + 4ca + c + 4ab ≥ 4 ab + bc + ca.

(Vasile Cîrtoaje, 2012)


354 Vasile Cîrtoaje

Solution. Without loss of generality, assume that

c = min{a, b, c}.

Using Minkowski’s inequality gives


s
p p €p p Š2 €p p Š2 €p p Šp
a + 4bc + b + 4ca ≥ a + b + 4c a+ b = a+ b 1 + 4c.

Therefore, it suffices to show that


€p p Šp p p
a+ b 1 + 4c ≥ 4 ab + bc + ca − c + 4ab.

By squaring, this inequality becomes


€ p Š Æ
a + b + 2 ab (1+4c)+8 (ab + bc + ca)(c + 4ab) ≥ 16(ab+ bc +ca)+c +4ab.

According to Lemma below, it suffices to show that


€ p Š
a + b + 2 ab (1 + 4c) + 8(2ab + bc + ca) ≥ 16(ab + bc + ca) + c + 4ab,

which is equivalent to
p p
a + b − c + 2 ab + 8c ab ≥ 4(ab + bc + ca).

Write this inequality in the homogeneous form


€ p Š p
(a + b + c) a + b − c + 2 ab + 16c ab ≥ 8(ab + bc + ca).

Due to homogeneity, we may assume that a + b = 1. Let us denote


p 1
d= ab, 0≤d ≤ .
2
We need to show that f (c) ≥ 0 for 0 ≤ c ≤ d, where

f (c) = (1 + c)(1 − c + 2d) + 16cd − 8d 2 − 8c


= (1 − 2d)(1 + 4d) + 2(9d − 4)c − c 2 .

Since f (c) is concave, it suffices to show that f (0) ≥ 0 and f (d) ≥ 0. Indeed,

f (0) = (1 − 2d)(1 + 4d) ≥ 0,

f (d) = (3d − 1)2 ≥ 0.


Thus, the proof is completed. The equality holds for a = b = 1 and c = 0 (or any
cyclic permutation).
Symmetric Nonrational Inequalities 355

Lemma (by Nguyen Van Quy). Let a, b, c be nonnegative real numbers such then

c = min{a, b, c}, a + b + c = 2.

Then, Æ
(ab + bc + ca)(c + 4ab) ≥ 2ab + bc + ca.
Proof. By squaring, the inequality becomes

c[ab + bc + ca − c(a + b)2 ] ≥ 0.

We need to show that

(a + b + c)(ab + bc + ca) − 2c(a + b)2 ≥ 0.

We have

(a + b + c)(ab + bc + ca) − 2c(a + b)2 ≥ (a + b)(b + c)(c + a) − 2c(a + b)2


= (a + b)(a − c)(b − c) ≥ 0.

P 2.57. If a, b, c are nonnegative real numbers, then


p p p p
a2 + b2 + 7ab + b2 + c 2 + 7bc + c 2 + a2 + 7ca ≥ 5 ab + bc + ca.

(Vasile Cîrtoaje, 2012)

Solution (by Nguyen Van Quy). Assume that

c = min{a, b, c}.

Using Minkowski’s inequality yields


s
p p €p p Š2
b2 + c 2 + 7bc + a2 + c 2 + 7ca ≥ (a + b)2 + 4c 2 + 7c a+ b .

Therefore, it suffices to show that


s
€p p Š2 p p
(a + b)2 + 4c 2 + 7c a + b ≥ 5 ab + bc + ca − a2 + b2 + 7ab.

By squaring, this inequality becomes


p Æ
2c 2 + 7c ab + 5 (a2 + b2 + 7ab)(ab + bc + ca) ≥ 15ab + 9c(a + b).
356 Vasile Cîrtoaje

1
Due to homogeneity, we may assume that a + b = 1, which implies c ≤ . Let us
2
1
denote x = ab. We need to show that f (x) ≥ 0 for c 2 ≤ x ≤ , where
4
p Æ
f (x) = 2c 2 + 7c x + 5 (1 + 5x)(c + x) − 15x − 9c.

Since
−7c 5(5c − 1)2
f 00 (x) = p − p <0
4 x 3 4 [5x 2 + (5c + 1)x + c]3
1
 ‹
f (c) is concave. Thus, it suffices to show that f (c ) ≥ 0 and f
2
≥ 0.
4
Write the inequality f (c 2 ) ≥ 0 as
Æ
5 (1 + 5c 2 )(c + c 2 ) ≥ 6c 2 + 9c.

By squaring, this inequality turns into

c(89c 3 + 17c 2 − 56c + 25) ≥ 0,

which is true since

89c 3 + 17c 2 − 56c + 25 ≥ 12c 2 − 56c + 25 = (1 − 2c)(25 − 6c) ≥ 0.


1
 ‹
Write the inequality f ≥ 0 as
4
€p Š
8c 2 − 22c + 15 4c + 1 − 1 ≥ 0.
p
Making the substitution t = 4c + 1, t ≥ 1, the inequality becomes

(t − 1)(t 3 + t 2 − 12t + 18) ≥ 0.

It is true since

t 3 + t 2 − 12t + 18 ≥ 2t 2 − 12t + 18 = 2(t − 3)2 ≥ 0.

Thus, the proof is completed. The equality holds for a = b and c = 0 (or any cyclic
permutation).

P 2.58. If a, b, c are nonnegative real numbers, then


p p p Æ
a2 + b2 + 5ab + b2 + c 2 + 5bc + c 2 + a2 + 5ca ≥ 21(ab + bc + ca).

(Nguyen Van Quy, 2012)


Symmetric Nonrational Inequalities 357

Solution. Without loss of generality, assume that c = min{a, b, c}. Using Minkowski’s
inequality, we have
s
Æ Æ €p p Š2
(a + c)2 + 3ac + (b + c)2 + 3bc ≥ (a + b + 2c)2 + 3c a+ b .

Therefore, it suffices to show that


s
€p p Š2 Æ p
(a + b + 2c) + 3c
2 a + b ≥ 21(ab + bc + ca) − a2 + b2 + 5ab.

By squaring, this inequality becomes


p Æ
2c 2 + 3c ab + 21(a2 + b2 + 5ab)(ab + bc + ca) ≥ 12ab + 7c(a + b).

Due to homogeneity, we may assume that a + b = 1. Let us denote x = ab. We


1
need to show that f (x) ≥ 0 for c 2 ≤ x ≤ , where
4
p Æ
f (x) = 2c 2 + 3c x + 21(1 + 3x)(c + x) − 12x − 7c.

Since p
−3c 21(3c − 1)2
f (x) = p − p
00
<0
4 x 3 4 [3x 2 + (3c + 1)x + c]3
1
 ‹
f (c) is concave. Thus, it suffices to show that f (c ) ≥ 0 and f
2
≥ 0.
4
Write the inequality f (c 2 ) ≥ 0 as
Æ
21(1 + 3c 2 )(c + c 2 ) ≥ 7(c + c 2 ).

By squaring, this inequality turns into

c(c + 1)(1 − 2c)(3 − c) ≥ 0,

which is clearly true.  ‹


1
Write the inequality f ≥ 0 as
4
Æ
8c 2 − 22c + 7 3(4c + 1) − 12 ≥ 0.
1
Using the substitution 3t 2 = 4c + 1, t ≥ p , the inequality becomes
3
(t − 1)2 (3t 2 + 6t − 4) ≥ 0.

This is true since p


3t 2 + 6t − 4 ≥ 1 + 2 3 − 4 > 0.
Thus, the proof is completed. The equality holds for a = b = c.
358 Vasile Cîrtoaje

P 2.59. Let a, b, c be nonnegative real numbers such that ab + bc + ca = 3. Prove


that v
p p p t2
a a2 + 5 + b b2 + 5 + c c 2 + 5 ≥ (a + b + c)2 .
3
(Vasile Cîrtoaje, 2010)

Solution. Write the inequality in the homogeneous form


X Æ p
a 3a2 + 5(ab + bc + ca) ≥ 2 (a + b + c)2 .

Due to homogeneity, we may assume that

ab + bc + ca = 1.

By squaring, the inequality becomes


X X Æ X X X
a4 +2 bc (3b2 + 5)(3c 2 + 5) ≥ 12 a2 b2 +19abc a+3 ab(a2 + b2 ).

Applying Lemma below for x = 3b2 , y = 3c 2 and d = 5, we have


Æ 9 2
2 (3b2 + 5)(3c 2 + 5) ≥ 3(b2 + c 2 ) + 10 − (b − c 2 )2 ,
20
hence
Æ 9
2bc (3b2 + 5)(3c 2 + 5) ≥ 3bc(b2 + c 2 ) + 10bc −
bc(b2 − c 2 )2 ,
20
X Æ X €X Š2 9 X
2 bc (3b2 + 5)(3c 2 + 5) ≥ 3 bc(b2 + c 2 ) + 10 bc − bc(b2 − c 2 )2
20
X X X 9 X
= 10 a b + 20abc
2 2
a+3 ab(a + b ) −
2 2
bc(b2 − c 2 )2 .
20
Therefore, it suffices to show that
X X X X 9 X
a + 10
4
a b + 20abc
2 2
a+3 ab(a + b ) − 2 2
bc(b2 − c 2 )2 ≥
20
X X X
≥ 12 a2 b2 + 19abc a+3 ab(a2 + b2 ),
which is equivalent to
X X X 9 X
4
a −2 a b + abc
2 2
a− bc(b2 − c 2 )2 ≥ 0.
20
To prove this inequality, we use the SOS method. Since
€X X X Š €X X Š €X X Š
2 a4 − 2 a2 b2 + abc a =2 a4 − a2 b2 −2 a2 b2 − abc a
Symmetric Nonrational Inequalities 359
X X
= (b2 − c 2 )2 − a2 (b − c)2 ,
we can write the inequality as
X
(b − c)2 Sa ≥ 0,

where
9
Sa = (b + c)2 − a2 − bc(b + c)2 .
10
In addition, since

bc(b + c)2
Sa ≥ (b + c)2 − a2 − bc(b + c)2 = (b + c)2 − a2 − ,
ab + bc + ca
a(b + c)3 − a2 (ab + bc + ca)
= ,
ab + bc + ca
it is enough to show that X
(b − c)2 Ea ≥ 0,
where
Ea = a(b + c)3 − a2 (ab + bc + ca).
Assume that
a ≥ b ≥ c, b>0
Since

E b = b(c + a)3 − b2 (ab + bc + ca) ≥ b(c + a)3 − b2 (c + a)(c + b)

≥ b(c + a)3 − b2 (c + a)2 = b(c + a)2 (c + a − b) ≥ 0,


Ec = c(a + b)3 − c 2 (ab + bc + ca) ≥ c(a + b)3 − c 2 (a + b)(b + c)
≥ c(a + b)3 − c 2 (a + b)2 = c(a + b)2 (a + b − c) ≥ 0
and
Ea E b (b + c)3 (c + a)3
+ = + − 2(ab + bc + ca)
a2 b2 a b
b3 + 2b2 c a3 + 2a2 c
≥ + − 2(ab + bc + ca)
a b
(a2 − b2 )2 + 2c(a + b)(a − b)2
= ≥ 0,
ab
we get

2 Ea Eb
X  ‹
(b − c) Ea ≥ (b − c) Ea + (a − c) E b ≥ a (b − c)
2 2 2 2
+ ≥ 0.
a2 b2
p
The equality holds for a = b = c = 1, and also for a = b = 3 and c = 0 (or any
cyclic permutation).
360 Vasile Cîrtoaje

Lemma. If x ≥ 0, y ≥ 0 and d > 0, then


Æ 1
2 (x + d)( y + d) ≥ x + y + 2d − (x − y)2 .
4d
Proof. We have
Æ 2x y + 2d(x + y) 2x y + 2d(x + y)
2 (x + d)( y + d) − 2d = p ≥ (x+d)+( y+d)
(x + d)( y + d) + d 2 +d

4x y + 4d(x + y) (x − y)2 (x − y)2


= =x+y− ≥x+y− .
x + y + 4d x + y + 4d 4d

P 2.60. Let a, b, c be nonnegative real numbers such that a2 + b2 + c 2 = 1. Prove that


p p p
a 2 + 3bc + b 2 + 3ca + c 2 + 3ab ≥ (a + b + c)2 .

(Vasile Cîrtoaje, 2010)

Solution. Write the inequality as


X p
a 2 + 3bc ≥ 1 + 2q,

where q = ab + bc + ca. By squaring, the inequality becomes


X X Æ
1 + 3abc a+2 bc (2 + 3ab)(2 + 3ac) ≥ 4q + 4q2 .

Applying Lemma from the preceding P 2.59 for x = 3ab, y = 3ac and d = 2, we
have Æ 9
2 (2 + 3ab)(2 + 3ac) ≥ 3a(b + c) + 4 − a2 (b − c)2 ,
8
hence Æ 9
2bc (2 + 3ab)(2 + 3ac) ≥ 3abc(b + c) + 4 − a2 bc(b − c)2 ,
8
X Æ X 9 X
2 bc (2 + 3ab)(2 + 3ac) ≥ 6abc a + 4q − abc a(b − c)2 .
8
Therefore, it suffices to show that
X X 9 X
1 + 3abc a + 6abc a + 4q − abc a(b − c)2 ≥ 4q + 4q2 ,
8
which is equivalent to
X 9 X
1 + 9abc a − 4q2 ≥ abc a(b − c)2 .
8
Symmetric Nonrational Inequalities 361

Since
X
a4 + b4 + c 4 = 1 − 2(a2 b2 + b2 c 2 + c 2 a2 ) = 1 − 2q2 + 4abc a,

from Schur’s inequality of fourth degree


X €X Š €X Š
a4 + b4 + c 4 + 2abc a≥ a2 ab ,

we get X
1 ≥ 2q2 + q − 6abc a.
Thus, it is enough to prove that
€ X Š X 9 X
2q2 + q − 6abc a + 9abc a − 4q2 ≥ abc a(b − c)2 ;
8
that is, € X Š X
8 q − 2q2 + 3abc a ≥ 9abc a(b − c)2 .
Since
X €X Š €X Š €X Š2 X
q − 2q2 + 3abc a= a2 ab − 2 ab + 3abc a
X X X
= bc(b2 + c 2 ) − 2 b2 c 2 = bc(b − c)2 ,
we need to show that X
bc(8 − 9a2 )(b − c)2 ≥ 0.
Since
8 − 9a2 = 8(b2 + c 2 ) − a2 ≥ b2 + c 2 − a2 ,
it suffices to prove the homogeneous inequality
X
bc(b2 + c 2 − a2 )(b − c)2 ≥ 0.

Assume that a ≥ b ≥ c. It is enough to show that

bc(b2 + c 2 − a2 )(b − c)2 + ca(c 2 + a2 − b2 )(c − a)2 ≥ 0.

This is true if

a(c 2 + a2 − b2 )(a − c)2 ≥ b(a2 − b2 − c 2 )(b − c)2 .

For the nontrivial case a2 − b2 − c 2 ≥ 0, this inequality follows from

a ≥ b, c 2 + a2 − b2 ≥ a2 − b2 − c 2 , (a − c)2 ≥ (b − c)2 .
1 1
The equality holds for a = b = c = p , and for a = 0 and b = c = p (or any
3 2
cyclic permutation).
362 Vasile Cîrtoaje

P 2.61. Let a, b, c be nonnegative real numbers such that a + b + c = 3. Prove that


v v v
t 2a + bc t 2b + ca t 2c + ab
(a) a +b +c ≥ 3;
3 3 3
v v v
t a(1 + b + c) t b(1 + c + a) t c(1 + a + b)
(b) a +b +c ≥ 3.
3 3 3
(Vasile Cîrtoaje, 2010)
Solution. (a) If two of a, b, c are zero, then the inequality is trivial. Otherwise, by
Hölder’s inequality, we have
‚ v Œ2 P 3
X t 2a + bc a 9
a ≥P =P a .
3 3a
2a + bc 2a + bc
Therefore, it suffices to show that
X a
≤ 1.
2a + bc
Since
2a bc
=1− ,
2a + bc 2a + bc
we can write this inequality as
X bc
≥ 1.
2a + bc
By the Cauchy-Schwarz inequality, we have
P 2 P 2
X bc bc bc
≥P = = 1.
2a + bc bc(2a + bc) 2abc a + b2 c 2
P P

3
The equality holds for a = b = c = 1, and for a = 0 and b = c = (or any cyclic
2
permutation).
(b) Write the inequality in the homogeneous form
X Æ
a a(a + 4b + 4c) ≥ (a + b + c)2 .

By squaring, the inequality becomes


X Æ X X
bc bc(b + 4c + 4a)(c + 4a + 4b) ≥ 3 b c + 6abc
2 2
a.

Applying the Cauchy-Schwarz inequality, we have


Æ Æ
(b + 4c + 4a)(c + 4a + 4b) = (4a + b + c + 3c)(4a + b + c + 3b)
Symmetric Nonrational Inequalities 363

p
≥ 4a + b + c + 3 bc,
hence
Æ p
bc bc(b + 4c + 4a)(c + 4a + 4b) ≥ (4a + b + c)bc bc + 3b2 c 2 ,
X Æ X p X
bc bc(b + 4c + 4a)(c + 4a + 4b) ≥ (4a + b + c)bc bc + 3 b2 c 2 .
Thus, it is enough to show that
X p X
(4a + b + c)bc bc ≥ 6abc a.

Replacing a, b, c by a2 , b2 , c 2 , respectively, this inequality becomes


X X
(4a2 + b2 + c 2 )b3 c 3 ≥ 6a2 b2 c 2 a2 ,
€X Š €X Š X X
a2 b3 c 3 + 3a2 b2 c 2 bc ≥ 6a2 b2 c 2 a2 ,
€X Š €X Š €X X Š
a2 a3 b3 − 3a2 b2 c 2 ≥ 3a2 b2 c 2 a2 − ab .
Use next the SOS method. Since
X €X Š €X X Š 1 €X Š X
a3 b3 − 3a2 b2 c 2 = ab a2 b2 − abc a = ab a2 (b − c)2 ,
2
and X X 1X
a2 − ab = (b − c)2 ,
2
we can write the inequality as
X
(b − c)2 Sa ≥ 0,

where €X Š €X Š
Sa = a 2 a2 ab − 3a2 b2 c 2 .
Assume that a ≥ b ≥ c. Since Sa ≥ S b ≥ 0 and
€X Š €X Š
S b + Sc = (b2 + c 2 ) a2 ab − 6a2 b2 c 2
€X Š €X Š
≥ 2bc a2 ab − 6a2 b2 c 2
€X Š
≥ 2bca2 ab − 6a2 b2 c 2 = 2a2 bc(ab + ac − 2bc) ≥ 0,

we get
X
(b − c)2 Sa ≥ (c − a)2 S b + (a − b)2 Sc ≥ (a − b)2 (S b + Sc ) ≥ 0.

The equality holds for a = b = c = 1, and also for a = 3 and b = c = 0 (or any
cyclic permutation).
364 Vasile Cîrtoaje

P 2.62. If a, b, c are nonnegative real numbers such that a + b + c = 3, then


Æ Æ Æ
8(a2 + bc) + 9 + 8(b2 + ca) + 9 + 8(c 2 + ab) + 9 ≥ 15.

(Vasile Cîrtoaje, 2013)

Solution. Use the SOS technique. Let q = ab + bc + ca and

A = (3a − b − c)2 + 8q, B = (3b − c − a)2 + 8q, C = (3c − a − b)2 + 8q.

Since

8(a2 + bc) + 9 = 8(a2 + q) + 9 − 8a(b + c) = 8(a2 + q) + 9 − 8a(3 − a)


= (4a − 3)2 + 8q = (3a − b − c)2 + 8q = A,

we can rewrite the inequality as follows:


Xp
A ≥ 15,
X p
[ A − (3a + b + c)] ≥ 0,
X 2bc − ca − ab
p ≥ 0,
A + 3a + b + c
X• b(c − a) c(b − a)
˜
p +p ≥ 0,
A + 3a + b + c A + 3a + b + c
X c(a − b) X c(b − a)
p + p ≥ 0,
B + 3b + c + a A + 3a + b + c
X p p p
c(a − b)( C + 3c + a + b)[ A − B + 2(a − b)] ≥ 0,
p 4(a + b − c)
X • ˜
c(a − b) ( C + 3c + a + b) p
2
p + 1 ≥ 0.
A+ B
Without loss of generality, assume that a ≥ b ≥ c. Since a + b − c > 0, it suffices to
show that
p 4(c + a − b)
• ˜
b(a − c) ( B + 3b + c + a) p
2
p +1 ≥
A+ C
p 4(a − b − c)
• ˜
a(b − c) ( A + 3a + b + c) p
2
p −1 .
B+ C
This inequality follows from the inequalities

b2 (a − c)2 ≥ a2 (b − c)2 ,
p p
a( B + 3b + c + a) ≥ b( A + 3a + b + c),
Symmetric Nonrational Inequalities 365

4(c + a − b) 4(a − b − c)
p p +1≥ p p − 1.
A+ C B+ C
Write the second inequality as

a2 B − b2 A
p p + (a − b)(a + b + c) ≥ 0.
a B+b A
Since

a2 B − b2 A = (a − b)(a + b + c)(a2 + b2 − 6ab + bc + ca) + 8q(a2 − b2 )


≥ (a − b)(a + b + c)(a2 + b2 − 6ab) ≥ −4ab(a − b)(a + b + c),

it suffices to show that


−4ab
p p + 1 ≥ 0.
a B+b A
p p p p
Indeed, from A > 8q ≥ 2 ab and B ≥ 8q ≥ 2 ab, we get
p p

p p p p p
a B + b A − 4ab > 2(a + b) ab − 4ab = 2 ab(a + b − 2 ab) ≥ 0.

The third inequality holds if

2(a − b − c)
1≥ p p .
B+ C
p p
It suffices to show that B ≥ a and C ≥ a. We have

B − a2 = 8q − 2a(3b − c) + (3b − c)2 ≥ 8ab − 2a(3b − c) = 2a(b + c) ≥ 0

and

C − a2 = 8q − 2a(3c − b) + (3c − b)2 ≥ 8ab − 2a(3c − b) = 2a(5b − 3c) ≥ 0.

The equality holds for a = b = c = 1, and also for a = 3 and b = c = 0 (or any
cyclic permutation).

9
P 2.63. Let a, b, c be nonnegative real numbers such that a + b + c = 3. If k ≥ ,
8
then p p p p
a2 + bc + k + b2 + ca + k + c 2 + ab + k ≥ 3 2 + k.

Solution. We will show that


XÆ XÆ Æ
8(a2 + bc + k) ≥ (3a + b + c)2 + 8k − 9 ≥ 6 2(k + 2).
366 Vasile Cîrtoaje

The right inequality is equivalent to


XÆ Æ
(2a + 3)2 + 8k − 9 ≥ 6 2(k + 2),

which follows immediately from Jensen’s inequality applied to the convex function
f : [0, ∞) → R defined by
Æ
f (x) = (2x + 3)2 + 8k − 9.

To prove the left inequality, we use the SOS method. By means of the substitutions

A1 = 8(a2 + bc + k), B1 = 8(b2 + ca + k), C1 = 8(c 2 + ab + k),

A2 = (3a + b + c)2 +8k −9, B2 = (3b + c + a)2 +8k −9, C2 = (3c + a + b)2 +8k −9,
we can write the inequality as follows:

A1 − A2 B1 − B2 C1 − C2
p +p p +p p ≥ 0,
A1 + A2 B1 + B2 C1 + C2
p

2bc − ca − ab 2ca − ab − bc 2ab − bc − ca


+ p + p ≥ 0,
A1 + A2 B1 + B2 C1 + C2
p p p p

X  b(c − a) c(b − a)

p +p ≥ 0,
A1 + A2 A1 + A2
p p

X c(a − b) X c(b − a)
+ p ≥ 0,
B1 + B2 A1 + A2
p p p

X
c(a − b)( C1 + C2 )[( A1 − B1 ) + ( A2 − B2 )] ≥ 0,
p p p p p p

2(a + b − c) 2a + 2b + c
X  
c(a − b) ( C1 + C2 ) p p +p
2
p p
≥ 0.
A1 + B1 A2 + B2
p

Without loss of generality, assume that a ≥ b ≥ c. Clearly, the desired inequality


is true for b + c ≥ a. Consider further the case b + c < a. Since a + b − c > 0, it
suffices to show that

2(b + c − a) 2b + 2c + a
 
a(b − c) ( A1 + A2 ) p p +p +
2
p p
B1 + C1 B2 + C2
p

2(c + a − b) 2c + 2a + b
 
+b(a − c) ( B1 + B2 ) p p +p
2
p p
≥ 0.
C1 + A1 C2 + AC2
p

Since
b2 (a − c)2 ≥ a2 (b − c)2 ,
Symmetric Nonrational Inequalities 367

it suffices to show that

2(b + c − a) 2b + 2c + a
 
b( A1 + A2 ) p p +p +
p p
B1 + C1 B2 + C2
p

2(c + a − b) 2c + 2a + b
 
+a( B1 + B2 ) p p +p
p p
≥ 0.
C1 + A1 C2 + A2
p

From
a2 B1 − b2 A1 = 8c(a3 − b3 ) + 8k(a2 − b2 ) ≥ 0
and

a2 B2 − b2 A2 = (a − b)(a + b + c)(a2 + b2 + 6ab + bc + ca) + (8k − 9)(a2 − b2 ) ≥ 0,


p p p p
we get a B1 ≥ b A1 and a B2 ≥ b A2 , hence

a( B1 + B2 ) ≥ b( A1 + A2 ).
p p p p

Therefore, it is enough to show that

2(b + c − a) 2b + 2c + a 2(c + a − b) 2c + 2a + b
p +p p +p p +p p ≥ 0.
B1 + C1 B2 + C2 C1 + A1 C2 + A2
p

This is true if
2b −2b
p +p p ≥0
B1 + C1 C1 + A1
p

and
−2a 2a 2a
p +p p +p p ≥ 0.
B1 + C1 C1 + A1 C2 + A2
p

The first inequality is true because A1 − B1 = 8(a − b)(a + b − c) ≥ 0. The second


inequality can be written as

1 1 1
+p ≥p .
C1 + C2 + B1 +
p p p p
A1 A2 C1

Since
1 1 4
+p ≥p ,
C1 + C2 + C1 + A1 + C2 +
p p p p p p
A1 A2 A2
it suffices to show that

4 B1 + 3 C1 ≥ A1 + A2 + C2 .
p p p p p

Taking account of
C1 − C2 = 4(2ab − bc − ca) ≥ 0,
C1 − B1 = 8(b − c)(a − b − c) ≥ 0,
368 Vasile Cîrtoaje

A2 − A1 = 4(ab − 2bc + ca) ≥ 0,


we have
B1 + 3 C1 − A1 − A2 − C2 ≥ 4 B1 + 2 C1 − A1 − A2
p p p p p p p p p
4
≥ 4 B1 + 2 B1 − A2 − A2
p p p p

= 2(3 B1 − A2 ).
p p

In addition,
9B1 − A2 = 64k − 8a2 + 72b2 − 4ab + 68ac
≥ 72 − 8a2 + 72b2 − 4ab + 68ac
= 8(a + b + c)2 − 8a2 + 72b2 − 4ab + 68ac
= 4(20b2 + 2c 2 + 3ab + 4bc + 21ac) ≥ 0.
Thus, the proof is completed. The equality holds for a = b = c = 1. If k = 9/8,
then the equality holds also for a = 3 and b = c = 0 (or any cyclic permutation).

P 2.64. If a, b, c are nonnegative real numbers such that a + b + c = 3, then


p p p p
a3 + 2bc + b3 + 2ca + c 3 + 2ab ≥ 3 3.
(Nguyen Van Quy, 2013)
Solution. Since
(a3 + 2bc)(a + 2bc) ≥ (a2 + 2bc)2 ,
it suffices to prove that
X a2 + 2bc p
p ≥ 3 3.
a + 2bc
By Hölder’s inequality, we have
‹2 X
a + 2bc
X 2 ”X —3
p (a2 + 2bc)(a + 2bc) ≥ (a2 + 2bc) = (a + b + c)6 .
a + 2bc
Therefore, it suffices to show that
X
(a + b + c)6 ≥ 27 (a2 + 2bc)(a + 2bc).
which is equivalent to
X
(a + b + c)4 ≥ (a2 + 2bc)(a2 + 6bc + ca + ab).
Indeed,
X X
(a + b + c) −
4
(a + 2bc)(a + 6bc + ca + ab) = 3
2 2
ab(a − b)2 ≥ 0.
The equality holds for a = b = c = 1, and also for a = 3 and b = c = 0 (or any
cyclic permutation).
Symmetric Nonrational Inequalities 369

P 2.65. If a, b, c are positive real numbers, then


p p p p
a2 + bc b2 + ca c 2 + ab 3 2
+ + ≥ .
b+c c+a a+b 2
(Vasile Cîrtoaje, 2006)

Solution. According to the well-known inequality

(x + y + z)2 ≥ 3(x y + yz + z x), x, y, z ≥ 0,

it suffices to show that


p
X (b2 + ca)(c 2 + ab) 3
≥ .
(c + a)(a + b) 2

Replacing a, b, c by a2 , b2 , c 2 , respectively, the inequality becomes


X Æ
2 (b2 + c 2 ) (b4 + c 2 a2 )(c 4 + a2 b2 ) ≥ 3(a2 + b2 )(b2 + c 2 )(c 2 + a2 ).

Multiplying the Cauchy-Schwarz inequalities


Æ
(b2 + c 2 )(b4 + c 2 a2 ) ≥ b3 + ac 2 ,
Æ
(c 2 + b2 )(c 4 + a2 b2 ) ≥ c 3 + ab2 ,
we get
Æ
(b2 + c 2 ) (b4 + c 2 a2 )(c 4 + a2 b2 ) ≥ (b3 + ac 2 )(c 3 + ab2 )
= b3 c 3 + a(b5 + c 5 ) + a2 b2 c 2 .
Therefore, it suffices to show that
X X
2 b3 c 3 + 2 a(b5 + c 5 ) + 6a2 b2 c 2 ≥ 3(a2 + b2 )(b2 + c 2 )(c 2 + a2 ).

This inequality is equivalent to


X X X
2 b3 c 3 + 2 bc(b4 + c 4 ) ≥ 3 b2 c 2 (b2 + c 2 ),
X
bc[2b2 c 2 + 2(b4 + c 4 ) − 3bc(b2 + c 2 )] ≥ 0,
X
bc(b − c)2 (2b2 + bc + 2c 2 ) ≥ 0.

The equality holds for a = b = c.


370 Vasile Cîrtoaje

P 2.66. If a, b, c are nonnegative real numbers, no two of which are zero,then


p p p
bc + 4a(b + c) ca + 4b(c + a) ab + 4c(a + b) 9
+ + ≥ .
b+c c+a a+b 2
(Vasile Cîrtoaje, 2006)

Solution. Let us denote

A = 4ab + bc + 4ca, B = 4ab + 4bc + ca, C = ab + 4bc + 4ca.

By squaring, the inequality becomes


p
X A X BC 81
+2 ≥ .
(b + c)2 (c + a)(a + b) 4
According to the known inequality Iran-1996, namely
X ab + bc + ca 9

(b + c)2 4
(see Remark from the proof of P 1.72), we have
X A X ab + bc + ca X a 9 X a
= + 3 ≥ + 3 .
(b + c)2 (b + c)2 b+c 4 b+c
On the other hand, from Lemma below, we have
p 2abc
BC ≥ 2ab + 4bc + 2ca + ,
b+c
p 2a(b2 + c 2 ) + 4bc(b + c) + 6abc
BC ≥ ,
b+c
p
4 a(b2 + c 2 ) + 8 bc(b + c) + 36abc
P P
X BC
2 ≥ ,
(c + a)(a + b) (a + b)(b + c)c + a)
p
12 bc(b + c) + 36abc
P
X BC
2 ≥ .
(c + a)(a + b) (a + b)(b + c)c + a)
Thus, it suffices to show that

12 bc(b + c) + 36abc
P
X a
3 + ≥ 18.
b+c (a + b)(b + c)c + a)
This is equivalent to Schur’s inequality of degree three
X X
a3 + 3abc ≥ bc(b + c).

The equality holds for a = b = c, and also for a = 0 and b = c (or any cyclic
permutation).
Symmetric Nonrational Inequalities 371

Lemma. If a, b, c are nonnegative real numbers, no two of which are zero, then
Æ 2abc
(4ab + 4bc + ca)(ab + 4bc + 4ca) ≥ 2ab + 4bc + 2ca + ,
b+c
with equality for b = c, and also for abc = 0.
Proof. We use the AM-GM inequality as follows:
Æ
(4ab + 4bc + ca)(ab + 4bc + 4ca) − 2ab − 4bc − 2ca =

abc(9a + 4b + 4c)
=p
(4ab + 4bc + ca)(ab + 4bc + 4ca) + 2ab + 4bc + 2ca
2abc(9a + 4b + 4c)

(4ab + 4bc + ca) + (ab + 4bc + 4ca) + 4ab + 8bc + 4ca
2abc(9a + 4b + 4c)
= .
9ab + 16bc + 9ca
Thus, it suffices to show that

9a + 4b + 4c 1
≥ .
9ab + 16bc + 9ca b+c
Indeed,

(9a + 4b + 4c)(b + c) − (9ab + 16bc + 9ca) = 4(b − c)2 ≥ 0.

P 2.67. If a, b, c are nonnegative real numbers, no two of which are zero,then


p p p
a a2 + 3bc b b2 + 3ca c c 2 + 3ab
+ + ≥ a + b + c.
b+c c+a a+b
(Cezar Lupu, 2006)

Solution. Using the AM-GM inequality, we have


p
a a2 + 3bc 2a(a2 + 3bc) 2a(a2 + 3bc) 2a3 + 6abc
= p ≥ = ,
b+c 2 (b + c)2 (a2 + 3bc) (b + c)2 + (a2 + 3bc) S + 5bc

where S = a2 + b2 + c 2 . Thus, it suffices to show that


X 2a3 + 6abc
≥ a + b + c.
S + 5bc
372 Vasile Cîrtoaje

Write this inequality as


X  2a2 + 6bc ‹
a − 1 ≥ 0,
S + 5bc
or, equivalently,
AX + BY + X Z ≥ 0,
where
1 1 1
A= , B= , C= ,
S + 5bc S + 5ca S + 5ab
X = a3 + abc − a(b2 + c 2 ), Y = b3 + abc − b(c 2 + a2 ), Z = c 3 + abc − c(a2 + b2 ).
Without loss of generality, assume that a ≥ b ≥ c. We have

A ≥ B ≥ C,

X = a(a2 − b2 ) + ac(b − c) ≥ 0, Z = c(c 2 − b2 ) + ac(b − a) ≤ 0


and, according to Schur’s inequality of third degree,
X X
X +Y +Z = a3 + 3abc − a(b2 + c 2 ) ≥ 0.

Therefore,

AX + BY + C Z ≥ BX + BY + BZ = B(X + Y + Z) ≥ 0.

The equality holds for a = b = c, and also for a = 0 and b = c (or any cyclic
permutation).
Remark. We can also prove the inequality AX +BY +X Z ≥ 0 by the SOS procedure.
Write this inequality as follows:
X a(a2 + bc − b2 − c 2 )
≥ 0,
S + 5bc
X a(a2 b + a2 c − b3 − c 3 )
≥ 0,
(b + c)(S + 5bc)
X ab(a2 − b2 ) + ac(a2 − c 2 )
≥ 0,
(b + c)(S + 5bc)
X ab(a2 − b2 ) X ba(b2 − a2 )
+ ≥ 0,
(b + c)(S + 5bc) (c + a)(S + 5ca)
X ab(a + b)(a − b)2 [S + 5c(a + b + c)]
≥ 0.
(b + c)(c + a)(S + 5bc)(S + 5ca)
Symmetric Nonrational Inequalities 373

P 2.68. If a, b, c are nonnegative real numbers, no two of which are zero,then


v v v
t 2a(b + c) t 2b(c + a) t 2c(a + b)
+ + ≥ 2.
(2b + c)(b + 2c) (2c + a)(c + 2a) (2a + b)(a + 2b)
(Vasile Cîrtoaje, 2006)
Solution. Making the substitution
p p p
x= a, y= b, z= c,

the inequality becomes


v
X t 2( y 2 + z 2 )
x ≥ 2.
(2 y 2 + z 2 )( y 2 + 2z 2 )
We claim that v
t 2( y 2 + z 2 ) y +z
≥ .
(2 y 2 + z 2 )( y 2 + 2z 2 ) y 2 + yz + z 2
Indeed, be squaring and direct calculation, this inequality reduces to

y 2 z 2 ( y − z)2 ≥ 0.

Thus, it suffices to show that


X x( y + z)
≥ 2,
y 2 + yz + z 2
which is just the inequality in P 1.69. The equality holds for a = b = c, and also
for a = 0 and b = c (or any cyclic permutation).

P 2.69. If a, b, c are nonnegative real numbers such that ab + bc + ca = 3, then


v s v v s v
t bc ca t ab t bc ca t ab
+ + ≤1≤ + + .
3a2 + 6 3b2 + 6 3c 2 + 6 6a2 + 3 6b2 + 3 6c 2 + 3
(Vasile Cîrtoaje, 2011)
Solution. By the Cauchy-Schwarz inequality, we have
‚ v Œ2 
1
X t bc X ‹ €X Š
≤ bc ,
3a2 + 6 3a2 + 6

hence ‚ v Œ2
Xt bc X 1
≤ .
3a2 + 6 a2 + 2
374 Vasile Cîrtoaje

Therefore, to prove the original left inequality, it suffices to show that


X 1
≤ 1.
a2 +2
This inequality is equivalent to
X a2
≥ 1.
a2 + 2
By the Cauchy-Schwarz inequality, we get
X a2 (a + b + c)2 (a + b + c)2
≥ = = 1.
a2 + 2 (a2 + 2) a2 + 6
P P

The equality occurs for a = b = c = 1.


To prove the original right inequality we apply Hölder’s inequality as follows:
‚ v Œ2
Xt bc ”X — €X Š3
b2 c 2 (6a2 + 3) ≥ bc .
6a + 3
2

Thus, it suffices to show that


X
(ab + bc + ca)3 ≥ b2 c 2 (6a2 + ab + bc + ca),

which is equivalent to
” X —
(ab + bc + ca) (ab + bc + ca)2 − b2 c 2 ≥ 18a2 b2 c 2 ,

2abc(ab + bc + ca)(a + b + c) ≥ 18a2 b2 c 2 ,


X
2abc a(b − c)2 ≥ 0.
The equality occurs for a = b = c = 1, and for a = 0 and bc = 3 (or any cyclic
permutation).

P 2.70. Let a, b, c be nonnegative real numbers such that ab + bc + ca = 3. If k > 1,


than
a k (b + c) + b k (c + a) + c k (a + b) ≥ 6.

Solution. Let
E = a k (b + c) + b k (c + a) + c k (a + b).
We consider two cases.
Symmetric Nonrational Inequalities 375

Case 1: k ≥ 2. Applying Jensen’s inequality to the convex function f (x) = x k−1 ,


x ≥ 0, we get

E = (ab + ac)a k−1 + (bc + ba)b k−1 + (ca + c b)c k−1


(ab + ac)a + (bc + ba)b + (ca + c b)c k−1
• ˜
≥ 2(ab + bc + ca)
2(ab + bc + ca)
˜k−1
a (b + c) + b2 (c + a) + c 2 (a + b)
• 2
=6 .
6

Thus, it suffices to show that

a2 (b + c) + b2 (c + a) + c 2 (a + b) ≥ 6.

Write this inequality as

(ab + bc + ca)(a + b + c) − 3abc ≥ 6,

a + b + c ≥ 2 + abc.
It is true since Æ
a+b+c ≥ 3(ab + bc + ca) = 3
and ‹3
a+b+c

abc ≤ = 1.
3

Case 2: 1 < k < 2. We have

E = a k−1 (3 − bc) + b k−1 (3 − ca) + c k−1 (3 − ab)


 
= 3(a k−1 + b k−1 + c k−1 ) − a k−1 b k−1 c k−1 (ab)2−k + (bc)2−k + (ca)2−k .
Since 0 < 2 − k < 1, f (x) = x 2−k is concave for x ≥ 0. Thus, by Jensen’s inequality,
we have
ab + bc + ca 2−k
 ‹
(ab) + (bc) + (ca)
2−k 2−k 2−k
≤3 = 3,
3
hence
E ≥ 3(a k−1 + b k−1 + c k−1 ) − 3a k−1 b k−1 c k−1 .
Consequently, it suffices to show that

a k−1 + b k−1 + c k−1 ≥ a k−1 b k−1 c k−1 + 2.

Due to symmetry, we may assume that

a ≥ b ≥ c,
376 Vasile Cîrtoaje

which involves
1
ab ≥ (ab + bc + ca) ≥ 1.
3
Let p
x= a k−1 b k−1 , x ≥ 1.
From p
2 ≥ 3 − ab = bc + ca ≥ 2c ab,
we get
1
c≤p ,
ab
hence
1
c k−1 ≤ .
x
Write the required inequality as

a k−1 + b k−1 − 2 ≥ a k−1 b k−1 − 1 c k−1 .

It suffices to show that

a k−1 b k−1 − 1
a k−1 + b k−1 − 2 ≥ .
x
Since p
a k−1 + b k−1 ≥ 2 a k−1 b k−1 = 2x,
we only need to prove that
x2 − 1
2x − 2 ≥ .
x
Indeed,
x 2 − 1 (x − 1)2
2x − 2 − = ≥ 0.
x x
The equality holds for a = b = c = 1.

P 2.71. Let a, b, c be nonnegative real numbers such that a + b + c = 2. If

2 ≤ k ≤ 3,

than
a k (b + c) + b k (c + a) + c k (a + b) ≤ 2.
Symmetric Nonrational Inequalities 377

Solution. Denote by Ek (a, b, c) the left hand side of the inequality, assume that

a ≤ b ≤ c,

and show that


Ek (a, b, c) ≤ Ek (0, a + b, c) ≤ 2.
The left inequality is equivalent to
ab k−1
(a + b k−1 ) ≤ (a + b)k − a k − b k .
c
Clearly, it suffices to consider c = b, when the inequality becomes

2a k + b k−1 (a + b) ≤ (a + b)k .

Since 2a k ≤ a k−1 (a + b), it remains to show that

a k−1 + b k−1 ≤ (a + b)k−1 ,

which is true since


a k−1 + b k−1  a k−1
 ‹k−1
b a b
= + ≤ + = 1.
(a + b)k−1 a+b a+b a+b a+b
Using the notation d = a + b, we can write the right inequality Ek (0, a + b, c) ≤ 2
in the form
cd(c k−1 + d k−1 ) ≤ 2,
where c + d = 2. By the Power-Mean inequality , we have
1/(k−1) ‹1/2
c k−1 + d k−1 c2 + d 2
 
≤ ,
2 2
‹(k−1)/2
c2 + d 2

c k−1
+d k−1
≤2 .
2
Thus, it suffices to show that
‹(k−1)/2
c2 + d 2

cd ≤ 1,
2
which is equivalent to
cd(2 − cd)(k−1)/2 ≤ 1.
Since 2 − cd ≥ 1, we have

cd(2 − cd)(k−1)/2 ≤ cd(2 − cd) = 1 − (1 − cd)2 ≤ 1.

The equality holds for a = 0 and b = c = 1 (or any cyclic permutation).


378 Vasile Cîrtoaje

P 2.72. Let a, b, c be nonnegative real numbers, no two of which are zero. If


m > n ≥ 0,
than
bm + c m c m + am am + bm
(b + c − 2a) + (c + a − 2b) + (a + b − 2c) ≥ 0.
bn + c n c n + an an + bn
(Vasile Cîrtoaje, 2006)
Solution. Write the inequality as
AX + BY + C Z ≥ 0,
where
bm + c m c m + am am + bm
A= , B = , C = ,
bn + c n c n + an an + bn
X = b + c − 2a, Y = c + a − 2b, Z = a + b − 2c, X + Y + Z = 0.
Without loss of generality, assume that
a ≤ b ≤ c,
which involves X ≥ Y ≥ Z and X ≥ 0. Since
2(AX + BY + C Z) = (2A − B − C)X + (B + C)X + 2(BY + C Z)
= (2A − B − C)X − (B + C)(Y + Z) + 2(BY + C Z)
= (2A − B − C)X + (B − C)(Y − Z),
it suffices to show that B ≥ C and 2A − B − C ≥ 0. The inequality B ≥ C can be
written as
b n c n (c m−n − b m−n ) + a n (c m − b m ) − a m (c n − b n ) ≥ 0,
b n c n (c m−n − b m−n ) + a n [c m − b m − a m−n (c n − b n )] ≥ 0.
This is true since c m−n ≥ b m−n and
c m − b m − a m−n (c n − b n ) ≥ c m − b m − b m−n (c n − b n ) = c n (c m−n − b m−n ) ≥ 0.
The inequality 2A − B − C ≥ 0 follows from
2A ≥ b m−n + c m−n , b m−n ≥ C, c m−n ≥ B.
Indeed, we have
(b n − c n )(b m−n − c m−n )
2A − b m−n − c m−n = ≥ 0,
bn + c n
a n (b m−n − a m−n )
b m−n − C = ≥ 0,
an + bn
a n (c m−n − a m−n )
c m−n
−B= ≥ 0.
c n + an
The equality holds for a = b = c, and also for a = 0 and b = c (or any cyclic
permutation).
Symmetric Nonrational Inequalities 379

P 2.73. Let a, b, c be positive real numbers such that abc = 1. Prove that
p p p
a2 − a + 1 + b2 − b + 1 + c 2 − c + 1 ≥ a + b + c.

(Vasile Cîrtoaje, 2012)

First Solution. Among a − 1, b − 1 and c − 1 there are two with the same sign. Let
(b − 1)(c − 1) ≥ 0, that is,

1
t≤ , t = b + c − 1.
a
By Minkowsky’s inequality, we have
v v
u ‹2 u ‹2
p p 1 3 1 3 p 2
− b+1+ −c+1= + + + ≥ t + 3.
t t
b2 c2 b− c−
2 4 2 4

Thus, it suffices to show that


p p
a2 − a + 1 + t 2 + 3 ≥ a + b + c,

which is equivalent to p
a2 − a + 1 + f (t) ≥ a + 1,
where p
f (t) = t 2 + 3 − t.
Clearly, f (t) is decreasing for t ≤ 0. Since

3
f (t) = p ,
t2 + 3 + t
1
 ‹
f (t) is also decreasing for t ≥ 0. Then, f (t) ≥ f , and it suffices to show that
a

1
p  ‹
a −a+1+ f
2 ≥ a + 1,
a

which is equivalent to
v
p t1 1
a2 − a + 1 + 2
+ 3 ≥ a + + 1.
a a

By squaring, this inequality becomes


v
1 2
t  ‹
2 (a − a + 1) 2 + 3 ≥ 3a + − 1.
2
a a
380 Vasile Cîrtoaje

Indeed, by the Cauchy-Schwarz inequality, we have


v ‹ v
1 1
t  t  ‹
2 (a − a + 1) 2 + 3 = [(2 − a) + 3a ] 2 + 3
2 2 2
a a
2−a 2
≥ + 3a = 3a + − 1.
a a
The equality holds for a = b = c.
Second Solution. If the inequality
1 3
p  ‹
x2 − x +1− x ≥ −1
2 x + x +1
2

holds for all x > 0, then it suffices to prove that


1 1 1
+ 2 + 2 ≥ 1,
a2 +a+1 b + b+1 c +c+1
which is just the known inequality in P 1.45. The above inequality in x is equivalent
to
1− x (1 − x)(2 + x)
p ≥ ,
x2 − x + 1 + x 2(x 2 + x + 1)
” p —
(x − 1) (x + 2) x − x + 1 − x − 2 ≥ 0,
2 2

3x 2 (x − 1)2
p ≥ 0.
(x + 2) x 2 − x + 1 + x 2 + 2

P 2.74. Let a, b, c be positive real numbers such that abc = 1. Prove that
p p p
16a2 + 9 + 16b2 + 9 + 16b2 + 9 ≥ 4(a + b + c) + 3.

(MEMO, 2012)
First Solution (by Vo Quoc Ba Can). Since
p 9
16a2 + 9 − 4a = p ,
16a2 + 9 + 4a
the inequality is equivalent to
X 1 1
p ≥ .
16a2 + 9 + 4a 3
By the AM-GM inequality, we have
p 16a2 + 9
2 16a2 + 9 ≤ + 2a + 3,
2a + 3
Symmetric Nonrational Inequalities 381

p 16a2 + 9 18(2a2 + 2a + 1)
2( 16a2 + 9 + 4a) ≤ + 10a + 3 = .
2a + 3 2a + 3
Thus, it suffices to show that
X 2a + 3
≥ 3.
2a2 + 2a + 1
If the inequality
2a + 3 3
≥ 8/5
2a2 + 2a + 1 a + a4/5 + 1
holds for all a > 0, then it suffices to show that
X 1
≥ 1,
a8/5 + a4/5 + 1
which follows immediately from the inequality in P 1.45. Therefore, using the
substitution x = a1/5 , x > 0, we need to show that

2x 5 + 3 3
≥ 8 ,
2x + 2x + 1
10 5 x + x4 + 1
which is equivalent to

2x 4 (x 5 − 3x 2 + x + 1) + x 4 − 4x + 3 ≥ 0.

This is true since, by the AM-GM inequality, we have


p
3
x 5 + x + 1 ≥ 3 x 5 · x · 1 = 3x 2

and p
4
x 4 + 3 = x 4 + 1 + 1 + 1 ≥ 4 x 4 · 1 · 1 · 1 = 4x.
The equality holds for a = b = c = 1.
Second Solution. Making the substitution
p p p
x= 16a2 + 9 − 4a, y= 16b2 + 9 − 4b, z = 16c 2 + 9 − 4c, x, y, z > 0,

which involves
9 − x2 9 − y2 9 − z2
a= , b= , c= ,
8x 8y 8z
we need to show that

(9 − x 2 )(9 − y 2 )(9 − z 2 ) = 512x yz

yields
x + y + z ≥ 3.
382 Vasile Cîrtoaje

Use the contradiction method. Assume that

x + y + z < 3,

and show that


(9 − x 2 )(9 − y 2 )(9 − z 2 ) > 512x yz.
According to the AM-GM inequality, we get
p p p
3 + x = 1 + 1 + 1 + x ≥ 4 x, 3 + y ≥ 4 4 y, 3 + z ≥ 4 z,
4 4

hence
p
(3 + x)(3 + y)(3 + z) ≥ 64 4 x yz.
Therefore, it suffices to prove that
p
(3 − x)(3 − y)(3 − z) > 8 x 3 y 3 z 3 .
4

Since  x + y + z 3
1> ≥ x yz,
3
we have

(3 − x)(3 − y)(3 − z) = 9(3 − x − y − z) + 3(x y + yz + z x) − x yz


> 3(x y + yz + z x) − x yz ≥ 9(x yz)2/3 − x yz
> 8(x yz)2/3 > 8(x yz)3/4 .

P 2.75. Let a, b, c be positive real numbers such that abc = 1. Prove that
p p p
25a2 + 144 + 25b2 + 144 + 25c 2 + 144 ≤ 5(a + b + c) + 24.

(Vasile Cîrtoaje, 2012)

First Solution. Since


p 144
25a2 + 144 − 5a = p ,
25a2 + 144 + 5a
the inequality is equivalent to
X 1 1
p ≤ .
25a2 + 144 + 5a 6
If the inequality
1 1
p ≤ p
25a + 144 + 5a 6 5a18/13 + 4
2
Symmetric Nonrational Inequalities 383

holds for all a > 0, then it suffices to show that


X 1
p ≤ 1,
5a18/13 + 4

which follows immediately from P 2.33. Using the substitution x = a1/13 , x > 0,
we only need to show that
p p
25x 26 + 144 + 5x 13 ≥ 6 5x 18 + 4.

By squaring, the inequality becomes


p
10x 13 ( 25x 26 + 144 + 5x 13 − 18x 5 ) ≥ 0.

This is true if
25x 26 + 144 ≥ (18x 5 − 5x 13 )2 ,
which is equivalent to
5x 18 + 4 ≥ 9x 10 .
By the AM-GM inequality, we have

5x 18 + 4 = x 18 + x 18 + x 18 + x 18 + x 18 + 1 + 1 + 1 + 1
p 9
≥ 9 x 18 · x 18 · x 18 · x 18 · x 18 · 1 · 1 · 1 · 1 = 9x 10 .

The equality holds for a = b = c = 1.


Second Solution. Making the substitution
p p p
8x = 25a2 + 144 − 5a, 8 y = 25b2 + 144 − 5b, 8z = 25c 2 + 144 − 5c,

which involves
9 − 4x 2 9 − 4 y2 9 − 4z 2 3
 ‹
a= , b= , c= , x, y, z ∈ 0, ,
5x 5y 5z 2

we need to show that

(9 − 4x 2 )(9 − 4 y 2 )(9 − 4z 2 ) = 125x yz

involves
x + y + z ≤ 3.
Use the contradiction method. Assume that

x + y + z > 3,

and show that


(9 − 4x 2 )(9 − 4 y 2 )(9 − 4z 2 ) < 125x yz.
384 Vasile Cîrtoaje

Since
12x 2 3( y + z − x)( y + z + 3x)
9 − 4x < 3(x + y + z) −
2
= ,
x + y +z x + y +z

it suffices to prove the homogeneous inequality

27AB ≤ 125x yz(x + y + z)3 ,

where
A = ( y + z − x)(z + x − y)(x + y − z),
B = ( y + z + 3x)(z + x + 3 y)(x + y + 3z).
Consider the nontrivial case A ≥ 0. By the AM-GM inequality, we have
125
B≤ (x + y + z)3 .
27
Therefore, it suffices to show that

A ≤ x yz,

which is a well known inequality (equivalent to Schur’s inequality of degree three).

P 2.76. If a, b are positive real numbers such that ab + bc + ca = 3, then


p p p
(a) a2 + 3 + b2 + 3 + b2 + 3 ≥ a + b + c + 3;
p p p p
(b) a + b + b + c + c + a ≥ 4(a + b + c) + 6.
(Lee Sang Hoon, 2007)

Solution. (a) First Solution (by Pham Thanh Hung). By squaring, the inequality
becomes XÆ
(b2 + 3)(c 2 + 3) ≥ 3(1 + a + b + c).
Since

(b2 + 3)(c 2 + 3) = (b + c)(b + a)(c + a)(c + b) = (b + c)2 (a2 + 3)


1
≥ (b + c)2 (a + 3)2 ,
4
we have
XÆ 1X 1€ X X Š
(b2 + 3)(c 2 + 3) ≥ (b + c)(a + 3) = 6 a+2 bc
2 2
= 3(a + b + c + 1).
Symmetric Nonrational Inequalities 385

The equality holds for a = b = c = 1.


Second Solution. Use the SOS method. Write the inequality as follows:
Æ Æ Æ
(a + b)(a + c) + (b + c)(b + a) + (c + a)(c + b) ≥ a + b + c + 3,
” Æ — X €p p Š2
2 a + b + c − 3(ab + bc + ca) ≥ a+b− a+c ,

1 X X (b − c)2
p (b − c)2 ≥ p p 2 ,
a+b+c+ 3(ab + bc + ca) a+b+ a+c
X Sa (b − c)2
p p 2 ≥ 0,
a+b+ a+c
where €p p Š2 Æ
Sa = a+b+ a+c −a−b−c− 3(ab + bc + ca).
The inequality is true since
Æ Æ
Sa = 3(a + b + c) + 2 (a + b)(a + c) − 3(ab + bc + ca)
Æ Æ
> 2 a2 + (ab + bc + ca) − 3(ab + bc + ca) > 0.
Third Solution. Use the substitution
p p p
x = a2 + 3 − a, y = b2 + 3 − b, z = c 2 + 3 − c, x, y, z > 0.

We need to show that


x + y + z ≥ 3.
We have
X X ”Æ — ”Æ —
yz = (b + a)(b + c) − b (c + a)(c + b) − c
X Æ X Æ X Æ X
= (b + c) (a + b)(a + c)− b (c + a)(c + b)− c (b + a)(b + c)+ bc
X Æ X Æ X Æ X
= (b + c) (a + b)(a + c)− c (a + b)(a + c)− b (a + c)(a + b)+ bc
X
= bc = 3.
Thus, we get Æ
x + y +z ≥ 3(x y + yz + z x) = 3.
(b) By squaring, we get the inequality in (a).
386 Vasile Cîrtoaje

P 2.77. If a, b, c are nonnegative real numbers such that a + b + c = 3, then


Æ Æ Æ
(5a2 + 3)(5b2 + 3) + (5b2 + 3)(5c 2 + 3) + (5c 2 + 3)(5a2 + 3) ≥ 24.

(Nguyen Van Quy, 2012)

Solution. Assume that

a ≥ b ≥ c, 1 ≤ a ≤ 3, b + c ≤ 2.

Using the notation

A = 5a2 + 3, B = 5b2 + 3, C = 5c 2 + 3,

we can write the inequality as follows:


p p p  p
A B + C + BC ≥ 24,
Ç p  p
A B + C + 2 BC ≥ 24 − BC.
p
Consider the nontrivial case BC < 24. The inequality is true if
p  p 2
A B + C + 2 BC ≥ 24 − BC ,

which is equivalent to
p 2
A(A + B + C + 48) ≥ A + 24 − BC .

Applying Lemma below for k = 5/3 and m = 4/15 yields


p
5 BC ≥ 25bc + 15 + 4(b − c)2 .

Therefore, it suffices to show that

25A(A + B + C + 48) ≥ [5A + 120 − 25bc − 15 − 4(b − c)2 ]2 ,

which is equivalent to

25(5a2 + 3)[5(a2 + b2 + c 2 ) + 57] ≥ [25a2 + 120 − 25bc − 4(b − c)2 ]2 .

Since

5(a2 + b2 + c 2 ) + 57 = 5a2 + 5(b + c)2 − 10bc + 57 = 2(5a2 − 15a + 51 − 5bc)

and

25a2 + 120 − 25bc − 4(b − c)2 = 25a2 + 120 − 4(b + c)2 − 9bc
= 3(7a2 + 8a + 28 − 3bc),
Symmetric Nonrational Inequalities 387

we need to show that

50(5a2 + 3)(5a2 − 15a + 51 − 5bc) ≥ 9(7a2 + 8a + 28 − 3bc)2 .

From bc ≤ (b + c)2 /4 and (a − b)(a − c) ≥ 0, we get


(3 − a)2
bc ≤ , bc ≥ a(b + c) − a2 = 3a − 2a2 .
4
Consider a fixed, a ≥ 1, and denote x = bc. So, we only need to prove that
f (x) ≥ 0 for
a2 − 6a + 9
3a − 2a2 ≤ x ≤ ,
4
where

f (x) = 50(5a2 + 3)(5a2 − 15a + 51 − 5x) − 9(7a2 + 8a + 28 − 3x)2 .


a − 6a + 9
 2 ‹
Since f is concave, it suffices to show that f (3a−2a ) ≥ 0 and f
2
≥ 0.
4
Indeed, we have

f (3a − 2a2 ) = 3(743a4 − 2422a3 + 2813a2 − 1332a + 198)


= 3(a − 1)2 [(a − 1)(743a − 193) + 5] ≥ 0,

a2 − 6a + 9 375
 ‹
f = (25a4 − 140a3 + 286a2 − 252a + 81)
4 16
375
= (a − 1)2 (5a − 9)2 ≥ 0.
16
Thus, the proof is completed. The equality holds for a = b = c = 1, and also for
a = 9/5 and b = c = 3/5 (or any cyclic permutation).
k
Lemma. Let b, c ≥ 0 such that b + c ≤ 2. If k > 0 and 0 ≤ m ≤ , then
2k + 2
Æ
(kb2 + 1)(kc 2 + 1) ≥ kbc + 1 + m(b − c)2 .

Proof. By squaring, the inequality becomes

(b − c)2 [k − 2m − 2kmbc − m2 (b − c)2 ] ≥ 0.

This is true since

k − 2m − 2kmbc − m2 (b − c)2 = k − 2m − 2m(k − 2m)bc − m2 (b + c)2


m(k − 2m)
≥ k − 2m − (b + c)2 − m2 (b + c)2
2
km
= k − 2m − (b + c)2 ≥ k − 2m − 2km ≥ 0.
2
388 Vasile Cîrtoaje

P 2.78. If a, b, c are nonnegative real numbers such that a + b + c = 3, then


v
p p p t 4(a2 + b2 + c 2 ) + 42
a2 + 1 + b2 + 1 + c 2 + 1 ≥ .
3
(Vasile Cîrtoaje, 2014)

Solution. Assume that

a ≥ b ≥ c, a ≥ 1, b + c ≤ 2.

By squaring, the inequality becomes


p p p  p a2 + b2 + c 2 + 33
A B + C + BC ≥ ,
6
p  p a2 + b2 + c 2 + 33
Ç
A B + C + 2 BC + BC ≥ ,
6
where
A = a2 + 1, B = b2 + 1, C = c 2 + 1.
Applying Lemma from the preceding problem P 2.77 for k = 1 and m = 1/4 gives
p 1
BC ≥ bc + 1 + (b − c)2 .
4
Therefore, it suffices to show that
v •
1 1 a2 + b2 + c 2 + 33
t ˜
A B + C + 2bc + 2 + (b − c) + bc + 1 + (b − c) ≥
2 2
,
2 4 6
which is equivalent to
Æ
6 2(a2 + 1)[3(b + c)2 + 8 − 4bc] ≥ 2a2 − (b + c)2 + 54 − 4bc,
Æ
6 2(a2 + 1)(3a2 − 18a + 35 − 4bc) ≥ a2 + 6a + 45 − 4bc.
From bc ≤ (b + c)2 /4 and (a − b)(a − c) ≥ 0, we get

(3 − a)2
bc ≤ , bc ≥ a(b + c) − a2 = 3a − 2a2 .
4
Consider a fixed, a ≥ 1, and denote x = bc. So, we only need to prove that
f (x) ≥ 0 for
a2 − 6a + 9
3a − 2a2 ≤ x ≤ ,
4
where

f (x) = 72(a2 + 1)(3a2 − 18a + 35 − 4x) − (a2 + 6a + 45 − 4x)2 .


Symmetric Nonrational Inequalities 389

a2 − 6a + 9
 ‹
Since f is concave, it suffices to show that f (3a−2a2 ) ≥ 0 and f ≥ 0.
4
Indeed,

f (3a − 2a2 ) = 9(79a4 − 228a3 + 274a2 − 180a + 55)


= 9(a − 1)2 (79a2 − 70a + 55 ≥ 0,

a2 − 6a + 9
 ‹
f = 144(a4 − 6a3 + 13a2 − 12a + 4)
4
= 144(a − 1)2 (a − 2)2 ≥ 0.

The equality holds for a = b = c = 1, and also for a = 2 and b = c = 1/2 (or any
cyclic permutation).

P 2.79. If a, b, c are nonnegative real numbers such that a + b + c = 3, then


p p p p
(a) a2 + 3 + b2 + 3 + c 2 + 3 ≥ 2(a2 + b2 + c 2 ) + 30;
p p p p
(b) 3a2 + 1 + 3b2 + 1 + 3c 2 + 1 ≥ 2(a2 + b2 + c 2 ) + 30.
(Vasile Cîrtoaje, 2014)

Solution. Assume that

a ≥ b ≥ c, a ≥ 1, b + c ≤ 2.

(a) By squaring, the inequality becomes


p p p  p a2 + b2 + c 2 + 21
A B + C + BC ≥ ,
2
p  p a2 + b2 + c 2 + 21
Ç
A B + C + 2 BC + BC ≥ ,
2
where
A = a2 + 3, B = b2 + 3, C = c 2 + 3.
Applying Lemma from problem P 2.77 for k = 1/3 and m = 1/9 gives
p 1
BC ≥ bc + 3 + (b − c)2 .
3
Therefore, it suffices to show that
v •
2 1 a2 + b2 + c 2 + 21
t ˜
A B + C + 2bc + 6 + (b − c) + bc + 3 + (b − c)2 ≥
2 ,
3 3 2
390 Vasile Cîrtoaje

which is equivalent to
Æ
2 3(a2 + 3)[5(b + c)2 + 36 − 8bc] ≥ 3a2 + (b + c)2 + 45 − 4bc,
Æ
3(a2 + 3)(5a2 − 30a + 81 − 8bc) ≥ 2a2 − 3a + 27 − 2bc.
From bc ≤ (b + c)2 /4 and (a − b)(a − c) ≥ 0, we get

(3 − a)2
bc ≤ , bc ≥ a(b + c) − a2 = 3a − 2a2 .
4

Consider a fixed, a ≥ 1, and denote x = bc. So, we only need to prove that
f (x) ≥ 0 for
a2 − 6a + 9
3a − 2a2 ≤ x ≤ ,
4
where

f (x) = 3(a2 + 3)(5a2 − 30a + 81 − 8x) − (2a2 − 3a + 27 − 2x)2 .

a2 − 6a + 9
 ‹
Since f is concave, it suffices to show that f (3a−2a ) ≥ 0 and f
2
≥ 0.
4
Indeed,
f (3a − 2a2 ) = 27a2 (a − 1)2 ≥ 0,

a2 − 6a + 9 27 4
 ‹
f = (a − 8a3 + 22a2 − 24a + 9)
4 4
27
= (a − 1)2 (a − 3)2 ≥ 0.
4

The equality holds for a = b = c = 1, and also for a = 3 and b = c = 0 (or any
cyclic permutation).
(b) By squaring, the inequality becomes

p p p  p 27 − a2 − b2 − c 2
A B + C + BC ≥ ,
2
Ç p  p 27 − a2 − b2 − c 2
A B + C + 2 BC + BC ≥ ,
2
where
A = 3a2 + 1, B = 3b2 + 1, C = 3c 2 + 1.
Applying Lemma from problem P 2.77 for k = 3 and m = 1/3 gives
p 1
BC ≥ 3bc + 1 + (b − c)2 .
3
Symmetric Nonrational Inequalities 391

Therefore, it suffices to show that


v •
2 1 27 − a2 − b2 − c 2
t ˜
A B + C + 6bc + 2 + (b − c) + 3bc + 1 + (b − c)2 ≥
2 ,
3 3 2

which is equivalent to
Æ
2 3(3a2 + 1)[11(b + c)2 + 12 − 8bc] ≥ 75 − 3a2 − 5(b + c)2 − 4bc,
Æ
3(3a2 + 1)(11a2 − 66a + 111 − 8bc) ≥ 15 + 15a − 4a2 − 2bc.
From bc ≤ (b + c)2 /4 and (a − b)(a − c) ≥ 0, we get

(3 − a)2
bc ≤ , bc ≥ a(b + c) − a2 = 3a − 2a2 .
4
Consider a fixed, a ≥ 1, and denote x = bc. So, we only need to prove that
f (x) ≥ 0 for
2 a2 − 6a + 9
3a − 2a ≤ x ≤ ,
4
where

f (x) = 3(3a2 + 1)(11a2 − 66a + 111 − 8x) − (15 + 15a − 4a2 − 2x)2 .

a − 6a + 9
 2 ‹
Since f is concave, it suffices to show that f (3a−2a ) ≥ 0 and f
2
≥ 0.
4
Indeed,
f (3a − 2a2 ) = 27(a − 1)2 (3a − 2)2 ≥ 0,

a2 − 6a + 9 27
 ‹
f = (9a4 − 48a3 + 94a2 − 80a + 25)
4 4
27
= (a − 1)2 (3a − 5)2 ≥ 0.
4
The equality holds for a = b = c = 1, and also for a = 5/3 and b = c = 2/3 (or any
cyclic permutation).
Remark. Similarly, we can prove the following generalization.
• Let a, b, c be nonnegative real numbers such that a + b + c = 3. If k > 0, then
v
p p p t 8k(a2 + b2 + c 2 ) + 3(9k2 + 10k + 9)
ka2 + 1 + kb2 + 1 + kc 2 + 1 ≥ ,
3(k + 1)

3k + 1 3k − 1
with equality for a = b = c = 1, and also for a = and b = c = (or any
2k 4k
cyclic permutation).
392 Vasile Cîrtoaje

P 2.80. If a, b, c are nonnegative real numbers such that a + b + c = 3, then


Æ Æ Æ
(32a2 + 3)(32b2 + 3) + (32b2 + 3)(32c 2 + 3) + (32c 2 + 3)(32a2 + 3) ≤ 105.

(Vasile Cîrtoaje, 2014)

Solution. Assume that

a ≤ b ≤ c, a ≤ 1, b + c ≥ 2.

Denote
A = 32a2 + 3, B = 32b2 + 3, C = 32c 2 + 3,
and write the inequality as follows:
p p p  p
A B + C + BC ≤ 105,
p Æ p p
A · B + C + 2 BC ≤ 105 − BC.
By Lemma below, we have
p
BC ≤ 5(b + c)2 + 12bc + 3 ≤ 8(b + c)2 + 3 ≤ 8(a + b + c)2 + 3 = 75 < 105.

Therefore, we can write the desired inequality as


p  p 2
A B + C + 2 BC ≤ 105 − BC ,

which is equivalent to
p
A(A + B + C + 210) ≤ (A + 105 − BC)2 .

According to Lemma below, it suffices to show that

A(A + B + C + 210) ≤ [A + 105 − 5(b2 + c 2 ) − 22bc − 3]2 ,

which is equivalent to

[32a2 + 105 − 5(b2 + c 2 ) − 22bc]2 ≥ (32a2 + 3)[32(a2 + b2 + c 2 ) + 219].

Since

32(a2 + b2 + c 2 )+219 = 32a2 +32(b + c)2 −64bc +219 = 64a2 −192a +507−64bc

and

32a2 +105−5(b2 +c 2 )−22bc = 32a2 +105−5(b+c)2 −12bc = 3(9a2 +10a+20−4bc),

we need to show that

9(9a2 + 10a + 20 − 4bc)2 ≥ (32a2 + 3)(64a2 − 192a + 507 − 64bc).


Symmetric Nonrational Inequalities 393

From bc ≤ (b + c)2 /4, we get

(3 − a)2
bc ≤ .
4
Consider a fixed, 0 ≤ a ≤ 1, and denote x = bc. So, we only need to prove that
f (x) ≥ 0 for
a2 − 6a + 9
0≤ x ≤ ,
4
where

f (x) = 9(9a2 + 10a + 20 − 4x)2 − (32a2 + 3)(64a2 − 192a + 507 − 64x).

Since

f 0 (x) = 72(4x − 9a2 − 10a − 20) + 64(32a2 + 3)


≤ 72[(a2 − 6a + 9) − 9a2 − 10a − 20) + 64(32a2 + 3)
= 8[184a(a − 1) + (44a − 75)] < 0,

a2 − 6a + 9
 ‹
f is decreasing, hence f (x) ≥ f . Therefore, it suffices to show that
4
a − 6a + 9
 2 ‹
f ≥ 0. We have
4
a − 6a + 9
 2 ‹
f =9[9a2 + 10a + 20 − (a2 − 6a + 9)]2
4
− (32a2 + 3)[64a2 − 192a + 507 − 16(a2 − 6a + 9)]
=9(8a2 + 16a + 11)2 − (32a2 + 3)(48a2 − 96a + 363)
=192a(a − 1)2 (18 − 5a) ≥ 0.

Thus, the proof is completed. The equality holds for a = b = c = 1, and also for
a = 0 and b = c = 3/2 (or any cyclic permutation).
Lemma. If b, c ≥ 0 such that b + c ≥ 2, then
Æ
(32b2 + 3)(32c 2 + 3) ≤ 5(b2 + c 2 ) + 22bc + 3.

Proof. By squaring, the inequality becomes

(5b2 + 5c 2 + 22bc)2 − 322 b2 c 2 ≥ 96(b2 + c 2 ) − 6(5b2 + 5c 2 + 22bc),

5(b − c)2 (5b2 + 5c 2 + 54bc) ≥ 66(b − c)2 .


It suffices to show that
5(5b2 + 5c 2 + 10bc) ≥ 100,
which is equivalent to the obvious inequality (b + c)2 ≥ 4.
394 Vasile Cîrtoaje

P 2.81. If a, b, c are positive real numbers, then

b+c c+a a+b


−3 + −3 + − 3 ≥ 2.
a b c
(Vasile Cîrtoaje, 2012)
Solution. Without loss of generality, assume that a ≥ b ≥ c.
Case 1: a > b + c. We have
b+c a+b c+a b+c b+c
−3 + −3 + −3 ≥ −3 =3− > 2.
a c b a a
Case 2: a ≤ b + c. We have
b+c a+b c+a b+c c+a
−3 + −3 + −3 ≥ −3 + −3
a c b a b

b+c c + a b+b b+a (a − b)(2b − a)


 ‹ 
= 3− + 3− ≥6− − =2+ ≥ 2.
a b a b ab
a
Thus, the proof is completed. The equality holds for = b = c (or any cyclic
2
permutation).

P 2.82. If a, b, c are real numbers such that abc 6= 0, then

b+c c+a a+b


+ + ≥ 2.
a b c

First Solution. Let


|a| = max{|a|, |b|, |c|}.
We have
b+c c+a a+b b+c c+a a+b
+ + ≥ + +
a b c a a a
|(−b − c) + (c + a) + (a + b)|
≥ = 2.
|a|
The equality holds for a = 1, b = −1 and |c| ≤ 1 (or any permutation).
Second Solution. Since the inequality remains unchanged by replacing a, b, c with
−a, −b, −c, it suffices to consider two cases: a, b, c > 0, and a < 0, b, c > 0.
Case 1: a, b, c > 0. We have

b+c c+a a+b


 ‹  ‹ 
a b b c c a
+ + = + + + + + ≥ 6.
a b c b a c b a c
Symmetric Nonrational Inequalities 395

Case 2: a < 0 and b, c > 0. Replacing a by −a, we need to show that

b + c |a − c| |a − b|
+ + ≥2
a b c
for all a, b, c > 0. Without loss of generality, assume that b ≥ c. There are three
case to consider: b ≥ c ≥ a, b ≥ a ≥ c and a ≥ b ≥ c.
For b ≥ c ≥ a, we have

b + c |a − c| |a − b| b+c
+ + ≥ ≥ 2.
a b c a
For b ≥ a ≥ c, we have

b + c |a − c| |a − b| b+c a−c (a − b)2 + c(b − a)


+ + −2≥ + −2= ≥ 0.
a b c a b ab
For a ≥ b ≥ c, we have

b + c |a − c| |a − b| b+c a−c a−b


+ + −2= + + −2
a b c a b c
a−b 1 1 (a − b)2 (a − b)(ab − c 2 )
 ‹  ‹
a b
= + −2 + +c − = + ≥ 0.
b a c a b ab abc

Third Solution. Using the substitution

b+c c+a a+b


x= , y= , z= ,
a b c
we need to show that

x + y + z + 2 = x yz, x, y, z ∈ R,

involves
|x| + | y| + |z| ≥ 2.
If x yz ≤ 0, then
−x − y − z = 2 − x yz ≥ 2,
hence
|x| + | y| + |z| ≥ |x + y + z| = | − x − y − z| ≥ −x − y − z ≥ 2.
If x yz > 0, then either x, y, z > 0 or only one of x, y, z is positive (for instance,
x > 0 and y, z < 0).
Case 1: x, y, z > 0. We need to show that x + y + z ≥ 2. We have

x yz = x + y + z + 2 > 2
396 Vasile Cîrtoaje

and, by the AM-GM inequality, we get


p p
3
x + y + z ≥ 3 3 x yz > 3 2 > 2,

Case 2: x > 0 and y, z < 0. Replacing y, z by − y, −z, we need to prove that

x − y − z + 2 = x yz

involves
x + y +z ≥2
for all x, y, z > 0. Since

x + y + z − 2 = x + y + z − (x yz − x + y + z) = x(2 − yz),

we need to show that yz ≤ 2. Indeed, we have


p
x + 2 = y + z + x yz ≥ 2 yz + x yz,
p
x(1 − yz) + 2(1 − yz ) ≥ 0,
p p
(1 − yz ) [x(1 + yz ) + 2] ≥ 0,
hence
yz ≤ 1 < 2.

P 2.83. Let a, b, c be nonnegative real numbers, no two of which are zero, and let

2a 2b 2c
x= , y= , z= .
b+c c+a a+b

Prove that
p p p
(a) xy+ yz + z x ≥ x yz + 2;
p p p
(b) x + y +z+ xy+ yz + z x ≥ 6;
p p p
x+ y+ z ≥ 8 + x yz;
p
(c)
p p p
yz zx xy
(d) + + ≥ 1.
x +2 y +2 z+2
Symmetric Nonrational Inequalities 397

Solution. (a) Since


p p p
p 2 bc(a + b)(c + a) 2 bc (a + bc)
yz = ≥
(a + b)(c + a) (a + b)(c + a)
p
2a(b + c) bc + 2bc(b + c) 4abc + 2bc(b + c)
= ≥ ,
(a + b)(b + c)(c + a) (a + b)(b + c)(c + a)

we have

12abc + 2 bc(b + c)
P
Xp
yz ≥
(a + b)(b + c)(c + a)
8abc
= + 2 = x yz + 2.
(a + b)(b + c)(c + a)

The equality holds for a = b = c, and also for a = 0 or b = 0 or c = 0.


(b) First Solution. Taking into account the inequality (a), it suffices to show
that
x + y + z + x yz ≥ 4,
which is equivalent to Schur’s inequality of degree three
X
a3 + b3 + c 3 + 3abc ≥ ab(a + b).

The equality holds for a = b = c, and also for a = 0 and b = c (or any cyclic
permutation).
Second Solution. We use the SOS technique. Write the inequality as
X X p p 2
4 (x − 1) ≥ y− z .

Since
X X (a − b) + (a − c) X a − b X b − a
(x − 1) = = +
b+c b+c c+a
X (a − b)2 X (b − c)2
= =
(b + c)(c + a) (a + b)(a + c)

and

p p 2 ( y − z)2 2(b − c)2 (a + b + c)2


y− z = p p 2 = p p 2 ,
y+ z (a + b)(a + c) b2 + ab + c 2 + ac

we can write the inequality as


X
(b − c)2 Sa ≥ 0,
398 Vasile Cîrtoaje

where  
(a + b + c)2
Sa = (b + c) 2 − p p 2  .
b + ab + c + ac
2 2

By Minkowski’s inequality, we have


€p p Š2 €p p Š2
b2 + ab + c 2 + ac ≥ (b + c)2 + a b+ c
≥ (b + c)2 + a(b + c) = (b + c)(a + b + c),

hence
a+b+c
 ‹
Sa ≥ (b + c) 2 − = b + c − a.
b+c
Thus, it suffices to show that
X
(b − c)2 (b + c − a) ≥ 0,

which is just Schur’s inequality of third degree.


Third Solution. Using the Cauchy-Schwarz inequality yields

a b c (a + b + c)2 (a + b + c)2
+ + ≥ = .
b+c c+a a+b a(b + c) + b(c + a) + c(a + b) 2(ab + bc + ca)
Also, using Hölder’s inequality, we have
‚s v Œ2
(a + b + c)3
s
a t b c
+ + ≥ 2 .
b+c c+a a+b a (b + c) + b2 (c + a) + c 2 (a + b)

Thus, it suffices to prove that

(a + b + c)2 2(a + b + c)3


+ 2 ≥ 12.
ab + bc + ca a (b + c) + b2 (c + a) + c 2 (a + b)
Due to homogeneity, we may assume that a + b + c = 1. Substituting

q = ab + bc + ca, 3q ≤ 1,

the inequality becomes


1 2
+ ≥ 12.
q q − 3abc
The fourth degree Schur’s inequality

6abcp ≥ (p2 − q)(4q − p2 ), p = a + b + c,

gives
6abc ≥ (1 − q)(4q − 1).
Symmetric Nonrational Inequalities 399

Therefore,

1 2 1 4 (1 − 3q)(1 − 4q)2
+ − 12 ≥ + − 12 = ≥ 0.
q q − 3abc q 2q − (1 − q)(4q − 1) q(4q2 − 3q + 1)

(c) By squaring, the inequality becomes


p p p
x + y + z + 2 x y + 2 yz + 2 z x ≥ 8 + x yz.

Based on the inequality in (a), it suffices to show that

x + y + z + 2(x yz + 2) ≥ 8 + x yz,

which is equivalent to
x + y + z + x yz ≥ 4,
X
a3 + b3 + c 3 + 3abc ≥ ab(a + b).
The last form is just Schur’s inequality of third degree. The equality holds for a =
b = c, and also for a = 0 and b = c (or any cyclic permutation).
(d) Write the inequality as
X p
(b + c) yz ≥ 2(a + b + c).

First Solution. Since


p p p
p 2 bc(a + b)(c + a) 2 bc (a + bc)
yz = ≥
(a + b)(c + a) (a + b)(c + a)
p
2a(b + c) bc + 2bc(b + c) 4abc + 2bc(b + c)
= ≥ ,
(a + b)(b + c)(c + a) (a + b)(b + c)(c + a)
it suffices to show that
X
(b + c)[2abc + bc(b + c)] ≥ (a + b + c)(a + b)(b + c)(c + a),

which is an identity. The equality holds for a = b = c, and also for a = 0 or b − 0


or c = 0.
Second Solution. Let
q = ab + bc + ca.
Since v 2b 2c
p t 2b 2c 2 · a+b · c+a 4bc
yz = · ≥ 2b = ,
a+b c+a a+b + 2c
c+a
bc + q
we can write the inequality as follows:
X 2bc(b + c)
≥ a + b + c,
bc + q
400 Vasile Cîrtoaje

X • 2bc(b + c) ˜
− a ≥ 0,
bc + q
X bc(b − a) + bc(c − a) + b(c 2 − a2 ) + c(b2 − a2 )
≥ 0,
bc + q
X c(b − a)(2b + a) + b(c − a)(2c + a)
≥ 0,
bc + q
X c(b − a)(2b + a) X c(a − b)(2a + b)
+ ≥ 0,
bc + q ca + q
2a + b 2b + a
X • ˜
c(a − b) − ≥ 0,
ca + q bc + q
X c(a − b)[q(a − b) − c(a2 − b2 )]
≥ 0,
(ca + q)(bc + q)
X (a − b)2
abc ≥ 0.
(ca + q)(bc + q)

P 2.84. Let a, b, c be nonnegative real numbers, no two of which are zero, and let
2a 2b 2c
x= , y= , z= .
b+c c+a a+b
Prove that p p p
1 + 24x + 1 + 24 y + 1 + 24z ≥ 15.
(Vasile Cîrtoaje, 2005)

Solution (by Vo Quoc Ba Can). Assume that c = min{a, b, c}, hence z ≤ 1. By


Hölder’s inequality
‚s v Œ2
a t b  2 
+ a (b + c) + b2 (c + a) ≥ (a + b)3 ,
b+c c+a

we get
p p 2 2(a + b)3 2(a + b)3
x+ y ≥ =
c(a2 + b2 ) + ab(a + b) c(a + b)2 + ab(a + b − 2c)
2(a + b)3 8(a + b) 8
≥ = = .
c(a + b)2 + 4 (a + b)2 (a + b − 2c) a + b + 2c
1
1+z

Using this result and Minkowski’s inequality, we have


v
p p q p p t 48
1 + 24x + 1 + 24 y ≥ (1 + 1)2 + 24( x + y)2 ≥ 2 1 + .
1+z
Symmetric Nonrational Inequalities 401

Therefore, it suffices to show that


v
t 48 p
2 1+ + 1 + 24z ≥ 15.
1+z
Using the substitution
p 1
1 + 24z = 5t, ≤ t ≤ 1,
5
the inequality turns into v
t t 2 + 47
2 ≥ 3 − t.
25t 2 + 23
By squaring, this inequality becomes

25t 4 − 150t 3 + 244t 2 − 138t + 19 ≤ 0,

which is equivalent to the obvious inequality

(t − 1)2 (5t − 1)(5t − 19) ≤ 0.

The equality holds for a = b = c, and also for a = b and c = 0 (or any cyclic
permutation).

P 2.85. If a, b, c are positive real numbers, then


v v v
t 7a t 7b t 7c
+ + ≤ 3.
a + 3b + 3c b + 3c + 3a c + 3a + 3b
(Vasile Cîrtoaje, 2005)

First Solution. Using the substitution


v v v
t 7a t 7b t 7c
x= , y= , z= ,
a + 3b + 3c b + 3c + 3a c + 3a + 3b
we have
(x − 7)a + 3x 2 b + 3x 2 c = 0
 2



3 y 2 a + ( y 2 − 7)b + 3 y 2 c = 0 ,


 3z a + 3z b + (z − 7)c = 0

 2 2 2

which involves
x 2 − 7 3x 2 3x 2
3 y2 y2 − 7 3 y2 =0 ;
2
3z 3z 2 z2 − 7
402 Vasile Cîrtoaje

that is,
F (x, y, z) = 0,
where X X
F (x, y, z) = 4x 2 y 2 z 2 + 8 x2 y2 + 7 x 2 − 49.
We need to show that F (x, y, z) = 0 involves x + y + z ≤ 3, where x, y, z > 0. To
do this, we use the contradiction method. Assume that x + y + z > 3 and show that
F (x, y, z) > 0. Since F (x, y, z) is strictly increasing in each of its arguments, it is
enough to prove that x + y + z = 3 involves F (x, y, z) ≥ 0. We will use the mixing
variables technique. Assume that x = max{x, y, z} and denote
y +z
t= , 0 < t ≤ 1 ≤ x.
2
We will show that
F (x, y, z) ≥ F (x, t, t) ≥ 0.
We have

F (x, y, z) − F (x, t, t) = (8x 2 + 7)( y 2 + z 2 − 2t 2 ) − 4(x 2 + 2)(t 4 − y 2 z 2 )


1
= (8x 2 + 7)( y − z)2 − (x 2 + 2)(t 2 + yz)( y − z)2
2
1
≥ (8x 2 + 7)( y − z)2 − 2(x 2 + 2)t 2 ( y − z)2
2
1
= (4x 2 − 1)( y − z)2 ≥ 0
2
and
3− x 3− x 1
 ‹
F (x, t, t) = F x, , = (x − 1)2 (x − 2)2 (x 2 − 6x + 23) ≥ 0.
2 2 4
a
The equality holds for a = b = c, and also for = b = c (or any cyclic permuta-
8
tion).
Second Solution. Due to homogeneity, we may assume that a + b + c = 3, when
the inequality becomes v
X t 7a
≤ 3.
9 − 2a
Using the substitution
v v v
t 7a t 7b t 7c
x= , y= , z= ,
9 − 2a 9 − 2b 9 − 2c
we need to show that if x, y, z are positive real numbers such that
X 1 1
= ,
2x 2 +7 3
Symmetric Nonrational Inequalities 403

then
x + y + z ≤ 3.
For the sake of contradiction, assume that x + y +z > 3 and show that F (x, y, z) < 0,
where X 1 1
F (x, y, z) = − .
2x + 7 3
2

Since F (x, y, z) is strictly decreasing in each of its arguments, it is enough to prove


that x + y + z = 3 involves F (x, y, z) ≤ 0. This is just the inequality in P 1.33.

P 2.86. If a, b, c are positive real numbers such that a + b + c = 3, then


Æ3
Æ
3
Æ
3
p
3
a2 (b2 + c 2 ) + b2 (c 2 + a2 ) + c 2 (a2 + b2 ) ≤ 3 2.
(Michael Rozenberg, 2013)
Solution. By Hölder’s inequality, we have
”X Æ 3
—3 ”X —2 X b 2 + c 2
a2 (b2 + c 2 ) ≤ a(b + c) · .
(b + c)2
Therefore, it suffices to show that
X b2 + c 2 27
≤ ,
(b + c)2 2(ab + bc + ca)2
which is equivalent to the homogeneous inequalities
X • b2 + c 2 ˜
p4
− 1 ≤ − 3,
(b + c)2 6q2
X 2bc p4
+ ≥ 3,
(b + c)2 6q2
where
p = a + b + c, q = ab + bc + ca.
According to P 1.62, the following inequality holds
X 2bc p2 9
+ ≥ .
(b + c)2 q 2
Thus, it is enough to show that
9 p2 p4
− + 2 ≥ 3,
2 q 6q
which is equivalent to
 2
p2
−3 ≥ 0.
q
The equality holds for a = b = c = 1.
404 Vasile Cîrtoaje

P 2.87. If a, b, c are nonnegative real numbers, no two of which are zero, then
1 1 1 1 2
+ + ≥ +p .
a+b b+c c+a a+b+c ab + bc + ca
(Vasile Cîrtoaje, 2005)
Solution. Using the notation

p = a + b + c, q = ab + bc + ca, r = abc,

we can write the inequality as

p2 + q 1 2
≥ +p .
pq − r p q

According to P 3.57-(a) in Volume 1, for fixed p and q, the product r = abc is


minimum when two of a, b, c are equal or one of a, b, c is zero. Therefore, it suffices
to prove the inequality for b = c = 1 and for a = 0. For a = 0, the inequality reduces
to
1 1 2
+ ≥p ,
b c bc
which is obvious. For b = c = 1, the inequality becomes as follows:
1 2 1 2
+ ≥ +p ,
2 a+1 a+2 2a + 1
1 1 2 2
− ≥p − ,
2 a+2 2a + 1 a + 1
p 
a 2 a + 1 − 2a + 1
≥ p ,
2(a + 2) (a + 1) 2a + 1
a 2a2
≥ p p .
2(a + 2) (a + 1) 2a + 1 a + 1 + 2a + 1
So, we need to show that
1 2a
≥ p p .
2(a + 2) (a + 1) 2a + 1 a + 1 + 2a + 1

Consider two cases: 0 ≤ a ≤ 1 and a > 1.


Case 1: 0 ≤ a ≤ 1. Since
p p p p p 
2a + 1(a + 1 + 2a + 1) ≥ 2a + 1 2a + 1 + 2a + 1 = 2(2a + 1),

it suffices to prove that


1 a
≥ ,
2(a + 2) (a + 1)(2a + 1)
Symmetric Nonrational Inequalities 405

which is equivalent to 1 − a ≥ 0.
Case 2: a > 1. Write the desired inequality as

1 2a
≥ p .
2(a + 2) (a + 1) (a + 1) 2a + 1 + 2a + 1


First, we will show that p


(a + 1) 2a + 1 > 3a.
Indeed, by squaring, we get the obvious inequality

a3 + a(a − 2)2 + 1 > 0.

Therefore, it suffices to show that


1 2a
≥ ,
2(a + 2) (a + 1)(3a + 2a + 1)

which is equivalent to (a − 1)2 ≥ 0.


The equality holds for a = 0 and b = c (or any cyclic permutation).

P 2.88. If a, b ≥ 1, then

1 1 1 1
p + ≥p +p .
3ab + 1 2 3a + 1 3b + 1

Solution. Using the substitution

2 2
x=p , y=p , x, y ∈ (0, 1],
3a + 1 3b + 1
the desired inequality can be written as
v
t 3
xy ≥ x + y − 1.
x y − x2 − y2 + 4
2 2

Consider the nontrivial case x + y − 1 ≥ 0, and denote

t = x + y − 1, p = x y.

We have
1 ≥ p ≥ t ≥ 0.
Since
x 2 + y 2 = (x + y)2 − 2x y = (t + 1)2 − 2p,
406 Vasile Cîrtoaje

we need to prove that


v
t 3
p ≥ t.
p2 + 2p − t 2 − 2t + 3

By squaring, we get the inequality

(p − t)[(3 − t 2 )p + t(1 − t)(3 + t)] ≥ 0,

which is clearly true. The equality holds for a = b = 1.

1
P 2.89. Let a, b, c be positive real numbers such that a + b + c = 3. If k ≥ p , then
2

(abc)k (a2 + b2 + c 2 ) ≤ 3.

(Vasile Cîrtoaje, 2006)

Solution. Since ‹3
a+b+c

abc ≤ = 1,
3
p
it suffices to prove the desired inequality for k = 1/ 2. Write the inequality in the
homogeneous form
‹3k+2
a+b+c

(abc) (a + b + c ) ≤ 3
k 2 2 2
.
3
According to P 3.57-(a) in Volume 1, for fixed a+ b+c and ab+ bc +ca, the product
a bc is maximum when two of a, b, c are equal. Therefore, it suffices to prove the
homogeneous inequality for b = c = 1; that is, f (a) ≥ 0, where

f (a) = (3k + 2) ln(a + 2) − (3k + 1) ln 3 − k ln a − ln(a2 + 2).

From
3k + 2 k 2a 2(a − 1)(ka2 − 2a + 2k)
f (a) =
0
− − 2 =
a+2 a a +2 a(a + 2)(a2 + 2)
p p 2
2(a − 1)(a − 2)
= ,
a(a + 2)(a2 + 2)

it follows that f is decreasing on (0, 1] and increasing on [1, ∞); therefore, f (a) ≥
f (1) = 0. This completes the proof. The equality holds for a = b = c = 1.
Symmetric Nonrational Inequalities 407

P 2.90. If a, b, c ∈ [0, 4] and ab + bc + ca = 4, then


p p p p
a+b+ b+c+ c+a ≤3+ 5.

(Vasile Cîrtoaje, 2019)

Solution. Assume that a ≥ b ≥ c, 1 ≤ a ≤ 4, and write the inequality as follows


p Ç Æ p
b+c+ (a + b) + (a + c) + 2 (a + b)(a + c) ≤ 3 + 5,
p q p p
b+c+ 2a + b + c + 2 a2 + 4 ≤ 3 + 5,
From 4 − a(b + c) = bc ≥ 0, we get

4
b+c ≤ .
a
Thus, it suffices to show that
v
2 t 4 p p
p + 2a + + 2 a2 + 4 ≤ 3 + 5,
a a

which is equivalent to
p
2 a + a2 + 4 p
p + p ≤ 3 + 5,
a a
p p p
a − 3 a + 2 ≤ 5a − a2 + 4,
p p (a − 1)(4 − a)
( a − 1)( a − 2) ≤ p p .
5a + a2 + 4
This is true if p p
( a + 1)( a + 2)
1≤ p p ,
5a + a2 + 4
that can be written in the obvious form
p p p
(a + 2 − a2 + 4) + (3 − 5) a ≥ 0.

The equality occurs for a = 4, b = 1 and c = 0 (or any permutation).

P 2.91. Let v
t a2 + b2 + c 2 a+b+c
F (a, b, c) = − ,
3 3
408 Vasile Cîrtoaje

where a, b, c are positive real numbers such that

a4 bc ≥ 1, a ≤ b ≤ c.

Then,
1 1 1
‹ 
F (a, b, c) ≥ F , , .
a b c

(Vasile Cîrtoaje and Vasile Mircea Popa, 2020)

Solution. Write the inequality as E(a, b, c) ≥ 0, where


v 
1 1 1 1 1 1
Æ t ‹
E(a, b, c) = 3(a + b + c ) − (a + b + c) − 3 2 + 2 + 2 + + + ,
2 2 2
a b c a b c

and show that


E(a, b, c) ≥ E(a, x, x) ≥ 0,
where p
x= bc ≥ a, a2 x ≥ 1, x ≥ 1.
Write the inequality E(a, b, c) ≥ E(a, x, x) it in the form

A − C ≥ B − D,

where
Æ Æ 3(b − c)2
A= 3(a2 + b2 + c 2 ) − 3(a2 + 2x 2 ) = p p
3(a2 + b2 + c 2 ) + 3(a2 + 2x 2 )

3(b − c)2
≥p ,
3(x 2 + b2 + c 2 ) + 3x
€p p Š2
B = (a + b + c) − (a + 2x) = b− c ,
v  ‹ v
1 1 1 t 1 2
t ‹
C= 3 2+ 2+ 2 − 3 2+ 2
a b c a x
3 (b − c)2
= 4·q  q
x 1 1
+ c12 + 3 1 2

3 a2 + b2 a2 + x2

3 (b − c)2 3 (b − c)2
≤ 4·q = 2·p ,
x 1 1 1 3 x 3(x 2 + c 2 + b2 ) + 3x

3 x2 + b2 + c2 + x
p p 2
1 1 1 1 2 b− c
D= + + − − = .
a b c a x x2
Symmetric Nonrational Inequalities 409

Thus, we need to show that


 
€p p Š2 1 1 1 x2 − 1
3 b+ c − 2·p ≥ .
x2
p
3(x 2 + b2 + c 2 ) + 3x x 3(x 2 + c 2 + b2 ) + 3x
This inequality is true if
€p p Š2 Æ
3 b + c ≥ 3(x 2 + b2 + c 2 ) + 3x,

that is equivalent to
p € p Š p
3 b + c + bc ≥ bc + b2 + c 2 ,

which is true.
Write now the inequality E(a, x, x) ≥ 0 in the form
v 
1 2 1 2
Æ t ‹
3(a + 2x ) − (a + 2x) ≥ 3 2 + 2 − − .
2 2
a x a x
Since both sides of the inequality are nonnegative and a2 x ≥ 1, it suffices to prove
the homogeneous inequality
–v  ™
1 2 1 2
Æ t ‹
3(a2 + 2x 2 ) − (a + 2x) ≥ (a2 x)2/3 3 2+ 2 − − .
a x a x
Due to homogeneity, we may set x = 1. Thus, we need to show that a ≤ x = 1
yields Æ ”Æ —
3(a2 + 2) − a − 2 ≥ a1/3 3(1 + 2a2 ) − 1 − 2a ,
which is equivalent to
2(a − 1)2 2(a − 1)2
p ≥ a1/3 p .
3(a2 + 2) + a + 2 3(1 + 2a2 ) + 1 + 2a
It is true if Æ ”Æ —
3(1 + 2a2 ) + 1 + 2a ≥ a1/3 3(a2 + 2) + a + 2 .
For t = a1/3 , t ∈ (0, 1], the inequality becomes
Æ Æ
3(1 + 2t 6 ) + 1 + 2t 3 ≥ 3(t 8 + 2t 2 ) + t 4 + 2t,
which is true because
1 + 2t 6 − (t 8 + 2t 2 ) = (1 − t 4 )(1 − t 2 )2 ≥ 0,
1 + 2t 3 − (t 4 + 2t) = (1 − t 2 )(1 − t)2 ≥ 0.
The equality occurs for a = b = c ≥ 1.
Remark. The inequality is true in the particular case a, b, c ≥ 1, which implies
a4 bc ≥ 1.
410 Vasile Cîrtoaje

P 2.92. Let v
t a2 + b2 + c 2 a+b+c
F (a, b, c) = − ,
3 3
where a, b, c are positive real numbers such that

a2 (b + c) ≥ 2, a ≤ b ≤ c.

Then,
1 1 1
 ‹
F (a, b, c) ≥ F , , .
a b c

(Vasile Cîrtoaje, 2020)

Solution. The proof follows the same way as the proof of the preceding P 2.91.
Write the inequality as E(a, b, c) ≥ 0, where
v
+ + t1 1 1 1 1 1 1
 ‹
p a b c
E(a, b, c) = a +b +c −
2 2 2 p − + + +p + + ,
3 a2 b2 c 2 3 a b c
and show that
E(a, b, c) ≥ E(a, x, x) ≥ 0,
where
b+c
x= ≥ b, a2 x ≥ 1, x ≥ 1.
2
Write the inequality E(a, b, c) ≥ E(a, x, x) it in the form

A + B ≥ C,

where p p
A= a2 + b2 + c 2 − a2 + 2x 2
(b − c)2 1
= ·p p
2 a2 + b2 + c 2 + a2 + 2x 2
(b − c)2 1
≥ ·p p ,
2 2b2 + c 2 + b2 + 2x 2
1 1 1 2 (b − c)2
 ‹
B=p + − =p ,
3 b c x 3bc(b + c)
v v
t1 1 1 t1 2
C= + + − +
a2 b2 c 2 a2 x 2
(b − c)2 (b2 + 4bc + c 2 ) 1
= ·q
b2 c 2 (b + c)2
q
1 1 1 1 2
a2 + b2 + c2 + a2 + x2
Symmetric Nonrational Inequalities 411

(b − c)2 (b2 + 4bc + c 2 ) 1


≤ ·q .
b2 c 2 (b + c)2
q
2 1 1 2
b2 + c2 + b2 + x2

Thus, we need to show that


1 1 1 b2 + 4bc + c 2 1
·p p +p ≥ 2 2 ·q .
2b2 + c 2 + b2 + 2x 2 3bc(b + c) b c (b + c)2
q
2 2 1 1 2
b2 + c2 + b2 + x2

Since
b2 + 4bc + c 2 = 4bc + (b2 + c 2 ),
it suffices to show that
1 4bc 1
p ≥ 2 2 ·q
3bc(b + c) b c (b + c)2
q
2 1 1 2
b2 + c2 + b2 + x2

and
1 1 b2 + c 2 1
·p p ≥ ·q .
2b2 + c 2 + b2 + 2x 2 b2 c 2 (b + c)2
q
2 2 1 1 2
b2 + c2 + b2 + x2

Write the first inequality as


–v v Œ
t2 1 t1 2 p
(b + c) + + + ≥ 4 3.
b2 c 2 b2 x 2

Since v v
t2 1 t1 2 1 2 1
 ‹
1 1 2
 ‹
+ + + ≥p + +p +
b2 c 2 b2 x 2 3 b c 3 b x
1 2 1 1 1 2 2 1 1 2
 ‹  ‹  ‹
≥p + +p + =p + +
3 b c 3 c x 3 b c b+c
p
2 4 2 4 3
 ‹
≥p + = ,
3 b+c b+c b+c
the inequality is proved.
The second inequality reduces to

bc(b + c)2 ≥ 2(b2 + c 2 ).

It is true if the following homogeneous inequality is true:


˜2/3
b2 (b + c)
•
bc(b + c) ≥ 2(b + c )
2 2 2
.
2
Due to homogeneity, we may set b = 1, hence c ≥ 1, when the inequality becomes

c + 1 2/3
 ‹
c(c + 1) ≥ 2(c + 1)
2 2
.
2
412 Vasile Cîrtoaje

It is true if
c 3 (c + 1)4 ≥ 2(c 2 + 1)3 ,
that is
c 7 + 2c 6 + 6c 5 − 2c 4 + c 3 − 6c 2 − 2 ≥ 0,
(c 7 + c 3 − 2) + 2c 4 (c 2 − 1) + 6c 2 (c 3 − 1) ≥ 0.
To complete the proof, we need to show that E(a, x, x) ≥ 0 for a2 x ≥ 1, x ≥ a.
This inequality was proved at the preceding P 2.91.
The equality occurs for a = b = c ≥ 1.

Remark. Since a4 bc ≥ 1 yields a2 (b + c) ≥ 2, the inequality in P 2.91 follows from


the inequality in P 2.92.

P 2.93. Let v
t a2 + b2 + c 2 a+b+c
F (a, b, c) = − ,
3 3
where a, b, c are positive real numbers such that

a4 (b2 + c 2 ) ≥ 2, a ≤ b ≤ c.

Then,
1 1 1
 ‹
F (a, b, c) ≥ F , , .
a b c

(Vasile Cîrtoaje, 2020)

Solution. The proof follows the same way as the proof of the preceding P 2.92.
Write the inequality as E(a, b, c) ≥ 0, where
v
+ + t1 1 1 1 1 1 1
 ‹
p a b c
E(a, b, c) = a +b +c −
2 2 2 p − + + +p + + ,
3 a2 b2 c 2 3 a b c
and show that
E(a, b, c) ≥ E(a, x, x) ≥ 0,
where v
t b2 + c 2
x= ≥ b, a2 x ≥ 1, x ≥ 1.
2
Write the inequality E(a, b, c) ≥ E(a, x, x) it in the form

A + B ≥ C,
Symmetric Nonrational Inequalities 413

where
2x − b − c (b − c)2
pA= =p ,
3 3 (2x + b + c)
1 1 1 2 (b − c)2 (b2 + c 2 + 4bc)
 ‹
B=p + − = p ,
3 b c x 2 3 b2 c 2 x 2 1b + 1c + 2x
v v
t1 1 1 t1 2
C= + + − +
a2 b2 c 2 a2 x 2
(b2 − c 2 )2 1
= 2 2 2
·q q
2b c x 1
+ 1
+ 1
+ 1 2
a2 b2 c2 a2 + x 2
p
3(b2 − c 2 )2 1
≤ · 1 1 1 1 2

a + b + c + a + x
2
2b c x2 2

p
3(b2 − c 2 )2 1
≤ · 3 1 2 .
b + c + x
2
2b c x2 2

Thus, we need to show that


1 b2 + c 2 + 4bc 3(b + c)2 1
+ ≥ · .
2x + b + c 2b2 c 2 x 2 1b + 1c + 2x 3
+ 1c + 2

2b2 c 2 x 2 b x

Since
b2 x ≥ a2 x ≥ 1,
it suffices to prove the homogeneous inequality
1 b2 + c 2 + 4bc 3(b + c)2 1
+ ≥ · .
(b2 x)3/2 (2x + b + c) 2b2 c 2 x 2 1b + 1c + 2x 3
+ 1c + 2

2b2 c 2 x 2 b x

Since
3 1 2 1 1 2 3 2 2
 ‹  ‹
2 + + −3 + + = − − ≥ 0,
b c x b c x b c x
it is enough to show that
1 b2 + c 2 + 4bc 2(b + c)2 1
+ ≥ · ,
(b2 x)2/3 (2x + b + c) 2b2 c 2 x 2 1b + 1c + 2x 1
+ 1c + 2

2b2 c 2 x 2 b x

that is
1 1 1
≥ · ,
(b2 x)2/3 (2x
+ b + c) + 1c + 2x
b2 c 2 1
b
2bc
 ‹  
c b+c+ ≥ b1/3 2x 5/3 + (b + c)x 2/3 .
x
Since x ≤ c, it suffices to show that
2bc
 ‹  
c b+c+ ≥ b1/3 2c x 2/3 + (b + c)x 2/3 ,
c
414 Vasile Cîrtoaje

that is
c(3b + c) ≥ (b + 3c)(bx 2 )1/3 .
Due to homogeneity, we may set c = 1, when 0 < b ≤ 1 and
v
t b2 + 1
x= .
2
Thus, we need to show that
v
3 b + b
t 3
3b + 1 ≥ (b + 3) ,
2
which is true if
2(3b + 1)3 ≥ b(b2 + 1)(b + 3)3 .
Since
(b + 3)3 = b3 + 39b2 + 27b + 27 ≤ 37b + 27 ≤ 32(b + 1),
it suffices to sow that
(3b + 1)3 ≥ 16(b2 + 1)(b + 1),
which is equivalent to
1 − 7b + 11b2 + 11b3 − 16b4 ≥ 0,
(1 − b)(1 − 6b + 5b2 + 16b3 ) ≥ 0.
This is true because
1 − 6b + 5b2 + 16b3 = (1 − 4b)2 + b(2 − 11b + 16b2 ) > 0.
To complete the proof, we need to show that E(a, x, x) ≥ 0 for a2 x ≥ 1, x ≥ a.
This inequality was proved at P 2.91.
The equality occurs for a = b = c ≥ 1.
Remark. Since a2 (b + c) ≥ 1 yields a4 (b2 + c 2 ) ≥ 2, the inequality in P 2.92 follows
from the inequality in P 2.93.

P 2.94. Let
p
3 3
F (a, b, c) = abc − 1
,
a + 1b + 1c
where a, b, c are positive real numbers such that
a4 b7 c 7 ≥ 1, a ≥ b ≥ c.
Then,
1 1 1
 ‹
F (a, b, c) ≥ F , , .
a b c

(Vasile Cîrtoaje and Vasile Mircea Popa, 2019)


Symmetric Nonrational Inequalities 415

Solution. By the AM-GM inequality, both sides of the inequality are nonnegative.
Denote p
x = bc.
We have
a ≥ 1, x ≤ a, a2 x 7 ≥ 1.
From
1 1
x≥ ≥ ,
a2/7 a1/2
it follows that
1
. a≥
x2
Write the inequality as E(a, b, c) ≥ 0, where
p
3 3 1 3
E(a, b, c) = abc − −p + ,
1
a + + 1
b
1
c
3
abc a+b+c

and prove that


E(a, b, c) ≥ E(a, x, x) ≥ 0.
1
We will show that the left inequality is true for a ≥ 1 and a ≥ . Write the
x2
inequality as follows
1 1 1 1
− ≥ p − ,
a+b+c 1
a + + 1
b
1
c a + 2 bc
1
a + p2bc

1 1 1 1
− ≥ p − ,
a + 2 bc a + b + c
1
a + p2bc 1
a + 1b + 1c
Š2
p p
€
1
− p1c
p
b ( b − c)2
€ Š ≥ p .
1
a + p
2 1
a + 1
b + 1
c
(a + 2 bc)(a + b + c)
bc
p p 2
After dividing by ( b − c) , we need to show that

2 1 2 1 b+c
 ‹ ‹
(a + 2x)(a + b + c) ≥ x + + 2 . (*)
a x a x
Write this inequality as
A(b + c) + B ≥ 0,
where
1 2 x 2 2x
A = a + 2x − − , B = a + 2ax − 2 −
2
.
a x a a
Clearly, A ≥ 0 for x ≥ 1. Also,A ≥ 0 for x ≤ 1, because

1 2 (1 − x)3 (1 + x)
A≥ + 2x − x 2
− = ≥0.
x2 x x2
416 Vasile Cîrtoaje

p
Since A ≥ 0 and b + c ≥ 2 bc, it suffices to replace b + c in (*) with 2x. So, we
need to show that

2 1 2 1 2
 ‹ ‹
(a + 2x)(a + 2x) ≥ x + + ,
a x a x
which is equivalent to
1 2
 ‹
a + 2x ≥ x + ,
a x
x
a + 2x ≥ + 2.
a
For x ≥ 1, we have

1 1 1
 ‹  ‹
x
a + 2x − − 2 = a − 2 + 2 − x ≥ a−2+ 2− =a− ≥0 ,
a a a a
and for x ≤ 1, we have

x 1 (1 − x)(1 + x − x 2 + x 3 + x 4 )
a + 2x − − 2 ≥ 2 + 2x − x 3 − 2 = ≥0.
a x x2
Write the right inequality E(a, x, x) ≥ 0, as follows
p
3 3ax 1 3
ax 2 − ≥p − .
2a + x 3
ax 2 a + 2x

Since a4/7 x 2 ≥ 1, it suffices to prove the homogeneous inequality

3ax   1 3
‹
4/7 2 7/9
p
3
2
ax − ≥ a x p − .
2a + x 3
ax 2 a + 2x
Setting x = 1 and substituting

a = d 9, d ≥ 1,

the inequality becomes

3d 9 1 3
 ‹
3 4
d − 9 ≥d − ,
2d + 1 d3 d9 + 2

d 2 (d 3 − 1)2 (2d 3 + 1) (d 3 − 1)2 (d 3 + 2)


≥ .
2d 9 + 1 d9 + 2
Thus, we need to show that

d 2 (2d 3 + 1)(d 9 + 2) ≥ (d 3 + 2)(2d 9 + 1) ,

that is
2(d 12 + 1)(d 2 − 1) + d 3 (d 8 − 1) − 4d 5 (d 4 − 1) ≥ 0 ,
Symmetric Nonrational Inequalities 417

(d 2 − 1)A ≥ 0,
where
A = 2(d 12 + 1) + d 3 (d 6 + d 4 + d 2 + 1) − 4d 5 (d 2 + 1)
= 2d 7 (d 5 − 1) + d 7 (d 2 − 1) − 3d 3 (d 2 − 1) − 2(d 3 − 1)
≥ 2d(d 5 − 1) + (d 2 − 1) − 3d 3 (d 2 − 1) − 2(d 3 − 1) = (d − 1)B ,
where

B = 2d(d 4 + d 3 + d 2 + d + 1) + (d + 1) − 3d 3 (d + 1) − 2(d 2 + d + 1)

= 2d 5 − d 4 − d 3 + d − 1 = (d − 1)(2d 4 + d 3 + 1) ≥ 0 .
The equality holds for a = b = c ≥ 1.
Remark. The inequality is true in the particular case a, b, c ≥ 1, which implies
a4 b7 c 7 ≥ 1.

P 2.95. Let p
4 4
F (a, b, c, d) = abcd − 1
,
a + + 1c +
1
b
1
d

where a, b, c, d are positive real numbers. If ab ≥ 1 and cd ≥ 1, then then

1 1 1 1
 ‹
F (a, b, c, d) ≥ F , , , .
a b c d

(Vasile Cîrtoaje, 2019)

Solution. Write the inequality as E(a, b, c, d) ≥ 0, where


p
4 4 1 4
E(a, b, c, d) = abcd − −p + ,
1
a
1
+ + +
b
1
c
1
d
4
abcd a+b+c+d

assume that
ab ≥ cd ≥ 1,
and show that
p p p p p p
E(a, b, c, d) ≥ E(a, b, cd, cd) ≥ E( ab, ab, cd, cd) ≥ 0.

Since p
cd cd
1− ≥1− ≥0
ab ab
and p
cd − 1 ≥ 0,
418 Vasile Cîrtoaje

p p
the left inequality E(a, b, c, d) ≥ p b, cd, pcd) follows
p E(a, p p fromp Lemma below,
point (a). The inequality E(a, b, cd, cd) ≥ E( p ab, ab, cd, cd) follows also
from Lemma below by replacing c and d with cd. We only need to show that
 p 
p p ab p
( cd + cd) 1 − + 2( ab − 1) ≥ 0,
cd

which is equivalent to the obvious inequality


‚v Œ
p t ab
( cd − 1) + 1 ≥ 0.
cd
p p p p
The inequality E( ab, ab, cd, cd) ≥ 0, is true if the inequality E(a, b, c, d) ≥ 0
holds for a = b = x 2 and c = d = y 2 , where x ≥ 1, y ≥ 1. We need to show that

4 1 4
xy− ≥ − 2 ,
2
x2 + 2
y2
x y 2x + 2 y 2

that is
(x 2 y 2 − 1)(x − y)2 ≥ 0.
This completes the proof. The equality holds for a = b = c = d ≥ 1, and for
a b = cd = 1.
Lemma. Let
p
4 4 1 4
E(a, b, c, d) = abcd − −p + ,
1
a + + + 1
b
1
c
1
d
4
abcd a+b+c+d

where a, b, c, d are positive real numbers such that ab ≥ 1 and cd ≥ 1.


(a) If  p 
cd p
(a + b) 1 − + 2( cd − 1) ≥ 0,
ab
then p p
E(a, b, c, d) ≥ E(a, b, cd, cd).
(b) If   p
ab p
(c + d) 1 − + 2( ab − 1) ≥ 0,
cd
then p p
E(a, b, c, d) ≥ E( ab, ab, c, d).
p p
Proof. (a) Write the inequality E(a, b, c, d) ≥ E(a, b, cd, cd) as follows:

1 1 1 1
− ≥ p − ,
a+b+c+d 1
a
1
+ + +
b
1
c
1
d a + b + 2 cd
1
a + + p2cd
1
b
Symmetric Nonrational Inequalities 419

1 1 1 1
− ≥ p − ,
a + b + 2 cd a + b + c + d
1 1 2 1 1 1 1
a + +
b
p
cd
+ + +
a b c d
p p p p
( c − d)2 ( c − d)2
€ Š ≥ p ,
cd 1a + 1b + p2cd 1a + 1b + 1c + d1 (a + b + 2 cd)(a + b + c + d)
p p
After dividing by ( c − d)2 , we need to show that

a+b p ‹a + b c + d ‹
p 
(a + b + 2 cd)(a + b + c + d) ≥ cd + 2 cd + , (*)
ab ab cd

that is
A(c + d) + B ≥ 0,
where
a+b 2 1 1
p  ‹ p ‹
A= a+ b+ cd − − p = (a + b) 1 − +2 cd − p ≥ 0,
ab cd ab cd
p
a+b
 
2 cd p
B = (a + b) cd + − a − b − 2 cd .
a2 b2 ab
Since p
A(c + d) + B ≥ 2A cd + b,
p
we need to show
p that 2A cd + b ≥ 0. This is equivalent to (*) if the sum c + d is
replaced by 2 cd:
p 
a+b p ‹ a + b 2 cd
 
p p
(a + b + 2 cd)(a + b + 2 cd) ≥ cd + 2 cd + ,
ab ab cd

that is
pa+b
p ‹2
(a + b + 2 cd) ≥ cd + 2 cd ,
2
ab
p a+b p
a + b + 2 cd ≥ cd + 2 cd,
ab
 p 
cd p
(a + b) 1 − + 2( cd − 1) ≥ 0.
ab
The last inequality is true by hypothesis.
(b) Due to symmetry, this follows from (a).

Remark. The inequality is true in the particular case a, b, c, d ≥ 1, which implies


a b ≥ 1 and cd ≥ 1.
420 Vasile Cîrtoaje

P 2.96. Let a, b, c, d be positive real numbers such that a2 + b2 + c 2 + d 2 = 1. Prove


that p p p
p p p p p
1 − a + 1 − b + 1 − c + 1 − d ≥ a + b + c + d.
(Vasile Cîrtoaje, 2007)

First Solution. We can obtain the desired inequality by summing the inequalities
p p p p
1 − a + 1 − b ≥ c + d,
p p p p
1 − c + 1 − d ≥ a + b.
Since p p Æ
4
1 − a + 1 − b ≥ 2 (1 − a)(1 − b)
and v v
p p tc + d t4 c + d
2 2
c+ d ≤2 ≤2 ,
2 2
the former inequality holds if

c2 + d 2
(1 − a)(1 − b) ≥ .
2
Indeed,

2(1 − a)(1 − b) − c 2 − d 2 = 2(1 − a)(1 − b) + a2 + b2 − 1 = (a + b − 1)2 ≥ 0.

Similarly, we can prove the second inequality. The equality holds for

1
a=b=c=d= .
2
Second Solution. We can obtain the desired inequality by summing the inequalities
p p 1 p p 1
1 − a − a ≥ p (1 − 4a2 ), 1− b− b ≥ p (1 − 4b2 ),
2 2 2 2
p p 1 p p 1
1 − c − c ≥ p (1 − 4c 2 ), 1 − d − d ≥ p (1 − 4d 2 ).
2 2 2 2
To prove the first inequality, we write it as

1 − 2a 1
p p ≥ p (1 − 2a)(1 + 2a).
1−a+ a 2 2

1
Case 1: 0 < a ≤ . We need to show that
2
p p p
2 2 ≥ (1 + 2a)( 1 − a + a).
Symmetric Nonrational Inequalities 421

p p p p
Since 1 − a + a ≤ 2[(1 − a) + a] = 2, we have
p p p p
2 2 − (1 + 2a)( 1 − a + a) ≥ 2(1 − 2a) ≥ 0.
1
Case 2: ≤ a < 1. We need to show that
2
p p p
2 2 ≤ (1 + 2a)( 1 − a + a).
p
Since 1 + 2a ≥ 2 2a, it suffices to prove that
Æ
1 ≤ a(1 − a) + a.

Indeed,
p
Æ p p p 1 − a (1 − 2a)
1−a− a(1 − a) = 1 − a ( 1 − a − a) = p p ≤ 0.
1−a+ a

P 2.97. Let a, b, c, d be positive real numbers. Prove that


p
A + 2 ≥ B + 4,

where
1 1 1 1
 ‹
A = (a + b + c + d) + + + − 16,
a b c d
1 1 1 1
 ‹
B = (a + b + c + d ) 2 + 2 + 2 + 2 − 16.
2 2 2 2
a b c d
(Vasile Cîrtoaje, 2004)

Solution. By squaring, the inequality becomes

A2 + 4A ≥ B.

Let us denote
x y z x 2 y 2 z2
f (x, y, z) = + + − 3, F (x, y, z) = 2 + 2 + 2 − 3,
y z x y z x
where x, y, z > 0. By the AM-GM inequality, it follows that

f (x, y, z) ≥ 0.

We can check that

A = f (a, b, c) + f (b, d, c) + f (c, d, a) + f (d, b, a)


= f (a, c, b) + f (b, c, d) + f (c, a, d) + f (d, a, b)
422 Vasile Cîrtoaje

and
B = F (a, b, c) + F (b, d, c) + F (c, d, a) + F (d, b, a).
Since

F (x, y, z) = [ f (x, y, z) + 3]2 − 2[ f (x, z, y) + 3] − 3


= f 2 (x, y, z) + 6 f (x, y, z) − 2 f (x, z, y),

we get
B = f 2 (a, b, c) + f 2 (b, d, c) + f 2 (c, d, a) + f 2 (d, b, a) + 4A,
4A − B = − f 2 (a, b, c) − f 2 (b, d, c) − f 2 (c, d, a) − f 2 (d, b, a).
Therefore,

A2 + 4A − B = [ f (a, b, c) + f (b, d, c) + f (c, d, a) + f (d, b, a)]2


− f 2 (a, b, c) − f 2 (b, d, c) − f 2 (c, d, a) − f 2 (d, b, a) ≥ 0.

The equality holds for a = b = c = d.

P 2.98. Let a1 , a2 , . . . , an be nonnegative real numbers such that a1 + a2 +· · ·+ an = 1.


Prove that
3a1 + 1 + 3a2 + 1 + · · · + 3an + 1 ≥ n + 1.
p p p

First Solution. Without loss of generality, assume that a1 = max{a1 , a2 , . . . , an }.


Write the inequality as follows:

( 3a1 + 1 − 2) + ( 3a2 + 1 − 1) + · · · + ( 3an + 1 − 1) ≥ 0,


p p p

a1 − 1 a2 an
+p + ··· + p ≥ 0,
3a1 + 1 + 2 3a2 + 1 + 1 3an + 1 + 1
p

a2 an a2 + · · · + an
+ ··· + p ≥p ,
3a2 + 1 + 1 3an + 1 + 1 3a1 + 1 + 2
p
   
1 1 1 1
a2 p −p +· · ·+an p −p ≥ 0.
3a2 + 1 + 1 3a1 + 1 + 2 3an + 1 + 1 3a1 + 1 + 2
The last inequality is clearly true. The equality holds for a1 = 1 and a2 = · · · = an =
0 (or any cyclic permutation).
Second Solution. We use the induction method. For n = 1, the inequality is an
equality. We claim that
Æ
3a1 + 1 + 3an + 1 ≥ 3(a1 + an ) + 1 + 1.
p p
Symmetric Nonrational Inequalities 423

By squaring, this inequality becomes


Æ Æ
(3a1 + 1)(an + 1) ≥ 3(a1 + an ) + 1,

which is equivalent to a1 an ≥ 0. Thus, to prove the original inequality, it suffices


to show that
Æ
3(a1 + an ) + 1 + 3a2 + 1 + · · · + 3an−1 + 1 ≥ n.
p p

Using the substitution b1 = a1 + an and b2 = a2 , . . . , bn−1 = an−1 , this inequality


turns into Æ Æ Æ
3b1 + 1 + 3b2 + 1 + · · · + 3bn−1 + 1 ≥ n
for b1 + b2 + · · · + bn−1 = 1. Clearly, this is true by the induction hypothesis.

P 2.99. Let a1 , a2 , . . . , an be positive real numbers such that a1 a2 · · · an = 1. Prove


that
1 1 1
+p + ··· + p ≥ 1.
1 + (n2 − 1)a1 1 + (n2 − 1)a2 1 + (n2 − 1)an
p

First Solution. For the sake of contradiction, assume that


1 1 1
+p + ··· + p < 1.
1 + (n2 1 + (n2 1 + (n2 − 1)an
p
− 1)a1 − 1)a2

It suffices to show that a1 a2 · · · an > 1. Let

1
xi = p , 0 < x i < 1, i = 1, 2, · · · , n.
1 + (n2 − 1)ai

1 − x i2
Since ai = for all i, we need to show that
(n2 − 1)x i2

x1 + x2 + · · · + x n < 1

implies
(1 − x 12 )(1 − x 22 ) · · · (1 − x n2 ) > (n2 − 1)n x 12 x 22 · · · x n2 .
Using the AM-GM inequality gives
Y Y h€X Š2 i Y
(1 − x 1 ) >
2
x1 − x1 =
2
(x 2 + · · · + x n )(2x 1 + x 2 + · · · + x n )
Y€ p q Š
≥ (n2 − 1)n x 12 x 2 · · · x n = (n2 − 1)n x 12 x 22 · · · x n2 .
n+1
n−1
x2 · · · x n ·
424 Vasile Cîrtoaje

The equality holds for a1 = a2 = · · · = an = 1.


Second Solution. We will show that
1 1

1 + (n − 1)x k
p
1 + (n2 − 1)x
n+1
for x > 0 and k = . By squaring, the inequality becomes
2n
(n − 1)x 2k−1 + 2x k−1 ≥ n + 1.

Applying the AM-GM inequality, we get


p
n+1
(n − 1)x 2k−1 + 2x k−1 ≥ (n + 1) x (n−1)(2k−1) · x 2(k−1) = n + 1.

Using this result, it suffices to show that


1 1 1
+ + ··· + ≥ 1.
1 + (n − 1)a1k 1 + (n − 1)a2k 1 + (n − 1)ank

Since a1k a2k · · · ank = 1, this inequality follows immediately from P 1.200-(a).

P 2.100. Let a1 , a2 , . . . , an be positive real numbers such that a1 a2 · · · an = 1. Prove


that n
X 1 1
≥ .
i=1 1 + 1 + 4n(n − 1)ai
p
2

First Solution. Write the inequality as follows:


n p
X 1 + 4n(n − 1)ai − 1
≥ 2n(n − 1),
i=1
ai
v
n u
X t1 4n(n − 1) X1
+ ≥ 2n(n − 1) + .
i=1
ai2 ai ai
By squaring, the inequality becomes
v – ™
X u t 1 4n(n − 1) 1 4n(n − 1) X 1
2
+ 2
+ ≥ 2n 2
(n − 1)2
+ .
1≤i< j≤n
ai ai aj aj aa
1≤i< j≤n i j

The Cauchy-Schwarz inequality gives


v – ™
t 1 4n(n − 1) 1 4n(n − 1) 1 4n(n − 1)
u
2
+ 2
+ ≥ + p .
ai ai aj aj ai a j ai a j
Symmetric Nonrational Inequalities 425

Thus, it suffices to show that


X 1 n(n − 1)
≥ ,
2
p
1≤i< j≤n
ai a j

which follows immediately from the AM-GM inequality. The equality holds for
a1 = a2 = · · · = an = 1.
Second Solution. For the sake of contradiction, assume that
n
X 1 1
< .
1+ 1 + 4n(n − 1)ai
p
i=1
2

It suffices to show that a1 a2 · · · an > 1. Using the substitution


xi 1
= , i = 1, 2, · · · , n,
2n 1 + 1 + 4n(n − 1)ai
p

which yields
n − xi
ai = , 0 < x i < n, i = 1, 2, · · · , n,
(n − 1)x i2
we need to show that
x1 + x2 + · · · + x n < n
implies
(n − x 1 )(n − x 2 ) · · · (n − x n ) > (n − 1)n x 12 x 22 · · · x n2 .
By the AM-GM inequality, we have
 x + x + · · · + x n
1 2 n
x1 x2 · · · x n ≤ <1
n
and
v
t x1 x2 · · · x n
n−1
n − x i > (x 1 + x 2 + · · · + x n ) − x i ≥ (n − 1) , i = 1, 2, · · · , n.
xi

Therefore, we get

(n − x 1 )(n − x 2 ) · · · (n − x n ) > (n − 1)n x 1 x 2 · · · x n > (n − 1)n x 12 x 22 · · · x n2 .

P 2.101. If a1 , a2 , . . . , an are positive real numbers such that a1 a2 · · · an = 1, then


v
t a2 + a2 + · · · + a2
1 2 n
a1 + a2 + · · · + an ≥ n − 1 + .
n
426 Vasile Cîrtoaje

Solution. Let us denote


v P
a1 + a2 + · · · + an t 2 1≤i< j≤n ai a j
u
a= , b= ,
n n(n − 1)

where a ≥ 1 and b ≥ 1 (by the AM-GM inequality). We need to show that


v
t n2 a2 − n(n − 1)b2
na − n + 1 ≥ .
n
By squaring, this inequality becomes

(n − 1)[n(a − 1)2 + b2 − 1] ≥ 0,

which is clearly true. The equality holds for a1 = a2 = · · · = an = 1.

P 2.102. If a1 , a2 , . . . , an are positive real numbers such that a1 a2 · · · an = 1, then


q Æ
(n − 1)(a12 + a22 + · · · + an2 ) + n − n(n − 1) ≥ a1 + a2 + · · · + an .

(Vasile Cîrtoaje, 2006)

Solution. We use the induction method. For n = 2, the inequality is equivalent to


the obvious inequality
1
a1 + ≥ 2.
a1
Assume now that the inequality holds for n − 1 numbers, n ≥ 3, and prove that it
holds also for n numbers. Let a1 = min{a1 , a2 , . . . , an }, and denote
a2 + a3 + · · · + an
x= y=
p
, n−1
a2 a3 · · · an ,
n−1
q Æ
f (a1 , a2 , . . . , an ) = (n − 1)(a12 + a22 + · · · + an2 )+n− n(n − 1)−(a1 +a2 +· · ·+an ).
By the AM-GM inequality, we have x ≥ y. We will show that

f (a1 , a2 , . . . , an ) ≥ f (a1 , y, · · · , y) ≥ 0. (*)

Write the left inequality as


q q p
a12 + a22 + · · · + an2 − a12 + (n − 1) y 2 ≥ n − 1 (x − y).

To prove this inequality, we use the induction hypothesis, written in the homoge-
neous form
q ” Æ —
(n − 2)(a22 + a32 + · · · + an2 ) + n − 1 − (n − 1)(n − 2) y ≥ (n − 1)x,
Symmetric Nonrational Inequalities 427

which is equivalent to
a22 + · · · + an2 ≥ (n − 1)A2 ,
where v
tn − 1
A = kx − (k − 1) y, k= .
n−2
So, we need to prove that
q q p
a12 + (n − 1)A2 − a12 + (n − 1) y 2 ≥ n − 1 (x − y).

Write this inequality as


A2 − y 2 x−y
Æ Æ ≥p .
a12 + (n − 1)A2 + a12 + (n − 1) y 2 n−1

Since x ≥ y and

A2 − y 2 = k(x − y)[kx − (k − 2) y] = k(x − y)(A + y),

we need to show that


k(A + y) 1
Æ Æ ≥p .
a12 + (n − 1)A2 + a12 + (n − 1) y 2 n−1

In addition, since a1 ≤ y, it suffices to show that


k(A + y) 1
p ≥p .
y 2 + (n − 1)A2 +
p
ny n−1
From

kA − y = k2 x − (k2 − k + 1) y ≥ k2 y − (k2 − k + 1) y = (k − 1) y > 0,

it follows that

y 2 + (n − 1)A2 < k2 A2 + (n − 1)2 = (n − 1)k2 A2 .

Therefore, it is enough to prove that


k(A + y) 1
p p ≥p ,
n − 1 kA + n y n−1
which is equivalent to p p 
k n − 1 − n y ≥ 0.
This is true since
p p n−1 p 1
k n−1− n= p − n= p > 0.
n−2 n − 1 + n(n − 2)
The proof is completed. The equality holds for a1 = a2 = · · · = an = 1.
428 Vasile Cîrtoaje

P 2.103. Let a1 , a2 , . . . , an be positive real numbers such that a1 a2 · · · an ≥ 1. If k > 1,


then
X a1k
≥ 1.
a1k + a2 + · · · + an
(Vasile Cîrtoaje, 2006)
p
First Solution. Let us denote r = n a1 a2 · · · an and bi = ai /r for i = 1, 2, · · · , n.
Note that r ≥ 1 and b1 b2 · · · bn = 1. The desired inequality becomes
X b1k
≥ 1,
b1k + (b2 + · · · + bn )/r k−1

and we see that it suffices to prove it for r = 1; that is, for a1 a2 · · · an = 1. On this
hypothesis, we will show that there exists a positive number p, 1 < p < k, such
that p
a1k a1
≥ p p p.
a1k + a2 + · · · + an a1 + a2 + · · · + an
Clearly, by adding this inequality and the analogous inequalities for a2 , . . . , an , we
get the desired inequality. Write the claimed inequality as
p
a2 + · · · + anp ≥ (a2 · · · an )k−p (a2 + · · · + an ).

Based on the AM-GM inequality


 a + · · · + a n−1
2 n
a2 · · · an ≤ ,
n−1
it suffices to show that
 a + · · · + a (n−1)(k−p)+1
p 2 n
a2 + ··· + anp ≥ (n − 1) .
n−1
Choosing
(n − 1)k + 1
p= , 1 < p < k,
n
the inequality becomes
 a + · · · + a p
p 2 n
a2 + · · · + anp ≥ (n − 1) ,
n−1
which is just Jensen’s inequality applied to the convex function f (x) = x p . The
equality holds for a1 = a2 = · · · = an = 1.
Second Solution. By the Cauchy-Schwarz inequality, we have
P k+1
2
2 k+1
a1 a1k+1 + 2 1≤i< j≤n (ai a j ) 2
P P
X a1k
≥P = .
a1k + a2 + · · · + an a1 (a1k + a2 + · · · + an ) a1 + 2 1≤i< j≤n ai a j
P k+1 P
Symmetric Nonrational Inequalities 429

Thus, it suffices to show that


X k+1
X
(ai a j ) 2 ≥ ai a j .
1≤i< j≤n 1≤i< j≤n

k+1
Jensen’s inequality applied to the convex function f (x) = x 2 yields
‚ P Œ k+1
X k+1 n(n − 1) 2 1≤i< j≤n ai a j 2
(ai a j ) 2 ≥ .
1≤i< j≤n
2 n(n − 1)

On the other hand, by the AM-GM inequality, we get


2 X 2
ai a j ≥ (a1 a2 · · · an ) n ≥ 1.
n(n − 1) 1≤i< j≤n

Therefore,
‚ P Œ k+1 ‚ P Œ k−1 P P
2 1≤i< j≤n ai a j 2 2 1≤i< j≤n ai a j 2 2 1≤i< j≤n ai a j 2 1≤i< j≤n ai a j
= · ≥ .
n(n − 1) n(n − 1) n(n − 1) n(n − 1)

hence P
X k+1 n(n − 1) 2 1≤i< j≤n ai a j X
(ai a j ) 2 ≥ · = ai a j .
1≤i< j≤n
2 n(n − 1) 1≤i< j≤n

P 2.104. Let a1 , a2 , . . . , an be positive real numbers such that a1 a2 · · · an ≥ 1. If


−2
≤ k < 1,
n−2
then
X a1k
≤ 1.
a1k + a2 + · · · + an
(Vasile Cîrtoaje, 2006)
p
Solution. Let us denote r = a1 a2 · · · an and bi = ai /r for i = 1, 2, · · · , n. Clearly,
n

r ≥ 1 and b1 b2 · · · bn = 1. The desired inequality becomes


X b1k
≤ 1,
b1k + (b2 + · · · + bn )r 1−k
and we see that it suffices to prove it for r = 1; that is, for a1 a2 · · · an = 1. On this
hypothesis, we will show that there exists a real number p such that
p
a1k a1
≤ p p p.
a1k + a2 + · · · + an a1 + a2 + · · · + an
430 Vasile Cîrtoaje

By adding this inequality and the analogous inequalities for a2 , . . . , an , we get the
desired inequality. Write the claimed inequality as
p k−p
a2 + · · · + an ≥ (a2 + · · · + anp )a1 ,

p
a2 + · · · + an ≥ (a2 + · · · + anp )(a2 · · · an ) p−k .
This inequality is homogeneous when 1 = p + (n − 1)(p − k); that is, for

(n − 1)k + 1 −1
p= , ≤ p < 1.
n n−2
Rewrite the homogeneous inequality as
1−p
p
a2 + · · · + an ≥ (a2 + · · · + anp )(a2 · · · an ) n−1 . (*)

To prove it, we use the weighted AM-GM inequality


m 1
ma2 + a3 + · · · + an ≥ (m + n − 2)a2m+n−2 (a3 · · · an ) m+n−2 , m ≥ 0,

which can be rewritten as


m−1 1
ma2 + a3 + · · · + an ≥ (m + n − 2)a2m+n−2 (a2 · · · an ) m+n−2 .

m−1
Choosing m such that = p, i.e.
m+n−2

1 + (n − 2)p
m= ≥ 0,
1−p

we get
1 + (n − 2)p n−1 p 1−p
a2 + a3 + · · · + a n ≥ a2 (a2 a3 · · · an ) n−1 .
1−p 1−p
Adding this inequality and the analogous inequalities for a3 , · · · , an yields the in-
equality (*). Thus, the proof is completed. The equality holds for a1 = a2 = · · · =
an = 1.

P 2.105. Let a1 , a2 , . . . , an be nonnegative real numbers such that a1 +a2 +· · ·+an ≥ n.


If 1 < k ≤ n + 1, then
X a1
≤ 1.
a1 + a2 + · · · + an
k

(Vasile Cîrtoaje, 2006)


Symmetric Nonrational Inequalities 431

Solution. Using the substitutions


a1 + a2 + · · · + an
s= ,
n
and
a1 a an
, x2 = 2 , · · · , x n = ,
x1 =
s s s
the desired inequality becomes
x1 xn
+ ··· + ≤ 1,
s k−1 x 1k + x2 + · · · + x n x 1 + x 2 + · · · + s k−1 x nk

where s ≥ 1 and x 1 + x 2 + · · · + x n = n. Clearly, if this inequality holds for s = 1,


then it holds for any s ≥ 1. Therefore, we only need to consider the case s = 1,
when a1 + a2 + · · · + an = n, and the desired inequality is equivalent to
a1 a2 an
+ + ··· + ≤ 1.
a1k − a1 + n a2k − a2 + n ank − an + n

By Bernoulli’s inequality, we have

a1k − a1 + n ≥ 1 + k(a1 − 1) − a1 + n = n − k + 1 + (k − 1)a1 ≥ 0.

Consequently, it suffices to prove that


n
X ai
≤ 1.
i=1
n − k + 1 + (k − 1)ai

For k = n+1, this inequality is an equality. Otherwise, for 1 < k < n+1, we rewrite
the inequality as
n
X 1
≥ 1,
i=1
n − k + 1 + (k − 1)a i

which follows from the AM-HM inequality as follows:


n
X 1 n2
≥ Pn = 1.
i=1
n − k + 1 + (k − 1)ai i=1
[n − k + 1 + (k − 1)a i ]

The equality holds for a1 = a2 = · · · = an = 1.

P 2.106. Let a1 , a2 , . . . , an be positive real numbers such that a1 a2 · · · an ≥ 1. If k > 1,


then X a1
≤ 1.
a1 + a2 + · · · + an
k

(Vasile Cîrtoaje, 2006)


432 Vasile Cîrtoaje

1
Solution. Consider two cases: 1 < k ≤ n + 1 and k ≥ n − .
n−1
Case 1: 1 < k ≤ n + 1. By the AM-GM inequality, we have

a1 + a2 + · · · + an ≥ n n a1 a2 · · · an ≥ n.
p

Thus, the desired inequality follows from the preceding P 2.105.


1 p
Case 2: k ≥ n − . Let r = n a1 a2 · · · an and bi = ai /r for i = 1, 2, · · · , n. Note
n−1
that r ≥ 1 and b1 b2 · · · bn = 1. The desired inequality can be rewritten as
X b1
≤ 1.
r k−1 b1k + b2 + · · · + bn

Obviously, it suffices to prove this inequality for r = 1; that is, for

a1 a2 · · · an = 1.

On this hypothesis, it suffices to show that there exists a real p such that
p
(n − 1)a1 a1
+ p p p ≤ 1.
a1k + a2 + · · · + an a1 + a2 + · · · + an

Then, adding this inequality and the analogous inequalities for a2 , · · · , an yields
p
the desired inequality. Let us denote t = n−1 a2 · · · an . By the AM-GM inequality, we
have
p
a2 + · · · + an ≥ (n − 1)t, a2 + · · · + anp ≥ (n − 1)t p .
Thus, it suffices to show that
p
(n − 1)a1 a1
+ p ≤ 1.
a1k + (n − 1)t a1 + (n − 1)t p

Since a1 = 1/t n−1 , this inequality is equivalent to

(n − 1)t q (t n − 1) − (t q−np − 1) ≥ 0,

where
q = (n − 1)(k − 1).
Choose p such that (n − 1)n = q − np, i.e.

(n − 1)(k − n − 1)
p= .
n
The inequality becomes as follows:
 
(n − 1)t q (t n − 1) − t n(n−1) − 1 ≥ 0,
Symmetric Nonrational Inequalities 433

2 2
(n − 1)t q (t n − 1) − (t n − 1)(t n −2n
+ tn −3n
+ · · · + 1) ≥ 0,
2 2
(t n − 1)[(t q − t n −2n
) + (t q − t n −3n
) + · · · + (t q − 1)] ≥ 0.
1
The last inequality is clearly true for q ≥ n2 − 2n; that is, for k ≥ n − .
n−1
The equality holds for a1 = a2 = · · · = an = 1.

P 2.107. Let a1 , a2 , . . . , an be positive real numbers such that a1 a2 · · · an ≥ 1. If


2
−1 − ≤ k < 1,
n−2
then X a1
≥ 1.
a1k + a2 + · · · + an
(Vasile Cîrtoaje, 2006)
p
Solution. Let us denote r = n a1 a2 · · · an and bi = ai /r for i = 1, 2, · · · , n. Note
that r ≥ 1 and b1 b2 · · · bn = 1. The desired inequality becomes
X b1
≥ 1,
b1k /r 1−k + b2 + · · · + b n

and we see that it suffices to prove it for r = 1; that is, for a1 a2 · · · an = 1. On this
hypothesis, by the Cauchy-Schwarz inequality, we have
P 2 2 P
X a1 a1 a1
≥P =P P 2 P 2 .
a1k + a2 + · · · + an a1 (a1k + a2 + · · · + an ) a11+k + a1 − a1

Thus, we still have to show that


X X
a12 ≥ a11+k .

Case 1: −1 ≤ k < 1. Using Chebyshev’s inequality and the AM-GM inequality yields
X 1 €X 1−k Š €X 1+k Š 1−k
X X
a12 ≥ a1 a1 ≥ (a1 a2 · · · an ) n a11+k = a11+k .
n
2
Case 2: −1 − ≤ k < −1. It is convenient to replace a1 , a2 , · · · , an by
n−1
(n−1)/2 (n−1)/2
a1 , a2 , · · · , an(n−1)/2 ,

respectively. Thus, we need to show that a1 a2 · · · an = 1 involves


X X
q
a1n−1 ≥ a1 ,
434 Vasile Cîrtoaje

where
(n − 1)(1 + k)
q= , −1 ≤ q < 0.
2
By the AM-GM inequality, we get
X X a n−1 + · · · + a n−1 X X 1
2 n
a1n−1 = ≥ a2 · · · a n = .
n−1 a1

Thus, it suffice to show that


X 1 X
q
≥ a1 .
a1
By Chebyshev’s inequality and the AM-GM inequality, we have
X 1 1 €X −1−q Š €X q Š €X Š X
q q
≥ a1 a1 ≥ (a1 a2 · · · an )−(1+q)/n a1 = a1 .
a1 n

Thus, the proof is completed. The equality holds for a1 = a2 = · · · = an = 1.

P 2.108. Let a1 , a2 , . . . , an be positive real numbers such that a1 a2 · · · an = 1. If k ≥ 0,


then X 1
≤ 1.
a1k + a2 + · · · + an
(Vasile Cîrtoaje, 2006)

Solution. Consider two cases: 0 ≤ k ≤ 1 and k ≥ 1.


Case 1: 0 ≤ k ≤ 1. By the Cauchy-Schwarz inequality and the AM-GM inequality,
we have
1 a11−k + 1 + · · · + 1
≤ p p p 2
a1k + a2 + · · · + an a1 + a2 + · · · + a n
a11−k + n − 1 a11−k + n − 1
=P ≤ ,
a1 + 2 1≤i< j≤n ai a j a1 + n(n − 1)
P p P

hence
a11−k + n(n − 1)
P
X 1
≤ P .
a1k + a2 + · · · + an a1 + n(n − 1)
Therefore, it suffices to show that
X X
a11−k ≤ a1 .

Indeed, by Chebyshev’s inequality and the AM-GM inequality, we have


X X 1 €X k Š €X 1−k Š €X Š X
a1 = a1k ·a11−k ≥ a1 a1 ≥ (a1 a2 · · · an )k/n a11−k = a11−k .
n
Symmetric Nonrational Inequalities 435

Case 2: k > 1. Write the inequality as


X p 
n−1 a1
+ p p p − 1 ≤ 0,
a1k + a2 + · · · + an a1 + a2 + · · · + an

where p > 0. It suffices to show that there exists a positive number p such that
p
n−1 a1
+ p p p ≤ 1.
a1k + a2 + · · · + an a1 + a2 + · · · + an

Let
x= x > 0.
p
n−1
a1 ,
By the AM-GM inequality, we have
n−1 n−1
a2 + · · · + an ≥ (n − 1) n−1 a2 · · · an = n−1
p =
p
a1 x
and
p
Æ n−1 n−1
a2 + · · · + anp ≥ (a2 · · · an ) p = n−1
Æ p=
n−1
.
a1 xp
Thus, it is enough to show that

n−1 x (n−1)p
+ ≤ 1,
x (n−1)k + n−1
x x (n−1)p + n−1
xp

which is equivalent to
x 1
≤ ,
x (n−1)k+1 +n−1 x np +n−1
x (n−1)k+1 − x np+1 − (n − 1)(x − 1) ≥ 0,
x np+1 (x (n−1)k−np − 1 − (n − 1)(x − 1) ≥ 0.
 

Choose p such that (n − 1)k − np = n − 1, i.e.


(k − 1)(n − 1)
p= > 0.
n
The inequality becomes as follows:
 
x np+1 (x n−1 − 1 − (n − 1)(x − 1) ≥ 0,
 
(x − 1) (x np+n−1 − 1) + (x np+n−2 − 1) + · · · + (x np+1 − 1) ≥ 0.
Since the last inequality is obvious true, the proof is completed. The equality holds
for a1 = a2 = · · · = an = 1.
436 Vasile Cîrtoaje

P 2.109. Let a1 , a2 , . . . , an be nonnegative real numbers such that a1 +a2 +· · ·+an ≤ n.


If 0 ≤ k < 1, then

1 1 1
+ + ··· + ≥ 1.
a1k + a2 + · · · + an a1 + a2k + · · · + an a1 + a2 + · · · + ank

Solution. By the AM-HM inequality


X 1 n2 n2
≥ =
a1k + a2 + · · · + an (a1k + a2 + · · · + an ) a1k + (n − 1) a1
P P P

and Jensen’s inequality


 X ‹k
X 1
a1k ≤n a1 ,
n
we get
X 1 n2
≥ ≥ 1.
a1k + a2 + · · · + an 1
k
+ (n − 1)
P P
n n a1 a1
The equality holds for a1 = a2 = · · · = an = 1.

P 2.110. Let a1 , a2 , . . . , an be positive real numbers. If k > 1, then

X ak + ak + · · · + ak n(a1k + a2k + · · · + ank )


2 3 n
≤ .
a2 + a3 + · · · + a n a1 + a2 + · · · + an

(Wolfgang Berndt and Vasile Cîrtoaje, 2006)

Solution. Due to homogeneity, we may assume that a1 + a2 + ... + an = 1. Write


the inequality as follows:
X a1
‹
1+ (a2k + a3k + · · · + ank ) ≤ n(a1k + a2k + · · · + ank );
a2 + a3 + · · · + a n

X a1 (a k + a k + · · · + a k )
2 3 n
≤ a1k + a2k + · · · + ank ;
a2 + a3 + · · · + an
X  a2
k
+ a3
k
+ · · · + a k
n
a1 a1k−1 − ≥ 0;
a2 + a3 + · · · + a n
X a1 a2 (a k−1 − a k−1 ) + a1 a3 (a k−1 − a k−1 ) + · · · + a1 an (a k−1 − a k−1 )
1 2 1 3 1 n
≥ 0;
a2 + a3 + · · · + an
aik−1 − a k−1 a k−1 − aik−1
‚ Œ
X j j
ai a j + ≥ 0;
1≤i< j≤n
1 − ai 1 − aj
Symmetric Nonrational Inequalities 437

X ai a j (aik−1 − a k−1
j
)(ai − a j )
≥ 0.
1≤i< j≤n
(1 − ai )(1 − a j )

Since the last inequality is true for k > 1, the proof is completed. The equality
holds for a1 = a2 = · · · = an .

P 2.111. Let f be a convex function on the closed interval [a, b], and let a1 , a2 , . . . , an ∈
[a, b] such that
a1 + a2 + · · · + an = pa + qb,
where p, q ≥ 0 such that p + q = n. Prove that

f (a1 ) + f (a2 ) + · · · + f (an ) ≤ p f (a) + q f (b).

(Vasile Cîrtoaje, 2009)

Solution. Consider the nontrivial case a < b. Since a1 , a2 , . . . , an ∈ [a, b], there
exist λ1 , λ2 , . . . , λn ∈ [0, 1] such that

ai = λi a + (1 − λi )b, i = 1, 2, . . . , n.

From
ai − b
λi = , i = 1, 2, . . . , n,
a−b
we have
n
‚ n Œ
X 1 X (pa + qb) − (p + q)b
λi = ai − nb = = p.
i=1
a − b i=1 a−b

Since f is convex on [a, b], we get


n
X n
X
f (ai ) ≤ [λi f (a) + (1 − λi ) f (b)]
i=1 i=1
‚ n
Œ
X
= λi [ f (a) − f (b)] + n f (b)
i=1
= p [ f (a) − f (b)] + (p + q) f (b)
= p f (a) + q f (b).
438 Vasile Cîrtoaje
Chapter 3

Symmetric Power-Exponential
Inequalities

3.1 Applications

3.1. If a, b are positive real numbers such that a + b = a4 + b4 , then


3 3
aa b b ≤ 1 ≤ aa b b .

3.2. If a, b are positive real numbers, then

a2a + b2b ≥ a a+b + b a+b .

3.3. If a, b are positive real numbers, then

aa + b b ≥ a b + ba .

3.4. If a, b are positive real numbers, then

a2a + b2b ≥ a2b + b2a .

3.5. If a, b are nonnegative real numbers such that a + b = 2, then

(a) a b + b a ≤ 1 + ab;

(b) a2b + b2a ≤ 1 + ab.

439
440 Vasile Cîrtoaje

2
3.6. If a, b are nonnegative real numbers such that ≤ a + b ≤ 2, then
3

a2b + b2a ≤ 1 + ab.

3.7. If a, b are nonnegative real numbers such that a2 + b2 = 2, then

a2b + b2a ≤ 1 + ab.

1
3.8. If a, b are nonnegative real numbers such that a2 + b2 = , then
4

a2b + b2a ≤ 1 + ab.

3.9. If a, b are positive real numbers, then


a+b
a a b b ≤ (a2 − ab + b2 ) 2 .

3.10. If a, b ∈ (0, 1], then

a a b b ≤ 1 − ab + a2 b2 .

3.11. If a, b are positive real numbers such that a + b ≤ 2, then


 a  b  b ‹a
+ ≤ 2.
b a

3.12. If a, b are positive real numbers such that a + b = 2, then

3
2a a b b ≥ a2b + b2a + (a − b)2 .
4

3.13. If a, b ∈ (0, 1] or a, b ∈ [1, ∞), then

2a a b b ≥ a2 + b2 .
Symmetric Power-Exponential Inequalities 441

3.14. If a, b are positive real numbers, then

2a a b b ≥ a2 + b2 .

3.15. If a ≥ 1 ≥ b > 0, then

2a a b b ≥ a2b + b2a .

3.16. If a ≥ e ≥ b > 0, then

2a a b b ≥ a2b + b2a .

3.17. If a, b are positive real numbers, then


‹ a+b
a2 + b2
 2
a b
a b ≥ .
2

3.18. If a, b are positive real numbers such that a2 + b2 = 2, then

1
2a a b b ≥ a2b + b2a + (a − b)2 .
2

3.19. If a, b ∈ (0, 1], then

1 1
 ‹
(a + b ) 2a + 2b
2 2
≤ 4.
a b

3.20. If a, b are positive real numbers such that a + b = 2, then

a b b a + 2 ≥ 3ab.

1
3.21. Let a, b be positive real numbers such that a + b = 2. If k ≥ , then
2
kb ka
a a b b ≥ 1.
442 Vasile Cîrtoaje

3.22. If a, b are positive real numbers such that a + b = 2, then


p p
a b
a b ≥ 1.

3.23. If a, b are positive real numbers such that a + b = 2, then


1
a a+1 b b+1 ≤ 1 − (a − b)4 .
48

3.24. If a, b are positive real numbers such that a + b = 2, then

a−a + b−b ≤ 2.

3.25. If a, b ∈ [0, 1], then

a b−a + b a−b + (a − b)2 ≤ 2.

3.26. If a, b are nonnegative real numbers such that a + b ≤ 2, then


7
a b−a + b a−b + (a − b)2 ≤ 2.
16

3.27. If a, b are nonnegative real numbers such that a + b ≤ 4, then

a b−a + b a−b ≤ 2.

3.28. If a, b are nonnegative real numbers such that a + b = 2, then

a2b + b2a ≥ a b + b a ≥ a2 b2 + 1.

3.29. If a, b are positive real numbers such that a + b = 2, then

a3b + b3a ≤ 2.

3.30. If a, b are nonnegative real numbers such that a + b = 2, then

a−b 4
 ‹
a +b +
3b 3a
≤ 2.
2
Symmetric Power-Exponential Inequalities 443

3.31. If a, b are positive real numbers such that a + b = 2, then


2 2
a a + b b ≤ 2.

3.32. If a, b are positive real numbers such that a + b = 2, then


3 3
a a + b b ≥ 2.

3.33. If a, b are positive real numbers such that a + b = 2, then


2 2
a5b + b5a ≤ 2.

3.34. If a, b are positive real numbers such that a + b = 2, then


p p
a2 b
+ b2 a
≤ 2.

3.35. If a, b are nonnegative real numbers such that a + b = 2, then

ab(1 − ab)2 ab(1 − ab)2


≤a b+1
+ b −2≤
a+1
.
2 3

3.36. If a, b are nonnegative real numbers such that a + b = 1, then

a2b + b2a ≤ 1.

3.37. If a, b are positive real numbers such that a + b = 1, then

2a a b b ≥ a2b + b2a .

3.38. If a, b are positive real numbers such that a + b = 1, then

a−2a + b−2b ≤ 4.

3.39. If a1 , a2 , . . . , an are positive real numbers such that a1 a2 · · · an = 1, then

1 a1 1 a2 1 an
 ‹  ‹  ‹
1− + 1− + ··· + 1 − ≤ n − 1.
n n n
444 Vasile Cîrtoaje
Symmetric Power-Exponential Inequalities 445

3.2 Solutions

P 3.1. If a, b are positive real numbers such that a + b = a4 + b4 , then


3 3
aa b b ≤ 1 ≤ aa b b .

(Vasile Cîrtoaje, 2008)

Solution. We will use the inequality

ln x ≤ x − 1, x > 0.

To prove this inequality, let us denote

f (x) = x − 1 − ln x, x > 0.

From
x −1
f 0 (x) = ,
x
it follows that f (x) is decreasing on (0, 1] and increasing on [1, ∞). Therefore,

f (x) ≥ f (1) = 0.

Using this inequality, we have

ln a a b b = a ln a + b ln b ≤ a(a − 1) + b(b − 1) = a2 + b2 − (a + b).

Therefore, the left inequality a a b b ≤ 1 is true if a2 + b2 ≤ a + b. We write this


inequality in the homogeneous form

(a2 + b2 )3 ≤ (a + b)2 (a4 + b4 ),

which is equivalent to the obvious inequality

ab(a − b)(a3 − b3 ) ≥ 0.

1
Taking now x = in the inequality ln x ≤ x − 1 yields
a
a ln a ≥ a − 1.

Similarly,
b ln b ≥ b − 1,
hence
3 3
ln a a b b = a3 ln a + b3 ln b ≥ a2 (a − 1) + b2 (b − 1) = a3 + b3 − (a2 + b2 ).
446 Vasile Cîrtoaje

3 3
Thus, to prove the right inequality a a b b ≥ 1, it suffices to show that a3 + b3 ≥
a2 + b2 , which is equivalent to the homogeneous inequality

(a + b)(a3 + b3 )3 ≥ (a4 + b4 )(a2 + b2 )3 .

We can write this inequality as


A − 3B ≥ 0,
where
A = (a + b)(a9 + b9 ) − (a4 + b4 )(a6 + b6 ),
B = a2 b2 (a2 + b2 )(a4 + b4 ) − a3 b3 (a + b)(a3 + b3 ).
Since
A = ab(a3 − b3 )(a5 − b5 ), B = a2 b2 (a − b)(a5 − b5 ),
we get
A − 3B = ab(a − b)3 (a5 − b5 ) ≥ 0.
Both inequalities become equalities for a = b = 1.

P 3.2. If a, b are positive real numbers, then

a2a + b2b ≥ a a+b + b a+b .

(Vasile Cîrtoaje, 2010)

Solution. Assume that a ≥ b and consider the following two cases.


Case 1: a ≥ 1. Write the inequality as

a a+b (a a−b − 1) ≥ b2b (b a−b − 1).

For b ≤ 1, we have

a a+b (a a−b − 1) ≥ 0 ≥ b2b (b a−b − 1).

For b ≥ 1, the inequality is also true since

a a+b ≥ a2b ≥ b2b , a a−b − 1 ≥ b a−b − 1 ≥ 0.

Case 2: a ≤ 1. Since
a2a + b2b ≥ 2a a b b ,
it suffices to show that
2a a b b ≥ a a+b + b a+b ,
which can be written as  a b  ‹a
b
+ ≤ 2.
b a
Symmetric Power-Exponential Inequalities 447

By Bernoulli’s inequality, we get


 ‹a 
 a b a−b b b−a a b(a − b) a(b − a)
‹  ‹
b
+ = 1+ + 1+ ≤1+ +1+ = 2.
b a b a b a
The equality holds for a = b.
Conjecture 1. If a, b are positive real numbers, then

a4a + b4b ≥ a2a+2b + b2a+2b .

Conjecture 2. If a, b, c are positive real numbers, then

a3a + b3b + c 3c ≥ a a+b+c + b a+b+c + c a+b+c .

Conjecture 3. If a, b, c, d are positive real numbers, then

a4a + b4b + c 4c + d 4d ≥ a a+b+c+d + b a+b+c+d + c a+b+c+d + d a+b+c+d .

P 3.3. If a, b are positive real numbers, then

aa + b b ≥ a b + ba .

(M. Laub, Israel, 1985, AMM)

Solution. Assume that a ≥ b. We will show that if a ≥ 1, then the inequality is


true. From
a a−b ≥ b a−b ,
we get
a b ba
bb ≥ .
aa
Therefore,

a b ba (a a − a b )(a a − b a )
aa + b b − a b − ba ≥ aa + − a b
− b a
= ≥ 0.
aa aa
Consider further the case 0 < b ≤ a < 1.
First Solution. Denoting
a
c = ab, d = bb, k= ,
b
where c ≥ d and k ≥ 1, the inequality becomes

c k − d k ≥ c − d.
448 Vasile Cîrtoaje

Since the function f (x) = x k is convex for x ≥ 0, from the well-known inequality

f (c) − f (d) ≥ f 0 (d)(c − d),

we get
c k − d k ≥ kd k−1 (c − d).
Thus, it suffices to show that
kd k−1 ≥ 1,
which is equivalent to
b1−a+b ≤ a.
Indeed, since 0 < 1 − a + b ≤ 1, by Bernoulli’s inequality, we get

b1−a+b = [1 + (b − 1)]1−a+b ≤ 1 + (1 − a + b)(b − 1) = a − b(a − b) ≤ a.

The equality holds for a = b.


Second Solution. Denoting

ba ab a
c= , d= , k= ,
a b + ba a b + ba b
where c + d = 1 and k ≥ 1, the inequality becomes

ck a + dk−b ≥ 1.

By the weighted AM-GM inequality, we have

ck a + dk−b ≥ k ac · k−bd = k ac−bd .

Thus, it suffices to show that ac ≥ bd; that is,

a1−b ≥ b1−a ,

which is equivalent to f (a) ≥ f (b), where


ln x
f (x) = .
1− x
It is enough to prove that f (x) is an increasing function. Since
g(x) 1
f 0 (x) = , g(x) = − 1 + ln x.
(1 − x)2 x
we need to show that g(x) ≥ 0 for x ∈ (0, 1). Indeed, from
x −1
g 0 (x) = < 0,
x2
it follows that g(x) is strictly decreasing, hence g(x) > g(1) = 0.
Symmetric Power-Exponential Inequalities 449

P 3.4. If a, b are positive real numbers, then

a2a + b2b ≥ a2b + b2a .

Solution. Without loss of generality, assume that a > b. We have two cases to
consider: a ≥ 1 and 0 < b < a < 1.
Case 1: a ≥ 1. From
a2(a−b) ≥ b2(a−b) ,
we get
a2b b2a
b2b ≥ .
a2a
Therefore,

a2b b2a (a2a − a2b )(a2a − b2a )


a2a + b2b − a2b − b2a ≥ a2a + − a 2b
− b 2a
= ≥0
a2a a2a
because a2a ≥ a2b and a2a ≥ b2a .
Case 2: 0 < b < a < 1. Denoting
a
c = ab, d = bb, k= ,
b
where c > d and k > 1, the inequality becomes

c 2k − d 2k ≥ c 2 − d 2 .

We will show that

c 2k − d 2k > k(cd)k−1 (c 2 − d 2 ) > c 2 − d 2 .

The left inequality follows from Lemma below for x = (c/d)2 . The right inequality
is equivalent to
k(cd)k−1 > 1,
b
(ab)a−b > ,
a
1+a− b
ln a > ln b.
1−a+ b
For fixed a, let us define
1+a− b
f (b) = ln a − ln b.
1−a+ b
If f 0 (b) < 0, then f (b) is strictly decreasing, and hence f (b) > f (a) = 0. Since
−2 1
f 0 (b) = ln a − ,
(1 − a + b)2 b
450 Vasile Cîrtoaje

we need to show that g(a) > 0, where

(1 − a + b)2
g(a) = 2 ln a + .
b
From
2 2(1 − a + b) 2(a − b)(a − 1)
g 0 (a) = − = < 0,
a b ab
it follows that g(a) is strictly decreasing, therefore g(a) > g(1) = b > 0. This
completes the proof. The equality holds for a = b.
Lemma. Let k and x be positive real numbers. If either k > 1 and x ≥ 1, or 0 < k < 1
and 0 < x ≤ 1, then
k−1
x k − 1 ≥ kx 2 (x − 1).
Proof. We need to show that f (x) ≥ 0, where
k−1
f (x) = x k − 1 − kx 2 (x − 1).

We have
1 k−3 k+1
f 0 (x) = kx 2 g(x), g(x) = 2x 2 − (k + 1)x + k − 1.
2
Since € Š
k−1
g (x) = (k + 1) x
0 2 − 1 ≥ 0,
g(x) is increasing. If x ≥ 1, then g(x) ≥ g(1) = 0, f (x) is increasing, hence
f (x) ≥ f (1) = 0. If 0 < x ≤ 1, then g(x) ≤ g(1) = 0, f (x) is decreasing, hence
f (x) ≥ f (1) = 0. The equality holds for x = 1.

Remark. The following more general results are valid (Vasile Cîrtoaje, 2006):
• Let 0 < k ≤ e.
(a) If a, b > 0, then
a ka + b kb ≥ a kb + b ka ;
(b) If a, b ∈ (0, 1], then
p
2 a ka b kb ≥ a kb + b ka .

Notice that these inequalities are known as the first and the second Vasc’s power
exponential inequalities.
Conjecture 1. If 0 < k ≤ e and either a, b ∈ (0, 4] or 0 < a ≤ 1 ≤ b, then
p
2 a ka b kb ≥ a kb + b ka .

Conjecture 2. If 0 < a ≤ 1 ≤ b, then


p
2 a3a b3b ≥ a3b + b3a .
Symmetric Power-Exponential Inequalities 451

Conjecture 3. If a, b ∈ (0, 5], then

2a a b b ≥ a2b + b2a .

Conjecture 4. If a, b ∈ [0, 5], then


‹ a+b
a2 + b2
 2

≥ a2b + b2a .
2

P 3.5. If a, b are nonnegative real numbers such that a + b = 2, then

(a) a b + b a ≤ 1 + ab;

(b) a2b + b2a ≤ 1 + ab.

Solution. Without loss of generality, assume that a ≥ b. Since

0 ≤ b ≤ 1, 0 ≤ a − 1 ≤ 1,

by Bernoulli’s inequality, we have

a b ≤ 1 + b(a − 1) = 1 + b − b2

and
b a = b · b a−1 ≤ b[1 + (a − 1)(b − 1)] = b2 (2 − b).
(a) We have

a b + b a − 1 − ab ≤ (1 + b − b2 ) + b2 (2 − b) − 1 − (2 − b)b = −b(b − 1)2 ≤ 0.

The equality holds for a = b = 1, for a = 2 and b = 0, and for a = 0 and b = 2.


(b) We have

a2b + b2a − 1 − ab ≤ (1 + b − b2 )2 + b4 (2 − b)2 − 1 − (2 − b)b


= b3 (b − 1)2 (b − 2) = −ab3 (b − 1)2 ≤ 0.

The equality holds for a = b = 1, for a = 2 and b = 0, and for a = 0 and b = 2.

2
P 3.6. If a, b are nonnegative real numbers such that ≤ a + b ≤ 2, then
3
a2b + b2a ≤ 1 + ab.

(Vasile Cîrtoaje, 2007)


452 Vasile Cîrtoaje

Solution. Assume that


a ≥ b.
p
From 2 ab ≤ a + b ≤ 2, we get
ab ≤ 1.
There are two cases to consider: a + b ≤ 1 and a + b ≥ 1.
2
Case 1: ≤ a + b ≤ 1. Since 2b ≤ 1, by Bernoulli’s inequality, we have
3
a2b ≤ 1 + 2b(a − 1) = 1 + 2ab − 2b.

Therefore, it suffices to show that

(1 + 2ab − 2b) + b2a ≤ 1 + ab,

which is equivalent to
ab + b2a ≤ 2b.
For 2a ≥ 1, this inequality is true since

ab ≤ b, b2a ≤ b.

For 2a ≤ 1, by Bernoulli’s inequality, we have

b2a ≤ 1 + 2a(b − 1) = 1 + 2ab − 2a.

Therefore, it suffices to show that

(1 + 2ab − 2b) + (1 + 2ab − 2a) ≤ 1 + ab,

which is equivalent to
1 + 3ab ≤ 2(a + b).
Indeed, we have

4 + 12ab − 8(a + b) ≤ 4 + 3(a + b)2 − 8(a + b)


= (a + b − 2)[3(a + b) − 2] ≤ 0.

Case 2: 1 ≤ a + b ≤ 2. For a, b ≤ 1, by Bernoulli’s inequality, we have

a2b = (a2 ) b ≤ 1 + b(a2 − 1) = 1 − b + a2 b,

b2a = (b2 )a ≤ 1 + a(b2 − 1) = 1 − a + ab2 ,


hence

a2b + b2a − 1 − ab ≤ (1 − b + a2 b) + (1 − a + ab2 ) − 1 − ab


= (1 − ab)(1 − a − b) ≤ 0.
Symmetric Power-Exponential Inequalities 453

Consider further that a ≥ 1 ≥ b. By Bernoulli’s inequality, we have

a b ≤ 1 + b(a − 1) = ab + 1 − b,

b2a = b a−1 · b a+1 ≤ b a+1 = b2 · b a−1 ≤ b2 [1 + (a − 1)(b − 1)]


= b2 (ab + 2 − a − b).

Therefore, it suffices to show that

(ab + 1 − b)2 + b2 (ab + 2 − a − b) ≤ 1 + ab,

which can be written as

1 + ab − (ab + 1 − b)2 ≥ b2 (ab + 2 − a − b).

Since
1 + ab − (ab + 1 − b)2 = bB,
where
B = (2 − a − b) + 2ab − a2 b ≥ 2ab − a2 b = ab(2 − a),
it is enough to prove that

ab2 (2 − a) ≥ b2 (ab + 2 − a − b),

which is equivalent to the obvious inequality

b2 (a − 1)(2 − a − b) ≥ 0.

The equality holds for a = 0 or b = 0. If a + b = 2, then the equality holds also for
a = b = 1.
Remark. Actually, the following extension is valid:
• If a, b are nonnegative real numbers such that
1
≤ a + b ≤ 2,
2
then
a2b + b2a ≤ 1 + ab.

P 3.7. If a, b are nonnegative real numbers such that a2 + b2 = 2, then

a2b + b2a ≤ 1 + ab.

(Vasile Cîrtoaje, 2007)


454 Vasile Cîrtoaje

Solution. Without loss of generality, assume that a ≥ 1 ≥ b. Applying Bernoulli’s


inequality gives
a b ≤ 1 + b(a − 1),
hence
a2b ≤ (1 + ab − b)2 .
Also, since 0 ≤ b ≤ 1 and 2a ≥ 2, we have

b2a ≤ b2 .

Therefore, it suffices to show that

(1 + ab − b)2 + b2 ≤ 1 + ab,

which can be written as

b(2 + 2ab − a − 2b − a2 b) ≥ 0.

So, we need to show that

2 + 2ab − a − 2b − a2 b ≥ 0,

which is equivalent to

4(1 − a)(1 − b) + a(2 − 2ab) ≥ 0,

4(1 − a)(1 − b) + a(a − b)2 ≥ 0.


Since a ≥ 1, it suffices to show that

4(1 − a)(1 − b) + (a − b)2 ≥ 0.

Indeed,

4(1 − a)(1 − b) + (a − b)2 = −4(a − 1)(1 − b) + [(a − 1) + (1 − b)]2


= [(a − 1) − (1 − b)]2 = (a + b − 2)2 ≥ 0.
p p
The equality holds for a = b = 1, for a = 2 and b = 0, and for a = 0 and b = 2.

1
P 3.8. If a, b are nonnegative real numbers such that a2 + b2 = , then
4

a2b + b2a ≤ 1 + ab.

(Vasile Cîrtoaje, 2007)


Symmetric Power-Exponential Inequalities 455

1
Solution. From a2 + b2 = , it follows that
4
1
a, b ≤ ,
2
1 1
ab = (a + b)2 − ,
2 8
p 1
a + b ≥ a2 + b2 = ,
2
Æ 1
a + b ≤ 2(a2 + b2 ) = p .
2
Applying Bernoulli’s inequality gives

a2b ≤ 1 + 2b(a − 1) = 1 − 2b + 2ab,

b2a ≤ 1 + 2a(b − 1) = 1 − 2a + 2ab.


Thus, it suffices to show that

(1 − 2b + 2ab) + (1 − 2a + 2ab) ≤ 1 + ab,

1 + 3ab ≤ 2(a + b),


3 3
1 + (a + b)2 − ≤ 2(a + b),
2 8
1 5
 ‹ ‹
a+b− a+b− ≤ 0.
2 6
The left inequality is true since
1 5
a+b≤ p < .
2 6
1 1
The equality holds for a = 0 and b = , and for a = and b = 0.
2 2
Remark. Actually, the following extended result is valid:
• If a, b are nonnegative real numbers such that
1
≤ a2 + b2 ≤ 2,
4
then
a2b + b2a ≤ 1 + ab.
1
This inequality is a consequence of Remark from P 3.6 (since ≤ a2 + b2 ≤ 2
4
1
involves ≤ a + b ≤ 2).
2
456 Vasile Cîrtoaje

P 3.9. If a, b are positive real numbers, then


a+b
a a b b ≤ (a2 − ab + b2 ) 2 .

Solution. By the weighted AM-GM inequality, we have


a b
a · a + b · b ≥ (a + b)a a+b b a+b ,
‹a+b
a2 + b2

≥ aa b b .
a+b
Thus, it suffices to show that
‹2
a2 + b2

a − ab + b ≥
2 2
,
a+b

which is equivalent to

(a + b)(a3 + b3 ) ≥ (a2 + b2 )2 ,

ab(a − b)2 ≥ 0.
The equality holds for a = b.

P 3.10. If a, b ∈ (0, 1], then

a a b b ≤ 1 − ab + a2 b2 .

(Vasile Cîrtoaje, 2010)

Solution. We claim that


x x ≤ 1 − x + x2
for all x ∈ (0, 1]. If this is true, then

1 − ab + a2 b2 − a a b b ≥ 1 − ab + a2 b2 − (1 − a + a2 )(1 − b + b2 )
= (a + b)(1 − a)(1 − b) ≥ 0.

Thus, it suffices to show that f (x) ≤ 0 for x ∈ (0, 1], where

f (x) = x ln x − ln(x 2 − x + 1).

We have
2x − 1
f 0 (x) = ln x + 1 − ,
x2 − x +1
Symmetric Power-Exponential Inequalities 457

(1 − x)(1 − 2x − x 2 − x 4 )
f 00 (x) = .
x(x 2 − x + 1)2
Let x 1 ∈ (0, 1) be the positive root of the equation x 4 + x 2 +2x = 1. Then, f 00 (x) > 0
for x ∈ (0, x 1 ) and f 00 (x) < 0 for x ∈ (x 1 , 1), hence f 0 is strictly increasing on (0, x 1 ]
and strictly decreasing on [x 1 , 1]. Since lim x→0 f 0 (x) = −∞ and f 0 (1) = 0, there
is x 2 ∈ (0, x 1 ) such that f 0 (x 2 ) = 0, f 0 (x) < 0 for x ∈ (0, x 2 ) and f 0 (x) > 0 for
x ∈ (x 2 , 1). Therefore, f is decreasing on (0, x 2 ] and increasing on [x 2 , 1]. Since
lim x→0 f (x) = 0 and f (1) = 0, it follows that f (x) ≤ 0 for x ∈ (0, 1]. The proof is
completed. The equality holds for a = b = 1.

P 3.11. If a, b are positive real numbers such that a + b ≤ 2, then


 a  b  b ‹a
+ ≤ 2.
b a

(Vasile Cîrtoaje, 2010)

Solution. Using the substitution a = t c and b = t d, where c, d, t are positive real


numbers such that c + d = 2 and t ≤ 1, we need to show that
 c t d  ‹t c
d
+ ≤ 2.
d c

Write this inequality as


f (t) ≤ 2,
where  c d  ‹c
d
f (t) = A + B ,
t t
A= , B= .
d c
Since f (t) is a convex function, we have

f (t) ≤ max{ f (0), f (1)} = max{2, f (1)}.

Therefore, we only need to show that f (1) ≤ 2; that is,

2c c d d ≥ c 2 + d 2 .

Setting c = 1 + x and d = 1 − x, where 0 ≤ x < 1, this inequality turns into

(1 + x)1+x (1 − x)1−x ≥ 1 + x 2 ,

which is equivalent to f (x) ≥ 0, where

f (x) = (1 + x) ln(1 + x) + (1 − x) ln(1 − x) − ln(1 + x 2 ).


458 Vasile Cîrtoaje

We have
2x
f 0 (x) = ln(1 + x) − ln(1 − x) − ,
1 + x2
1 1 2(1 − x 2 ) 8x 2
f 00 (x) = + − = .
1 + x 1 − x (1 + x 2 )2 (1 − x 2 )(1 + x 2 )2
Since f 00 (x) ≥ 0 for x ∈ [0, 1), it follows that f 0 is increasing, f 0 (x) ≥ f 0 (0) = 0,
f (x) is increasing, hence f (x) ≥ f (0) = 0. The proof is completed. The equality
holds for a = b.

P 3.12. If a, b are positive real numbers such that a + b = 2, then

3
2a a b b ≥ a2b + b2a + (a − b)2 .
4
(Vasile Cîrtoaje, 2010)

Solution. According to the inequalities in P 3.5-(b) and P 3.11 (for a + b = 2), we


have
a2b + b2a ≤ 1 + ab
and
2a a b b ≥ a2 + b2 .
Therefore, it suffices to show that

3
a2 + b2 ≥ 1 + ab + (a − b)2 .
4
which is an identity. The equality holds for a = b = 1.

P 3.13. If a, b ∈ (0, 1] or a, b ∈ [1, ∞), then

2a a b b ≥ a2 + b2 .

Solution. For a = x and b = 1, the desired inequality becomes

2x x ≥ x 2 + 1, x > 0.

If this inequality is true, then

4a a b b − 2(a2 + b2 ) ≥ (a2 + 1)(b2 + 1) − 2(a2 + b2 ) = (a2 − 1)(b2 − 1) ≥ 0.


Symmetric Power-Exponential Inequalities 459

To prove the inequality 2x x ≥ x 2 + 1, we show that f (x) ≥ 0, where

f (x) = ln 2 + x ln x − ln(x 2 + 1).

We have
2x
f 0 (x) = ln x + 1 − ,
x2 + 1
x 2 (x + 1)2 + (x − 1)2
f (x) =
00
.
x(x 2 + 1)2
Since f 00 (x) > 0 for x > 0, f 0 is strictly increasing. Since f 0 (1) = 0, it follows that
f 0 (x) < 0 for x ∈ (0, 1) and f 0 (x) > 0 for x ∈ (1, ∞). Therefore, f is decreasing on
(0, 1] and increasing on [1, ∞), hence f (x) ≥ f (1) = 0 for x > 0. This completes
the proof. The equality holds for a = b = 1.

P 3.14. If a, b are positive real numbers, then

2a a b b ≥ a2 + b2 .

(Vasile Cîrtoaje, 2014)


Solution. By Lemma below, it suffices to show that

(a4 − 2a3 + 4a2 − 2a + 3)(b4 − 2b3 + 4b2 − 2b + 3) ≥ 8(a2 + b2 ),

which is equivalent to A ≥ 0, where

A =a4 b4 − 2a3 b3 (a + b) + 4a2 b2 (a2 + b2 + ab) − [2ab(a3 + b3 ) + 8a2 b2 (a + b)]


+ [3(a4 + b4 ) + 4ab(a2 + b2 ) + 16a2 b2 ] − [6(a3 + b3 ) + 8ab(a + b)]
+ 4(a2 + b2 + ab) − 6(a + b) + 9.

We can check that

A = [a2 b2 − ab(a + b) + a2 + b2 − 1]2 + B,

where

B =a2 b2 (a + b)2 − 6a2 b2 (a + b) + [2(a4 + b4 ) + 4ab(a2 + b2 ) + 16a2 b2 ]


− [6(a3 + b3 ) + 10ab(a + b)] + [6(a2 + b2 ) + 4ab] − 6(a + b) + 8.

Also, we have
B = [ab(a + b) − 3ab + 1]2 + C,
where

C =[2(a4 + b4 ) + 4ab(a2 + b2 ) + 7a2 b2 ] − [6(a3 + b3 ) + 12ab(a + b)]


+ [6(a2 + b2 ) + 10ab] − 6(a + b) + 7,
460 Vasile Cîrtoaje

and
C = (ab − 1)2 + 2D,
where

D =[a4 + b4 + 2ab(a2 + b2 ) + 3a2 b2 ] − [3(a3 + b3 ) + 6ab(a + b)]


+ 3(a + b)2 − 3(a + b) + 3,

It suffices to show that D ≥ 0. Indeed,

D =[(a + b)4 − 2ab(a + b)2 + a2 b2 ] − 3[(a + b)3 − ab(a + b)]


+ 3(a + b)2 − 3(a + b) + 3
=[(a + b)2 − ab]2 − 3(a + b)[(a + b)2 − ab] + 3(a + b)2 − 3(a + b) + 3
˜2 ‹2
3 a+b
• 
= (a + b)2 − ab − (a + b) + 3 − 1 ≥ 0.
2 2
This completes the proof. The equality holds for a = b = 1.
Lemma. If x > 0, then
1
x x ≥ x + (x − 1)2 (x 2 + 3).
4
Proof. We need to show that f (x) ≥ 0 for x > 0, where

f (x) = ln 4 + x ln x − ln g(x), g(x) = x 4 − 2x 3 + 4x 2 − 2x + 3.

We have
2(2x 3 − 3x 2 + 4x − 1)
f 0 (x) = 1 + ln x − ,
g(x)
x 8 + 6x 4 − 32x 3 + 48x 2 − 32x + 9 (x − 1)2 h(x)
f 00 (x) = = ,
g 2 (x) g 2 (x)
where
h(x) = x 6 + 2x 5 + 3x 4 + 4x 3 + 11x 2 − 14x + 9.
Since
h(x) > 7x 2 − 14x + 7 = 7(x − 1)2 ≥ 0,
we have f 00 (x) ≥ 0, hence f 0 is strictly increasing on (0, ∞). Since f 0 (1) = 0, it
follows that f 0 (x) < 0 for x ∈ (0, 1) and f 0 (x) > 0 for x ∈ (1, ∞). Therefore, f is
decreasing on (0, 1] and increasing on [1, ∞), hence f (x) ≥ f (1) = 0 for x > 0.

P 3.15. If a ≥ 1 ≥ b > 0, then

2a a b b ≥ a2b + b2a .
Symmetric Power-Exponential Inequalities 461

Solution. Taking into account the inequality 2a a b b ≥ a2 + b2 from the preceding


P 3.14, it suffices to show that

a2 + b2 ≥ a2b + b2a .

This inequality follows immediately from a2 ≥ a2b and b2 ≥ b2a . The equality holds
for a = b = 1.

P 3.16. If a ≥ e ≥ b > 0, then

2a a b b ≥ a2b + b2a .

Solution. It suffices to show that a a b b ≥ a2b and a a b b ≥ b2a . Write the first in-
equality as
 a b
a a−b ≥ ,
b
a
a x−1 ≥ x, x = ≥ 1.
b
Since a x−1 ≥ e x−1 , we only need to show that

e x−1 ≥ x,

which is equivalent to f (x) ≥ 0 for x ≥ 1, where

f (x) = x − 1 − ln x.

From
1
f 0 (x) = 1 − ≥ 0,
x
it follows that f is increasing on [1, ∞), therefore f (x) ≥ f (1) = 0.
Write the second inequality as
 ‹a
b
b a−b ≤ 1,
a
b
x b1−x ≤ 1, x= ≤ 1.
a
Since b1−x ≤ e1−x , we only need to show that

x e1−x ≤ 1,

which is equivalent to f (x) ≤ 0 for x ≤ 1, where

f (x) = ln x + 1 − x.
462 Vasile Cîrtoaje

Since
1
f 0 (x) =− 1 ≥ 0,
x
f is increasing on (0, 1], therefore f (x) ≤ f (1) = 0. This completes the proof. The
equality holds for a = b = e.

P 3.17. If a, b are positive real numbers, then


‹ a+b
a2 + b2
 2
a b
a b ≥ .
2

First Solution. Using the substitution a = bx, where x > 0, the inequality be-
comes as follows:
‹ bx+b
b x + b2 2
 2 2
(bx) b ≥
bx b
,
2
x+1
+
 2 2 2‹ 2
b x b
(bx) x b ≥ ,
2
‹ x+1
+
 2
x 1 2

b x+1 x x ≥ b x+1 ,
2
‹ x+1
x +1 2
 2
x
x ≥ .
2
It is true if f (x) ≥ 0 for all x > 0, where

x 1 x2 + 1
f (x) = ln x − ln .
x +1 2 2
We have
1 1 x g(x)
f 0 (x) = ln x + − = ,
(x + 1)2 x + 1 x 2 + 1 (x + 1)2
where
x2 − 1
g(x) = ln x − .
x2 + 1
Since
(x 2 − 1)2
g 0 (x) = ≥ 0,
x(x 2 + 1)2
g is strictly increasing, therefore g(x) < 0 for x ∈ (0, 1), g(1) = 0, g(x) > 0
for x ∈ (1, ∞). Thus, f is decreasing on (0, 1] and increasing on [1, ∞), hence
f (x) ≥ f (1) = 0. This completes the proof. The equality holds for a = b.
Symmetric Power-Exponential Inequalities 463

Second Solution. Write the inequality in the form

a + b a2 + b2
a ln a + b ln b ≥ ln .
2 2
Without loss of generality, consider a + b = 2k, k > 0, and denote

a = k + x, b = k − x, 0 ≤ x < k.

We need to show that f (x) ≥ 0, where

f (x) = (k + x) ln(k + x) + (k − x) ln(k − x) − k ln(x 2 + k2 ).

We have
2kx
f 0 (x) = ln(k + x) − ln(k − x) − ,
x2+ k2

1 1 2k(x 2 − k2 )
f (x) =
00
+ +
k+x k−x (x 2 + k2 )2
2 2
8k x
= 2 .
(k − x 2 )(x 2 + k2 )2

Since f 00 (x) ≥ 0 for x ≥ 0, f 0 is increasing, hence f 0 (x) ≥ f 0 (0) = 0. Therefore, f


is increasing on [0, k), hence f (x) ≥ f (0) = 0.

Remark. For a + b = 2, this inequality can be rewritten as

2a a b b ≥ a2 + b2 ,
 a  b  b ‹a
2≥ + .
b a
Also, for a + b = 1, the inequality becomes

2a2a b2b ≥ a2 + b2 ,
 a 2b  ‹2a
b
2≥ + .
b a

P 3.18. If a, b are positive real numbers such that a2 + b2 = 2, then

1
2a a b b ≥ a2b + b2a + (a − b)2 .
2
(Vasile Cîrtoaje, 2010)
464 Vasile Cîrtoaje

Solution. According to the inequalities in P 3.7 and P 3.17, we have

a2b + b2a ≤ 1 + ab

and
a a b b ≥ 1.
Therefore, it suffices to show that

1
2 ≥ 1 + ab + (a − b)2 ,
2
which is an identity. The equality holds for a = b = 1.

P 3.19. If a, b ∈ (0, 1], then

1 1
 ‹
(a + b ) 2a + 2b
2 2
≤ 4.
a b

(Vasile Cîrtoaje, 2014)

Solution. For a = x and b = 1, the desired inequality becomes

1 + x2
x 2x ≥ , x ∈ (0, 1].
3 − x2
If this inequality is true, it suffices to show that

3 − a2 3 − b2
 ‹
(a + b )
2 2
+ ≤ 4,
1 + a2 1 + b2

which is equivalent to

a2 b2 (2 + a2 + b2 ) + 2 − (a2 + b2 ) − (a2 + b2 )2 ≥ 0,

(2 + a2 + b2 )(1 − a2 )(1 − b2 ) ≥ 0.
1 + x2
To prove the inequality x 2x ≥ , we show that f (x) ≥ 0, where
3 − x2
1 1
f (x) = x ln x + ln(3 − x 2 ) − ln(1 + x 2 ), x ∈ (0, 1].
2 2
We have
x x
f 0 (x) = 1 + ln x − − ,
3− x 2 1 + x2
Symmetric Power-Exponential Inequalities 465

1 3 + x2 1 − x2
f 00 (x) = − −
x (3 − x 2 )2 (1 + x 2 )2
(1 − x)(9 + 6x − x 3 ) 1 − x2
= − .
x(3 − x)2 (1 + x 2 )2

We will show that f 00 (x) > 0 for 0 < x < 1. This is true if

9 + 6x − x 3 1+ x
− > 0.
x(3 − x)2 (1 + x 2 )2
Indeed,
9 + 6x − x 3 1+ x 9 1+ x 1
− > − = > 0.
x(3 − x)2 (1 + x )
2 2 9x x(1 + x)2 1+ x
Since f 00 (x) > 0, f 0 is strictly increasing on (0, 1]. Since f 0 (1) = 0, it follows that
f 0 (x) < 0 for x ∈ (0, 1), f is strictly decreasing on (0, 1], hence f (x) ≥ f (1) = 0.
This completes the proof. The equality holds for a = b = 1.

P 3.20. If a, b are positive real numbers such that a + b = 2, then

a b b a + 2 ≥ 3ab.

(Vasile Cîrtoaje, 2010)

Solution. Setting
a = 1 + x, b = 1 − x, 0 ≤ x < 1,
the inequality is equivalent to

(1 + x)1−x (1 − x)1+x ≥ 1 − 3x 2 .
1
Consider further the nontrivial case 0 ≤ x < p , and write the desired inequality
3
as f (x) ≥ 0, where

f (x) = (1 − x) ln(1 + x) + (1 + x) ln(1 − x) − ln(1 − 3x 2 ).

We have
1− x 1+ x 6x
f 0 (x) = − ln(1 + x) + ln(1 − x) + − + ,
1 + x 1 − x 1 − 3x 2
1 00 −1 2(x 2 + 1) 3(3x 2 + 1)
f (x) = − + .
2 1 − x 2 (1 − x 2 )2 (1 − 3x 2 )2
Making the substitution
1
t = x 2, 0≤t < ,
3
466 Vasile Cîrtoaje

we get
1 00 3(3t + 1) t +3 4t(5 − 9t)
f (x) = − = > 0.
2 (3t − 1)2 (t − 1)2 (t − 1)2 (3t − 1)2
Therefore, f 0 (x) is strictly increasing, f 0 (x) ≥ f 0 (0) = 0, f (x) is strictly increasing,
hence f (x) ≥ f (0) = 0. This completes the proof. The equality holds for a = b = 1.

1
P 3.21. Let a, b be positive real numbers such that a + b = 2. If k ≥ , then
2
kb ka
a a b b ≥ 1.

(Vasile Cîrtoaje, 2010)

Solution. Setting
a = 1 + x, b = 1 − x, 0 ≤ x < 1,
the inequality can be written as

(1 + x)k(1−x) ln(1 + x) + (1 − x)k(1+x) ln(1 − x) ≥ 0.

Consider further the nontrivial case 0 < x < 1, and write the desired inequality as
f (x) ≥ 0, where

f (x) = k(1 − x) ln(1 + x) − k(1 + x) ln(1 − x) + ln ln(1 + x) − ln(− ln(1 − x)).

It suffices to show that f 0 (x) > 0. Indeed, if this is true, then f (x) is strictly
increasing, hence
f (x) > lim f (x) = 0.
x→0

We have

2k(1 + x 2 ) 1 1
f 0 (x) = − k ln(1 − x 2 ) + +
1− x 2 (1 + x) ln(1 + x) (1 − x) ln(1 − x)
2k 1 1
> + +
1− x 2 (1 + x) ln(1 + x) (1 − x) ln(1 − x)
1 1 1
≥ + +
1 − x 2 (1 + x) ln(1 + x) (1 − x) ln(1 − x)
g(x)
= ,
(1 − x 2 ) ln(1 + x) ln(1 − x)

where

g(x) = ln(1 + x) ln(1 − x) + (1 + x) ln(1 + x) + (1 − x) ln(1 − x).


Symmetric Power-Exponential Inequalities 467

It is enough to how that g(x) < 0. We have


−x
g 0 (x) = h(x),
1 − x2
where
h(x) = (1 + x) ln(1 + x) + (1 − x) ln(1 − x).
Since
1+ x
h0 (x) = ln > 0,
1− x
h(x) is strictly increasing, h(x) > h(0) = 0, g 0 (x) < 0, g(x) is strictly decreasing,
and hence g(x) < g(0) = 0. This completes the proof. The equality holds for
a = b = 1.

P 3.22. If a, b are positive real numbers such that a + b = 2, then


p p
a b
a b ≥ 1.

(Vasile Cîrtoaje, 2010)

Solution. Assume that a > 1 > b. Taking logarithms of both sides, the inequality
becomes in succession: p p
a ln a + b ln b ≥ 0,
p p
a ln a ≥ b(− ln b),
1 1
ln a + ln ln a ≥ ln b + ln(− ln b).
2 2
Substituting
a = 1 + x, b = 1 − x, 0 < x < 1,
we need to show that f (x) ≥ 0, where
1 1
f (x) = ln(1 + x) − ln(1 − x) + ln ln(1 + x) − ln(− ln(1 − x)).
2 2
We have
1 1 1
f 0 (x) = + + .
1− x 2 (1 + x) ln(1 + x) (1 − x) ln(1 − x)
As shown in the proof of the preceding P 3.21, we have f 0 (x) > 0. Therefore, f (x)
is strictly increasing, therefore

f (x) > lim f (x) = 0.


x→0

The equality holds for a = b = 1.


468 Vasile Cîrtoaje

P 3.23. If a, b are positive real numbers such that a + b = 2, then


1
a a+1 b b+1 ≤ 1 − (a − b)4 .
48
(Vasile Cîrtoaje, 2010)

Solution. Putting

a = 1 + x, b = 1 − x, 0 ≤ x < 1,

the inequality becomes


1 4
(1 + x)2+x (1 − x)2−x ≤ 1 − x .
3
Write this inequality as f (x) ≤ 0, where

1 4
 ‹
f (x) = (2 + x) ln(1 + x) + (2 − x) ln(1 − x) − ln 1 − x .
3
We have
2x 4x 3
f 0 (x) = ln(1 + x) − ln(1 − x) − + ,
1 − x2 3 − x4
2 2(1 + x 2 ) 4x 2 (x 4 + 9)
f 00 (x) = − +
1 − x 2 (1 − x 2 )2 (3 − x 4 )2
−4x 2 4x 2 (x 4 + 9) −8x 4 [x 4 + 1 + 8(1 − x 2 )]
= + = ≤ 0.
(1 − x 2 )2 (3 − x 4 )2 (1 − x 2 )2 (3 − x 4 )2

Therefore, f 0 (x) is decreasing, f 0 (x) ≤ f 0 (0) = 0, f (x) is decreasing, f (x) ≤


f (0) = 0. The equality holds for a = b = 1.

P 3.24. If a, b are positive real numbers such that a + b = 2, then

a−a + b−b ≤ 2.

(Vasile Cîrtoaje, 2010)

Solution. Consider a ≥ b, when we have

0 < b ≤ 1 ≤ a < 2,

and write the inequality as

aa − 1 b b − 1
+ ≥ 0.
aa bb
Symmetric Power-Exponential Inequalities 469

According to Lemma from the proof of P 3.4, we have


a+1 b+1
aa − 1 ≥ a 2 (a − 1), bb − 1 ≥ b 2 (b − 1).

Therefore, it suffices to show that


1−a 1−b
a 2 (a − 1) + b 2 (b − 1) ≥ 0,

which is equivalent to
1−a 1−b
a 2 ≥b 2 ,
1−b
(ab) 2 ≤ 1,
ab ≤ 1,
(a − b)2 ≥ 0.
The equality holds for a = b = 1.

P 3.25. If a, b ∈ [0, 1], then

a b−a + b a−b + (a − b)2 ≤ 2.

(Vasile Cîrtoaje, 2010)

Solution (by Vo Quoc Ba Can). Without loss of generality, assume that a ≥ b. Us-
ing the substitution
c = a − b,
we need to show that
(b + c)−c + b c + c 2 ≤ 2
for
0 ≤ b ≤ 1 − c, 0 ≤ c ≤ 1.
If c = 1, then b = 0, and the inequality is an equality. Also, for c = 0, the inequality
is an equality. Consider further that

0 < c < 1.

We need to show that f (x) ≤ 0, where

f (x) = (x + c)−c + x c + c 2 − 2, x ∈ [0, 1 − c].

We claim that f 0 (x) > 0 for x > 0. On this assumption, f (x) is strictly increasing
on [0, 1 − c], hence

f (x) ≤ f (1 − c) = (1 − c)c − (1 − c 2 ).
470 Vasile Cîrtoaje

By Bernoulli’s inequality, we have

f (x) ≤ 1 + c(−c) − (1 − c 2 ) = 0.

Since
c[(x + c)1+c − x 1−c ]
f (x) =
0
,
(x + c)1+c x 1−c
the inequality f 0 (x) > 0 holds for x > 0 if and only if
1−c
x + c > x 1+c .

For any d > 0, using the weighted AM-GM inequality yields


d
c 1
 c  1+d
x + c = x + d · ≥ (1 + d)x 1+d .
d d

Choosing
2c
d= ,
1−c
we get
c−1
1+c 1−c
 ‹ 1+c
1−c
x +c ≥ x 1+c .
2 2
Thus, it suffices to show that
‹ 1−c
1+c 1 − c 1+c

≥ .
2 2

Indeed, using Bernoulli’s inequality, we get


‹ 1−c ‹ 1−c
1−c 1 + c 1+c 1−c 1+c 1+c
 1+c

= 1− ≤1− · = .
2 2 1+c 2 2

The equality holds for a = b, for a = 1 and b = 0, and for a = 0 and b = 1.

P 3.26. If a, b are nonnegative real numbers such that a + b ≤ 2, then

7
a b−a + b a−b + (a − b)2 ≤ 2.
16

(Vasile Cîrtoaje, 2010)


Symmetric Power-Exponential Inequalities 471

Solution. Assume that a ≥ b. Using the substitution

c = a − b,

we need to show that


7 2
a−c + (a − c)c + c ≤2
16
for
c
0 ≤ c ≤ 2, c ≤ a ≤1+ .
2
For c = 0 and c = 2 (which involves a = 2), the inequality is an equality. Therefore,
we only need to show that f (x) ≤ 0 for 0 < c < 2, where

7 2 h ci
f (x) = x −c + (x − c)c + x ∈ c, 1 +
c − 2, .
16 2

3
• ˜
In the case c = 1, we need to show that f (x) ≤ 0 for x ∈ 1, ; indeed, we have
2

1 41 2 3 41 −19
f (x) = +x− ≤ + − = < 0.
x 16 3 2 16 48

Consider next that


c ∈ (0, 1) ∪ (1, 2).

The derivative
c[x 1+c − (x − c)1−c ]
f 0 (x) =
x 1+c (x − c)1−c
has the same sign as

g(x) = (1 + c) ln x − (1 − c) ln(x − c).

We have
c(2x − 1 − c)
g 0 (x) = .
x(x − c)
 ci
Case 1: 0 < c < 1. We claim that g(x) > 0 for x ∈ c, 1 + . On this assumption,
2
h ci
f is strictly increasing on c, 1 + , hence
2
 c
f (x) ≤ f 1 + .
2
 c
Thus, we need to show that f 1 + ≤ 0, which is just the inequality in Lemma
2
4 below.
472 Vasile Cîrtoaje

1+c
 ˜
From the expression of g (x), it follows that g(x) is decreasing on c,
0
,
2
1+c
• ˜
c  ci
and increasing on , 1 + . Then, to show that g(x) > 0 for x ∈ c, 1 + ,
2 2 2
it suffices to prove that
1+c
 ‹
g > 0,
2
which is equivalent to
1 + c 1+c 1 − c 1−c
 ‹  ‹
> .
2 2
This inequality follows from Bernoulli’s inequality, as follows:

1 + c 1+c 1 − c 1+c (1 + c)(1 − c) 1 + c 2


 ‹  ‹
= 1− >1− =
2 2 2 2
and
1 − c 1−c 1 + c 1−c (1 − c)(1 + c) 1 + c 2
 ‹  ‹
= 1− <1− = .
2 2 2 2
Case 2: 1 < c < 2. Since

2x − 1 − c ≥ 2c − 1 − c = c − 1 > 0,

it follows that g 0 (x) > 0, hence g(x) is strictly increasing. For x → c, we have
g(x) → −∞. If g(1 + c/2) ≤ 0, then g(x) ≤ 0, hence f is decreasing. If g(1 +
c/2) > 0, then there exists x 1 ∈ (c, 1 + c/2) such that g(x 1 ) = 0, g(x) < 0 for
x ∈ [c, x 1 ) and g(x) > 0 for x ∈ (x 1 , 1 + c/2], hence f is decreasing on [c, x 1 ]
and
 increasing on [x 1 , 1 + c/2]. Therefore, it suffices to show that f (c) ≤ 0 and
c
f 1+ ≤ 0. These inequalities follow respectively from Lemma 1 and Lemma 4
2
below.
The proof is completed. The equality holds for a = b, for a = 2 and b = 0, and
for a = 0 and b = 2.
Lemma 1. If 1 ≤ c ≤ 2, then
7 2
c −c + c ≤ 2,
16
with equality for c = 2.
Proof. The desired inequality is equivalent to h(c) ≥ 0, where
7 2
 ‹
h(c) = c ln c + ln 2 − c , c ∈ [1, 2].
16
We have
14c
h0 (c) = 1 + ln c − ,
32 − 7c 2
1 14(32 + 7c 2 )
h00 (c) = − .
c (32 − 7c 2 )2
Symmetric Power-Exponential Inequalities 473

Since h00 is strictly decreasing, h00 (1) = 79/625 and h00 (2) = −52, there exists c1 ∈
(1, 2) such that h00 (c1 ) = 0, h00 (c) > 0 for c ∈ [1, c1 ) and h00 (c) < 0 for c ∈ (c1 , 2],
hence h0 is strictly increasing on [1, c1 ] and strictly decreasing on [c1 , 2]. Since
h0 (1) = 11/25 and h0 (2) = ln 2 − 6 < 0, there exists c2 ∈ (1, 2) such that h0 (c2 ) = 0,
h0 (c) > 0 for c ∈ [1, c2 ) and h0 (c) < 0 for c ∈ (c2 , 2], hence h is strictly increasing
on [1, c2 ] and strictly decreasing on [c2 , 2]. Thus, it suffices to show that h(1) ≥ 0
and h(2) ≥ 0. Indeed, h(1) = ln 25 − ln 16 > 0 and h(2) = 0.
Lemma 2. If 0 ≤ x ≤ 2, then
 x −x 3 2
1+ + x ≤ 1,
2 16
with equality for x = 0 and x = 2.
Proof. We need to show that f (x) ≤ 0, where

3 2
 ‹
 x
f (x) = −x ln 1 + − ln 1 − x , x ∈ [0, 2].
2 16
We have  x x(3x 2 + 6x − 4)
f 0 (x) = − ln 1 + + ,
2 (2 + x)(16 − 3x 2 )
g(x)
f 00 (x) = ,
(2 + x)2 (16 − 3x 2 )2
where
g(x) = −9x 5 − 18x 4 + 168x 3 + 552x 2 + 128x − 640.
Since g(x 1 ) = 0 for x 1 ≈ 0, 88067, g(x) < 0 for x ∈ [0, x 1 ) and g(x) > 0 for
x ∈ (x 1 , 2], f 0 is strictly decreasing on [0, x 1 ] and strictly increasing on [x 1 , 2].
5
Since f 0 (0) = 0 and f 0 (2) = − ln 2+ > 0, there is x 2 ∈ (x 1 , 2) such that f 0 (x 2 ) = 0,
2
f 0 (x) < 0 for x ∈ (0, x 2 ), and f 0 (x) > 0 for x ∈ (x 2 , 2]. Therefore, f is decreasing
on [0, x 2 ] and increasing on [x 2 , 2]. Since f (0) = f (2) = 0, it follows that f (x) ≤ 0
for x ∈ [0, 2].
Lemma 3. If 0 ≤ x ≤ 2, then
 x x 1 2
1− + x ≤ 1,
2 4
with equality for x = 0 and x = 2.
Proof. We need to show that f (x) ≤ 0, where

1 2
 ‹
 x
f (x) = x ln 1 − − ln 1 − x , x ∈ [0, 2).
2 4
We have  x x2
f (x) = ln 1 −
0
− ,
2 4 − x2
474 Vasile Cîrtoaje

−1 8x
f 00 (x) = − .
2 − x (4 − x 2 )2
Since f 00 < 0 for x ∈ [0, 2), f 0 is strictly decreasing, hence f 0 (x) ≤ f 0 (0) = 0, f is
strictly decreasing, therefore f (x) ≤ f (0) = 0 for x ∈ [0, 2).
Lemma 4. If 0 ≤ x ≤ 2, then
 x −x  x x 7 2
1+ + 1− + x ≤ 2,
2 2 16
with equality for x = 0 and x = 2.
Proof. By Lemma 2 and Lemma 3, we have
 x −x 3 2
1+ + x ≤1
2 16
and  x x 1 2
1− + x ≤ 1.
2 4
The desired inequality follows by adding up these inequalities.
1
Conjecture. If a, b are nonnegative real numbers such that a + b = , then
4

a2(b−a) + b2(a−b) ≤ 2.

P 3.27. If a, b are nonnegative real numbers such that a + b ≤ 4, then

a b−a + b a−b ≤ 2.

(Vasile Cîrtoaje, 2010)

Solution. Without loss of generality, assume that a ≥ b. Consider first that a − b ≥


2. We have
a ≥ a − b ≥ 2,
and from
4 ≥ a + b = (a − b) + 2b ≥ 2 + 2b,
we get b ≤ 1. Clearly, the desired inequality is true because

a b−a < 1, b a−b ≤ 1.

Since the case a − b = 0 is trivial, consider further that 0 < a − b < 2 and use the
substitution
c = a − b.
Symmetric Power-Exponential Inequalities 475

So, we need to show that


a−c + (a − c)c ≤ 2
for
c
0 < c < 2, c ≤ a ≤2+ .
2
Equivalently, we need to show that f (x) ≤ 0 for 0 < c < 2, where
h ci
f (x) = x −c + (x − c)c − 2, x ∈ c, 2 + .
2
The derivative
c[x 1+c − (x − c)1−c ]
f 0 (x) =
x 1+c (x − c)1−c
has the same sign as

g(x) = (1 + c) ln x − (1 − c) ln(x − c).

We have
c(2x − 1 − c)
g 0 (x) = .
x(x − c)
5
˜ •
Case 1: c = 1. We need to show that x − 3x + 1 ≤ 0 for x ∈ 1, . Indeed, we
2
2
have
2(x 2 − 3x + 1) = (x − 1)(2x − 5) + (x − 3) < 0.
 ci
Case 2: 0 < c < 1. We will show that g(x) > 0 for x ∈ c, 2 + . From
2
c(2x − 1 − c)
g 0 (x) = ,
x(x − c)

1+c 1+c
 ˜ • ˜
c
it follows that g(x) is decreasing on c, and increasing on ,2 + .
2 i 2 2
 c
Then, to show that g(x) > 0 for x ∈ c, 1 + , it suffices to prove that
2
1+c
 ‹
g > 0,
2

which is equivalent to
‹1+c ‹1−c
1+c 1−c
 
> .
2 2
This inequality follows from Bernoulli’s inequality, as follows:
‹1+c
1+c 1 − c 1+c (1 + c)(1 − c) 1 + c 2
  ‹
= 1− >1− =
2 2 2 2
476 Vasile Cîrtoaje

and ‹1−c
1−c 1 + c 1−c (1 − c)(1 + c) 1 + c 2
  ‹
= 1− <1− = .
2 2 2 2
Since g(x) > 0 involves f 0 (x) > 0, it follows that f (x) is strictly increasing on
h ci
c, 2 + , and hence
2  c
f (x) ≤ f 2 + .
2
 c
So, we need to show that f 2 + ≤ 0 for 0 < c < 1, which follows immediately
2
from Lemma 3 below.
Case 3: 1 < c < 2. Since

2x − 1 − c ≥ 2c − 1 − c > 0,

we have g 0 (x) > 0, hence g(x) is strictly increasing. Since g(x) → −∞ when
x → c and
 c  c  c
g 2+ = (1 + c) ln 2 + + (c − 1) ln 2 −
2 2 2
 c
> (c − 1) ln 2 − > 0,
2
 c
there exists x 1 ∈ c, 2 + such that g(x 1 ) = 0, g(x) < 0 for x ∈ (c, x 1 ) and
2 
 c
g(x) > 0 for x ∈ x 1 , 2 + . Thus, f (x) is decreasing on [c, x 1 ] and increasing
2
h c i  c
on x 1 , 2 + . Then, it suffices to show that f (c) ≤ 0 and f 2 + ≤ 0. The first
2 2
inequality is true because

f (c) = c −c − 2 < 1 − 2 < 0,

while the second inequality follows immediately from Lemma 3 below.


The proof is completed. The equality holds for a = b.
Lemma 1. If x < 4, then
xh(x) ≤ 0,
where
1 2
 ‹
 x x
h(x) = ln 2 − − ln 2 − − x .
2 4 32
Proof. From
−x 2
h0 (x) = ≤ 0,
16(4 − x)
it follows that h(x) is decreasing. Since h(0) = 0, we have h(x) ≥ 0 for x ≤ 0, and
h(x) ≤ 0 for x ∈ [0, 4); that is, xh(x) ≤ 0 for x < 4.
Symmetric Power-Exponential Inequalities 477

Lemma 2. If
−2 ≤ x ≤ 2,
then  x x x3
2− ≤ 1 + x ln 2 − .
2 9
Proof. We have
ln 2 ≈ 0.693 < 7/9.
If x ∈ [0, 2], then
x3 x3 8
1 + x ln 2 − ≥1− ≥ 1 − > 0.
9 9 9
Also, if x ∈ [−2, 0], then
x3 7x x 3 8 + 7x − x 3
1 + x ln 2 − ≥1+ − >
9 9 9 9
2(x + 2)2 + (−x)(x + 1)2
= > 0.
9
So, we can write the desired inequality as f (x) ≥ 0, where
x3
 ‹  x
f (x) = ln 1 + d x − − x ln 2 − , d = ln 2.
9 2
We have
9d − 3x 2 x  x
f (x) =
0
+ − ln 2 − .
9 + 9d x − x 3 4 − x 2
Since f (0) = 0, it suffices to show that f 0 (x) ≤ 0 for x ∈ [−2, 0], and f 0 (x) ≥ 0 for
x ∈ [0, 2]; that is, x f 0 (x) ≥ 0 for x ∈ [−2, 2]. We have

f 0 (x) = g(x) − h(x),

where
9d − 3x 2 1 2
 ‹
x x
g(x) = + − d− − x ,
9 + 9d x − x 3 4 − x 4 32
1 2
 ‹
 x x
h(x) = ln 2 − − d− − x .
2 4 32
According to Lemma 1,

x f 0 (x) = x g(x) − xh(x) ≥ x g(x).

Therefore, to show that x f 0 (x) ≥ 0, it suffices to prove that x g(x) ≥ 0. We have


9d − 3x 2 1 2
 ‹  ‹
x x
g(x) = −d + + + x
9 + 9d x − x 3 4 − x 4 32
• 2
d x − 3x − 9d 2 64 − 4x − x 2
˜
=x + ,
9 + 9d x − x 3 32(4 − x)
478 Vasile Cîrtoaje

hence
x 2 g1 (x)
x g(x) = ,
32(4 − x)(9 + 9d x − x 3 )
where

g1 (x) =32(4 − x)(d x 2 − 3x − 9d 2 ) + (64 − 4x − x 2 )(9 + 9d x − x 3 )


=x 5 + 4x 4 − (64 + 41d)x 3 + (87 + 92d)x 2 + 12(24d 2 + 48d − 35)x
+ 576(1 − 2d 2 ).

Since g1 (x) ≥ 0 for x ∈ [a1 , b1 ], where a1 ≈ −12.384 and b1 =≈ 2.652, we have


g1 (x) ≥ 0 for x ∈ [−2, 2].
Lemma 3. If 0 ≤ c ≤ 2, then
 c −c  c c
2+ + 2− ≤ 2.
2 2
Proof. According to Lemma 2, the following inequalities hold for c ∈ [0, 2]:
 c −c c3
2+ ≤ 1 − c ln 2 + ,
2 9
 c c c3
2− ≤ 1 + c ln 2 − .
2 9
Summing these inequalities, the desired inequality follows.

P 3.28. If a, b are nonnegative real numbers such that a + b = 2, then

a2b + b2a ≥ a b + b a ≥ a2 b2 + 1.

(Vasile Cîrtoaje, 2010)

Solution. Since a, b ∈ [0, 2] and

(1 − a)(1 − b) = −(1 − a)2 ≤ 0,

from Lemma below, we have

b(ab + 3 − a − b)(a − 1) b(ab + 1)(a − 1)


ab − 1 ≥ =
2 2
and
a(ab + 1)(b − 1)
ba − 1 ≥ .
2
Symmetric Power-Exponential Inequalities 479

Based on these inequalities, we get


a b + b a − a2 b2 − 1 = (a b − 1) + (b a − 1) + 1 − a2 b2
b(ab + 1)(a − 1) a(ab + 1)(b − 1)
≥ + + 1 − a2 b2
2 2
= (ab + 1)(ab − 1) + 1 − a2 b2 = 0
and
a2b + b2a − a b − b a = a b (a b − 1) + b a (b a − 1)
a b b(ab + 1)(a − 1) b a a(ab + 1)(b − 1)
≥ +
2 2
ab(ab + 1)(a − b)(a b−1 − b a−1 )
= .
4
Under the assumption that a ≥ b, we only need to show that a b−1 ≥ b a−1 , which is
equivalent to
b−a a−b a−b
a 2 ≥b 2 , 1 ≥ (ab) 2 , 1 ≥ ab, (a − b)2 ≥ 0.
For both inequalities, the equality holds when a = b = 1, when a = 0 and b = 2,
and when a = 2 and b = 0.
Lemma. If x, y ∈ [0, 2] such that (1 − x)(1 − y) ≤ 0, then
y(x y + 3 − x − y)(x − 1)
xy −1 ≥ ,
2
with equality for x = 1, and also for y = 0, y = 1 and y = 2.
Proof. For y = 0, y = 1 and y = 2, the inequality is an identity. For fixed
y ∈ (0, 1) ∪ (1, 2),
let us define
y(x y + 3 − x − y)(x − 1)
f (x) = x y − 1 − .
2
We have
x y + 3 − x − y (x − 1)( y − 1)
• ˜
f (x) = y x
0
− y−1
− ,
2 2
f 00 (x) = y( y − 1)(x y−2 − 1).
Since x y−2 − 1 has the same sign as 1 − x, it follows that f 00 (x) ≥ 0 for x ∈ (0, 2],
therefore f 0 is increasing. There are two cases to consider.
Case 1: x ≥ 1 > y. We have f 0 (x) ≥ f 0 (1) = 0, f (x) is increasing, hence
f (x) ≥ f (1) = 0.
Case 2: y > 1 ≥ x. We have f 0 (x) ≤ f 0 (1) = 0, f (x) is decreasing, hence
f (x) ≥ f (1) = 0.
480 Vasile Cîrtoaje

P 3.29. If a, b are positive real numbers such that a + b = 2, then

a3b + b3a ≤ 2.

(Vasile Cîrtoaje, 2007)

Solution. Without loss of generality, assume that a ≥ b. Using the substitution

a = 1 + x, b = 1 − x, 0 ≤ x < 1,

we can write the inequality as

e3(1−x) ln(1+x) + e3(1+x) ln(1−x) ≤ 2.

Applying Lemma below, it suffices to show that f (x) ≤ 2, where


2 3 2 3
   
3(1−x) x− x2 + x3 −3(1+x) x+ x2 + x3
f (x) = e +e .

Since f (0) = 2, it suffices to show that f 0 (x) ≤ 0 for x ∈ [0, 1). From

15 2
 ‹
9x 2 5x 3 4
f (x) = 3 − 9x +
0
x − 4x e3x− 2 + 2 −x
3
2
15 2
 ‹
9x 2 5x 3 4
− 3 + 9x + x + 4x e−3x− 2 − 2 −x ,
3
2

it follows that f 0 (x) ≤ 0 is equivalent to

−6x−5x 3 6 − 18x + 15x 2 − 8x 3


e ≥ .
6 + 18x + 15x 2 + 8x 3

For the nontrivial case 6 − 18x + 15x 2 − 8x 3 > 0, we rewrite this inequality as
g(x) ≥ 0, where

g(x) = −6x − 5x 3 − ln(6 − 18x + 15x 2 − 8x 3 ) + ln(6 + 18x + 15x 2 + 8x 3 ).

Since g(0) = 0, it suffices to show that g 0 (x) ≥ 0 for x ∈ [0, 1). From

1 0 (6 + 8x 2 ) − 10x (6 + 8x 2 ) + 10x
g (x) = −2 − 5x 2 + + ,
3 6 + 15x 2 − (18x + 8x 3 ) 6 + 15x 2 + (18x + 8x 3 )

it follows that g 0 (x) ≥ 0 is equivalent to

2(6 + 8x 2 )(6 + 15x 2 ) − 20x(18x + 8x 3 ) ≥ (2 + 5x 2 )[(6 + 15x 2 )2 − (18x + 8x 3 )2 ].

Since

(6 + 15x 2 )2 − (18x + 8x 3 )2 ≤ (6 + 15x 2 )2 − 324x 2 − 288x 4 ≤ 4(9 − 36x 2 ),


Symmetric Power-Exponential Inequalities 481

it suffices to show that

(3 + 4x 2 )(6 + 15x 2 ) − 5x(18x + 8x 3 ) ≥ (2 + 5x 2 )(9 − 36x 2 ).

This reduces to 6x 2 + 200x 4 ≥ 0, which is clearly true. The equality holds for
a = b = 1.
Lemma. If t > −1, then
t2 t3
ln(1 + t) ≤ t −
+ .
2 3
Proof. We need to prove that f (t) ≥ 0, where

t2 t3
f (t) = t − + − ln(1 + t).
2 3
Since
t3
f 0 (t) =
,
t +1
f (t) is decreasing on (−1, 0] and increasing on [0, ∞). Therefore,

f (t) ≥ f (0) = 0.

P 3.30. If a, b are nonnegative real numbers such that a + b = 2, then


‹4
a−b

a 3b
+b 3a
+ ≤ 2.
2

(Vasile Cîrtoaje, 2007)

Solution (by M. Miyagi and Y. Nishizawa). Using the substitution

a = 1 + x, b = 1 − x, 0 ≤ x ≤ 1,

we can write the inequality as

(1 + x)3(1−x) + (1 − x)3(1+x) + x 4 ≤ 2.

By Lemma below, we have

1
(1 + x)1−x ≤ (1 + x)2 (2 − x 2 )(2 − 2x + x 2 ),
4
1
(1 − x)1+x ≤ (1 − x)2 (2 − x 2 )(2 + 2x + x 2 ).
4
482 Vasile Cîrtoaje

Therefore, it suffices to show that

(1 + x)6 (2 − x 2 )3 (2 − 2x + x 2 )3 + (1 − x)6 (2 − x 2 )3 (2 + 2x + x 2 )3 + 64x 4 ≤ 128,

which is equivalent to

x 4 (1 − x 2 )[x 6 (x 6 − 8x 4 + 31x 2 − 34) − 2(17x 6 − 38x 4 + 16x 2 + 8)] ≤ 0.

Thus, it suffices to show that

t 3 − 8t 2 + 31t − 34 < 0

and
17t 3 − 38t 2 + 16t + 8 > 0
for all t ∈ [0, 1]. Indeed, we have

t 3 − 8t 2 + 31t − 34 < t 3 − 8t 2 + 31t − 24 = (t − 1)(t 2 − 7t + 24) ≤ 0,

17t 3 − 38t 2 + 16t + 8 = 17t(t − 1)2 + (−4t 2 − t + 8) > 0.

Lemma. If −1 ≤ t ≤ 1, then

1
(1 + t)1−t ≤ (1 + t)2 (2 − t 2 )(2 − 2t + t 2 ),
4
with equality for t = −1, t = 0 and t = 1.
Proof. It suffices to consider that

−1 < t ≤ 1.

Rewrite the inequality as

(1 + t)1+t (2 − t 2 )(2 − 2t + t 2 ) ≥ 4,

which is equivalent to f (t) ≥ 0, where

f (t) = (1 + t) ln(1 + t) + ln(2 − t 2 ) + ln(2 − 2t + t 2 ) − ln 4.

We have
2t 2(t − 1)
f 0 (t) = 1 + ln(1 + t) − + ,
2 − t 2 2 − 2t + t 2
t 2 g(t)
f 00 (t) = ,
(1 + t)(2 − t 2 )2 (2 − 2t + t 2 )2
where
g(t) = t 6 − 8t 5 + 12t 4 + 8t 3 − 20t 2 − 16t + 16.
Symmetric Power-Exponential Inequalities 483

Case 1: 0 ≤ t ≤ 1. From

g 0 (t) = 6t 5 − 40t 4 + 48t 3 + 24t 2 − 40t − 16


= 6t 5 − 8t − 16 − 8t(5t 3 − 6t 2 − 3t + 4)
= (6t 5 − 8t − 16) − 8t(t − 1)2 (5t + 4) < 0,

it follows that g is strictly decreasing on [0, 1]. Since g(0) = 16 and g(1) = −7,
there exists a number c ∈ (0, 1) such that g(c) = 0, g(t) > 0 for 0 < t < c and
g(t) < 0 for c < t ≤ 1. Therefore, f 0 is strictly increasing on [0, c] and strictly
decreasing on [c, 1]. From f 0 (0) = 0 and f 0 (1) = ln 2 − 1 < 0, it follows that
there exists a number d ∈ (0, 1) such that f 0 (d) = 0, f 0 (t) > 0 for 0 < t < d and
f 0 (t) < 0 for d < t ≤ 1. As a consequence, f is strictly increasing on [0, d] and
strictly decreasing on [d, 1]. Since f (0) = 0 and f (1) = 0, we have f (t) ≥ 0 for
0 ≤ t ≤ 1.
Case 2: −1 < t ≤ 0. From

g(t) = t 4 (t − 2)(t − 6) + 4(t + 1)(2t 2 − 7t + 3) + 4 > 0,

it follows that f 0 is strictly increasing on (−1, 0]. Since f 0 (0) = 0, we have f 0 (t) < 0
for −1 < t < 0, hence f is strictly decreasing on (−1, 0]. From f (0) = 0, it follows
that f (t) ≥ 0 for −1 < t ≤ 0.
Conjecture. If a, b are nonnegative real numbers such that a + b = 2, then

a−b 2
 ‹
a +b +
3b 3a
≥ 2.
2

P 3.31. If a, b are positive real numbers such that a + b = 2, then


2 2
a a + b b ≤ 2.

(Vasile Cîrtoaje, 2008)


Solution. Without loss of generality, assume that

0 < a ≤ 1 ≤ b < 2,

and write the inequality as


1 1
 +
1 1/a
 ≤ 2.
1 2/b
a2 b

By Bernoulli’s inequality, we have


 ‹1/a
1 1 1 a3 − a2 + 1
 ‹
≥ 1 + − 1 = ,
a2 a a2 a3
484 Vasile Cîrtoaje

 ‹2/b
1 2 1 b2 − 2b + 2
 ‹
≥1+ −1 = .
b b b b2
Therefore, it suffices to show that

a3 b2
+ ≤ 2,
a3 − a2 + 1 b2 − 2b + 2
which is equivalent to
a3 (2 − b)2
≤ ,
a3 − a2 + 1 b2 − 2b + 2
a3 a2
≤ ,
a3 − a2 + 1 a2 − 2a + 2
a2 (a − 1)2 ≥ 0.
The equality happens for a = b = 1.

P 3.32. If a, b are positive real numbers such that a + b = 2, then


3 3
a a + b b ≥ 2.

(Vasile Cîrtoaje, 2008)

Solution. Assume that a ≤ b; that is,

0 < a ≤ 1 ≤ b < 2.
3 3
There are two cases to consider: 0 < a ≤ and ≤ a ≤ 1.
5 5
3 7
Case 1: 0 < a ≤ . From a + b = 2, we get ≤ b < 2. Let
5 5
3
f (x) = x x , 0 < x < 2.

Since
3
f 0 (x) = 3x x −2 (1 − ln x) > 0,
7
 ‹
f (x) is increasing on (0, 2), hence f (b) ≥ f ; that is,
5
 ‹15/7
3 7
b ≥
b .
5
Using Bernoulli’s inequality gives
 ‹15/7
7 7 2 8/7 7 16 51
 ‹  ‹
= 1+ > 1+ = > 2,
5 5 5 5 35 25
Symmetric Power-Exponential Inequalities 485

therefore
3 3
a a + b b > 2.

3 7
Case 2: ≤ a ≤ 1. From a + b = 2, we get 1 ≤ b ≤ . By Lemma below, we have
5 5
3
2a a ≥ 3 − 15a + 21a2 − 7a3

and
3
2b b ≥ 3 − 15b + 21b2 − 7b3 .
Summing these inequalities, we get
€ 3 3
Š
2 a a + b b ≥ 6 − 15(a + b) + 21(a2 + b2 ) − 7(a3 + b3 )
= 6 − 15(a + b) + 21(a + b)2 − 7(a + b)3 = 4.

This completes the proof. The equality holds for a = b = 1.


3
Lemma. If ≤ x ≤ 2, then
5
3
2x x ≥ 3 − 15x + 21x 2 − 7x 3 ,

with equality for x = 1.


Proof. First, we show that h(x) > 0, where

h(x) = 3 − 15x + 21x 2 − 7x 3 .

From
h0 (x) = 3(−5 + 14x − 7x 2 ),
 s s   s 
2 2 2
it follows that h(x) is increasing on 1 − ,1 + , and decreasing on 1 + ,∞ .
7 7 7
3
 ‹
Then, it suffices to show that f ≥ 0 and f (2) ≥ 0. Indeed
5
3 6
 ‹
f = , f (2) = 1.
5 125
Write now the desired inequality as f (x) ≥ 0, where

3 3
f (x) = ln 2 + ln x − ln(3 − 15x + 21x 2 − 7x 3 ), ≤ x ≤ 2.
x 5
We have
x2 0 x 2 (7x 2 − 14x + 5)
f (x) = g(x), g(x) = 1 − ln x + ,
3 3 − 15x + 21x 2 − 7x 3
486 Vasile Cîrtoaje

g1 (x)
g 0 (x) = ,
x(3 − 15x + 21x 2 − 7x 3 )2
where

g1 (x) = −49x 7 + 245x 6 − 280x 5 − 147x 4 + 471x 3 − 321x 2 + 90x − 9.

In addition,

g1 (x = (x − 1)2 g2 (x), g2 (x) = −49x 5 + 147x 4 + 63x 3 − 168x 2 + 72x − 9,

g2 (x) = 11x 5 + 3g3 (x), g3 (x) = −20x 5 + 49x 4 + 21x 3 − 56x 2 + 24x − 3,
g3 (x) = (4x − 1)g4 (x), g4 (x) = −5x 4 + 11x 3 + 8x 2 − 12x + 3,
g4 (x) = x 5 + g5 (x), g5 (x) = −6x 4 + 11x 3 + 8x 2 − 12x + 3,
g5 (x) = (2x − 1)g6 (x), g6 (x) = −3x 3 + 4x 2 + 6x − 3,
g6 (x) = 1 + (2 − x)(3x 2 + 2x − 2).
Therefore, we get in succession g6 (x) > 0, g5 (x) > 0, g4 (x) > 0, g3 (x) > 0,
g2 (x) > 0, g•1 (x) ‹≥ 0, g 0 (x) ≥ 0, g(x) is increasing. Since g(1) = 0,• we ˜have
3 3
g(x) < 0 on , 1 and g(x) > 0 on (1, 2]. Then, f (x) is decreasing on , 1 and
5 5
increasing on [1, 2], hence f (x) ≥ f (1) = 0.

P 3.33. If a, b are positive real numbers such that a + b = 2, then


2 2
a5b + b5a ≤ 2.

(Vasile Cîrtoaje, 2010)

Solution. Assume that a ≥ b. For a = 2 and b = 0, the inequality is obvious.


Otherwise, using the substitution a = 1 + x and b = 1 − x, 0 ≤ x < 1, we can write
the desired inequality as
2 2
e5(1−x) ln(1+x)
+ e5(1+x) ln(1−x)
≤ 2.

According to Lemma below, it suffices to show that f (x) ≤ 2, where

f (x) = e5(u−v) + e−5(u+v) ,

7 3 31 5 5 17 4 9 6
u= x+ x + x , v = x2 + x + x .
3 30 2 12 20
If f 0 (x) ≤ 0, then f (x) is decreasing, hence

f (x) ≤ f (0) = 2.
Symmetric Power-Exponential Inequalities 487

Since
f 0 (x) = 5(u0 − v 0 )e5(u−v) − 5(u0 + v 0 )e−5(u+v) ,
31 4 17 3 27 5
u0 = 1 + 7x 2 + x , v 0 = 5x + x + x ,
6 3 10
the inequality f 0 (x) ≤ 0 becomes

e−10u (u0 + v 0 ) ≥ u0 − v 0

For the nontrivial case u0 − v 0 > 0, we rewrite this inequality as g(x) ≥ 0, where

g(x) = −10u + ln(u0 + v 0 ) − ln(u0 − v 0 ).

If g 0 (x) ≥ 0, then g(x) is increasing, hence

g(x) ≥ f (0) = 0.

We have
u00 + v 00 u00 − v 00
g 0 (x) = −10u0 + − 0 ,
u0 + v 0 u − v0
where
62 3 27 4
u00 = 14x + x , v 00 = 5 + 17x 2 + x .
3 2
Write the inequality g 0 (x) ≥ 0 as

u0 v 00 − v 0 u00 ≥ 5u0 (u0 + v 0 )(u0 − v 0 ),

a1 t + a2 t 2 + a3 t 3 + a4 t 4 + a5 t 5 + a6 t 6 + a7 t 7 ≥ 0,
where t = x 2 , 0 ≤ t < 1, and

a1 = 2, a2 = 321.5, a3 ≈ 152.1, a4 ≈ −498.2,

a5 ≈ −168.5, a6 ≈ 356.0, a7 ≈ 188.3.


This inequality is true if

300t 2 + 150t 3 − 500t 4 − 200t 5 + 250t 6 ≥ 0.

Since the last inequality is equivalent to the obvious inequality

50t 2 (1 − t)(6 + 9t − t 2 − 5t 3 ) ≥ 0,

the proof is completed. The equality holds for a = b = 1.


Lemma. If −1 < t < 1, then

5 2 7 3 17 4 31 5 9 6
(1 − t)2 ln(1 + t) ≤ t − t + t − t + t − t .
2 3 12 30 20
488 Vasile Cîrtoaje

Proof. We show that

1 2 1 3 1 4 1 5
 ‹
(1 − t) ln(1 + t) ≤ (1 − t) t − t + t − t + t
2 2
2 3 4 5
5 2 7 3 17 4 31 5 9 6
≤t− t + t − t + t − t .
2 3 12 30 20
The left inequality is equivalent to f (t) ≥ 0, where

1 2 1 3 1 4 1 5
f (t) = t − t + t − t + t − ln(1 + t).
2 3 4 5
Since
t5
f 0 (t) = ,
1+ t
f (t) is decreasing on (−1, 0] and increasing on [0, 1); therefore, f (t) ≥ f (0) = 0.
The right inequality is equivalent to t 6 (t − 1) ≤ 0, which is clearly true.

P 3.34. If a, b are positive real numbers such that a + b = 2, then


p p
a2 b
+ b2 a
≤ 2.

(Vasile Cîrtoaje, 2010)

Solution. Assume that a ≥ b. For a = 2 and b = 0, the inequality is obvious.


Otherwise, using the substitution a = 1 + x and b = 1 − x, 0 ≤ x < 1, we can write
the desired inequality as f (x) ≤ 2, where
p p p p
f (x) = (1 + x)2 1−x
+ (1 − x)2 1+x
= e2 1−x ln(1+x)
+ e21+x ln(1−x)
.

There are two cases to consider.


Case 1: 13/20 ≤ x < 1. If f is decreasing on [13/20, 1), then
 ‹  ‹p7/5  ‹p33/5  ‹5/4  ‹2
13 33 7 5 1
f (x) ≤ f = + < + < 2.
20 20 20 3 4
p
Since the function (1 − x)2 1+x
is decreasing, it suffices to show that
p
g(x) = (1 + x)2 1−x

is decreasing. This is true if g 0 (x) ≤ 0 for x ∈ [13/20, 1), that is equivalent to


h(x) ≤ 0, where
2(1 − x)
h(x) = − ln(1 + x).
1+ x
Symmetric Power-Exponential Inequalities 489

Clearly, h is decreasing, hence


13 14 33
‹ 
h(x) ≤ h = − ln < 0.
20 33 20
Case 2: 0 ≤ x ≤ 13/20. By Lemma below, it suffices to show that g(x) ≤ 2, where
2
+ 11 3 1 4 2
+ 11 3 1 4
12 x + 2 x )
g(x) = e2x−2x 12 x − 2 x + e−(2x+2x .

If g 0 (x) ≤ 0 for x ∈ [0, 13/20], then g is decreasing, hence g(x) ≤ g(0) = 2. Since
11 2 2 11 3 1 4
g 0 (x) =(2 − 4x + x − 2x 3 )e2x−2x + 12 x − 2 x
4
11 2 2 11 3 1 4
− (2 + 4x + x + 2x 3 )e−(2x+2x + 12 x + 2 x ) ,
4
the inequality g 0 (x) ≤ 0 is equivalent to

11 3 8 − 16x + 11x 2 − 8x 3
e−4x− 6 x ≥ .
8 + 16x + 11x 2 + 8x 3
For the nontrivial case 8−16x +11x 2 −8x 3 > 0, rewrite this inequality as h(x) ≥ 0,
where
11 3
h(x) = −4x − x − ln(8 − 16x + 11x 2 − 8x 3 ) + ln(8 + 16x + 11x 2 + 8x 3 ).
6
If h0 ≥ 0, then h is increasing, hence h(x) ≥ h(0) = 0. From

11 2 (16 + 24x 2 ) − 22x (16 + 24x 2 ) + 22x


h0 (x) = −4 − x + + ,
2 8 + 11x 2 − (16x + 8x 3 ) 8 + 11x 2 + (16x + 8x 3 )
it follows that h0 (x) ≥ 0 is equivalent to
1
(16+24x 2 )(8+11x 2 )−22x(16x +8x 3 ) ≥ (8+11x 2 )[(8+11x 2 )2 −(16x +8x 3 )2 ].
4
Since

(8 + 11x 2 )2 − (16x + 8x 3 )2 ≤ (8 + 11x 2 )2 − 256x 2 − 256x 4 ≤ 16(4 − 5x 2 ),

it suffices to show that

(4 + 6x 2 )(8 + 11x 2 ) − 11x(8x + 4x 3 ) ≥ (8 + 11x 2 )(4 − 5x 2 ).

This inequality reduces to 77x 4 ≥ 0. The proof is completed. The equality holds
for a = b = 1.
13
Lemma. If −1 < t ≤ , then
20
p 11 3 1 4
1 − t ln(1 + t) ≤ t − t 2 + t − t .
24 4
490 Vasile Cîrtoaje

Proof. Consider two cases.


13
Case 1: 0 ≤ t ≤ . We can prove the desired inequality by multiplying the follow-
20
ing inequalities
p 1 1 1 3
1 − t ≤ 1 − t − t2 − t ,
2 8 16
1 1 1 1
ln(1 + t) ≤ t − t 2 + t 3 − t 4 + t 5 ,
2 3 4 5
1 1 1 3 1 1 1 1 11 3 1 4
 ‹ ‹
1 − t − t2 − t t − t2 + t3 − t4 + t5 ≤ t − t2 + t − t .
2 8 16 2 3 4 5 24 4

The first inequality is equivalent to f (t) ≥ 0, where


1 1 2 1 3 1
 ‹
f (t) = ln 1 − t − t − t − ln(1 − t).
2 8 16 2
Since
1 8 + 4t + 3t 2 5t 3
f 0 (t) = − = ≥ 0,
2(1 − t) 16 − 8t − 2t 2 − t 3 2(1 − t)(16 − 8t − 2t 2 − t 3 )
f is increasing, hence f (t) ≥ f (0) = 0.
The second inequality is equivalent to f (t) ≥ 0, where
1 2 1 3 1 4 1 5
f (t) = t − t + t − t + t − ln(1 + t).
2 3 4 5
Since
1 t5
f (t) = 1 − t + t − t + t −
0 2 3
=4
≥ 0,
1+ t 1+ t
f (t) is increasing, hence f (t) ≥ f (0) = 0.
The third inequality is equivalent to

t 4 (160 − 302t + 86t 2 + 9t 3 + 12t 4 ) ≥ 0.

This is true since

160 − 302t + 86t 2 + 9t 3 + 12t 4 ≥ 2(80 − 151t + 43t 2 ) > 0.

Case 2: −1 < t ≤ 0. Write the desired inequality as


p 11 3 1 4
− 1 − t ln(1 + t) ≥ −t + t 2 − t + t .
24 4
This is true if
p 1 1
1 − t ≥ 1 − t − t 2,
2 8
Symmetric Power-Exponential Inequalities 491

1 3 1 4
t + t ,
− ln(1 + t) ≥ −t + t 2 −
3 4
1 1 1 1 11 3 1 4
 ‹ ‹
1 − t − t 2 −t + t 2 − t 3 + t 4 ≥ −t + t 2 − t + t .
2 8 3 4 24 4

The first inequality is equivalent to f (t) ≥ 0, where

1 1 1 2
 ‹
f (t) = ln(1 − t) − ln 1 − t − t .
2 2 8
Since
−1 2(2 + t) −3t 2
f 0 (t) = + = ≤ 0,
2(1 − t) 8 − 4t − t 2 2(1 − t)(8 − 4t − t 2 )
f is decreasing, hence f (t) ≥ f (0) = 0.
The second inequality is equivalent to f (t) ≥ 0, where

1 2 1 3 1 4
f (t) = t − t + t − t − ln(1 + t).
2 3 4
Since
1 −t 4
f 0 (t) = 1 − t + t 2 − t 3 − = ≤ 0,
1+ t 1+ t
f is decreasing, hence f (t) ≥ f (0) = 0.
The third inequality reduces to the obvious inequality

t 4 (10 − 8t − 3t 2 ) ≥ 0.

P 3.35. If a, b are nonnegative real numbers such that a + b = 2, then

ab(1 − ab)2 ab(1 − ab)2


≤ a b+1 + b a+1 − 2 ≤ .
2 3
(Vasile Cîrtoaje, 2010)

Solution. Assume that a ≥ b, which yields 1 ≤ a ≤ 2 and 0 ≤ b ≤ 1.


(a) To prove the left inequality we apply Lemma 1 below. For x = a and k = b,
we have
b(1 + b) b(1 + b)(1 − b)
a b+1 ≥ 1 + (1 + b)(a − 1) + (a − 1)2 − (a − 1)3 ,
2 6
b(1 + b) b(1 + b)
a b+1 ≥ a − b + ab + (a − 1)2 − (a − 1)4 . (*)
2 6
492 Vasile Cîrtoaje

Also, for x = b and k = a − 1, we have


a(a − 1) a(a − 1)(2 − a)
b a ≥ 1 + a(b − 1) + (b − 1)2 − (b − 1)3 ,
2 6
a ab
b a ≥ 1 − a + ab + (a − 1)3 + (a − 1)4 ,
2 6
ab ab2
b a+1 ≥ b − ab + ab2 + (a − 1)3 + (a − 1)4 . (**)
2 6
Summing up (*) and (**) gives
b(3 − ab) b(1 + b − ab)
a b+1 + b a+1 − 2 ≥ −b(a − 1)2 + (a − 1)2 − (a − 1)4
2 6
b b(1 + b − ab)
= (a − 1)4 − (a − 1)4
2 6
ab(1 + b) ab ab(1 − ab)2
= (a − 1)4 ≥ (a − 1)4 = .
6 6 6
The equality holds for a = b = 1, for a = 2 and b = 0, and for a = 0 and b = 2.
(b) To prove the right inequality we apply Lemma 2 below. For x = a and
k = b, we have
(b + 1)b (b + 1)b(b − 1)
a b+1 ≤ 1 + (b + 1)(a − 1) + (a − 1)2 + (a − 1)3
2 6
(b + 1)b(b − 1)(b − 2)
+ (a − 1)4 ,
24
b(b + 1) b(b + 1) ab(b + 1)
a b+1 ≤ 1 + (b + 1)(a − 1) + (a − 1)2 − (a − 1)4 + (a − 1)5 .
2 6 24
Also, for x = b and k = a, we have
a(a + 1) a(a + 1) ab(a + 1)
b a+1 ≤ 1 + (a + 1)(b − 1) + (b − 1)2 − (b − 1)4 + (b − 1)5 .
2 6 24
Summing up these inequalities and having in view that

(b + 1)(a − 1)5 + (a + 1)(b − 1)5 = −2(a − 1)5 ≤ 0

give
a2 + b2 + 2 a2 + b2 + 2
a b+1 + b a+1 − 2 ≤ −2(a − 1)2 + (a − 1)2 − (a − 1)4
2 6
a2 + b2 − 2 a 2
+ b 2
+ 2
≤ (a − 1)2 − (a − 1)4
2 6
a2 + b2 + 2
= (a − 1) −
4
(a − 1)4
6
ab ab(1 − ab)2
= (a − 1)4 = .
3 3
Symmetric Power-Exponential Inequalities 493

The equality holds for a = b = 1, for a = 2 and b = 0, and for a = 0 and b = 2.


Lemma 1. If x ≥ 0 and 0 ≤ k ≤ 1, then

k(1 + k) k(1 + k)(1 − k)


x k+1 ≥ 1 + (1 + k)(x − 1) + (x − 1)2 − (x − 1)3 ,
2 6
with equality for x = 1, for k = 0 and for k = 1.
Proof. For k = 0 and k = 1, the inequality is an identity. For fixed k, 0 < k < 1, let
us define
k(1 + k) k(1 + k)(1 − k)
f (x) = x k+1 − 1 − (1 + k)(x − 1) − (x − 1)2 + (x − 1)3 .
2 6
We need to show that f (x) ≥ 0. We have

1 k(1 − k)
f 0 (x) = x k − 1 − k(x − 1) + (x − 1)2 ,
1+k 2
1
f 00 (x) = x k−1 − 1 + (1 − k)(x − 1),
k(1 + k)
1
f 000 (x) = −x k−2 + 1.
k(1 + k)(1 − k)

Case 1: 0 ≤ x ≤ 1. Since f 000 ≤ 0, f 00 is decreasing, f 00 (x) ≥ f 00 (1) = 0, f 0 is


increasing, f 0 (x) ≤ f 0 (1) = 0, f is decreasing, hence f (x) ≥ f (1) = 0.
Case 2: x ≥ 1. Since f 000 ≥ 0, f 00 is increasing, f 00 (x) ≥ f 00 (1) = 0, f 0 is increasing,
f 0 (x) ≥ f 0 (1) = 0, f is increasing, hence f (x) ≥ f (1) = 0.

Lemma 2. If either x ≥ 1 and 0 ≤ k ≤ 1, or 0 ≤ x ≤ 1 and 1 ≤ k ≤ 2, then

(k + 1)k (k + 1)k(k − 1)
x k+1 ≤ 1 + (k + 1)(x − 1) + (x − 1)2 + (x − 1)3
2 6

(k + 1)k(k − 1)(k − 2)
+ (x − 1)4 ,
24
with equality for x = 1, for k = 0, for k = 1 and for k = 2.
Proof. For k = 0, k = 1 and k = 2, the inequality is an identity. For fixed k,
k ∈ (0, 1) ∪ (1, 2), let us define

(k + 1)k (k + 1)k(k − 1)
f (x) = x k+1 − 1 − (k + 1)(x − 1) − (x − 1)2 − (x − 1)3
2 6

(k + 1)k(k − 1)(k − 2)
− (x − 1)4 .
24
494 Vasile Cîrtoaje

We need to show that f (x) ≤ 0. We have

1 k(k − 1) k(k − 1)(k − 2)


f 0 (x) = x k − 1 − k(x − 1) − (x − 1)2 − (x − 1)3 ,
k+1 2 6
1 (k − 1)(k − 2)
f 00 (x) = x k−1 − 1 − (k − 1)(x − 1) − (x − 1)2 ,
k(k + 1) 2
1
f 000 (x) = x k−2 − 1 − (k − 2)(x − 1),
k(k + 1)(k − 1)
1
f (4) (x) = x k−3 − 1.
k(k + 1)(k − 1)(k − 2)

Case 1: x ≥ 1, 0 < k < 1. Since f (4) (x) ≤ 0, f 000 (x) is decreasing, f 000 (x) ≤ f 000 (1) =
0, f 00 is decreasing, f 00 (x) ≤ f 00 (1) = 0, f 0 is decreasing, f 0 (x) ≤ f 0 (1) = 0, f is
decreasing, hence f (x) ≤ f (1) = 0.
Case 2: 0 ≤ x ≤ 1, 1 < k < 2. Since f (4) ≤ 0, f 000 is decreasing, f 000 (x) ≥ f 000 (1) =
0, f 00 is increasing, f 00 (x) ≤ f 00 (1) = 0, f 0 is decreasing, f 0 (x) ≥ f 0 (1) = 0, f is
increasing, hence f (x) ≤ f (1) = 0.

P 3.36. If a, b are nonnegative real numbers such that a + b = 1, then

a2b + b2a ≤ 1.

(Vasile Cîrtoaje, 2007)

Solution. Without loss of generality, assume that


1
0≤ b≤ ≤ a ≤ 1.
2
Applying Lemma 1 below for c = 2b, 0 ≤ c ≤ 1, we get

a2b ≤ (1 − 2b)2 + 4ab(1 − b) − 2ab(1 − 2b) ln a,

which is equivalent to

a2b ≤ 1 − 4ab2 − 2ab(a − b) ln a.

Similarly, applying Lemma 2 below for d = 2a − 1, d ≥ 0, we get

b2a−1 ≤ 4a(1 − a) + 2a(2a − 1) ln(2a + b − 1),

which is equivalent to

b2a ≤ 4ab2 + 2ab(a − b) ln a.


Symmetric Power-Exponential Inequalities 495

Adding up these inequalities, the desired inequality follows. The equality holds for
a = b = 1/2, for a = 0 and b = 1, and for a = 1 and b = 0.
Lemma 1. If 0 < a ≤ 1 and c ≥ 0, then

a c ≤ (1 − c)2 + ac(2 − c) − ac(1 − c) ln a,

with equality for a = 1, for c = 0 and for c = 1.


Proof. Making the substitution

a = e−x , x ≥ 0,

we need to prove that f (x) ≥ 0, where

f (x) = (1 − c)2 e x + c(2 − c) + c(1 − c)x − e(1−c)x ,

f 0 (x) = (1 − c)[(1 − c)e x + c − e(1−c)x ].


If f 0 ≥ 0 on [0, ∞), then f is increasing, and hence f (x) ≥ f (0) = 0. In order to
prove that f 0 ≥ 0, we consider two cases.
Case 1: 0 ≤ c ≤ 1. By the weighted AM-GM inequality, we have

(1 − c)e x + c ≥ e(1−c)x ,

hence f 0 (x) ≥ 0.
Case 2: c ≥ 1. By the weighted AM-GM inequality, we have

(c − 1)e x + e(1−c)x ≥ c,

which yields
f 0 (x) = (c − 1)[(c − 1)e x + e(1−c)x − c] ≥ 0.

Lemma 2. If 0 ≤ b ≤ 1 and d ≥ 0, then

b d ≤ 1 − d 2 + d(1 + d) ln(b + d),

with equality for b = 0 and for d = 0.


Proof. Consider 0 < b ≤ 1 and d > 0, and write the inequality as

(1 + d)[1 − d + d ln(b + d)] ≥ b d .

Since
1 − d + d ln(b + d) > 1 − d + d ln d ≥ 0,
we can rewrite the inequality in the form

ln(1 + d) + ln[1 − d + d ln(b + d)] ≥ d ln b.


496 Vasile Cîrtoaje

Using the substitution

b = e−x − d, − ln(1 + d) ≤ x < − ln d,

we need to prove that f (x) ≥ 0, where

f (x) = ln(1 + d) + ln(1 − d − d x) + d x − d ln(1 − de x ).

Since
d 2 (e x − 1 − x)
f 0 (x) = ≥ 0,
(1 − d − d x)(1 − de x )
f is increasing, hence

f (x) ≥ f (− ln(1 + d)) = ln[1 − d 2 + d(1 + d) ln(1 + d)].

To complete the proof, we only need to show that −d 2 + d(1 + d) ln(1 + d) ≥ 0; that
is,
(1 + d) ln(1 + d) ≥ d.
−d
This inequality follows from e x ≥ 1 + x, where x = .
1+d
Conjecture. If a, b are nonnegative real numbers such that 1 ≤ a + b ≤ 15, then

a2b + b2a ≤ a a+b + b a+b .

P 3.37. If a, b are positive real numbers such that a + b = 1, then

2a a b b ≥ a2b + b2a .

Solution. Taking into account the inequality a2b + b2a ≤ 1 from the preceding P
3.36, it suffices to show that
2a a b b ≥ 1.
Write this inequality as
2a a b b ≥ a a+b + b a+b ,
 a  b  b ‹a
2≥ + .
b a
Since a < 1 and b < 1, we apply Bernoulli’s inequality as follows:
 a  b  b ‹a a  
b
‹
+ ≤1+ b −1 +1+a − 1 = 2.
b a b a
Thus, the proof is completed. The equality holds for a = b = 1/2.
Symmetric Power-Exponential Inequalities 497

P 3.38. If a, b are positive real numbers such that a + b = 1, then

a−2a + b−2b ≤ 4.

Solution. Applying Lemma below, we have

a−2a ≤ 4 − 2 ln 2 − 4(1 − ln 2)a,

b−2b ≤ 4 − 2 ln 2 − 4(1 − ln 2)b.


Adding these inequalities, the desired inequality follows. The equality holds for
a = b = 1/2.
Lemma. If x ∈ (0, 1], then

x −2x ≤ 4 − 2 ln 2 − 4(1 − ln 2)x,

with equality for x = 1/2.


Proof. Write the inequality as
1 −2x 1
x ≤ 1 − c − (1 − 2c)x, c= ln 2 ≈ 0.346.
4 2
This is true if f (x) ≤ 0, where

f (x) = −2 ln 2 − 2x ln x − ln[1 − c − (1 − 2c)x].

We have
1 − 2c
f 0 (x) = −2 − 2 ln x + ,
1 − c − (1 − 2c)x
2 (1 − 2c)2 g(x)
f 00 (x) = − + = ,
x [1 − c − (1 − 2c)x]2 x[1 − c − (1 − 2c)x]2
where
g(x) = 2(1 − 2c)2 x 2 − (1 − 2c)(5 − 6c)x + 2(1 − c)2 .
Since

g 0 (x) = (1 − 2c)[4(1 − 2c)x − 5 + 6c] ≤ (1 − 2c)[4(1 − 2c) − 5 + 6c]


= (1 − 2c)(−1 − 2c) < 0,

g is decreasing on (0, 1], hence g(x) ≥ g(1) = −2c 2 +4c −1 > 0, f 00 (x) > 0 for x ∈
(0, 1], f 0 is increasing. Since f 0 (1/2) = 0, we have f 0 (x) ≤ 0 for x ∈ (0, 1/2] and
f 0 (x) ≥ 0 for x ∈ [1/2, 1]. Therefore, f is decreasing on (0, 1/2] and increasing
on [1/2, 1], hence f (x) ≥ f (1/2) = 0.

Remark. According to the inequalities in P 3.36 and P 3.38, the following inequality
holds for all positive numbers a, b such that a + b = 1:
 1 1
‹
a +b
2b 2a
+ ≤ 4.
a2a b2b
498 Vasile Cîrtoaje

Actually, this inequality holds for all a, b ∈ (0, 1]. In this case, it is sharper than the
inequality in P 3.19.

P 3.39. If a1 , a2 , . . . , an are positive real numbers such that a1 a2 · · · an = 1, then

1 a1 1 a2 1 an
 ‹  ‹  ‹
1− + 1− + ··· + 1 − ≤ n − 1.
n n n

(Vasile Cîrtoaje, 2004)

Solution. We will prove the more general inequality

1 a1 1 a2 1 an 1 a
 ‹  ‹  ‹  ‹
1− + 1− + ··· + 1 − ≤ n 1− , (*)
n n n n
p
where a = n a1 a2 · · · an ≤ 1. Using the substitution
n
x i = ai ln , i = 1, 2, . . . , n,
n−1
the inequality becomes as follows:

e−x 1 + e−x 2 + · · · + e−x n ≤ ne−r , (**)

where
n
r=
p
n
x 1 x 2 · · · x n ≤ ln
.
n−1
To prove this inequality, we use the induction technique. For n = 1, (**) is an
equality. Consider now that (**) holds for n − 1 numbers, n ≥ 2, and show that it
also holds for n numbers. Assume that

x1 ≤ x2 ≤ · · · ≤ x n,

and denote
x=
p
n−1
x 1 x 2 · · · x n−1 .
Because
n n−1
x ≤ r ≤ ln < ln ,
n−1 (n − 1) − 1
the induction hypothesis yields

e−x 1 + e−x 2 + · · · + e−x n−1 ≤ (n − 1)e−x .

Thus, we only need to show that

e−x n + (n − 1)e−x ≤ ne−r ,


Symmetric Power-Exponential Inequalities 499

which is equivalent to
f (x) ≤ ne−r
for
n
0 < x ≤ r ≤ ln < 1,
n−1
where
n
/x n−1
f (x) = e−r + (n − 1)e−x .
We have
x n e r /x
n n−1
n
/x n−1 −x
f 0 (x) = g(x), g(x) = r n − x n e r ,
n−1
n
/x n−1
e x−r g 0 (x) = h(x), h(x) = x n − nx n−1 + (n − 1)r n ,
h0 (x) = nx n−2 (x − n + 1).
Since h0 (x) < 0, h is strictly decreasing, and from

h(0) = (n − 1)r n > 0, h(r) = nr n−1 (r − 1) < 0,

it follows that there exists x 1 ∈ (0, r) such that h(x 1 ) = 0, h(x) > 0 for x ∈ (0, x 1 ),
h(x) < 0 for x ∈ (x 1 , r]. Therefore, g is strictly increasing on (0, x 1 ] and strictly de-
creasing on [x 1 , r]. Since g(0+ ) = −∞ and g(r) = 0, there exists x 2 ∈ (0, x 1 ) such
that g(x 2 ) = 0, g(x) < 0 for x ∈ (0, x 2 ), g(x) > 0 for x ∈ (x 2 , r]. Consequently, f
is strictly decreasing on (0, x 2 ] and strictly increasing on [x 2 , r], hence

f (x) ≤ max{ f (0+ ), f (r)} = max{n − 1, ne−r } = ne−r .

Thus, the proof is completed. The inequality (**) is an equality for


n
x 1 = x 2 = · · · = x n ≤ ln ,
n−1
the inequality (*) for
a1 = a2 = · · · = an ≤ 1,
and the original inequality for

a1 = a2 = · · · = an = 1.
500 Vasile Cîrtoaje
Appendix A

Glosar

1. AM-GM (ARITHMETIC MEAN-GEOMETRIC MEAN) INEQUALITY

If a1 , a2 , . . . , an are nonnegative real numbers, then

a1 + a2 + · · · + an ≥ n n a1 a2 · · · an ,
p

with equality if and only if a1 = a2 = · · · = an .

2. WEIGHTED AM-GM INEQUALITY

Let p1 , p2 , . . . , pn be positive real numbers satisfying

p1 + p2 + · · · + pn = 1.

If a1 , a2 , . . . , an are nonnegative real numbers, then


p p
p1 a1 + p2 a2 + · · · + pn an ≥ a1 1 a2 2 · · · anpn ,

with equality if and only if a1 = a2 = · · · = an .

3. AM-HM (ARITHMETIC MEAN-HARMONIC MEAN) INEQUALITY

If a1 , a2 , . . . , an are positive real numbers, then

1 1 1
 ‹
(a1 + a2 + · · · + an ) + + ··· + ≥ n2 ,
a1 a2 an

with equality if and only if a1 = a2 = · · · = an .

501
502 Vasile Cîrtoaje

4. POWER MEAN INEQUALITY

The power mean of order k of positive real numbers a1 , a2 , . . . , an , that is


  1
a k +a k +···+ank k
 1 2n
 , k 6= 0
Mk = p ,
 n a1 a2 · · · an ,
 k=0

is an increasing function with respect to k ∈ R. For instant, M2 ≥ M1 ≥ M0 ≥ M−1


is equivalent to
v
t a2 + a2 + · · · + a2
1 2 n a1 + a2 + · · · + an p n
≥ ≥ n a1 a2 · · · an ≥ .
n n 1 1 1
+ + ··· +
a1 a2 an

5. BERNOULLI’S INEQUALITY

For any real number x ≥ −1, we have


a) (1 + x) r ≥ 1 + r x for r ≥ 1 and r ≤ 0;
b) (1 + x) r ≤ 1 + r x for 0 ≤ r ≤ 1.
If a1 , a2 , . . . , an are real numbers such that either a1 , a2 , . . . , an ≥ 0 or

−1 ≤ a1 , a2 , . . . , an ≤ 0,

then
(1 + a1 )(1 + a2 ) · · · (1 + an ) ≥ 1 + a1 + a2 + · · · + an .

6. SCHUR’S INEQUALITY

For any nonnegative real numbers a, b, c and any positive number k, the inequality
holds
a k (a − b)(a − c) + b k (b − c)(b − a) + c k (c − a)(c − b) ≥ 0,
with equality for a = b = c, and for a = 0 and b = c (or any cyclic permutation).
For k = 1, we get the third degree Schur’s inequality, which can be rewritten as
follows
a3 + b3 + c 3 + 3abc ≥ ab(a + b) + bc(b + c) + ca(c + a),
(a + b + c)3 + 9abc ≥ 4(a + b + c)(ab + bc + ca),
9abc
a2 + b2 + c 2 + ≥ 2(ab + bc + ca),
a+b+c
(b − c)2 (b + c − a) + (c − a)2 (c + a − b) + (a − b)2 (a + b − c) ≥ 0.
Symmetric Power-Exponential Inequalities 503

For k = 2, we get the fourth degree Schur’s inequality, which holds for any real
numbers a, b, c, and can be rewritten as follows

a4 + b4 + c 4 + abc(a + b + c) ≥ ab(a2 + b2 ) + bc(b2 + c 2 ) + ca(c 2 + a2 ),

a4 + b4 + c 4 − a2 b2 − b2 c 2 − c 2 a2 ≥ (ab + bc + ca)(a2 + b2 + c 2 − ab − bc − ca),


(b − c)2 (b + c − a)2 + (c − a)2 (c + a − b)2 + (a − b)2 (a + b − c)2 ≥ 0,
6abcp ≥ (p2 − q)(4q − p2 ), p = a + b + c, q = ab + bc + ca.
A generalization of the fourth degree Schur’s inequality, which holds for any
real numbers a, b, c and any real number m, is the following (Vasile Cirtoaje, 2004)
X
(a − mb)(a − mc)(a − b)(a − c) ≥ 0,

where the equality holds for a = b = c, and for a/m = b = c (or any cyclic
permutation). This inequality is equivalent to
X X X X
a4 + m(m + 2) a2 b2 + (1 − m2 )abc a ≥ (m + 1) ab(a2 + b2 ),
X
(b − c)2 (b + c − a − ma)2 ≥ 0.
A more general result is given by the following theorem (Vasile Cirtoaje, 2008).
Theorem. Let
X X X X
f4 (a, b, c) = a4 + α a2 b2 + β abc a−γ ab(a2 + b2 ),

where α, β, γ are real constants such that 1 + α + β = 2γ. Then,


(a) f4 (a, b, c) ≥ 0 for all a, b, c ∈ R if and only if

1 + α ≥ γ2 ;

(b) f4 (a, b, c) ≥ 0 for all a, b, c ≥ 0 if and only if

α ≥ (γ − 1) max{2, γ + 1}.

7. CAUCHY-SCHWARZ INEQUALITY
If a1 , a2 , . . . , an and b1 , b2 , . . . , bn are real numbers, then

(a12 + a22 + · · · + an2 )(b12 + b22 + · · · + bn2 ) ≥ (a1 b1 + a2 b2 + · · · + an bn )2 ,

with equality for


a1 a an
= 2 = ··· = .
b1 b2 bn
Notice that the equality conditions are also valid for ai = bi = 0, where 1 ≤ i ≤ n.
504 Vasile Cîrtoaje

8. HÖLDER’S INEQUALITY
If x i j (i = 1, 2, · · · , m; j = 1, 2, · · · n) are nonnegative real numbers, then
‚ n Œ v !m
m
Y X n uY
X m
m
≥ .
t
xi j xi j
i=1 j=1 j=1 i=1

9. CHEBYSHEV’S INEQUALITY
Let a1 ≥ a2 ≥ · · · ≥ an be real numbers.

a) If b1 ≥ b2 ≥ · · · bn , then
n
‚ n
Œ‚ n
Œ
X X X
n ai bi ≥ ai bi ;
i=1 i=1 i=1

b) If b1 ≤ b2 ≤ · · · ≤ bn , then
n
‚ n
Œ‚ n
Œ
X X X
n ai bi ≤ ai bi .
i=1 i=1 i=1

10. CONVEX FUNCTIONS


A function f defined on a real interval I is said to be convex if

f (αx + β y) ≤ α f (x) + β f ( y)

for all x, y ∈ I and any α, β ≥ 0 with α + β = 1. If the inequality is reversed, then


f is said to be concave.
If f is differentiable on I, then f is (strictly) convex if and only if the derivative f 0
is (strictly) increasing. If f 00 ≥ 0 on I, then f is convex on I.
Jensen’s inequality. Let p1 , p2 , . . . , pn be positive real numbers. If f is a convex
function on a real interval I, then for any a1 , a2 , . . . , an ∈ I, the inequality holds

p1 f (a1 ) + p2 f (a2 ) + · · · + pn f (an ) p1 a1 + p2 a2 + · · · + pn an


 ‹
≥f .
p1 + p2 + · · · + pn p1 + p2 + · · · + pn

For p1 = p2 = · · · = pn , Jensen’s inequality becomes


a + a + ··· + a 
1 2 n
f (a1 ) + f (a2 ) + · · · + f (an ) ≥ n f .
n
Symmetric Power-Exponential Inequalities 505

11. KARAMATA’S MAJORIZATION INEQUALITY


Let f be a convex function on a real interval I. If a decreasingly ordered sequence

A = (a1 , a2 , . . . , an ), ai ∈ I,

majorizes a decreasingly ordered sequence

B = (b1 , b2 , . . . , bn ), bi ∈ I,

then
f (a1 ) + f (a2 ) + · · · + f (an ) ≥ f (b1 ) + f (b2 ) + · · · + f (bn ).
We say that a sequence A = (a1 , a2 , . . . , an ) with a1 ≥ a2 ≥ · · · ≥ an majorizes a
sequence B = (b1 , b2 , . . . , bn ) with b1 ≥ b2 ≥ · · · ≥ bn , and write it as

A  B,

if
a1 ≥ b1 ,
a1 + a2 ≥ b1 + b2 ,
·····················
a1 + a2 + · · · + an−1 ≥ b1 + b2 + · · · + bn−1 ,
a1 + a2 + · · · + an = b1 + b2 + · · · + bn .

12. SYMMETRIC INEQUALITIES OF DEGREE THREE, FOUR OR FIVE

Theorem (Vasile Cirtoaje, 2010) Let f n (a, b, c) be a symmetric homogeneous


polynomial of degree n.
(a) The inequality f4 (a, b, c) ≥ 0 holds for all real numbers a, b, c if and only if
f4 (a, 1, 1) ≥ 0 for all real a;
(b) For n ∈ {3, 4, 5}, the inequality f n (a, b, c) ≥ 0 holds for all a, b, c ≥ 0 if and
only if f n (a, 1, 1) ≥ 0 and f n (0, b, c) ≥ 0 for all a, b, c ≥ 0.

13. SYMMETRIC HOMOGENEOUS INEQUALITIES OF DEGREE SIX

Any sixth degree symmetric homogeneous polynomial f6 (a, b, c) can be written in


the form
f6 (a, b, c) = Ar 2 + B(p, q)r + C(p, q),
where A is called the highest coefficient of f6 , and

p = a + b + c, q = ab + bc + ca, r = abc.

Theorem (Vasile Cirtoaje, 2010). Let f6 (a, b, c) be a sixth degree symmetric


homogeneous polynomial having the highest coefficient A ≤ 0.
506 Vasile Cîrtoaje

(a) The inequality f6 (a, b, c) ≥ 0 holds for all real numbers a, b, c if and only if
f6 (a, 1, 1) ≥ 0 for all real a;
(b) The inequality f6 (a, b, c) ≥ 0 holds for all a, b, c ≥ 0 if and only if f6 (a, 1, 1) ≥
0 and f6 (0, b, c) ≥ 0 for all a, b, c ≥ 0.

This theorem is also valid for the case where B(p, q) and C(p, q) are homoge-
neous rational functions.
For A > 0, we can use the highest coefficient cancellation method (Vasile Cirtoaje,
2010). This method consists in finding some suitable real numbers B, C and D such
that the following sharper inequality holds
 2
q2
f6 (a, b, c) ≥ A r + Bp3 + C pq + D .
p

Because the function g6 defined by


 2
q2
g6 (a, b, c) = f6 (a, b, c) − A r + Bp3 + C pq + D
p

has the highest coefficient A1 = 0, we can prove the inequality g6 (a, b, c) ≥ 0 using
Theorem above.
Notice that sometimes it is useful to break the problem into two parts, p2 ≤ ξq
and p2 > ξq, where ξ is a suitable real number.
A symmetric homogeneous polynomial of degree six in three variables has the
form
X X X
f6 (a, b, c) = A1 a + A2
6
ab(a + b ) + A3
4 4
a2 b2 (a2 + b2 )
X X X
+A4 a3 b3 + A5 abc a3 + A6 abc ab(a + b) + 3A7 a2 b2 c 2 ,
where A1 , . . . , A7 are real constants. In order to write this polynomial as a function
of p, q and r, the following relations are useful:
X
a3 = 3r + p3 − 3pq,
X
ab(a + b) = −3r + pq,
X
a3 b3 = 3r 2 − 3pqr + q3 ,
X
a2 b2 (a2 + b2 ) = −3r 2 − 2(p3 − 2pq)r + p2 q2 − 2q3 ,
X
ab(a4 + b4 ) = −3r 2 − 2(p3 − 7pq)r + p4 q − 4p2 q2 + 2q3 ,
X
a6 = 3r 2 + 6(p3 − 2pq)r + p6 − 6p4 q + 9p2 q2 − 2q3 ,
(a − b)2 (b − c)2 (c − a)2 = −27r 2 + 2(9pq − 2p3 )r + p2 q2 − 4q3 .
Symmetric Power-Exponential Inequalities 507

According to these relations, the highest coefficient A of the polynomial f6 (a, b, c)


is
A = 3(A1 − A2 − A3 + A4 + A5 − A6 + A7 ).
The polynomials
X
P1 (a, b, c) = (A1 a2 + A2 bc)(B1 a2 + B2 bc)(C1 a2 + C2 bc),
X
P2 (a, b, c) = (A1 a2 + A2 bc)(B1 b2 + B2 ca)(C1 c 2 + C2 ab)
and
P3 (a, b, c) = (A1 a2 + A2 bc)(A1 b2 + A2 ca)(A1 c 2 + A2 ab)
has the highest coefficients

P1 (1, 1, 1), P2 (1, 1, 1), P3 (1, 1, 1),

respectively. The polynomial

P4 (a, b, c) = (a2 + mab + b2 )(b2 + mbc + c 2 )(c 2 + mca + a2 )

has the highest coefficient


A = (m − 1)3 .

14. VASC’S POWER EXPONENTIAL INEQUALITIES

Theorem. Let 0 < k ≤ e.


(a) If a, b > 0, then (Vasile Cîrtoaje, 2006)

a ka + b kb ≥ a kb + b ka ;

(b) If a, b ∈ (0, 1], then (Vasile Cîrtoaje, 2010)


p
2 a ka b kb ≥ a kb + b ka .
508 Vasile Cîrtoaje
Appendix B

Bibliography

[1] Andreescu T., Cîrtoaje V., Dospinescu G., Lascu M., Old and New Inequalities,
GIL Publishing House, 2004.
[2] Bin X., Boreico I., Can V.Q.B., Bulj A., Lascu M., Opympiad Inequalities, GIL
Publishing House, 2015.
[3] Bin X., Boreico I., Can V.Q.B., Cîrtoaje V., Lascu M., An Introduction to Inequali-
ties, GIL Publishing House, 2015.
[4] Can V.Q.B., Pohoaţă C., Old and New Inequalities, GIL Publishing House, 2008.
[5] Can V.Q.B., Anh T. Q., Su Dung Phuong Phap Cauchy-Schwarz De Chung Minh
Bat Dang Thuc, Nha Xuat Ban Dai Hoc Su Pham, 2010.
[6] Cîrtoaje V., Asupra unor inegalitati cu restrictii, Revista Matematica din Timisoara,
Nr. 1, pag. 3-7, 1990.
[7] Cîrtoaje V., A Generalization of Jensen’s Inequality, Gazeta Matematica-A, Nr. 2,
pp. 124-138, 2005.
[8] Cîrtoaje V., Algebraic Inequalities-Old and New Methods, GIL Publishing House,
2006.
[9] Cîrtoaje V., The Proof of Three Open Inequalities, Crux Mathematicorum, Volume
34, Issue 4, pp. 231-238, 2008.
[10] Cîrtoaje V., Can V. Q. B., Anh T. Q., Inequalities with Beautiful Solutions, GIL
Publishing House, 2009.
[11] Cîrtoaje V., On Some Inequalities with Power-Exponential Functions, Journal of
Inequalities In Pure and Applied Mathematics, Volume 10, Issue 1, 2009.
[12] Cîrtoaje V., The Best Lower Bound Depended on Two Fixed Variables for Jensen’s
Inequality with Ordered Variables, Journal of Inequalities and Applications, Volume
2010.
[13] Cîrtoaje V., Proofs of Three Open Inequalities with Power-Exponential Functions,
Journal of Nonlinear Sciences and Applications, Volume 4, no. 2, 2011.
[14] Cîrtoaje V., The Best Upper Bound for Jensen’s Inequality, Australian Journal of
Mathematical Analysis and Aplications, Volume 7, Issue 2, Art. 22, 2011.

509
510 Vasile Cîrtoaje

[15] Cîrtoaje V., Necessary and Sufficient Conditions for Symmetric Homogeneous
Polynomial Inequalities of Degree Four and Six in Real Variables, Journal of Nonlinear
Science and Applications, Volume 5, Issue 5, 2012.
[16] Cîrtoaje V., Necessary and Sufficient Conditions for Symmetric Homogeneous
Polynomial Inequalities in Nonnegative Real Variables, Mathematical Inequalities and
Applications, Volume 16, No. 2, 2013.
[17] Cîrtoaje V., The Best Lower Bound for Jensen’s Inequality with three fixed ordered
variables Journal, Banach Journal of Mathematical Analysis, Volume 7, Issue 1,
2013.
[18] Cîrtoaje V., A Strong Method for Symmetric Homogeneous Polynomial Inequali-
ties of Degree Six in Nonnegative Real Variables, British Journal of Mathematical and
Computers Science, 4(5), 2014.
[19] Cîrtoaje V., Mathematical Inequalities - Volumme 1, Symmetric Polynomial In-
equalities, Lambert Academic Publishing, 2018.
[20] Cvetkovski Z., Inequalities: Theorems, Techniques and Selected Problems, Springer-
Verlag Berlin Heidelberg, 2012.
[21] Drimbe M.O., Inegalitati - Idei si Metode, GIL Publishing House, 2003.
[22] Engel A., Problem-Solving Strategies, Springer, 1998.
[23] Hung P. K., Secrets in Inequalities, Volume 1: Basic Inequalities, GIL Publishing
House, 2007.
[24] Hung P. K., Secrets in Inequalities, Volume 2: Advanced Inequalities, GIL Pub-
lishing House, 2008.
[25] Littlewood G. H., Polya, J. E., Inequalities, Cambridge University Press, 1967.
[26] Mitrinovic̆ D. S., Pecaric̆ J. E., Fink A. M., Classical and New Inequalities in
Analysis, Kluwer, 1993.
[27] Panaitopol L., Băndilă V., Lascu M., Inegalităţi, GIL Publishing House, 1995.

You might also like